You are on page 1of 260

Chapter 1 Pronouns

Pronouns
1- Case 2- Number 3- Antecedent

1- Case
Reflexive &
Subjective Case Objective Case Possessive case
Intensive pronouns
I me my mine myself
we us our ours ourselves
you you your yours yourself / yourselves
they them their theirs themselves
he him his his himself
she her her hers herself
it it its its itself

Indefinite Pronouns
Singular Plural
someone everything all
anyone no one some
everyone one both
somebody each few
anybody neither many
everybody either most
something none several
anything much

Demonstrative Pronouns
Singular Plural
this that these those

Relative Pronouns
who whom which that whose

Top Score
GET READY TO STUDY
Chapter 1 Pronouns

Subjective Case VS Objective Case


Subjective case VERB Objective case
Examples:

• My sister, my brother and I invited them to the party.

• The article cited Brown and her as the best examples.

Exercises:

1. He / him and his family are looking for a new home.


2. It is difficult to see how the rule will affect you and I / me.
3. My parents said that my sister had taken both my brother and they / them to the new
restaurant.
4. Ask whether, after all our hard work, Suzie and I / me are adequately prepared for the
competition.

SAT Question:
The owners hired my husband and I to manage the inn because we had more experience than the
other applicants.
A) No change
B) my husband and my
C) my husband and me
D) both my husband and I

Than / as + Subjective Case


Examples:

• He works harder than she.


• Nader is as fast as I

Exercises:

1. Now that they have won the lottery, they are as rich as we / us.
2. They received the news later than she / her.
3. John is as tall as Susan; she is younger than he / him.
Top Score
GET READY TO STUDY
Chapter 1 Pronouns

SAT Questions:
My grandmother leads a more active life than many other retirees who are younger than her.
(A) No change
(B) them
(C) her or him
(D) she

Preposition + Objective Case


*Prepositions: (by – to – with – from – in – at – about – against – except, etc.)

Examples:

• Mother told father about Tom and me.


• I think Joan has an eye on her.

Exercises:

1. Everyone except he / him was well prepared for the test.


2. They received the news from the neighbor before they heard from my girlfriend and I /
me.
3. I gave the new book to a friend who promised to deliver it to we / us eager readers.
SAT Question:
By order of the Student Council, the wearing of slacks by we girls in school has been permitted.
(A) No change
(B) us girls
(C) ours girls
(D) most of we girls

Who VS Whom
*Both refer to persons
*Who / whoever + verb
*Whom / whomever + subject + verb
Examples:

• They will give the award to whoever deserves it.


• I watched the man whom you defeated yesterday.

Top Score
GET READY TO STUDY
Chapter 1 Pronouns

Exercises:

1. We did not know who / whom would be able to repair the hard drive.
2. They will take the money from whoever / whomever they think most able to afford it.
3. The lawyer who / whom you chose to represent you is doing an excellent job.
SAT Questions:
The novelists who readers choose as their favorites are not always the most skilled writers.
(A) No change
(B) whom are reading
(C) whom readers
(D) who readers may

Who VS Which/That
*WHO refers to persons whereas WHICH & THAT refer to things
Examples:
• People who live in glass houses shouldn't throw stones.
• Walden Pond, which was written in the mid 1800s, remains a popular book among
romantic and individualistic Americans.
Exercises:
1. She prefers to watch movies which / who make her cry.
2. He bought all the books that / who are required for the course.
3. Tashonda Viereck's children, which / who all graduated from college, came home for her
eighteenth birthday.
SAT Questions:
1- The volleyball players which arrived for the team picture in the uniforms that they had worn to
practice were irritable and tired.
(A) No change
(B) whom arrived
(C) which arrives
(D) who arrived

2- The advertisement for Neil Simon's play Rumors features three sets of mechanical false teeth who
appear to be talking to each other.
(A) No change
(B) which appear
(C) whom appear
(D) whom they appear

Top Score
GET READY TO STUDY
Chapter 1 Pronouns

Its OR It's
*None of the possessive forms of pronouns have apostrophes.

Its  possessive case: My wallet was lost; its color is brown.

It's  it is: It will rain because it's cloudy.

Exercises:

1. Its / it’s my birthday today.


2. The cat carried it's / its kitten in it's / its mouth.
3. It's / its one of the hardest courses in it's / its history
SAT Questions:
1- The Watergate scandal may be a thing of the past but the Republicans will feel it's effects for a long
time to come.
(A) No change
(B) its effects
(C) their effects
(D) there effects

Their OR They're OR There


Their  possessive case:
The students are very worried waiting to know their score.
They're  They are + Adj / Gerund / Noun
The students are very worried because they're waiting to know their scores.
There  Adverb Or Pronoun
*Adverb: You cannot park your car here, but it is better to park it there.
*Pronoun: It may rain today because there is much fog.
SAT Question
Perhaps the most extra ordinary feature of the African elephants is there amazingly versatile trunk.
(A) No change
(B) their
(C) they're
(D) theirs

Top Score
GET READY TO STUDY
Chapter 1 Pronouns

2- Number
• Pronoun/noun and pronoun/noun (plural)
EX: Ahmed and Ali are coming now.
• Pronoun/noun or pronoun/noun (singular)
EX: Ahmed or Ali is coming now.
• None of / no one / everyone / anyone / each,etc. (singular)

Exercises:
- Although Yara and Sara were sick, (she–they) came to work yesterday.
- The students jumped from (her or his–their) places after they had known the scores.
- Each of the applicants presented (his or her–their) required certificates.
SAT Questions:
1- I sincerely believe that a person intelligent enough to be in business by themselves should have the
ability to recognize a dangerous investment.
(A) No change
(B) by them
(C) by himself
(D) with his own skills

2- The survey indicated that workers in the United States hope that his or her wages will
keep pace with the rising cost of living.
(A) No change
(B) one's
(C) there
(D) their

• This / that + singular (This book–that dog, etc.)


• These / those + plural (These flowers / those boys, etc.)

SAT Questions:
Most students like to read these kind of books during their spare time.
(A) No change
(B) these kind of book
(C) this kind of books
(D) this kinds of books

Top Score
GET READY TO STUDY
Chapter 1 Pronouns

3- Antecedent
(Ambiguous pronoun – Confusing pronoun – pronoun shifting)

Ambiguous Pronoun
Examples:
- They went to Alex in the previous vacation.
They this is a pronoun without clear reference
*To solve the AMBIGUITY remove the pronoun OR name it
SAT Questions:
1- The Socialist Party is now powerful enough to worry the governing Christian Democrats, and they
may win fifty seats in the next election.
(A) No change
(B) and they might win
(C) and the Socialists may win
(D) and the party
2- In the Trenton News, they report that there are as many as 10,000 homeless people in Washington
DC.
(A) No change
(B) In the Trenton News, they issued the report that
(C) The Trenton News reports that
(D) The Trenton News makes the report that

Confusing pronoun

• Singular noun, singular noun ……………. Singular pronoun ×


Example:
- Khaled waited for Sam in his car. ×

SAT Questions:
Neither Ms. Perez nor Ms. Tanaka believes that watching as much television as her son Sam does
will lead to anything productive.
(A) No change
(B) hers
(C) their son Sam
(D) Tanaka's son Sam

Top Score
GET READY TO STUDY
Chapter 1 Pronouns

• Plural noun, plural noun ……………. plural pronoun ×


Example:

- I think bananas are better than apples because they have much Potassium. ×
SAT Questions:
Bears, mountain lions, beavers, dear, squirrels and coyotes inhabit Sequoia Park, and they may be
dangerous to campers.
(A) No change
(B) and it
(C) mountain lions
(D) and them

• Singular noun, Singular noun ……………. plural pronoun √


Examples:
- My cousin invited my brother to go on picnic, but they didn't ask me to come with
them. √

SAT Questions:
Neither Roger nor his sister was able to understand what the conductor was saying because they did
not know how to speak French.
(A) No change
(B) them
(C) one
(D) him

Pronoun shifting
Examples:

- Even after you have endured a cold winter in subzero weather, you find it possible to
become acclimated to tropical temperatures in the summer.
- If Tom wins the lottery jackpot, he will have enough money to pay off his debt.
- You will be able to get into the cinema even if you arrive before the film starts.
- The Ancient Egyptians built pyramids as tombs for the pharaohs and their queens, and
they shaped them in many different styles.

Top Score
GET READY TO STUDY
Chapter 1 Pronouns

Exercises: Find the mistake and correct it


- If we linked our ideas, you / we would get good results. ……………….
- Even though Bob studied very hard, we / he still failed in French tests. ……………

SAT Questions:
If we plan carefully and waste no time on irrelevant topics, you should be able to
finish the essay in twenty minutes.
(A) No change
(B) you
(C) he
(D) one

Walking on coral from their earliest years, the Kitawans develop soles as tough as
leather, and they find the tender feet of westerners very odd.
(A) No change
(B) them
(C) it
(D) one

NB:

 , Which √ , that ×
 Where & in which & in that refer to places.
- I went to The United States where I enjoyed my time.
 When refers to time.
- She wrote the first novel when she was twelve.

Top Score
GET READY TO STUDY
Chapter 1 Pronouns

Practices
1- Therefore, I admire organizations that speak up 6- If westerners acknowledge that the eastern
for fairness and democratic principles, even if it United States has wilderness areas, one probably
is motivated by selfishness. thinks of the Blue Ridge Mountains or perhaps
(A) No change Maine.
(B) even when their motives are selfish. (A) No change
(C) even when the motive is to be for its own (B) one may think
selfish aims. (C) you probably think
(D) whether or not it's for their own selfish (D) they probably think
aims.
7- Many of the compounds that can be produced
2- Encouraged by the success of its weekly from the leaves of this plant are dangerous, but
Spanish-language programs, MTV will launch a the plant themselves cannot be called toxic.
twenty-four hour Spanish language cable (A) No change
network. (B) the plant itself
(A) No change (C) the plant oneself
(B) it's (D) the plant himself
(C) their
(D) them 8- If one digs beneath the snow in the garden, you
may find the scarlet flowers of the hardy
3- When the new ambassador first arrived cyclamen.
in the capital, though late in May, they found
(A) No change
that the large lakes were still frozen.
(B) you may come upon
(A) No change
(C) one may find
(B) them found
(D) we may find
(C) him found
(D) he found
9- Everywhere you look in New York City,
4- That Naomi, the book's main character and one sees the white brick apartment buildings
possibly its hero, is a mere child during the that sprang up after the war like mushrooms.
Revolution makes the story even more appealing (A) No change
to young readers. (B) one will see
(A) No change (C) they saw
(B) it's (D) you see
(C) their
(D) them 10- In English literature James Boswell is the prime
example of a biographer who, by ensuring the
5- If the election results are as Harris predicts, the immortality of another author, has achieved
new senator will be the man which the people immortality for himself.
believed made the better showing in the (A) No change
televised debate. (B) oneself
(A) No change (C) yourself
(B) whom people (D) themselves
(C) who people
(D) which is

Top Score
GET READY TO STUDY
Chapter 1 Pronouns

11- In Hawaii, they are emphasizing the Hawaiian 16- Psychologists advise that before making any
language as a part of a renaissance in the native major changes in your life, a person needs to
culture, including music and dance. focus on one's goals.
(A) No change (A) No change
(B) the emphasis on the Hawaiian language is (B) their life, a person needs to focus on their
part of goals.
(C) they are putting emphasis on Hawaiian as a (C) one's life, you should focus on your goals.
language as a part of (D) their lives, people should focus on their
(D) the emphasis on the Hawaiian language is to goals.
them a part of
17- As the mayor was evaluating the proposed tax,
12- Zookeepers have expanded one's definition of he was less interested in the revenue it
care to include concern for the animal's mental would generate than in whether they would
state as well as for its physical well-being. disproportionately affect certain income groups.
(A) No change
(A) No change (B) whether them would
(B) their (C) whether it would
(C) there (D) whether this would
(D) our
18- Ethel Payne, the tough investigative reporter
13- Last summer, when Mary's aunt and uncle which is well known for her coverage of the
flew from Turkey to visit their relatives and tour Civil Rights movement, first received critical
the United States, Mary invited Sandhya and I to acclaim for a diary she kept while living in
her house to meet them. Japan after the Second World War.
(A) No change (A) No change
(B) and my (B) which are
(C) and me (C) who is
(D) and mine (D) whom are

14- Because the American Indian rodeo includes 19- The plans proposed by the students for the
games and exhibitations developed as early as home-combing celebration are perfectly clear
the seventeenth century, they predate by a few and it is well within our budget; however, the
hundred years the form of rodeo now seen on plans are not likely to be approved by the
television. administration.
(A) No change (A) No change
(B) them predate (B) and they are
(C) this predate (C) and them are
(D) it predates (D) and one is

15- Hearing the unexpected loud noise, Cindy, 20- During rehearsals, the director praised the actors
Leroy, and me were so startled that we almost which had supporting roles more often than
jumped out of our seats. those with the most demanding roles.
(A) No change (A) No change
(B) Cindy, Leroy, and me was (B) whom had
(C) Cindy, Leroy, and I were (C) which have
(D) Cindy, Leroy; and I was (D) who had

Top Score
GET READY TO STUDY
Chapter 1 Pronouns

21- Our modern solar calendar, established in 1582, 26- The governor favored the new laws because
is based on the Julian calendar, Julius Caesar it would allow the state to save more than
introduced it in 46 B.C. ten million dollars.
(A) No change (A) No change
(B) calendar; then Julius Caesar introduced it (B) its
(C) calendar, with an introduction by Julius (C) they
Caesar (D) them
(D) calendar, introduced by Julius Caesar 27- The success of the governor in attracting
new industry to the rural area can be
22- The African violet is among the most popular attributed to their low cost of living.
houseplants because their flowers bloom (A) No change
throughout the year and come in a wide array of (B) them
colors. (C) it
(A) No change (D) its
(B) it's flowers 28- It is believed that small fish cluster together
(C) its flowers when confronted by a predator in order to
(D) there flowers confuse them and thereby protect themselves
from harm.
23- Though delightful to the adult palate, the flavors (A) NO CHANGE
of the dishes are pungent and spicy, so Nadia (B) they
takes care not to feed it to very young children. (C) it
(A) No change (D) it's
(B) it's 29- Recently adapted as emblem of humanitarian
(C) they aid, the Red Crystal is, unlike its
(D) them counterparts the Red Cross and the Red
24- Biologists fear that if the northern snakehead Crescent, entirely devoid of religious
fish establishes itself in the United States, they connotation.
will join other introduced species that threaten (A) NO CHANGE
native plants and animals in and around water. (B) it's
(A) No change (C) their
(B) them (D) there
(C) it 30- France is well-known throughout the world
(D) this for their numerous mountain ranges, which
25- Perhaps the most extra ordinary feature of the include the Alps, the Pyrenees, and the
African elephant is their amazingly versatile Vosges.
trunk. (A) NO CHANGE
(E) No change (B) there
(F) there (C) its
(G) its (D) it's
(H) it's

Top Score
GET READY TO STUDY
Chapter 1 Pronouns

Homework
1. Ocean currents that start in the warm waters of 5. Since it opened in 1956, The Zelinski restaurant
the Pacific and Indian Oceans are initially low in has become famous for their unusual fare and its
salt content but become more saline as it flows elaborately decorated dining room.
northward into colder regions. (A) No change
(A) No change (B) them
(B) it is flowing (C) it's
(C) they flow (D) its
(D) them flow 6. The codfish inhabits the shallow waters of the
2. In addition to having a theoretical understanding ocean; they are especially plentiful in such areas
of disease, medical students must have as the North American continental shelf, where
experience in treating patients if he or she is the depth is rarely greater than 200 or 300 feet.
to learn to diagnose illnesses accurately. (A) No change
(A) No change (B) them are
(B) them are (C) it is
(C) they are (D) its
(D) each are 7. Though I am acquainted with Mr. Bartholomew
3. Professor Corey, a man genuinely interested in and have long known of his interest in the
others ideas, told the students in his government painting that he has gone to such lengths
class that they were free to voice all of their to obtain it astonishes me.
opinions, no matter how controversial it was. (A) No change
(A) No change (B) them astonishes me
(B) it is (C) it astonishes I
(C) its was (D) them astonishes I
(D) they were 8. My mother always gave my sister and I the same
4. The United States Supreme Court is the treats when we were little so that neither of us
final interpreter of the constitution, and they have would be jealous of the other.
the power to declare a state law unconstitutional. (A) No change
(A) No change (B) my sister and me
(B) it has (C) my sister and they
(C) them have (D) my sister and my
(D) those have
Top Score
GET READY TO STUDY
Chapter 1 Pronouns

9. Everyone who attends a concert at the sports 14. The Governor asked the attorney to head the
arena Knows that they will be searched for drugs committee because he was convinced that the
committee needed to start work immediately.
before entering.
(A) No change
(A) No change
(B) because he is
(B) they are
(C) because the attorney was
(C) he or she will be
(D) because of them
(D) you will be
15. As an incentive to attend the local college, our
10. Most gardeners like to cultivate these kind of father told my brother and I that we could use
flowers in the early spring. his company car for transportation.
(A) No change
(A) No change
(B) my brother and me
(B) these kind of flower
(C) both my brother and I
(C) those kind of flower
(D) we
(D) this kind of flowers
16. All the aspiring young writers submitted their
11. Its my opinion that learning the correct stories, each hoping that they would win first
pronunciation should precede attempt to learn prize.
the correct spelling of a word. (A) No change
(A) No change (B) their
(B) It's (C) them
(C) Those are (D) he or she
(D) They are 17. Since we first started high school, there has been
12. Every typist in the office except she was out sick great competition for grades between him and I.
at least one day during the past month. (A) No change
(A) No change (B) between he and I
(B) except her (C) between him and me
(C) but not her (D) between he and me
(D) apart from she 18. The man who Mexican authorities believe to be
13. I appreciate you offering to help me with my the country's number I drug trafficker
research project, but the honor system prevents has been arrested in a Pacific resort area.
students from giving and receiving assistance. (A) No change
(A) No change (B) whose
(B) yours (C) whom
(C) your (D) which
(D) you are

Top Score
GET READY TO STUDY
Chapter 1 Pronouns

19. If you still due west from the city of Perth in 23. For much of the last century, the building
Australia, one will not encounter land until the industry relied heavily on the availability of
coast of Africa is reached. timber, but now that forest resources are being
(A) NO CHANGE depleted rapidly, builders must decide what to
(B) you do about it.
(C) we (A) NO CHANGE
(D) they (B) its
20. Scientists and researchers which are respected in (C) them
their fields have conducted studies indicating (D) they
that the good feelings produced by eating 24. The conscientious board of directors has done
chocolate may be attributed to its effect on a everything it can to address the issues raised by
brain chemical called an andamide. their critics.
(A) NO CHANGE (A) NO CHANGE
(B) whom are (B) your
(C) who are (C) one's
(D) who is (D) its
21. Although she knew that the seat was reserved 25. John and Bob had been friends since
for someone, Mary interrupted the conversation kindergarten because he was good at soccer
between John and I to ask if she could sit with
and ready to laugh at any joke.
us.
(A) NO CHANGE
(A) NO CHANGE
(B) between John and me
(B) he or she
(C) between John and myself (C) each
(D) between both John and I
(D) they
22. France is well-known throughout the world for 26. Although polls showed that many voters felt
their numerous mountain ranges, which include sympathetic toward the embattled politician, he
the Alps, the Pyrenees, and the Vosges. or she voted him out of office in the next
(A) NO CHANGE election.
(B) there (A) NO CHANGE
(C) its (B) them
(D) it's (C) they
(D) he

Top Score
GET READY TO STUDY
Chapter 1 Pronouns

27. African grey parrots particularly noted for their 30. Users of advanced software often think that the
cognitive abilities, which are believed to have initial sections of their support manuals provide
evolved as a consequence of it's history of them with all the information you will ever
cooperative feeding on the ground. need.
(A) NO CHANGE (A) NO CHANGE
(B) it (B) I
(C) their (C) they
(D) one's (D) he or she
28. Before the election, there was a long period of 31. The tour guide warned us to lower their heads
disagreement among my friends and me, when entering the old castle, since the old
because we all had conflicting opinions from doorway was very low.
which we would not be swayed. (A) No changes
(A) NO CHANGE (B) one's heads
(C) your head
(B) they
(D) our heads
(C) he
(D) it
29. The first public buildings to have air-
conditioning were often movie theaters and
restaurants, whose owners tried to attract
customers by promising that you would be
cooler inside.
(A) NO CHANGE
(B) your
(C) they
(D) he or she

Top Score
GET READY TO STUDY
Chapter 1 Pronouns
Mini Exam
. 1 .. .
Dying Woman Seeks Home For Dogs
(A) NO CHANGE
This is What's Trending Today: (B) there
(C) their
Wednesday was National Puppy Day. Lots of (D) it's
people used social media to post adorable photos
. 2 .. .
of . 1 . our furry companions. Americans
(A) NO CHANGE
definitely love . 2 . there dogs. But what happens (B) their
(C) they
when dogs outlive . 3 . its owners? (D) them
That is going to happen to Kathleen
. 3 .. .
Zuidema, a dog-lover from the U.S. state of
(A) NO CHANGE
Washington. Zuidema has lung cancer and does (B) their
(C) it's
not expect to live much longer. . 4 . She has three (D) they
dogs, Australian shepherds. Zuidema is making
. 4 .. .
plans for the end of . 5 . their life, and . 6 . they
(A) NO CHANGE
wrote . 7 . these post on Facebook in March:"My (B) her
(C) they
heart is breaking and I'm sobbing as . 8 . me write (D) hers
this …. "
. 5 .. .

(A) NO CHANGE
(B) her
(C) our
(D) your

. 6 .. .

(A) NO CHANGE
(B) her
(C) she
(D) them

. 7 .. .

(A) NO CHANGE
(B) they
(C) this
(D) those

. 8 .. .

(A) NO CHANGE
(B) I
(C) my
(D) you

Top Score
GET READY TO STUDY
Chapter 1 Pronouns

Zuidema said . 9 . they needs someone to take


. 9 .. .
all three dogs, and not break .10. they up. The new
(A) NO CHANGE
owner must have a large yard for the dogs to run, (B) he or she
jump and play. The yard must have a fence so the (C) one
(D) she
dogs are safe and cannot get loose. But Zuidema
.10.. .
lives on a small island in the cold waters off the
(A) NO CHANGE
coast of Washington state. (B) her
There are few people on San Juan Island .11. (C) they
(D) them
who would be able to take in three full-grown
.11.. .
dogs together. For about three weeks, Zuidema′s
(A) NO CHANGE
post was passed from person to person. It was (B) which
shared more than 30,000 times. Neighbors being (C) that
(D) Delete the underlined portion.
neighbors, another islander, Danielle Cochran,
.12.. .
walked over to Zuidema′s home and asked if she
(A) NO CHANGE
could help. She did not know Zuidema, but .12. (B) her
they knew her dogs from seeing .13. them around (C) she
(D) hers
town.
.13.. .
Zuidema′s last wishes will not go
(A) NO CHANGE
unanswered. A local television station says (B) their
Cochran and .14. their husband will take the two (C) our
(D) your
younger dogs soon. But the third, and oldest, will
.14.. .
stay at .15. their side on .16. her deathbed until
(A) NO CHANGE
Zuidema dies. "I got everything I wanted ... except (B) her
for the dying part," Zuidema says. And that′s (C) they
(D) them
what′s trending today.
.15.. .

(A) NO CHANGE
(B) they
(C) these
(D) her

.16.. .

(A) NO CHANGE
(B) me
(C) my
(D) you

Top Score
GET READY TO STUDY
Chapter 1 Pronouns

Answers
Explanation:
Page 5: He / me / them / I / C / we / she / he

Page 6: D / him / me / us / B

Page 7: Who / whomever / whom / C / which / that / who / D / B

Page 8: It's / its - its / it's - its / B / B

Page 9: They / their / his or her / C / D / C

Page 10: C / C / D

Page 11: C / A / we / he

Page 12: B / A

Practice:
1 B / 2 A / 3 D / 4 A / 5 B / 6 D / 7 B / 8 C / 9 D / 10 A / 11 B / 12 B / 13 C / 14 D / 15 C

16 D / 17 C / 18 C / 19 B / 20 D / 21 D / 22 C / 23 D / 24 C / 25 C / 26 C / 27 D / 28 C / 29 A / 30 C

Homework:
1 C / 2 C / 3 D / 4 B / 5 D / 6 C / 7 A / 8 B / 9 C / 10 D / 11 B / 12 B / 13 C / 14 C / 15 B / 16 D

17 C / 18 C / 19 B / 20 C / 21 B / 22 C / 23 C / 24 D / 25 C / 26 C / 27 C / 28 A / 29 C / 30 C / 31 D

Mini Exam:
1 C / 2 B / 3 B / 4 A / 5 B / 6 C / 7 C / 8 B / 9 D / 10 D / 11 A / 12 C / 13 A / 14 B / 15 D / 16 A

Top Score
GET READY TO STUDY
Chapter 2 Modifiers

Modifiers
*Simply, MODIFIER is a phrase modifies a word in 3 ways:
A) Phrase ………… , Subject + Verb
The phrase here modifies the Subject after the Comma.
EX:
- Walking in the park, some photos were taken by Sara. X
Walking in the park, Sara t s e t s √

- A brilliant song writer who is able to adjust her style as she ages, the songs of
Madonna always seem to fit with the music of the time. X
- A brilliant song writer who is able to adjust her style as she ages, Madonna always
rites s ngs t t it it t e si t e ti e √

B) Subject , Phrase …………… , Verb


The phrase here modifies the Subject before the Comma.
EX:
- Some photos, walking in the park, were taken by Sara. X
- Sara, walking in the park, t s e t s √

- The songs of Madonna, a brilliant song writer who is able to adjust her style as she
ages, always seem to fit with the music of the time. X
- Madonna, a brilliant song writer who is able to adjust her style as she ages, always
rites s ngs t t it it t e si t e ti e √

C) Subject + Verb + Object Phrase ……………………


The phrase here modifies the Object (without comma).
- I saw some people riding horses smoking cigarettes. X

D) Subject + Verb + Object ( , or linker ) + Phrase ……………………


The phrase here modifies the Subject or the Object.
EX:
- I saw some people riding horses while smoking cigarettes. √
- Dorothy Crowfoot Hodgkin used X-Ray to discover the chemical structures of
penicillin and vitamin B12, winning a Noble Prize for her work. √

Top Score
GET READY TO STUDY
Chapter 2 Modifiers

- The domed shell of a box turtle is hinged at the bottom allowing the animals to close
their shell tightly to evade predators. X
- The domed shell of a box turtle is hinged at the bottom, allowing the animals to close
their shell tightly to evade predators. √
- Aunt Marian prepared her garden with unusual care disappointed by the results of
last year's plantings and determined to do better this year. X
- Aunt Marian prepared her garden with unusual care, disappointed by the results of
last year's plantings and determined to do better this year. √
- Aunt Marian, disappointed by the results of last year's plantings and determined to
do better this year, prepared her garden with unusual care √

- Frustrated and bored by rambling lectures, Professor Moore's introductory philosophy


course was dropped by many students before the middle of the semester.
(A)NO CHANGE
(B) Before the middle of the semester, many students, frustrated and bored by rambling
lectures dropped Professor's Moore's introductory philosophy course
(C) Before the middle of the semester, many students dropped introductory philosophy
course of Professor's Moore frustrated and bored by rambling lectures
(D)Before the middle of the semester, Professor Moore's introductory philosophy
course was dropped by many students frustrated and bored by rambling lectures

Kinds of Modifiers:
1. Gerund Phrase (A phrase beginning with GERUND):
- Having no additional funds to spend, the meeting of the budget committee was
promptly concluded.
(A) No change
(B) The conclusion of the budget committee meeting was prompt.
(C) The budget committee promptly concluded its meeting.
(D) The budget committee's meeting was promptly concluded.

2. Participle Phrase (A phrase beginning with past participle):


- Burdened with three pieces of luggage and a pair of skis, Sarah's search for a baggage
cart was desperate.
(A) No change
(B) Sarah's desperate search was for a baggage cart.
(C) A baggage cart was what Sarah desperately searched for.
(D) Sarah searched desperately for a baggage cart.

Top Score
GET READY TO STUDY
Chapter 2 Modifiers

3. Prepositional Phrase (A phrase beginning with a preposition):


- By investigating Saturn's rings, it is hoped by scientists to achieve greater insight into
the composition of stars and planetary systems.
(A) No change
(B) Scientists hope to achieve greater insight
(C) It is hoped that scientists' insight will be achieved more greatly achieved
(D) The insight of scientists will be achieved more greatly, hopefully
- Of all the people playing roles in the musical, mine is the smallest part.
(A) No change
(B) the smallest part is mine
(C) I have a part smaller than theirs
(D) I have the smallest part

4. Definition:
EX: A / an + noun + who / which / that / where / gerund + verb
- A writer who well understood the plight of the underprivileged, many acclaim Richard
Wright as the novelist of the downtrodden.
(A) No change
(B) A writer who well understood the plight of the underprivileged, many have
acclaimed Richard Wright as the novelist of the downtrodden.
(C) Richard Wright, a writer who well understood the plight of the underprivileged,
has been widely acclaimed as the novelist of the downtrodden.
(D) Many, a writer who well understood the plight of the underprivileged, acclaim
Richard Wright as the novelist of the downtrodden.

The most / the best / the first + noun + to / once / as a / as an


- The most notorious quick-draw gunman of the Texas frontier, John Wesley Hardin's
exploits were written about in an autobiography published posthumously.
(A) No change
(B) John Wesley Hardin's exploits are recorded
(C) John Wesley Hardin wrote about his exploits
(D) the exploits of John Wesley Hardin were written about

Top Score
GET READY TO STUDY
Chapter 2 Modifiers

- As a student at Penn State, where Julie grew to love history with a passion, ultimately
leading to a job as a historian at the United States Library of Congress.
(A) No change
(B) Julie grew to love history with a passion that ultimately led
(C) Where she grew to love history with a passion, as a result leading Julie ultimately
(D) Julie's love for history grew with a passion and it ultimately led her

5. Infinitive Phrase (To + the base of the verb: to go / to write / to


solve…..etc):
- To help freshman and sophomores in selecting their courses, candid reviews of
courses and instructors compiled by juniors and seniors.
(A) No change
(B) And to compile candid reviews of courses and instructors by juniors and seniors
(C) Juniors and seniors have compiled candid reviews of courses and instructors
(D) With juniors and seniors compiled candid reviews of courses and instructors

6. Adjective Phrase (A phrase beginning with adjective):


- Familiar with the terrain from previous visits, the explorer's search for the abandoned
mine site was a success.
(A) No change
(B) the success of the explorer's search for the abandoned mine site was assured
(C) the explorer succeeded in finding the abandoned mine site
(D) the explorer in his search for the abandoned mine site was a success

7. Linker + Phrase:
- When creating collections of new clothing, styles of the past have often influenced
contemporary fashion designers.

(A) No change
(B) styles of the past often have an influence on contemporary fashion designers.
(C) contemporary fashion designers have often been influenced from styles of the past.
(D) contemporary fashion designers are often influenced by styles of the past.

Top Score
GET READY TO STUDY
Chapter 2 Modifiers

- After watching Jaws for the fifth time, the ocean looked incredibly menacing to
tourists Kathleen and Kevin.

(A) No change
(B) After watching Jaws for the fifth time, incredibly menacing the ocean looked to
tourists Kathleen and Kevin.
(C) The ocean looked incredibly menacing to tourists Kathleen and Kevin who had
just watched Jaws for the fifth time.
(D) Tourists Kathleen and Kevin, after watching Jaws for the fifth time, thought that
the ocean looked incredibly menacing.

8. Comparisons:
Using: like / unlike / similar to / compare to …..
EX:
- Compared to South Carolina, the climate of Arizona is drier. X
- Compared to the climate of South Carolina, the climate of Arizona is drier. √
- Unlike Egypt, the flag of Jamaica is more colorful. X
- Unlike that of Egypt, t e g i is re r √

- Unlike several decades ago, today's librarians teach students to evaluate the accuracy
and objectivity of online resources in addition to helping them find particular books.

(A) No change
(B) Unlike them of
(C) Unlike those of
(D) Contrary to

- My roommate, a drama major, claimed that Ibsen's plays, unlike Ionesco, are totally
conventional in their style.
(A) No change
(B) unlike that of Ionesco
(C) unlike with Ionesco
(D) unlike Ionesco's

Top Score
GET READY TO STUDY
Chapter 2 Modifiers

Nota Bene
1- Gerund / Having + pp …………, the doer of the gerund
EX:
Bob, reviewing the notes he had taken at the meeting, prepared a report to be
distributed to all members of the council.
Having Reviewed the notes he had taken at the meeting, Bob prepared a report to be
distributed to all members of the council.

2- PP / Having + been + p.p …………., the receiver of the verb


EX:
Written in easy language, the book of Mr. Sam is the most important math book.

- After receiving critical acclaim for his abstract paintings, Philip Gaston surprised and
alarmed the art world in 1970 with an exhibition of stark and cartoonlike figurative
paintings.
(A) No change
(B) him receiving
(C) he receives
(D) he is receiving

- Despite watering it daily, the plant continued to shed its leaves, probably because it
had been placed in such a dimly lit corner of the room.
(A)No change
(B) watered
(C) he is watering it
(D)to water it

3- Verb to be + WH Word always INCORRECT


(is what / is why / was when / are how …….etc).
The economist is who studies or has a special knowledge of trade, industries, or money. X
The economist is a person who studies or has a special knowledge of trade, industries, or
money. √

Top Score
GET READY TO STUDY
Chapter 2 Modifiers

4- Avoid all kinds of The Wordiness.


- Barely able to speak because of the cold, the communication among the two explorers
had to be done through the gestures.
(A) No change
(B) the two explorers had to communicate through the gestures
(C) the two explorers had to communicate between themselves from gestures
(D) gestures were used to communicate between the two explorers

5- Modifier………………, Noun's + been modified


- Having thought the problem through with some care, that the committee did not
understand her solution frustrated the chairperson extremely.
(A) No change
(B) Having thought the problem through with some care, the chairperson's extreme
frustration resulted from the committee not understanding her solution
(C) The chairperson's frustration at the committee's failing to understand her solution,
having thought the problem through with some care, was extreme
(D) The chairperson, having thought the problem through with some care, was
extremely frustrated by the committee's failure to understand her solution

- Failing to anticipate the sharp downturn in the economy, millions of dollars were lost
through the investment fund's managers' slowness to act.
(A) No change
(B) millions of dollars were lost due to the slowness of the investment fund's managers
to act
(C) the investment fund's managers were slow to act, millions of dollars were lost as a
result
(D) the investment fund's managers were slow to act and thus lost millions of dollars

Top Score
GET READY TO STUDY
Chapter 2 Modifiers

Practice
1- Although long considered a vestigial organ 4- An oversized men's suit popular in the late
that has no function in humans, the appendix, 1930s and 1940s, it was known as the zoot
some scientists believe, may have a significant suit, was a declaration of rebellious self-
role as part of the body's immune system. assertion for its wearers.
(A) NO CHANGE (A) NO CHANGE
(B) The scientists, although long considered a (B) The zoot suit, an oversized men's suit
vestigial organ that has no function in popular in the late 1930s and 1940s, was a
humans, believe that the appendix may declaration of rebellious self-assertion for
have a significant role as part of the body's its wearers.
immune system. (C) An oversized men's suit popular in the late
(C) The appendix is believed by some 1930s and 1940s, they knew it as the zoot,
scientists as perhaps having a significant it was a declaration of rebellious self-
role as part of the body's immune system, assertion for its wearers.
although long considered a vestigial organ (D) The zoot suit was known as the zoot suit,
that has no function in humans, was a declaration of rebellious self-
(D) Although long considered a vestigial assertion for its wearers, an oversized
organ that has no function in humans, men's suit popular in the late 1930s and
some scientists believe the appendix may 1940s
have a significant role as part of the body's
5- Lecturing at the university, read the poetry of
immune system. Margret Atwood was the advice professor
2- Looking down through the boat's glass bottom, Clark gave her audience.
a school of yellow fish was seen swimming (A) No change
along with the turtles.
(B) Lecturing at the university, her audience
(A) NO CHANGE
was advised by professor Clark Atwood
(B) a school of yellow fish were seen
(C) Margaret Atwood's poetry, lecturing at the
(C) we saw a school of yellow fish
university, advised professor Clark, was
(D) we seen a school of yellow fish what her audience should read
3- Among the most flavorful cuisines in the
(D) Lecturing at the university, Professor
United States, New Orleans has also
Clark advised her audience to read the
become one of the most popular.
poetry of Margret Atwood
(A) NO CHANGE
(B) New Orleans has also become famous as
(C) the cuisine of New Orleans is also
(D) also the cuisine of New Orleans is

Top Score
GET READY TO STUDY
Chapter 2 Modifiers

6- By simply entering an internet website or 9- Spread by rat fleas, millions of people in


calling a toll-free number, a catalog order can medieval Europe were killed by bubonic
be placed by you for almost anything from plague.
cheesecakes to fully equipped desktop (A) NO CHANGE
computers. (B) This led to the killing of millions of
(A) NO CHANGE medieval European by bubonic plague
spread by rat fleas
(B) A catalog order can be placed for almost
anything from cheesecakes to fully (C) Bubonic plague, spread by rat fleas, killed
equipped desktop computers entering an millions of people in medieval Europe
internet website or calling a toll-free (D) Bubonic plague in medieval Europe was
number why millions of people were killed, spread
(C) They, by simply entering an internet by rat fleas
website or calling a toll-free number, will 10- Frustrated with the lack of fashionable
place your catalog order for almost clothing at decent price in most retail stores,
anything from cheesecakes to fully a new discount store was opened on Main
equipped desktop computers, Street by a local clothing designer.
(D) By simply entering an internet website or (A) NO CHANGE
calling a toll-free number, you can place a
(B) stores, a local clothing designer opened a
catalog order for almost anything from
new discount store on Main Street
cheesecakes to fully equipped desktop
computers. (C) stores; on Main Street a local clothing
designer opened a new discount store
7- In order to save money and offer competitive
ticket prices, meals are no longer being served (D) stores led a local clothing designer to
open a new discount store on Main Street
on many airlines, even on long flights.
11- Feeling that their votes do not matter, the
(A) NO CHANGE number of young people going to the polls
(B) even on long flights, meals are not served are becoming increasingly smaller.
any longer by many airlines (A) NO CHANGE
(C) many airlines, even on long flights, no (B) Feeling that their votes do not matter,
longer serving meals increasingly smaller number of young
(D) many airlines are no longer serving meals, people are going to the polls.
even on long flights (C) Young people, feeling that their votes do
8- Recalling how difficult it is to park a car at the not matter, are going to the polls in
airport, our decision was taking the train. increasingly smaller numbers.
(A) NO CHANGE (D) Young people, who in increasingly
(B) our decision was to take the train smaller numbers, are going to the polls
feeling that their votes do not matter.
(C) we decided to take the train
(D) taking the train is what we decided to do

Top Score
GET READY TO STUDY
Chapter 2 Modifiers

12- Saying nothing further, the table was cleared 15- Acting on a tip from an anonymous
by the family in preparation for the evening informant, a cache of counterfeit money and
meal. stolen credit cards were seized by federal
(A) NO CHANGE agents during a raid on a local nightclub
early yesterday morning.
(B) Nothing further was said, and the family
clears the table in preparation for the (A) NO CHANGE
evening meal (B) Acting on a tip from an anonymous
(C) The family said nothing further, the table informant, there was a cache of
was cleared in preparation for the counterfeit money and stolen credit cards
evening meal that federal agents seized

(D) Saying nothing further, the family (C) Federal agents would seize a cache of
cleared the table in preparation for the counterfeit money and stolen credit cards
evening meal acting on a tip from an anonymous
informant
13- When visiting the rain forest, the
preservation of the environment should be (D) Federal agents seized a cache of
the tourist's concern. counterfeit money and stolen credit
cards, acting on a tip from an anonymous
(A) NO CHANGE informant
(B) When visiting the rain forest, the 16- Finding the Baltimore waterfront fascinating,
environment and its preservation should all that there was to see was thoroughly
be the tourist's concern explored by Antonio.
(C) The tourist should be concerned with the (A) NO CHANGE
preservation of the environment when
visiting the rain forest (B) Antonio found the Baltimore waterfront
fascinating, he thoroughly explored all
(D) The tourist, when visiting the rain forest, that there was to see
should be concerned with the
preservation of the environment (C) Finding the Baltimore waterfront
fascinating, Antonio thoroughly explored
14- While trimming his forsythia bush, an all that there was to see
overgrown mass of vegetation that had just
begun to flower, an indignant bee stung poor (D) The Baltimore waterfront is fascinating
Mr. Nettleworth on the wrist. and is why Antonio thoroughly explored
all that there was to see.
(A) NO CHANGE
(B) there was an indignant bee that stung
poor Mr. Nettleworth on his wrist
(C) poor Mr. Nettleworth was stung on the
wrist by an indignant bee
(D) poor Mr. Nettleworth's wrist was stung
from an indignant bee

Top Score
GET READY TO STUDY
Chapter 2 Modifiers

17- Once a popular form of entertainment in 20- Once almost entirely devoted to pineapple
arcades, shops, and saloons across the United production, tourists, who enjoy its mountain
States, collectors prize coin-operated trails and nature preserves, are now attracted
mechanical games as emblems of the nation's to the island of Lanai
past. (A) NO CHANGE
(A) NO CHANGE (B) mountain trails and nature preserves,
(B) collectors who prize coin-operated once almost entirely devoted to
mechanical games pineapple production, are enjoyed by
(C) coin-operated mechanical games that are tourists, who are now attracted to the
prized by collectors island of Lanai

(D) coin-operated mechanical games are (C) it now attracts tourists, who enjoy the
prized by collectors mountain trails and nature preserves on
the island of Lanai once almost entirely
18- Sometimes called "the founder of art in devoted to pineapple production
Texas", a sculpture of William Jennings
Bryan was completed by Elisabeth Ney in (D) Once almost entirely devoted to
her Austin studio in 1899. pineapple production, the island of Lanai
now attracts tourists, who enjoy its
(A) NO CHANGE mountain trails and nature preserves
(B) Sometimes called "the founder of art in 21- Seeking to reduce pollution from
Texas", William Jennings Bryan's conventional electricity plants, ways to
sculpture was completed by Elisabeth convert the energy of ocean waves into
Ney in her Austin studio in 1899 usable power are being considered by the
(C) in her Austin studio, Elisabeth Ney state of Oregon.
completed a sculpture of William (A) NO CHANGE
Jennings Bryan called "the founder of art
in Texas" in 1899 (B) Seeking to reduce pollution from
conventional electricity plants, the
(D) Elisabeth Ney, sometimes called "the conversion of the energy of ocean waves
founder of art in Texas", completed a into usable power has been considered
sculpture of William Jennings Bryan in by the state of Oregon
her Austin studio in 1899
(C) The state of Oregon is considering ways
19- An orphan raised in humble surrounding of ocean wave energy being converted
during the early nineteenth century, the novel into usable power seeking to reduce
Great Expectations has a hero who is given a pollution from conventional electricity
fortune and sets out to become a gentleman. plants
(A) NO CHANGE (D) The state of Oregon is considering ways
(B) Great Expectations is a novel the hero of convert the energy of ocean waves
into usable power, seeking to reduce
(C) The hero of the novel Great Expectations pollution from conventional electricity
(D) The hero of the novel, that is Great plants
Expectations

Top Score
GET READY TO STUDY
Chapter 2 Modifiers

22- Carried by the strong, dry winds of the 24- Having spent seven years correcting errors in
stratosphere, the 1980 eruption of Mount the text, a new edition of James Joyce's
Saint Helens caused dust that crossed the Ulysses has been published by Hans Walter
United States in three days and circled the Gabbler.
globe in two weeks. (A) NO CHANGE
(A) NO CHANGE (B) Hans Walter Gabbler, having spent seven
(B) Mount Saint Helens' eruption in 1980 years correcting errors in the text, has
caused dust that carried by the strong, published a new edition of James Joyce's
dry winds of the stratosphere and crossed Ulysses
the United States in three days and (C) Having spent seven years correcting
circled the globe in two weeks errors in the text, Hans Walter Gabbler's
(C) Dust from the 1980 eruption of Mount new edition of James Joyce's Ulysses has
Saint Helens, carried by the strong, dry been published
winds of the stratosphere, crossed the (D) Having spent seven years correcting
United States in three days and circled errors in the text, James Joyce's Ulysses
the globe in two weeks has been published in a new edition by
(D) Carried by the strong, dry winds of the Hans Walter Gabbler
stratosphere, there was dust from the 25- Determined to locate the famous statue, the
1980 eruption of Mount Saint Helens and museum's confusing map was scrutinized by
it crossed the United States in three days Howard and Christine.
and circled the globe in two weeks (A) NO CHANGE
23- In 1972, to reduce pollution in the Great (B) Determined to locate the famous statue,
Lakes, limits having been set by the United Howard and Christine scrutinized the
States and Canada in the amount of museum's confusing map
phosphorus that could be discharged into
(C) Determined to locate the famous statue,
Lakes Erie and Ontario
museum's map, which was confusing,
(A) NO CHANGE was scrutinized by Howard and Christine
(B) to reduce pollution in the Great Lakes, (D) confused, Howard and Christine
limits have been set by the United States scrutinized the museum's map
and Canada determined to locate the famous statue,
(C) to reduce pollution in the Great Lakes,
the United States and Canada have set
limits
(D) the United States and Canada set limits
to reduce pollution in the Great Lakes

Top Score
GET READY TO STUDY
Chapter 2 Modifiers

26- The chair of the school board announced a 28- In the Netherlands, a man found more than
plan to build two new elementary schools 100 ancient Celtic coins that were in a
during an interview with a local news cornfield using a metal detector.
reporter. (A) NO CHANGE
(A) NO CHANGE (B) Using a metal detector, more than 100
(B) The chair of the school board announced ancient Celtic coins were found by a man
that two new elementary schools were in a cornfield in Netherlands.
planned to be built during an interview (C) A metal detector in the Netherland was
with a local news reporter used to find more than 100 ancient
Celtic coins in a cornfield by a man.
(C) During an interview with a local news
(D) A man using a metal detector found more
reporter, The chair of the school board
than 100 ancient Celtic coins in a
announced a plan to build two new cornfield in the Netherlands.
elementary schools
29- Selected as an astronaut by NASA in 1990,
(D) During an interview with a local news over 719 hours in space were spent by Dr.
reporter, The chair of the school board Ellen Ochoa on three flights by 2001.
announcing a plan to build two new (A) NO CHANGE
elementary schools (B) Selected as an astronaut by NASA in
27- By painting them this afternoon, the walls 1990, three flights and 719 hours were
would be completely dry by tomorrow spent by Dr. Ellen Ochoa in space by
evening. 2001
(A) NO CHANGE (C) Dr. Ellen Ochoa, by 2001 spending over
(B) The walls would be completely dry by 719 hours in space on three flights
tomorrow evening, by painting them this selected by NASA in 1990 as an
afternoon. astronaut
(C) After having been painted this afternoon, (D) Dr. Ellen Ochoa had spent over 719
the walls would be completely dry by hours in space on three flights by 200, an
tomorrow evening. astronaut who selected by NASA in 1990
(D) They would be painted, the walls would
be completely dry by tomorrow evening.

Top Score
GET READY TO STUDY
Chapter 2 Modifiers

Homework
1- To satisfy her high school's community 3- Traveling through Yosemite, the scenery of
service requirement, patients at a hospital waterfalls and granite peaks, which we
were visited by Jan three afternoons a week. photographed, was beautiful.
(A) No change
(A) No change
(B) Traveling through Yosemite, the
(B) To satisfy her high school's community waterfalls and granite peaks were the
service requirement, Hospital patients beautiful scenery we photographed
were visited by Jan three afternoons a
(C) We, traveling through Yosemite,
week. photographed the beautiful scenery of
(C) To satisfy her high school's community waterfalls and granite peaks
service requirement, patients at a hospital (D) What we photographed was the
were visited by Jan three afternoons a beautiful scenery of waterfalls and
week. granite peaks traveling through
Yosemite
(D) Jan visited patients at a hospital three
4- Agreeing upon it beforehand, the new
afternoons a week to satisfy her high
municipal budget passed with little
school's community service requirement opposition when it was put before the city
2- A beautifully written narrative of the author's council.
boyhood in South America, his descriptions (A) No change
of animal life in the plains region are (B) They agreed upon it beforehand,
fascinating therefore
(A) No change (C) It being agreed upon beforehand, so
(B) his descriptions of animal life in the (D) Having been agreed upon it beforehand,
plains region is fascinating 5- Unable to see more than three inches in front
(C) he fascinatingly describes animal life in of her nose without corrective lenses, Mary's
the plains region search for her missing glasses was frantic.

(D) the book contains fascinating (A) No change


descriptions of animal life in the plains (B) Mary's frantic search was for her
region missing glasses was
(C) Mary frantically searched for her
missing glasses
(D) Her missing glasses was that for which
Mary frantically searched

Top Score
GET READY TO STUDY
Chapter 2 Modifiers

6- Examining the principal movement sweeping 9- Before going on the senior class trip, a
through the world, it can be seen that they are parental permission slip must be filled out
being accelerated by the war. for each student.
(A) No change
(A) No change
(B) Having examined the principal
movement sweeping through the world, (B) Before going on the senior class trip, a
it can be seen by us that they are being student must have their parental
accelerated by the war permission slips filled out
(C) Examining the principal movement (C) Before going on the senior class trip,
sweeping through the world can be seen their parents must fill out a permission
by us that they are being accelerated by slip for each student
the war
(D) Before going on the senior class trip, a
(D) We can see that they are being
accelerated by the war, examining the student must have a parental permission
principal movement sweeping through slip filled out
the world 10- While passing the fire department building,
7- Despite writing more than 1,700 poems, only the siren began to screech loud, which
seven were published during Emily scared me.
Dickinson’s lifetime.
(A) No change
(A) No change (B) While passing the fire department
(B) Emily Dickinson, despite writing more building, the siren began screeching
than 1,700 poems, published only seven loudly, which scared me
during her (C) I was scared by the loud screech of the
siren while passing the fire department
(C) Despite writing more than 1,700 poems, building
the only published seven during Emily
(D) I was scared by the loud screech of the
Dickinson’s
siren passing the fire department
(D) seven of them were the only ones building
published during Emily Dickinson’s 11- Essential for doing business or just staying
writing more than 1,700 poems in touch with family and friends, cell
8- Standing on the bridge of the ship, there blew phones, they are increasingly popular.
the most strong winds that I had seen at sea (A) No change
for at least a decade
(B) cell phones are increasingly popular
(A) No change
(C) their popularity is growing
(B) the strongest winds were blowing
(D) they have become more popular
(C) I experienced the most strong winds
(D) I observed the strongest winds

Top Score
GET READY TO STUDY
Chapter 2 Modifiers

12- While walking down Market Street, that 14- Flying at a very low altitude in an effort to
was when Clarissa sighted her old guitar in avoid radar detection, the townspeople were
the pawn shop window. startled by the jet fighter’s supersonic
(A) No change boom.

(B) Clarissa sighted her old guitar in the (A) No change


pawn shop window walking down (B) Jet fighter, flying at a very low altitude
Market Street in an effort to avoid radar detection,
(C) Clarissa sighted her old guitar in the startled the townspeople with its
pawn shop window while walking supersonic boom
down Market Street (C) Flying at a very low altitude in an effort
(D) While walking down Market Street, to avoid radar detection, jet fighter’s
Clarissa's old guitar was sighted in the supersonic boom startled the
pawn shop window townspeople

13- Having a mother who plays in a symphony (D) Jet fighter and its supersonic boom
orchestra and a father who teaches in high startled the townspeople flying at a very
school, the violin and the piano are two of low altitude in an effort to avoid radar
the instrument that Rosie learned at an early detection
age 15- The octopus emits a cloud of ink acting as a
(A) No change smokescreen when fleeing a predator.

(B) Having a mother who plays in a (A) No change


symphony orchestra and a father who (B) When fleeing a predator, the cloud of
teaches in high school, violin and piano ink emitted by an octopus acts as
were taught to Rosie at an early age smokescreen.
(C) Having a mother who plays in a (C) The cloud of ink emitted by an octopus
symphony orchestra and a father who acts as a smokescreen when fleeing a
teaches in high school, the violin and predator.
the piano were the two instruments (D) When fleeing a predator, the octopus
which Rosie learned to play at an early emits a cloud of ink that acts as a
age smokescreen.
(D) Rosie learned the playing of both violin 16- Having order the cup of black coffee at the
and piano at an early age, having a Starbucks counter, the fumes smelled
mother who plays in a symphony pleasing to Howard.
orchestra and a father who teaches in
high school (A) No change
(B) the fume's smell pleased Howard
(C) smells from the fumes pleased Howard
(D) Howard smelled the fumes pleasingly

Top Score
GET READY TO STUDY
Chapter 2 Modifiers

17- When donating money to charity, a 19- Renowned as an expert in linguistics,


nonprofit organization that will use your modern intellectual also acknowledge
gift wisely should be your priority. Pinker's many contributions to psychology.
(A) No change (A) No change
(B) When donating money to charity, you (B) Pinker's many contributions have been
should make it your priority to choose a also acknowledged by modern
intellectual to psychology
nonprofit organization that will use
your gift wisely (C) Pinker being also acknowledged for his
many contributions to psychology and
(C) When donating money to charity, a by modern intellectuals
nonprofit organization should be your
(D) Pinker is also acknowledged by modern
priority that will use your gift wisely intellectuals for his many contributions
(D) using your gift wisely should be your to psychology
priority when choosing a nonprofit 20- Although numbered among the most
organization when donating money to technically demanding pieces ever created
charity for piano, Frederic Chopin wrote
18- Brought up in a homogeneous, all white compositions that emphasize nuance and
suburb, it was only when I moved to San expressive depth over mere technical
Francisco that I realized how exciting life in display.
an ethnically diverse community can be. (A) No change
(A) No change (B) Although numbered among the most
(B) I did not realize how exciting life in an technically demanding pieces ever
ethnically diverse community can be created for piano, Frederic Chopin
until I moved to San Francisco brought wrote compositions emphasizing
up in a homogeneous, all white suburb nuance and expressive depth over mere
(C) When I moved to San Francisco I, technical display
brought up in a homogeneous, all white
(C) Frederic Chopin’s compositions that
suburb, realized how exciting life in an
ethnically diverse community can be emphasized nuance and expressive
depth over mere technical display
(D) Brought up in a homogeneous, all white
suburb, An exciting life in an ethnically although numbered among the most
diverse community was unrealized by technically demanding pieces ever
me until I moved to San Francisco created for piano,
(D) Frederic Chopin’s compositions,
although numbered among the most
technically demanding pieces ever
created for piano, emphasized nuance
and expressive depth over mere
technical display

Top Score
GET READY TO STUDY
Chapter 2 Modifiers

21- One of the most important agriculture 24- Without speaking so much as a syllable,
resources of the Philippines, the coconut Chaplin's emotions and intentions were
tree faces the threat of extinction within clearly portrayed on the screen.
three years due to coconut leaf beetle (A) No change
infestation.
(B) Chaplin clearly on the screen portrayed
(A) No change his emotional intentions
(B) the coconut tree would face the threat (C) Chaplin's emotions and intentions on
of extinction the screen were clearly portrayed by
(C) the threat of extinction faced by the him
coconut tree is (D) Chaplin clearly portrayed his emotions
(D) the coconut tree, facing the extinction and intentions on the screen
threat 25- Having run with little effort for over an
22- Attracted by the movement of the red cape, hour, Jane was disheartened to feel a
the matador starts the fight with a series of sudden pain in his knee.
passes as the bull charges. (A) No change
(A) No change (B) It was disheartening for Jane to feel a
(B) The matador starts the fight with a sudden pain in her knee
series of passes as the bull attracted by (C) Jane's sudden pain in her knee was
the movement of the red cape charges disheartening to her
(C) The bull charges as the matador (D) Jane's sudden pain in her knee
attracted by the movement of the red disheartened her
cape starts the fight with a series of
passes 26- Receiving no advice to the contrary from
his brother, Timmy's decision was to take a
(D) Attracted by the movement of the red year off from medical school.
cape, the fight with the bull s
started by the matador with the (A) No change
series of passes (B) Taking a year off from medical school
was Timmy's decision
23- Taking time off from her job as an attorney, (C) Timmy deciding to take a year off from
it was Courtney's intention to teach math to medical school
middle school students in Boston. (D) Timmy decided to take a year off from
(A) No change medical school
(B) The intention of Courtney was to teach
(C) Courtney had the intention for teaching
(D) Courtney intended to teach

Top Score
GET READY TO STUDY
Chapter 2 Modifiers

27- Indicating their desire to extend free 30- Unlike flying squirrels, which may leave
enterprise, Canadians elected a member of their young in a tree cavity while foraging,
the Progressive Conservative Party, Kim the babies of flying lemurs are usually
Campbell, as Prime Minister in 1993. carried with them.
(A) No change (A) No change
(B) Canadians' election of member of the (B) the flying lemur's babies are usually
Progressive Conservative Party as carried
Prime Minister was Kim Campbell (C) flying lemurs usually carry their babies
(C) Kim Campbell of the Progressive (D) flying lemurs' babies are usually carried
Conservative Party was elected Prime
Minister of Canada 31- Prized for their brilliance and durability,
people will spend thousands of dollars on
(D) The Progressive Conservative Party's high-quality diamonds.
Kim Campbell was elected Prime
Minister of Canadians (A) No change
28- This legend about Admiral Nelson, like (B) Prized for their brilliance and
other naval heroes, are based only partially durability, thousands of dollars are
on fact. spent by people on high- quality
diamonds.
(A) No change
(C) Prizing them for their brilliance as well
(B) like those of other naval heroes, are as their durability, thousands of dollars
can be spent by people on high-quality
(C) like legends about other naval heroes, diamonds
are (D) Prizing the brilliance and durability of
high-quality diamonds, people will
(D) like legends about other naval heroes, is spend thousands of dollars for them.
29- Unlike her sister Heather, who would 32- Differing only slightly from the Greeks
always put spiders safely outside if she were the Roman theatres, which were often
found them in the house, Joanne's fear kept freestanding rather than part of a hillside.
her from going anywhere near the creatures. (A) No change
(A) No change (B) Differing only slightly from Greek
(B) Joanne's fear is what kept her from theaters, Roman theaters
going anywhere near the creatures (C) Differing only in the slightest from the
(C) fear is why Joanne had not gone Greeks were the Roman theaters, which
anywhere near them (D) The Greeks differed only slightly from
(D) Joanne was too afraid to go anywhere the Romans, they
near the creatures

Top Score
GET READY TO STUDY
Chapter 2 Modifiers

Mini Exam
Lucille-Clifton - The award winning poet . 1 .. .

Lucille Clifton began writing poetry when she (A) NO CHANGE


(B) Because of her mother, the interest in
was about ten years old. . 1 . Because of her
poetry had developed, Thelma Sayles
mother, she had developed an interest in poetry, (C) Her mother developed her interest in poetry,
Thelma Sayles. Her mother was also a poet Thelma Sayles
(D) She had developed an interest in poetry
although her poems were never published. . 2 .
because of her mother, Thelma Sayles
Lucille would sit on her mother's lap and listen as
. 2 .. .
she read poetry as a child. She learned to love
(A) NO CHANGE
words and the power of words. That stayed with
(B) Lucille, as a child, would sit on her
her as she grew. mother's lap and listen as she read
There was another experience that stayed with poetry
her, too. Once, her mother was offered a chance to (C) On her mother's lap, Lucille would sit
and listen as she read poetry as a child
publish her poetry. But her husband, Samuel
(D) As she read poetry, on her mother's lap,
Sayles, ordered her not to do it. . 3 . In anger, and Lucille would sit and listen
sorrow, Missus Sayles' poems were thrown into a . 3 .. .
fire. That memory also stayed with Lucille. She
(A) NO CHANGE
would write about it years later in her poem called (B) Missus Sayles threw her poems, in
fury. . 4 . Similar to Lucille Clifton, fury is anger and sorrow, into a fire
personal. It deals with her own experiences. (C) In anger and sorrow, Missus Sayles
threw her poems into a fire
(D) In angry way and sorrow manner, Missus
Sayles threw her poems into a fire

. 4 .. .

(A) NO CHANGE
(B) fury, like Lucille Clifton, is a personal
poem and it deals with her own
experiences
(C) Like Lucille Clifton, fury is a personal
poem and it deals with her own
experiences
(D) Like many of Lucille Clifton's poems,
fury is personal. It deals with her own
experiences

Top Score
GET READY TO STUDY
Chapter 2 Modifiers
. 5 .. .
Lucille Clifton believed that it was
important for poets to write about their own (A) NO CHANGE
memories. She said poetry comes out of the (B) Deal with life and death, religion and
life of the poet. That, she said, is the only way politics, motherhood and family, Lucille
that poetry can reach other people. . 5 . Lucille Clifton wrote her poems
Clifton's poems deal with life and death, (C) There are many ideas related to life and
religion and politics, motherhood and family. death, religion and politics, motherhood and
They tell stories of racism, sexism and family included in Lucille Clifton's poems
(D) Lucille Clifton wrote poems and they deal
injustice. They tell of terrible things done to
with life and death, religion and politics,
humans by humans.
motherhood and family
In one poem she calls it the extraordinary
evil in ordinary men. . 6 . She takes a different . 6 .. .
look at violence in the poem Cruelty.
(A) NO CHANGE
Lucille Clifton was born Thelma (B) Different look was taken by her at violence
Lucille Sayles in Depew, New York in ninety in the poem Cruelty
thirty-six. She was named Thelma after her (C) In the poem Cruelty, she takes a different
mother. Lucille was the name of one of her look at violence
father's ancestors. Neither mother nor daughter (D) In a different look at violence, she wrote the
was happy with the name Thelma. When the poem Cruelty
younger one got older, she chose to call
. 7 .. .
herself Lucille.
. 7 . Unlike her mother's, Lucille (A) NO CHANGE
Clifton’s poetry was anything but traditional. (B) Unlike her mother, Lucille Clifton’s poetry
Her poems do not rhyme or follow a special was anything but traditional
(C) Lucille Clifton’s poetry, unlike her mother,
kind of pattern. They do not use fancy words.
was anything but traditional
They do not deal in make believe. Her poetry
(D) Similar to her mother, Lucille Clifton’s
is known for being simple, truthful, and direct.
poetry was anything but traditional
. 8 . After one year of publishing her
first book of poetry, the first children's books . 8 .. .
were released in 1970. Her book “Some of the (A) NO CHANGE
Days of Everett Anderson” became the first in (B) She released her first children’s books in
a series of books about a young African- 1970, a year after her first book of poetry
American boy growing up in the city. was published
(C) Her first book of poetry was published, one
year later, she released her first children's
books in 1970
(D) A year after her first book of poetry was
published, she released her first children’s
books in 1970

Top Score
GET READY TO STUDY
Chapter 2 Modifiers

Answers
Explanation:
Page 2: D / C / D

Page 3: B / D / C / C

Page 4: B / C / C / D

Page 5: D / C / D

Page 6: A / B

Page 7: B / D / D

Practice:
1 A / 2 C / 3 C / 4 B / 5 D / 6 D / 7 D / 8 C / 9 C / 10 B / 11 C / 12 D / 13 D / 14 C / 15 D
16 C / 17 D / 18 D / 19 C / 20 D / 21 D / 22 C / 23 D / 24 B / 25 B / 26 C / 27 C / 28 D / 29 D

Homework:
1 D / 2 D / 3 C / 4 D / 5 C / 6 D / 7 B / 8 D / 9 D / 10 C / 11 B / 12 C / 13 D / 14 B / 15 D / 16 D
17 B / 18 C / 19 D / 20 D / 21 A / 22 B / 23 D / 24 D / 25 A / 26 D / 27 A / 28 D / 29 D / 30 C
31 D / 32 B

Mini Exam:
1D/2B/3C/4D/5A/6C/7A/8B

Top Score
GET READY TO STUDY
Chapter 3 Combining Sentences

Combining Sentences
Types of sentence:
Clause: Subject + Verb + Complement
- Marie Curie was the first woman to win the Noble Prize.
- The committee considered the proposals and made its decision.

Phrase: No Subject and no verb (review modifiers)

Issues of Combining Sentence:


A) Run-on Sentence:Run-on sentence ocures when two or more clauses are joined
without an appropriate punctuation or linker.

Cases of run-on sentence:


1- Clause (,) Clause OR Subject + Verb (,) Subject + Verb
- It was a hot summer day, Dina played in the pool.
- All of my friends went to the concert last Saturday, I couldn’t go.

2- Subject + verb + subject + verb (without comma)


- My uncle rang the doorbell our dog barked.
- He deserves a raise he completed the project on schedule and under budget.

Ways to revise the run-on sentences:


1- Subject + verb (;) the same Subject + verb
- We have paid our dues; we expect all the privileges listed in the contract.
2- *Subject + verb (:) Subject + verb (Colon = because)
- I have very little time to learn the language: my new job starts in five weeks.

*Subject + verb (:) List = , such as + List


- You may be required to bring many things: sleeping bags, pans, utensils, and warm
clothing.
- You may be required to bring many things, such as sleeping bags, pans, utensils, and
warm clothing.
Top Score
GET READY TO STUDY
Chapter 3 Combining Sentences

*Explanation (:) what explained


- Arsenal bought a new perfect player: Sanchez
- The president will talk today about an important topic: the unemployment
*Sentence (:) Explanation "phrase"
- I hope that my son will be an economist: a person who studies or has a special
knowledge of trade, industries, or money.

NB:
- ; /: + preposition (in – on – with ….) / wh word (who – which – when – while – what)
/ gerund x
- Subject + such as + list + verb √ (Such as without using comma)
EX:
Nonprofit enterprises such as the Organized Crime and Corruption Reporting Project have
begun to fill the void created by staff losses at newspapers and magazines.

- Subject + verb + object , such as + list (Such as with comma)


EX:
The spaces are usually stocked with standard office equipment, such as photocopiers,
printers, and fax machines.
: / ; + such as X , such as, X such as + ; / : X

Phrase; X /….. phrase X


Phrase: √ /….. phrase √
- The Ducal Palace is connected with the state prison by The Bridge of Sighs; a partially
enclosed bridge which built in Venice in the sixteenth century. X
- The Ducal Palace is connected with the state prison by The Bridge of Sighs: a partially
enclosed bridge which built in Venice in the sixteenth century. √
Decide which of the following is correct and which is incorrect.
- All of my friends went to the concert last Saturday; while I couldn′t go. ................
- Our next-door neighbor rang the doorbell, our dog barked. ................
- Tom realized he had no pen he asked Bob if he could borrow one. ................
- He deserves a raise: he completed the project on schedule and under budget…………
- Today is the most important day in my life; my wedding party…………..

Top Score
GET READY TO STUDY
Chapter 3 Combining Sentences

SAT Question:
- Tickets are available at the box office they can be picked up one hour before the
performance.
(A) No change
(B) at the box office; they can be picked up one hour before the performance
(C) one hour before the performance, they can be picked up at the box office
(D) at the box office, one hour before the performance is when they can be picked up

- The students found field work in the state forest more exciting and dangerous than any of
them had anticipated; having to be rescued by helicopter during a fire.
(A) No change
(B) anticipated; when they had to be
(C) anticipated: they had to be
(D) anticipated: among which was their

- There are numerous other reasons to buy organic food, such as, a desire to protect the
environment from potentially damaging pesticides or a preference for the taste of
organically grown foods.
(A) NO CHANGE
(B) food such as:
(C) food such as,
(D) food, such as
3- Dashes: — Definition…… — / — Definition…… .
*We use dashes as a super-comma or a set of super-commas to set off parenthetical
elements, especially when those elements contain internal forms of punctuation:
EX:
- Without dash: All four of them, Bob, Jeffry, Jason, and Brett, did well in college. X
- With dash: All four of them—Bob, Jeffry, Jason, and Brett—did well in college. √
- When I arrived to Kuala Lumpur, the capital of Malaysia, I impressed by Petronas Twin
Towers, a pair of glass-and-steel-clad skyscrapers with Islamic motifs.
- When I arrived to Kuala Lumpur, I impressed by Petronas Twin Towers—a pair of glass-
and-steel-clad skyscrapers with Islamic motifs.

*We chose dash from choices only when we find dash in the sentence:
- When we think about animals depicted in well-known works of art, the image of dogs
playing poker—popularized in a series of paintings by American artist C. M. Coolidge,
may be the first and only one that comes to mind.
(A) NO CHANGE
(B) Coolidge—
(C) Coolidge;
(D) Coolidge

Top Score
GET READY TO STUDY
Chapter 3 Combining Sentences

*We use dashes to give examples:


- The designer envisions the game′s fundamental elements: the settings, characters, and
plots that make each game unique, and is thus a primary creative force behind a video
game.
(A) NO CHANGE
(B) elements: the settings, characters, and plots that make each game unique—
(C) elements—the settings, characters, and plots that make each game unique—
(D) elements; the settings, characters, and plots that make each game unique;

4- Subject + verb (,) FANBOYS + Subject + verb


(for / and / nor / but / or / yet / so)
All of my friends went to the concert last Saturday, but I couldn’t go.
Our next-door neighbor rang the doorbell, so our dog barked.

- (,) FANBOYS + Subject + Verb (When the FANBOYS linkers are preceded by
COMMA, SUBJECT + VERB must follow them).
- FANBOYS + Verb (when the FANBOYS linkers aren’t preceded by COMMA,
ONLY VERB must follow them).
- ; or : FANBOYS ×

SAT Question
- Ulysses wants to play for U-Conn: but he has had trouble meeting the academic
requirements.
(A) NO CHANGE
(B) U-Conn; but
(C) U-Conn, but
(D) U-Conn, but,

- Tashonda sent in her applications and waited by the phone for a response.
(A) NO CHANGE
(B) applications: and
(C) applications; and
(D) applications, and

Top Score
GET READY TO STUDY
Chapter 3 Combining Sentences

5- Subject + verb (;) CONJUNCTIVE ADVERBS (,) + Subject + verb


Subject + verb (;) Subject + verb (,) CONJUNCTIVE ADVERBS
Run-on: Joan lived in Tokyo for ten years, she can't speak any Japanese.
Revised: Joan lived in Tokyo for ten years; nevertheless, she can't speak any Japanese.
Joan lived in Tokyo for ten years; she can't speak any Japanese, nevertheless.

NB:
SAT Question:
- The Basque language is not closely related to any other language in the world, its origins
therefore cannot be determined scholars.
(A) No change
(B) ;therefore, scholars cannot determine its origins
(C) :therefore scholars therefore not determining its origins
(D) , scholars therefore not determining its origins

- It has been raining for the last five day, therefore the drought predicted by experts has
averted, if only temporarily.
(A) No change
(B) ; therefore, the drought
(C) , the drought
(D) , therefore, the drought

- The swiftest animal on earth, the cheetah can reach a top speed of about 70 miles
per hour, it can maintain that speed for no more than 300 yards.
(A) No change
(B) hour but can
(C) hour, but they can
(D) hour, however, it can

- The Pony Express was an ingenious system for carrying mail; it was in existence only
briefly, however, before the telegraph system made it obsolete.
(A) No change
(B) mail, for it was in existence only briefly, however,
(C) mail; however, existing only briefly
(D) mail, but was existing only briefly

Top Score
GET READY TO STUDY
Chapter 3 Combining Sentences

6- Conjunction + subject + verb (,) Subject + verb


Run-on: It rained every day, we enjoyed our camping holiday.
Revised: Although it rained every day, we enjoyed our camping holiday.
Run-on: our next-door neighbor rang the doorbell, our dog barked.
Revised: Because our next-door neighbor rang the doorbell, our dog barked.

SAT Question
- The Ancient Egyptians had a varied diet and ate no refined sugar, they did not suffer
from the tooth decay associated with the consumption of processed sugar.
(A) No change
(B) The Ancient Egyptians, who had varied diet and did not eat refined sugar, and
(C) With a varied diet and them not eating refined sugar, the Ancient Egyptians
(D) Because the Ancient Egyptians had a varied diet that was free of refined sugar, they

7- Subject + verb + Conjunction + subject + verb


We enjoyed our camping holiday although it rained every day.
Our dog barked because our next-door neighbor rang the doorbell.

NB:
; or : + LINKER ......... most likely INCORRECT
LINKER............................. (,) LINKER................................... IS INCORRECT
LINKER..............................(;)......................................... IS INCORRECT

Answer the following questions:


- Many psychologists do not use hypnosis in their practices, it is because they know very
little about it and are wary of it as a result
(A) No change
(B) practices because they know very little about it and are therefore wary of it
(C) practices because of knowing very little about it and therefore they are wary of it
(D) practices, their knowledge

- Sugar alcohols-sugar substitutes used in various brands of sugar-free chewing gum-do


not cause tooth decay, however, they contain calories.
(A) No change
(B) decay, they contain calories though
(C) decay; although they contain calories
(D) decay but do contain calories

Top Score
GET READY TO STUDY
Chapter 3 Combining Sentences

- Although both birds are known for their distinctive songs, but the Rufus song lark has a
sweeter song than has the brown song lark.
(A) No change
(B) Both birds are known for their distinctive songs,
(C) Those birds are known for their distinctive songs;
(D) Many birds are known for their distinctive songs

- While females were not allowed to compete in the Olympic games of ancient Greece;
they were not even allowed to watch them.
(A) No change
(B) Even though females were not allowed
(C) Females were not allowed
(D) They did not allow females

8- Phrase (,) Clause OR Clause (,) Phrase (MODIFIERS CASE)


(We studied this point in the Modifiers chapter in details)

*To convert the CLAUSE into PHRASE:

1) Remove the SUBJECT AND THEN convert the VERB into GERUND or PP.

Example:

- Antonio found the Baltimore waterfront fascinating, Antonio thoroughly explored all that
there was to see. (there is a run-on sentence error, so to avoid it you can convert either
of the two sentences into a phrase).
- Finding the Baltimore waterfront fascinating, Antonio thoroughly explored all that there
was to see.
2) Start the second sentence with (, who / , which / , whom / , whose /, of which / ,
each Or that "without comma").
- Mr. Sam taught us some new lessons, these lessons were very hard. X
- Mr. Sam taught us some new lessons, which were very hard. √
- Mr. Sam taught us some new lessons, some of which were very hard. √
- Nick is very grateful for his grandfather, his grandfather taught him many things in life. X
- Nick is very grateful for his grandfather, who taught him many things in life. √

3) Convert the second sentence into definition: Revise Modifiers Chapter


- Yesterday I met Mr. Sam, he is a teacher who helped me is SAT in my high school. X
- Yesterday I met Mr. Sam, a teacher who helped me is SAT in my high school. √

Top Score
GET READY TO STUDY
Chapter 3 Combining Sentences

SAT Question:

- I had never encountered such arrogance before, I did not know how to react.
(A) No change
(B) I never encountered
(C) Never had I encountered
(D) Never having encountered

- Textile production has a serious environmental impact, it consumes a significant amount


of all the water used worldwide.
(A) No change
(B) its consumption is
(C) this consumes
(D) consuming

- 1-MCP works by limiting a fruit’s production of ethylene, it is a chemical that causes


fruit to ripen and eventually rot.
(A) NO CHANGE
(B) being
(C) that is
(D) DELETE the underlined portion.

Which choice most effectively combines the two sentences at the underlined
portion?
- In neighborhoods throughout the United States, one can encounter hundreds of
different rope-jumping games. These rope-jumping games have their own rules.
(A) rope-jumping games; having their own rules.
(B) rope-jumping games, each with its own rules
(C) rope-jumping games, and the rope-jumping games have their own rules
(D) rope-jumping games, when they each have their own rules

COORDINATING
SUBORDINATING
CONJUNCTIONS CONJUNCTIVE ADVERBS
CONJUNCTIONS
(FAN BOYS)
For ……. F Therefore – thereby - thus Although - Even though
Though – Because – as
And……. A Consequently – hence
+ Clause
Nor……. N Subsequently – however because of - Despite –
But…….. B Nonetheless – nevertheless in spite of - due to + Phrase
Or……… O Instead – still After - before – When - If
Yet…….. Y Also – moreover While + Clause or Phrase
So………. S Furthermore - besides

Top Score
GET READY TO STUDY
Chapter 3 Combining Sentences

B) Inappropriate linker: it is about the meaning of the linker.


Yesterday I was sick, so I went to my work. X
Yesterday I was sick, but I went to my work. √
Although the typhoon, my parents ended their trip one day early. X
Because of the typhoon, my parents ended their trip one day early. √

Exercises:
1. She is a graceful dancer, (but / and) people enjoy watching her.
2. Other dancers try to imitate her style, (yet / and) they have not succeeded.
3. This room is old (but / and) comfortable.
4. I was very tired, (and / so) I went to bed early yesterday.
5. This old woman spoke neither Polish (and / nor) Russian.
6. She tried to learn Chinese, (and / but) it was too difficult.
7. Last night I was very tired, (so / and) I went to sleep.
8. This is Mary. She is very rich, (so / but) she isn't happy.
9. She is good at maths, (and / yet) her favourite subject is history.
10. He easily passed the exams, (for / but) he studied quite thoroughly.
11. I'm Polish, (but / and) my mother is Russian.
12. Chris needed some money, (so / and) he borrowed some from his parents.

SAT questions:
The stoic school of thought was founded in the 4th century BCE, and its most well-known
follower lived and wrote much later.
(A) NO CHANGE
(B) BCE, so its
(C) BCE, however its
(D) BCE, but its
The Stoics, unlike the Epicureans, believed that a divine will they called the logos influenced
all events.
(A) NO CHANGE
(B) divine will, they called
(C) divine will: they called
(D) divine will–they called
Suspicious for not wanting to cause a false alarm, he did not say anything to the coachmen.
(A) NO CHANGE
(B) nor
(C) but
(D) so

Top Score
GET READY TO STUDY
Chapter 3 Combining Sentences

Practice
5. Our modern solar calendar, established in
1. Although some members of the jury
suspected that defendant was guilty, but 1582, is based on the Julian calendar, Julius
none of the evidence against him turned Caesar introduced it in 46 B.C.
out to be more than circumstantial. (A) No change
(A) No change (B) calendar, which is what was introduced
(B) , therefore none by Julius Caesar
(C) , none (C) calendar, with an introduction by Julius
(D) ; none Caesar
(D) calendar, introduced by Julius Caesar
2. One of only a few venomous mammals, 6. The new medical school is accepting
the Slow Loris coats the fur of its young applications from around the world. There
offspring with toxic saliva. This saliva will be 200 places in the entering class.
protects them from predators. (A) world, there will be
(A) No change (B) world, and there would be
(B) saliva, it protects them (C) world; with
(C) saliva, protecting them (D) world for the
(D) saliva, they are protected 7. A recent report indicates that sleep- deprived
drivers caused more than 100,000 accidents
3. Because European filmmaking all but
last year, they fall asleep at the wheel.
shut down during the First World War, so
(A) No change
the film industry in the United States rose
(B) year, and they fall
to prominence.
(C) year by falling
(A) No change
(D) year, and falling
(B) Because European filmmaking all but
shut down during the First World 8. Alice Guy Blaché, an early filmmaker,
War, introduced close-ups and double exposures,
(C) European filmmaking all but shut also she sets cars on fire, used rats in special
down during the First World War, effects sequences, and ran film backward.
(D) The Fact that European (A) No change
filmmaking nearly shut down (B) exposures, and also she sets
during the First (C) exposures, she sets
World War is why (D) exposures; she also set

4. Whereas the participating candidates 9. The fabled city of Timbuktu was founded by
described the debate as vigorous, it nomads, they were seeking shelter from the
seemed downright hostile. Sahara desert on the cool banks of the Niger
(A) No change river.
(B) but to some members of the audience (A) No change
it seemed (B) nomads, seeking
(C) and it seemed to some members of the (C) nomads; seeking
audience (D) nomads which sought
(D) and seemed to some members of
the audience

Top Score
GET READY TO STUDY
Chapter 3 Combining Sentences

10. The charges against the organization are 15. Diamond is harder than any other naturally
being investigated by a committee. It occurring mineral, and the reason is
includes several senators. because its carbon atoms are arranged in a
(A) committee, it includes strongly bonded crystal structure.
(B) committee; it including (A) No change
(C) committee, and it will include (B) mineral and because
(D) committee that includes (C) mineral because
11. Although one might think that contact (D) mineral by
lenses are a recent innovation, but in fact 16. Relatively little is known about Suminoe
were developed more than a century ago. Oyster, which is new to the
(A) No change Chesapeake Bay, This has resulted in it
(B) but they were in fact being difficult for scientists to decide
(C) they have in fact been whether it may disturb the ecosystem there.
(D) they were in fact (A) No change
12. A poetic form congenial to Robert (B) Bay, which have resulted in difficulty
Browning was the dramatic monologue, it (C) Bay; they result in difficulty
let him explore a character's mind without (D) Bay; as a result, it is difficult
the simplification demanded by stage
productions. 17. Which choice most effectively combines
(A) No change the two sentences?
(B) monologue, which let him explore The visiting professor's application for an
(C) monologue that lets him explore extended appointment was dismissed rudely
(D) monologue; letting him explore and abruptly by the chancellor of
13. We generally think of Canada as the
the university. The chancellor considered
northern neighbor of the United States. the professor unfit for the job.
More than half of the states extend farther (A) NO CHANGE
north than Canada’s southernmost point. (B) university, who considered
(A) No change (C) university, who considers
(D) university, and he considered
(B) States, and it is the case that more than
half of the states extend 18. Dorothy Crowfoot Hodgkin used X-Ray to
(C) States, but more than half of the states discover the chemical structures of
extending penicillin and vitamin B12, winning a
Noble Prize for her work.
(D) States; however, more than half of the
(A) No change
states extend (B) she then won a Nobel Prize for her
14. Scientists predict technological changes work
in the next century, they will be as (C) this work won her a Nobel Prize
dramatic as was the development of the (D) she won a Nobel Prize for her work
transcontinental railroad in the last
century.
(A) No change
(B) century, these changes will be
as dramatic as
(C) century; being as dramatic as was
(D) century will be as dramatic as

Top Score
GET READY TO STUDY
Chapter 3 Combining Sentences

Homework
1. The brains of infants have many more 5. Chipmunks do not technically hibernate,
neural connections than the brains of adults but they do remain in their dens or burrows
do, but they are much less efficient. during cold periods.
(A) No change (A) No change
(B) but it is much less efficient (B) hibernate, but remaining
(C) but the efficiency being lower (C) hibernate, but they remained
(D) hibernate, they remain
(D) their efficiency is much lower, though
6. Though the Savannah was the first
2. Maya Lin is best known for designing the steamship to cross the Atlantic Ocean, it
Vietnam Veterans Memorial, but her work relied on its sail, rather than its steam
also including buildings, furniture, and engine, for most of the voyage.
sculptures in stone, wood, and glass. (A) No change
(A) No change (B) they relied
(B) but her work also includes (C) but it relied
(C) but her work also having included (D) but relying
(D) her work also includes 7. The film was full of suspense, this made
3. In his Star Wars films, director Riya keep her hands gripped tightly to her
George Lucas is a contemporary seat.
mythmaker, Plus being a master of special (A) No change
effects. (B) The film, which was full of suspense,
keeping Riya's
(A) No change
(C) The suspenseful film made Riya keep
(B) Lucas, a contemporary mythmaker, her
moreover (D) Being full of suspense, Riya found that
(C) Lucas is a contemporary mythmaker as film made her keep her
well as 8. Because many Szechuan recipes require for
(D) Lucas is a contemporary mythmaker, one to cook without there having to be
he is also interruption, it is a good idea to measure all
4. Because its early history is not fully known, in gradients advance.
Origami, the art of folding objects out of (A) No change
paper without cutting, pasting, or (B) Because many Szechuan recipes
decorating, seems to have developed from require that one cook without
the older art of folding cloth. interruption,
(A) No change (C) Being that many Szechuan recipes
(B) Since require you to cook and not be
(C) Although interrupted,
(D) As (D) Many Szechuan recipes require that
one cook without interruption and

Top Score
GET READY TO STUDY
Chapter 3 Combining Sentences

9. We generally think of Canada as the 13. Because fiscal problems will force some
northern neighbor of the United States. cities to lay off firefighters, and so the state
More than half of the states extend farther legislature must decide whether to provide
those cities with financial aid.
north than Canada′s southernmost point.
(A) No change
(A)No change
(B) firefighters is a matter
(B) States, and it is the case that more than
(C) firefighters,
half of the states extend
(D) firefighters; this is a problem
(C) States, but more than half of the states
extending 14. At lunchtime, Kevin paid for
(D)States; however, more than half of the Anita′s hamburger; the reason is because he
states extend owed her money.
10. Henry Wadsworth Longfellow was a
(A) No change
descent of John and Priscilla Alden, whose (B) hamburger; the reason is because he
owed
romance he celebrated in the narrative
(C) hamburger because he owed
poem "The Courtship of Miles Standish".
(D) hamburger because of the fact of owing
(A) No change
(B) he celebrated their romance 15. Having already become famous for his
(C) their romance was celebrated by him Arctic expedition, polar explorer Frid
(D) it was their romance he celebrates Nansen won the Nobel Peace Prize in 1922
for his humanitarian work.
11. A discovery in New Jersey actually
contributed to the early economic (A) No change
development of America and, in 1714, a (B) His having already become
worker uncovered a green rock containing (C) He had already been
copper. (D) He was already
(A) No change 16. You cannot expect to treat your friends
(B) America when, in 1714, a worker badly and no one notice.
uncovered (A) No change
(C) America, thus, in 1714, a worker (B) and have no one notice
uncovered (C) without notice by someone
(D) America, that being a worker in 1714 (D) without the result of somebody noticing
uncovering
17. Increased competition in this market will
12. Nancy and Carlos will represent Central
certainly lower prices, which some
High in the swimming competition, their
economists are warning that it may also
work in having been excellent this year.
lower the quality of service.
(A) No change
(B) competition, they have done excellent (A) No change
work this year in this (B) however, some economists have
(C) competition, for their swimming has warned
been excellent this year (C) but some economists warn
(D) competition, their work as swimmers (D) although some economists warning
having been excellent this year.
Top Score
GET READY TO STUDY
Chapter 3 Combining Sentences

18. The beaver is the largest rodent in North 21. The television show Soul Train reached the
America, it has a large, flat, nearly hairless height of its popularity n the 1970s and
tail: webbed hind feet; and short front legs 1980s, it was showcasing the latest songs
with hairy claws.
by outstanding rhythm-and-blues, soul, and
(A) No change hip-hop artists.
(B) The beaver, the largest rodent in North
America, which (A) No change
(C) The largest rodent in North America, (B) showcasing
the beaver (C) when it showcases
(D) The largest rodents in North America, (D) when they showcased
the beaver
22. We had never seen anything like this style
19. Although Daniel Gabriel Fahrenheit was far
of architecture before, we thought we were
from being the only eighteenth-century
looking at giant sculpture, not buildings.
scientist to propose temperature scale, but
(A) No change
he was one of the few that were widely
(B) We never saw
used.
(C) Never having seen
(A) No change (D) Never seeing
(B) though he was one of the few that were
23. Many college students think that school
widely used
policies should not be dictated by
(C) whereas he was one of the few that
administrative officers but instead they
were widely used
should be submitted to the approval of the
(D) he was one of the few that were widely
students.
used
20. A recent study by two professors at the (A) No change
University of California, Santa Cruz, Chris (B) but they should have been submitted to
Wilmers and James Estes, suggests, that the approval of the students.
kelp forests protected by sea otters can (C) but should be submitted to the students
absorb as much as twelve times the amount
for approval
of carbon dioxide.
(A) NO CHANGE (D) however, they ought to be submitted to
(B) Estes suggests—that the students for their approval
(C) Estes suggests, that 24. As a choreographer, Judith Jamison has
(D) Estes suggests that enriched the world of dance, she uses as her
work inspiration African American culture.

(A) No change
(B) Jamison has enriched the world of dance
with works inspired by
(C) Jamison, who has enriched the world of
dance by works whose inspiration are
(D) Jamison enriches the world of dance
through works that had the inspiration of

Top Score
GET READY TO STUDY
Chapter 3 Combining Sentences

Official Questions
*The heading of the question:
"Which choice most effectively combines the two sentences at the underlined portion?"

TIP 1: Choose the right type of linking, right linker, and concise answer "review Run-on Sentence"

Official Examples:

Typically, the ice sheet begins to show Which choice most effectively combines the
evidence of thawing in late .13. summer. This follows two sentences at the underlined portion?
several weeks of higher temperatures. A) summer, following
B) summer, and this thawing follows
C) summer, and such thawing follows
D) summer and this evidence follows

Also, studies have found that those students Which choice most effectively combines the
who major in philosophy often do better than students sentences at the underlined portion?
A) writing as
from other majors in both verbal reasoning and
B) writing, and these results can be
analytical .39. writing. These results can be measured
C) writing, which can also be
by standardized test scores. D) writing when the results are

The plainer rooms are more sparsely .18. furnished. Which choice most effectively combines the
Their architectural features, furnishings, and sentences at the underlined portion?
decorations are just as true to the periods they A) furnished by their
represent. B) furnished, but their
C) furnished: their
D) furnished, whereas

Top Score
GET READY TO STUDY
Chapter 3 Combining Sentences

TIP 2: In the long sentence, when the sentence is very long and the question asks you to rearrange it, you
have to separate the sentence to parts, and find the same parts in the choices.
Let's see…..

In context, which choice best combines the


..7.. Artificial light sources are also costly aside from underlined sentences?
lowering worker productivity. They typically A) Aside from lowering worker productivity,
constitute anywhere from 25 to 50 percent of a artificial light sources are also costly, typically
building’s energy use. constituting anywhere from 25 to 50 percent of a
building’s energy use.
B) The cost of artificial light sources, aside from
lowering worker productivity, typically
constitutes anywhere from 25 to 50 percent of a
building’s energy use.
C) Typically constituting 25 to 50 percent of a
building’s energy use, artificial light sources
lower worker productivity and are costly.
D) Artificial lights, which lower worker productivity
and are costly, typically constitute anywhere
from 25 to 50 percent of a building’s energy use.

First: We will separate the sentence to parts like that:


1- Artificial light sources are also costly
2- aside from lowering worker productivity
3- They typically constitute anywhere
4- from 25 to 50 percent of a building’s energy use
Second: Eliminate any choice which has different parts:
Choice (B): part 1 is different: The cost of artificial light sources
Choice (C): part 1 and 2 are different from the original: artificial light sources lower worker productivity
and are costly.
Choice (D): part 1 and 2 are different from the original: which lower worker productivity and are costly
Third: Find the nearest answer to the original:
Choice (A): Aside from lowering worker productivity / artificial light sources are also costly / typically
constituting anywhere / from 25 to 50 percent of an energy use.

Top Score
GET READY TO STUDY
Chapter 3 Combining Sentences

Official Examples:

Which choice most effectively combines the


.23. 1-MCP lengthens storage life by three to four underlined sentences?
times when applied to apples. This extended life A) When applied to apples, 1-MCP lengthens
allows producers to sell their apples in the off-season, storage life by three to four times, allowing
producers to sell their apples in the off-season,
months after the apples have been harvested..
months after the apples have been harvested.
B) Producers are allowed to sell their apples
months after they have been harvested—in the
off-season—because 1-MCP, when applied to
apples, lengthens their storage life by three to
four times.
C) 1-MCP lengthens storage life, when applied
to apples, by three to four times, allowing
producers to sell their apples months after the
apples have been harvested in the off-season.
D) Months after apples have been harvested,
producers are allowed to sell their apples, in the
off-season, because 1-MCP lengthens storage life
when applied to apples by three to four times.

Which choice most effectively combines the


..8.. The result was an explosion of mural painting underlined sentences?
that spread throughout California and the A) The result was an explosion, the Chicano mural
southwestern United States in the 1970s. It was the movement, of mural painting that spread
Chicano mural movement. throughout California and the southwestern
United States in the 1970s.
B) The result was the Chicano mural movement, an
explosion of mural painting that spread
throughout California and the southwestern
United States in the 1970s.
C) The explosion of mural painting that spread
throughout California and the southwestern
United States in the 1970s was the resulting
Chicano mural movement.
D) An explosion of mural painting resulted and it
spread throughout California and the
southwestern United States in the 1970s; it was
the Chicano mural movement.

Top Score
GET READY TO STUDY
Chapter 3 Combining Sentences

Exercises:
Which choice most effectively combines the
An SLP who specializes in speech may work on underlined sentences?
articulation or phonation, though some of these A) NO CHANGE
specialists will also work with attention and memory. B) memory; they work in particular with the
In particular, they work with the components of those components
practices that deal with language. C) memory. Particularly the components
D) memory, particularly the components

Which choice most effectively combines the


Over twenty cities throughout the empire bear his underlined sentences?
name. Alexandria, Egypt, perhaps the most famous of A) thrive; it is
these cities, continues to thrive. It is the second- B) thrive, is
largest city in the modern nation of Egypt. C) thrive. It's
D) thrive and is

Which choice most effectively combines the


This time of great political and social change also saw underlined sentences?
an artistic revolution. The Impressionist movement A) Revolution, and in addition The
arose to challenge the conventions of art. Impressionist movement also
B) Revolution, such as The Impressionist
movement that
C) Revolution, as The Impressionist movement
D) Revolution, but The Impressionist
movement

Which choice most effectively combines the


Echolocation thus enables microbats to hunt in the underlined sentences?
dead of night. This is a time when darkness keeps A) night, as
them safe from predators. B) night, being a time when
C) night, considering that
D) night, when

Which choice most effectively combines the


That's part of the appeal of this career. Each day underlined sentences?
offers unique opportunities and unique challenge. A) career each,
B) career each
C) career, each
D) career: each

Top Score
GET READY TO STUDY
Chapter 3 Combining Sentences

Which choice most effectively combines the


Vitamin C is important for good health. People underlined sentences?
should make sure they get the recommended amount A) good health., making sure
by eating a balanced diet. B) good health, but people should make sure
C) good health, and people should make sure
D) good health; however making sure

Which choice most effectively combines the


Critics argue that modern streetcars aren't any faster underlined sentences?
than local buses. Critics argue that this is the reason A) Critics argue that modern streetcars aren't
modern streetcars will never be cost-effective. any faster than local buses and therefore
will never be cost-effective.
B) Critics argue that modern streetcars aren't
any faster than local buses and that they will
never be cost-effective.
C) Critics argue that modern streetcars aren't
any faster than local buses and Critics argue
they will never be cost-effective.
D) Critics argue that modern streetcars aren't
any faster than local buses, so critics say
they will never be cost-effective.

Which choice most effectively combines the


The critical impact to the environment must also be underlined sentences?
taken into account. Proper regulations were either not A) be taken into account, and proper
in place or not enforced. regulations
B) be taken into account since without proper
regulations
C) be taken into account, as proper regulations
D) be taken into account; however, proper
regulations

Which choice most effectively combines the


It was signed in 1987. This document created underlined sentences?
restrictions on chemicals that were known to be A) Signed in 1987, this document
dangerous to the protective barrier that the ozone B) Because it was signed in 1987, this
layer offers earth. document
C) It was signed in 1987, and this document
D) It was signed in 1987 so this document

Top Score
GET READY TO STUDY
Chapter 3 Combining Sentences

Mini Exam
Seeking Support at Emory University
. 1 .. .
Ruofei Chen’s family is from China . 1 .for
she grew up in Lima, Peru. She knew those (A) NO CHANGE
(B) and
parts of the world . 2 . and studied for (C) so
(D) nor
her undergraduate degree in Asia and Latin
America. . 2 .. .
(A) NO CHANGE
. 3 . However, Ruofei Chen chose to start (B) , and, studied
a master’s degree program in 2015, she says (C) and—studied
(D) ,and studied
she wanted something really different.
Ruofei Chen chose a new field of study . 3 .. .
(A) NO CHANGE
called development practice. This field tries to (B) Although
(C) When
explain how economic development can help
(D) Delete the underlined portion
solve poverty and other issues around the
. 4 .. .
world. (A) NO CHANGE
As this field is so . 4 . new, but only a few (B) new; only a few
(C) new: only a few
universities around the world offer a degree
(D) new, only a few
program for it. For Ruofei Chen, it has also
. 5 .. .
meant new challenges. (A) NO CHANGE
Because the U.S. is involved in (B) world, therefore Ruofei Chen wanted
(C) world; Ruofei Chen wanted
development around . 5 . world: Ruofei Chen
(D) world, Ruofei Chen wanted
wanted to study at a U.S. university. She
. 6 .. .
chose . 6 . Emory University; a private (A) NO CHANGE
research university just outside of Atlanta, (B) Emory University:

Georgia. (C) Emory University—


(D) Emory University and
Founded by the Methodist Episcopal . 7 .
Church in 1836, the school is now home to . 7 .. .
(A) NO CHANGE
over 14,700 students. The founders of the (B) Church in 1836; the school
Coca-Cola Company helped build the current (C) Church in 1836: the school
(D) Church in 1836 and the school
campus in 1914.

Top Score
GET READY TO STUDY
Chapter 3 Combining Sentences

. 8 . She had no idea what to expect and. . 8 .. .


This caused some problems for her right away Which choice most effectively combines the
when Ruofei Chen arrived at Emory. underlined sentences?
"Grad school in (the) U.S. can be really (A) NO CHANGE
busy - like, really busy, depending on how (B) When Ruofei Chen arrived at Emory,
much elective classes you are enrolled in." This caused some problems for her
Ruofei Chen studies in the hours between right away and the reason is she had no
her classes, at night and on the weekends. She idea what to expect
also has to make connections with people and (C) When Ruofei Chen arrived at Emory,
work on group projects. she had no idea what to expect: This
She is also planning a trip to Nicaragua in caused some problems for her right
away.
the summer of 2016 to help prevent . 9 . (D) Some problems were caused immediately
diseases. The diseases affect people in .10. once Ruofei Chen arrived at Emory and
poverty; This is part of an Emory Global she had no idea what to expect

Health Institute project. . 9 .. .

All of this happens in English, .11. and it is (A) NO CHANGE


(B) diseases that affect people
not her native language. But Ruofei Chen has
(C) diseases ,and these diseases affect
found that Emory wants her to succeed. For people
over 20 years, Emory’s Laney Graduate (D) diseases which can also affect people
School English Language Support . 10 .. .

Program has offered special English lessons. (A) NO CHANGE


(B) Poverty,
Once a week Ruofei Chen meets one-on-
(C) Poverty—
one with .12. Peggy Wagner—a teacher with (D) Poverty:
the program. Together, they work on more . 11 .. .
than just English grammar or vocabulary. (A)NO CHANGE
Wagner helps Ruofei Chen examine how her (B)who
voice sounds and how to use the language in (C)which
(D)Delete the underlined portion
different ways.
12
(A) NO CHANGE
(B) Peggy Wagner; a teacher with the
program
(C) Peggy Wagner. A teacher with the
program
(D) Peggy Wagner, a teacher with the

Top Score
GET READY TO STUDY
Chapter 3 Combining Sentences

Answers

Explanation:
Page 2: √ / X / X / X / √ / X
Page 3: B / C / D / B
Page 4: C / C / A
Page 5: B / B / B / A
Page 6: D / B / D
Page 7: B / C / D
Page 8: D / B
Page 9: and / yet / but / so / nor / but / so / but / yet / for / but / so / D / A / C

Practice:
1 C / 2 C / 3 B / 4 A / 5 D / 6 D / 7 C / 8 D / 9 B / 10 D / 11 D / 12 B / 13 D / 14 D / 15 C / 16 D
17 B / 18 A

Homework
1 A / 2 B / 3 C / 4 C / 5 A / 6 A / 7 C / 8 B / 9 D / 10 A / 11 B / 12 C / 13 C / 14 C/ 15 A / 16 B
17 C / 18 C / 19 D / 20 D / 21 B/ 22 C / 23 C / 24 B

Official Questions
Page 1: A / A / B
Page 2: A
Page 3: A / B
Page 4: D / D / C / D / D
Page 5: C / A / D / A

MINI EXAM
1 B / 2 A / 3 C / 4 D / 5 D / 6 B / 7 A / 8 C / 9 B / 10 D / 11 C / 12 D

Top Score
GET READY TO STUDY
Chapter 4 Parallelism

Parallelism
1) Lists
All items in a list or series should be in the same grammatical form. Look for items separated by
commas.
er nd, ger nd, nd ger nd √
Gerund, gerund, but infinitive x
Verb, verb, or noun x
Examples:
- I am interested in studying geology, playing chess, and going to the theatre.

SAT Questions:
- All year round, tourists come to Oregon to hike the tree-covered hills, explore the pine forests,
ski on the snowy mountains, visit the sandy coast, and swimming in the chilly ocean.
(A) No change
(B) and to swim
(C) and swam
(D) and swim

- Joseph’s weekend plans so far included finishing his research paper, paying his rent and utilities,
stopping in for a visit with his grandparents, and to make out a new class schedule.
(A) No change
(B) and make out
(C) and making out
(D) and made out

- After the tree was cut down, the workers spent the next few hours cutting off the numerous
branches, sweeping up the debris, and pick up the piles of leaves left on the ground.
(A) No change
(B) and picking up
(C) and picked up
(D) and to pick up

Top Score
GET READY TO STUDY
Chapter 4 Parallelism

2) Correlative linkers
NOT ONLY + structure + BUT ALSO + the same structure

- The process known as thermal conversion is advantageous because it not only breaks down plastic
waste and also converts it into oil.
(A) NO CHANGE
(B) but converts
(C) but also converts
(D) and also converting
- Like her nonfiction, Jean Craighead George's fiction draws extensively not only from published
material but also she had firsthand observations of animals and ecological systems.
(A) NO CHANGE
(B) but also from
(C) and also from
(D) but also having

NOT JUST / MERELY + structure + BUT ALSO + the same structure

- The book is useful because it offers not just philosophy and theory but also tells you what and how
to live every day.
(A) NO CHANGE
(B) but also it gives ways of everyday living
(C) but also advice for everyday living
(D) but also it gives practical advice for everyday life

NEITHER+ structure + NOR + the same structure


EITHER+ structure + OR + the same structure
1- Although Marion had made a commitment to painting by September, she had neither the energy or
the inclination continue working on it.
(A) NO CHANGE
(B) or she had the inclination
(C) nor the inclination
(D) and the inclination

Top Score
GET READY TO STUDY
Chapter 4 Parallelism

2- Travel writing often describes a journey and endurance, a trip that is risky either because of natural
hazards but also because of political unrest.
(A) NO CHANGE
(B) but also due to
(C) or because there was
(D) or because of

BETWEEN + structure + AND + the same structure


When I learned that both events were scheduled for the same evening, I found it difficult to choose
between goings to the basketball game or attend the rock concert
(A) NO CHANGE
(B) or to attend
(C) and attending
(D) and to attend

SO MUCH + structure + AS + the same structure

- The article praised Isaac Dinesen not so much for her genius as a storyteller but because she was
remarkably defiant of society's expectations of women during the early twentieth century.
(A) NO CHANGE
(B) but because of her remarkable defiance
(C) as for her remarkable defiance
(D) as it did her remarkable defiance

BOTH + structure + AND + the same structure

- In contemporary fiction, ambiguous endings both allow readers to imagine their own stories and it
forces them to recognize life's complexity.
(A) NO CHANGE
(B) force them to recognize
(C) forcing them to recognize
(D) to force them to recognize

Top Score
GET READY TO STUDY
Chapter 4 Parallelism

THAT + structure + AND + the same structure

- The German choreographer Pina Bausch Created dances that incorporated every day Human
gestures and alternating between highly stylized, precise movement and more flowing, expressive
ones.
(A) NO CHANGE
(B) and to alternate
(C) and alternated
(D) and also alternating

TO+ structure + AND + the same structure

- In their search for ways to extend the human life span and warding off diseases, scientists find
themselves focusing not on expanding the diet but rather on limiting it.
(A) NO CHANGE
(B) ward
(C) to ward
(D) also ward

MUST + structure + OR / AND + the same structure

- Unlike bears and some other carnivorous animals that can survive on plants when meat is scarce,
wild cats must capture prey or to go hungry.
(A) NO CHANGE
(B) going
(C) went
(D) go

Structure + THAN / RATHER THAN + the same structure

- In a study published in 2010, Japanese scientists found that people experience greater health
benefits from walking in the forest than if they walk in the city.
(A) NO CHANGE
(B) than from walking in the city
(C) compared to a walk in the city
(D) than walks in the city

Top Score
GET READY TO STUDY
Chapter 4 Parallelism

- When being interviewed for a job, you should emphasize what you can do for the employer rather
than what can the employer do for you.
(A) NO CHANGE
(B) does the employer do for you
(C) the employer can do for you
(D) did the employer do for you

JUST AS + structure + SO + the same structure

- Many environmentalists think that it is just as important to use existing energy sources efficiently
than it is to develop pollution-free means of generating energy.
(A) NO CHANGE
(B) so
(C) but
(D) but also

Structure + LINKER + the same structure

- Explaining modern art is impossible, partly because of its complexity but largely because of it
rapidly changing.
(A) NO CHANGE
(B) it makes rapid changes
(C) of the rapidity with which it changes
(D) changing it is rapid

Gerund + FANBOYS + gerund

- In addition to cutting hair, barbers in Europe during The Middle Ages had the special function of
performing surgery and they pulled teeth.
(A) NO CHANGE
(B) they pulling
(C) pulled
(D) pulling

Top Score
GET READY TO STUDY
Chapter 4 Parallelism

3) Clear Comparisons
Noun of / in / at /Noun – compared to – Noun of / in / at / Noun
Noun of / in / at / Noun – compared to – that/those of /in/at/ Noun

The climate of Arizona is drier than South Carolina. (The sentence incorrectly compares a climate to a
state.)
 The climate of Arizona is drier than the climate of South Carolina.
 The climate of Arizona is drier than that of South Carolina.
Exercises: Correct the following sentences
1. Fresh vegetables at a farmers’ market are sometimes lower in price than a grocery.
…………………………………………………………………………………………………………..
…………………………………………………………………………………………………………..
2. Muscles in the leg are stronger than the arm.
…………………………………………………………………………………………………………..
…………………………………………………………………………………………………………..
SAT Question:
- In some species of birds, such as the peafowl, the plumage of the male is more colorful and more
variegated than the female.
(A) NO CHANGE
(B) than are the females
(C) than that of the female
(D) compared to the female

Noun's noun – Compared to – Noun's noun / Noun’s


Noun's noun – Compared to – that / those of noun
This year s drought was much worse than last year. (The sentence incorrectly compares
drought to a last year)
 This year s drought was much worse than last year's drought.
 This year s drought was much worse than last year's.
 This year s drought was much worse than that of last year.
SAT Question:
- It is thought that a dog's sense of smell is generally 10,000 to 100,000 times better than humans.
(A) NO CHANGE
(B) humans' are
(C) humans have
(D) a human's
Top Score
GET READY TO STUDY
Chapter 4 Parallelism

Subject + verb – compared to – subject + verb / do + subject


I wrote to Sally more often than Carlos. . (The sentence incorrectly compares wrote to Carlos)
 I wrote to Sally more often than did Carlos.

Exercises: Correct the following sentences


I finished my assignment earlier and more perfectly than Sara.
…………………………………………………………………………………………………………..

SAT Question:
- Modern discus throwers use much the same technique of Ancient Greece.
(A) NO CHANGE
(B) of Ancient Greece
(C) as Ancient Greece did
(D) as they did in Ancient Greece

V+ing – compared to – V + ing


Running outdoors burns about five percent more calories than running on a treadmill.
SAT Question:
- Eating food that has a high concentration of fat causes essentially the same reaction in the stomach
than if you eat too fast.
(A) NO CHANGE
(B) than to eat
(C) as if one eats
(D) as eating

Top Score
GET READY TO STUDY
Chapter 4 Parallelism

Practice
1- Hiroko was sitting at the breakfast table, 5- Although Marion had made a commitment to
engrossed in a novel, when the telephone painting by September, she had neither the
rang and abruptly recalling her to the energy or the inclination to continue working
present. on it.
(A) NO CHANGE (A) NO CHANGE
(B) recalled (B) and
(C) to recall (C) but
(D) she recalled (D) nor
2- The ancient Sumerians wrote on clay tablets 6- Unlike Thomas, neither Leslie or her
which were either hardened to preserve the younger brother Philip has an interest in a
writing and left unhardened so that they career in law.
could be inscribed again. (A) NO CHANGE
(A) NO CHANGE (B) nor
(B) or (C) but also
(C) nor (D) and
(D) but also
7- The charm of Lofting's book lies in the
3- The flowers that Jane and Jonathan ordered humorous reversal of roles—the animals
to be sent to their mother were less fresh and guide, assist, and generally they take care of
much more expensive than Carr's flower the helpless humans.
shop. (A) NO CHANGE
(A) NO CHANGE (B) to take
(B) than Carr (C) taking
(C) than that of Carr (D) take
(D) than Carr's flowers
8- When people gave up the hunter—gatherer
4- From its modest beginnings as a series of way of life and began to cultivate the soil
brief vignettes to its establishment as the and grow their food they often became less
longest running prime-time series on mobile, built more substantial residences,
television, The Simpsons transformed the and they developed more effective means of
way both the audiences and television storage.
programmers view the animated sitcom. (A) NO CHANGE
(A) NO CHANGE (B) developed
(B) both the audience and the television (C) developing
(C) both audience and the television (D) to develop
(D) both the audience plus the television

Top Score
GET READY TO STUDY
Chapter 4 Parallelism

9- The introduction of elevators in hotels meant 14- Used in sculpture, carving is the process of
that previously undesirable rooms on the top reducing substances such as stone, wood, or
floors, away from the bustle and noise of the ivory to a desired shape by cutting or to chip
street, became sought and more expensive away unnecessary parts.
than the lower floors. (A) NO CHANGE
(A) NO CHANGE (B) chip
(B) rooms on the lower floors (C) chipping
(C) that of lower floors (D) when chip
(D) those of lower floors 15- Studies have suggested that eating nuts—
10- The flavor of a Meyer lemon, a fruit native almonds in particular—might help to lower
to China, is sweeter and less acidic than that blood cholesterol levels in humans and
of a common lemon, and a Meyer lemon's reducing the risk of heart disease by
skin is edible. protecting the blood vessels.
(A) NO CHANGE (A) NO CHANGE
(B) a common lemon (B) to reduce
(C) those of a common lemon (C) by reducing
(D) any flavors (D) reduce
11- The proposed environmental legislation 16- The hummingbird may appear to be a
would devote billions of dollars in federal delicate creature, but its body is
grants to conservation activities such as proportionately more muscular than any
acquiring new public lands, preserving bird.
coastal habitats, and to establish urban parks. (A) NO CHANGE
(A) NO CHANGE (B) more muscular than the other birds
(B) established (C) more muscular than that of any other
(C) also establish bird
(D) establishing (D) the most muscular of any other bird
12- Experts who decipher ancient written 17- According to some critics, the title character
languages report that the Indus civilization's of the Greek tragedy Oedipus Rex saw
script is more difficult to decode than other himself as the savior of his people and
civilizations. believing erroneously that he could do no
(A) NO CHANGE wrong.
(B) any other civilizations' scripts (A) NO CHANGE
(C) many civilizations (B) believed
(D) any scripts (C) believes
13- The revolt against Victorianism was perhaps (D) to believe
even more marked in poetry than either
fiction or drama.
(A) NO CHANGE
(B) either fiction or in drama
(C) either in fiction or drama
(D) in either fiction or drama

Top Score
GET READY TO STUDY
Chapter 4 Parallelism

18- The Arctic ringed seal employs its sharp- 23- When a steel mill is shut down because its
flippers not only to carve breadline holes in production methods have become antiquated
the thick sea ice and also to build its lair in what is lost is not only jobs and also a piece
snowdrifts over such holes. of industrial history.
(A) NO CHANGE (A) NO CHANGE
(B) but also building (B) but also
(C) but also to build (C) than
(D) but to build (D) or
19- It is thought that a dog's sense of smell is 24- Born around 46 C.E. in Greece, Plutarch is
generally 10,000 to 100,000 times better known primarily for parallel Lives, a series
than humans. of biographies in which he paired famous
(A) NO CHANGE Romans with famous Greeks and then
(B) humans' are comparing them in short essays.
(C) humans have (A) NO CHANGE
(D) a human's (B) to compare
20- Learning a foreign language is difficult not (C) by comparing
only because most languages contain an (D) compared
enormous number of words and because 25- Members of the Alvin Ailey Dance
people need to a language often to become Company have once again shown how the
comfortable with it. combination of strength and being agile can
(A) NO CHANGE produce beautiful movements.
(B) and also because people (A) NO CHANGE
(C) or because people (B) to be agile
(D) but also because people (C) agility
21- The computer company has increased sales (D) how to be agile
for three consecutive years, both by lowering 26- A museum in Cologne Germany, not only
prices on its personal computers and it added has exhibits on the history of chocolate and
new product lines. it offers a view of a factory floor where
(A) NO CHANGE visitors can watch a chocolate bar being
(B) adding made
(C) by adding (A) NO CHANGE
(D) to add (B) chocolate but also offers
22- Eating food that has a high concentration of (C) chocolate, they also offers
fat causes essentially the same reaction in the (D) chocolate and offers
stomach than if you eat too fast. 27- Because of its innovativeness and its
(A) NO CHANGE effective presentation, Mary's science project
(B) than to eat received more judges than did Jim.
(C) as if one eats (A) NO CHANGE
(D) as eating (B) Jim's
(C) did those of Jim
(D) did Jim's science project

Top Score
GET READY TO STUDY
Chapter 4 Parallelism

28- Lynn Margulis's theory that evolution is a 32- Because the high-altitude where it lives has
process involving interdependency rather cold temperatures, dry soil, and growing
than competition among organisms differs seasons are short, the bristlecone pine grows
dramatically from most biologists. slowly.
(A) NO CHANGE (A) NO CHANGE
(B) most biologists' theories (B) Short growing seasons
(C) that of most biologists (C) Short seasons are growing
(D) any other biologists (D) Short seasons
33- In some species of birds, such as the
29- The primatologist has argued that sustained peafowl, the plumage of the male is more
observation of a few animals provides better colorful and more variegated than the
behavioral data than does intermittent female.
observation of many animals. (A) NO CHANGE
(A) NO CHANGE (B) than are the females
(B) provides better behavioral data than (C) than that of the female
many animals are observed (D) compared to the female
intermittently
(C) providing better behavioral data than 34- Gustave Eiffel is famous not only for
does intermittent observation of many building the Eiffel Tower but also for
animals designing the internal structure of the statue
(D) do provide better behavioral data than of liberty.
intermittent observation of many animals (A) NO CHANGE
do (B) Building the Eiffel Tower but also that
he designed
30- Hubble's law has great significance not only (C) Building the Eiffel Tower, he also
because it describes expansion of the designed
universe, it can also be used to calculate the (D) Having Build the Eiffel Tower and also
age of the cosmos. for designing
(A) NO CHANGE
(B) universe, but it also can 35- For many a brilliant architect, being free to
(C) universe but also because it can innovate is more important than being well
(D) universe but because it also can paid.
(A) NO CHANGE
31- The owner's manual for the refrigerator (B) having freedom of innovation is more
explained how to connect the ice maker, important than
replacing the light bulb, and positioning the (C) there is more importance in the freedom
shelves. to innovate than
(A) NO CHANGE (D) freedom to innovate has more
(B) both replacing the light bulb as well as importance than
positioning the shelves
(C) with replacing the light bulb and then
positioning the shelves
(D) replace the light bulb, and position the
shelves

Top Score
GET READY TO STUDY
Chapter 4 Parallelism

Homework
1- Both Dorothy Sayers plus Carolyn 5- Meals prepared by the Algonquin Indians,
Heilbronn have written scholarly works as who were farmers as well as hunters,
well as popular murder mysteries, included more maize and pumpkin than
demonstrating a diversity of talents and other Indian tribes.
interests. (A) NO CHANGE
(A) NO CHANGE (B) pumpkin than did those prepared by
(B) but also other Indian tribes
(C) and (C) pumpkin than that which other Indian
(D) to tribes did
2- The bookstore sold three times as many (D) pumpkin; and other Indian tribes did not
textbooks this week as they ordinarily do in prepare meals in this way
an entire month. 6- The current generation of high school
(A) NO CHANGE students has more educational opportunities,
(B) as they ordinarily sell job training possibilities, and career options
(C) as it ordinarily sells to consider before entering the work force
(D) than they ordinarily sell than any other generation has had
3- The nineteenth-century lying of the first (A) NO CHANGE
telegraph cables between Europe and (B) do any generation
America was a project plagued by cable (C) for other generations
breakage, financial setbacks, and (D) would any generation
overcoming logistical problem. 7- The landscape artist who designed Central
(A) NO CHANGE Park New York City's central park believed
(B) plagued by cable the breakage and that providing scenic settings accessible to
financial setbacks, and by overcoming all would not only benefit the public's
logistical problems physical and mental health and also foster a
(C) that was plagued by the occurrence of sense democracy.
cable breakage and s financial setbacks, (A) NO CHANGE
with logistical problems (B) and would foster a sense of democracy
(D) plagued by cable breakage, financial also
setbacks, and logistical problems (C) but also foster a sense of democracy
4- One reason that an insect can walk on walls (D) and foster a sense of democracy also
while a human cannot is that the mass of its
tiny body is far lower than humans.
(A) NO CHANGE
(B) far lower than that of the human's body
(C) lower by far than humans
(D) far lower than a human

Top Score
GET READY TO STUDY
Chapter 4 Parallelism

8- A fine orchestral performance will exhibit 13- Confident that she was fully prepared, Ellen
the skills of the musicians, their abilities to decided to spend the night before the recital
work as an ensemble, and how he or she reading and relaxing but not be practicing.
responds to the conductor. (A) NO CHANGE
(A) NO CHANGE (B) more than to practice
(B) their responding (C) rather than practicing
(C) their responses (D) rather than having practiced
(D) they respond 14- During the 1920s many Cuban painters who
9- The bristlecone pine has a maximum life had traveled abroad returned to their
span of about 5000 years, which is much homeland to produce paintings that were
longer than almost any other tree. more angular and stylized than conventional
(A) NO CHANGE Western painters.
(B) and this is much longer than (A) NO CHANGE
(C) much longer than that of (B) west painters who were conventional
(D) much the longest of (C) conventions of Western painting
10- Studying dance for many years, practicing (D) conventional Western paintings
difficult steps every day, and frequent 15- When, after bleak and lonely years in an
performances has enabled the young dancer English public school, he returned to India,
the leading role in the spring production. there was suddenly perceived by himself a
(A) NO CHANGE strong to write about the people and land he
(B) a frequent performances, the young love.
dancer being enabled (A) NO CHANGE
(C) frequent performances, these enabled the (B) suddenly the feeling that came to him
young dancer being
(D) Performing frequently enabled the (C) he suddenly felt
young dancer (D) suddenly he had had the feeling of
11- The furnace exploded, blowing off the door,
spraying greasy soot all over the basement 16- I read recently that classical music is more
floor, and it would rattle furniture and popular among European high school
windowpanes throughout the building. students than students in America.
(A) NO CHANGE (A) NO CHANGE
(B) it rattled (B) than among American high school
(C) causing the rattling of students
(D) rattling (C) than the same music is among American
12- The development of an all-electric airplane high schools
is more problematic and complex than an all- (D) than is its popularity among American
electric car, several aeronautical companies students
are working to design and build all electric
airplanes.
(A) NO CHANGE
(B) all-electric cars
(C) those of an all-electric car
(D) the development of an all-electric car

Top Score
GET READY TO STUDY
Chapter 4 Parallelism

17- When chronological order is followed too 21- The judge ruled that the magazine must not
mechanically, they are obscuring rather than publish or in any way profiting from the
clarifying important relationships. illegally obtained photographs
(A) NO CHANGE (A) NO CHANGE
(B) chronological order, if too mechanically (B) should they profit
followed, obscures rather than it (C) to profit
clarifying important relationships (D) profit
(C) chronological order, if followed too 22- The art classes at the village museum are
mechanically, obscures rather than more important for their social value than for
clarifies important relationships their educational contribution.
(D) if you follow a too mechanical (A) NO CHANGE
chronological order, it obscures rather (B) than in the contribution they make
than clarifying important relationships educationally
18- In his book on the Harlem Renaissance, (C) rather than for the contribution they
Huggins makes clear determination not only make educationally
to emulate his intellectual forebears and (D) instead of for their educational
recognizing that their challenges were contributions
unique.
(A) NO CHANGE 23- To persuade his parents to let him study
(B) while recognizing that abroad, Kenneth described other students'
(C) but also to recognize that positive experiences, explains how foreign
(D) but also his recognition of how study would benefit his future career, and
19- The study found that the environmental costs assured them that he could get financial aid.
of producing and recycling paper cups were (A) NO CHANGE
as high as to produce and recycle (B) explained how foreign study would
polystyrene cups. benefit his future career, and assured
(A) NO CHANGE (C) explaining how foreign study is
(B) producing and recycling beneficial to his future career, and
(C) the production and recycling of assures
(D) those of producing and recycling (D) he explained how foreign study would
benefit his future career, and assuring
20- The survival of many species of marine life
may depend both the enforcement of waste- 24- The paintings by the junior high students
disposal regulations and the education of the displayed more sophisticated color schemes
public about the fragility of ocean resources. than the elementary school students.
(A) NO CHANGE (A) NO CHANGE
(B) educating the public (B) schemes than did those by the
(C) and the public being educated elementary school students
(D) in combination with public education (C) schemes as that which the elementary
school students did
(D) schemes, not like the elementary school
students

Top Score
GET READY TO STUDY
Chapter 4 Parallelism

25- Through his novels Thomas Wolfe reveals to 29- To complete the music program, a student
us both the pain and the beauty of his must present one vocal performance, one
boyhood in the American south. instrumental performance, and composing
(A) NO CHANGE one original work.
(B) Not only the pain is revealed to us in (A) NO CHANGE
Thomas Wolfe's novels but he also (B) and one original composition
describes (C) with one original composition
(C) Thomas Wolfe, though the medium of (D) and to compose one original work
his novels , reveals to us both the pain 30- Some Tibetan monks consider butter to be
with not only a delicacy and also a substance of
(D) As a novelist, Thomas Wolfe thus spiritual value.
revealing to us the pain and (A) NO CHANGE
26- Most famous for her record-breaking trip (B) as well as
around the world, Nellie (1864-1922) also (C) but also
pioneered a new type of undercover (D) besides being
investigative journalism and inventing a type 31- Daring a conference with Pravika's parents,
of steel barrel. the teacher mentioned that Pravika had
(A) NO CHANGE demonstrated considerable ability in math
(B) invents a type of steel barrel and to learn foreign languages.
(C) invented a type of steel barrel (A) NO CHANGE
(D) a type of steel barrel was invented by her (B) considerable ability in math and that she
27- The underside of the starfish is covered could do it well in
hundreds of tube feet, which it uses to walk (C) ability that was considerable math as
around, for attaching tightly to rocks, and well as in learning
holding on to prey. (D) considerable ability in math and in
(A) NO CHANGE 32- Lorena Ochoa has been recognized not only
(B) to walk around, for its tight attachment as a champion golfer but also being a
to rocks, and to hold humanitarian, having created a foundation to
(C) for walking around, to attach tightly to support young people in Mexico.
rocks, and holding (A) NO CHANGE
(D) for walking around, attaching tightly to (B) but also as
rocks, and holding (C) she also is
(D) also that she is
28- William Dalrymple's book is not only a
gripping tale of politics and power and also
evidence of the complexity of cross- cultural
relationships.
(A) NO CHANGE
(B) but also
(C) as well as
(D) and

Top Score
GET READY TO STUDY
Chapter 4 Parallelism

33- The 1922 silent film the toll of the sea is 37- A hot-air balloon rises when the air inside it
notable both for being the first commercial becomes warm, drops when that air cools
movie shot entirely in Technicolor and and moves sideways when blown by the
Chinese American actor Anna Wong was wind.
starring in it. (A) NO CHANGE
(A) NO CHANGE (B) moving sideways when the wind blows
(B) Chinese American actor Anna Way (C) when the wind blows, the movement is
Wong was starred in the film as well sideways
(C) for starring Anna May Wong, a Chinese (D) caused to move sideways when blown
American actor by the wind
(D) its star was Chinese American actor 38- A promising food source, the winged bean is
Anna May Wong rich in protein, needs no fertilizer, and even
34- The mansion house, once home to a thriving enriches the soil in which it grows.
utopian community, remained closed to the (A) NO CHANGE
public for over a century and reopening as a (B) enriching the soil where they grow
museum in 1987. (C) where they grow they enrich the soil
(A) NO CHANGE (D) the soil is enriched where they grow it
(B) century it reopened
(C) century but reopened 39- Running outdoors burns about five percent
(D) century, when reopening more calories than if you are running on a
35- After she won a gold medal in figure skating treadmill, in part this is because there is
at the 1956 Olympic Games, Tinley Albright greater wind resistance outdoors.
attended medical school and becoming a (A) NO CHANGE
surgeon. (B) if you run on treadmill, in part because
(A) NO CHANGE of the
(B) school and became (C) when running on a treadmill, in part
(C) school, she became because of the
(D) school; also becoming (D) running on the treadmill, in part because
there is
36- Veterans of World War II received greater
support from the public than the Korean and 40- Between 1508 to 1512, Michelangelo,
Vietnam Wars. working on a scaffold 60 feet above the
(A) NO CHANGE floor, painted the vaulted ceiling of the
(B) than did Sistine chapel in Rome with hundreds of
(C) than did veterans of giant figures that represented his vision of
(D) than from the support of the word's creation.
(A) NO CHANGE
(B) or
(C) and
(D) but

Top Score
GET READY TO STUDY
Chapter 4 Parallelism

Mini Exam
Making Bamboo a Common Building Material . 1 .. .

Bamboo is one of the world's most useful (A) NO CHANGE


(B) to building material
plants. For thousands of years bamboo has been
(C) building material
used in many different ways-from food to (D) also building material
medicine, to clothing and, in small ways. 1 . as a
. 2 .. .
building material.
However, bamboo is not often used as a (A) NO CHANGE
building material or . 2 .as food in the developed (B) to feed
(C) it used as food
world. It is not made according to officially set
(D) for food
measurements, or . 3 . in standardized way. It
simply grows in the ground. A professor at the . 3 .. .
University of Pittsburgh in Pennsylvania is trying (A) NO CHANGE
to change that. (B) standardizing
The professor and his students are testing and (C) to standardize
. 4 . to measure the strength of bamboo. They are (D) standards
testing its radial load, or how much pressure or . 4 .. .
. 5 . weigh it can take before breaking. That
(A) NO CHANGE
pressure was too much. (B) for measuring
First, it is strong. The strength of at least (C) measuring
three species of bamboo is . 6 . similar to steel. (D) measure
Besides its strength, Harries says that bamboo has . 5 .. .
other features that make it a great building
material. He says it is resilient, meaning it keeps (A) NO CHANGE
(B) weight
not only its shape . 7 . and also its strength even
(C) to weigh
under pressure. (D) its weight
"It's a remarkably strong material,
. 6 .. .
remarkably resilient. It's optimized to be very tall.
Obviously bamboo material, something like this, (A) NO CHANGE
might grow 20 meters and it supports its own (B) similar to those of the steel
(C) similar to the strength of the steel
weight, and, of course, it also supports huge wind
(D) than that of the steel
loads ... ."
. 7 .. .

(A) NO CHANGE
(B) but also its strength
(C) and its strength
(D) but also being strong

Top Score
GET READY TO STUDY
Chapter 4 Parallelism

Bamboo also grows quickly. Bamboo . 8 .. .


that is suitable for construction needs (A) NO CHANGE
much less resources than . 8 . that of wood. (B) wooden objects
The harvest cycle of bamboo is about 3 (C) any kind of woods
(D) does wood
years. Softwoods such as cedar, pine and
. 9 . spruce have a harvest cycle of about 10 . 9 .. .
years. And hardwoods that come from
(A) NO CHANGE
flowering plants such as oak, maple and (B) what called spruce have a harvest cycle
walnut need more than 30 years. (C) spruce have a harvest cycle
(D) that of spruce have a harvest cycle

. 10 .. .

(A) NO CHANGE
(B) standardizing
(C) to standardize
(D) standards

. 11 .. .

(A) NO CHANGE
(B) for measuring
(C) measuring
(D) measure

. 12 .. .

(A) NO CHANGE
(B) weight
(C) to weigh
(D) its weight

. 13 .. .

(A) NO CHANGE
(B) similar to those of the steel
(C) similar to the strength of the steel
(D) than that of the steel

. 14 .. .

(A) NO CHANGE
(B) but also its strength
(C) and its strength
(D) but also being strong

Top Score
GET READY TO STUDY
Chapter 4 Parallelism

Answers
Explanation:
Page 1: D / C / B

Page 2: C/ B / C / C / D

Page 3: C / C / B / C / B

Page 4: D / B / C / B / C

Page 5: D / C / Fresh vegetables at a farmers’ market are sometimes lower in price than fresh vegetables
at a grocery / Fresh vegetables at a farmers’ market are sometimes lower in price than those at a
grocery / Muscles in the leg are stronger than muscles in the arm / Muscles in the leg are stronger than
those in the arm / C

Page 6: D / I finished my assignment earlier and more perfectly than Sara did OR did Sara / C / D

Practice:
1 B / 2 B / 3 D / 4 B / 5 D / 6 B / 7 D / 8 B / 9 B / 10 A / 11 D / 12 B / 13 D / 14 C / 15 D / 16 C / 17 B / 18 C / 19 D / 20
D / 21 C / 22 D / 23 B / 24 D / 25 C / 26 B / 27 D / 28 B / 29 A / 30 C / 31 D / 32 B / 33 C / 34 A / 35 A

Homework:
1 C/ 2 C/ 3 A / 4 B / 5 B / 6 A / 7 C / 8 C / 9 C / 10 D / 11 D / 12 D / 13 C / 14 D / 15 C / 16 B / 17 C / 18 C / 19 D / 20 A /
21 D / 22 A / 23 B / 24 B / 25 A / 26 C / 27 D / 28 B / 29 B / 30 C / 31 D / 32 B / 33 C / 34 C / 35 B / 36 C / 37 A / 38 A /
39 D / 40 C

Mini Exam:
1B/2A/3D/4C/5B/6C/7B/8D/9C

Top Score
GET READY TO STUDY
Chapter 5 Verb-subject Agreement

Verb-subject Agreement
Quick Revision:

Verb to be
Plural Singular
are / were is / was

Verb to have
Plural Singular
Have has

Verb to do
Plural Singular
Do does

Present simple
Plural Singular
Infinitive Infinitive + s

Examples:
1- Martina has / have her breakfast early.
2- Leaf, flower, and seedpod were / was glimmering with frost.
3- My cousin always arrives / arrive home at five o'clock.
4- Benito doesn't / don't know the answer.
5- George and Tamara doesn't / don't want to see that movie.
6- The committee debates / debate these questions carefully.

Top Score
GET READY TO STUDY
Chapter 5 Verb-subject Agreement

1) Subject + prep phrase + Verb


 Singular noun + prep phrase + singular verb
 Plural noun + prep phrase + plural verb
NB: Prep: {of / from / about / to / with / as well as / along with / in addition to / together with, etc.}
Examples:
The murmur of bees buzzing around the grassy meadows sounds delightful.
The talks about burning of the embassy aren't true.

Exercises:
The independence of both Jerusalem and Golan was / were our best dream.
The harmful effects of excessive alcohol intake on the hepatic system is / are extensively
documented by medical studies.
The movement to establish women's issues as important subjects of study have / has had a
profound impact on the curricula offered in colleges today.
The cardamom, along with the cinnamon and apple, give / gives the dish its unusual flavor.

SAT Question:

- The theories of planetary motion espoused by Galileo was seen as heretical by members of the
Inquisition, who used their power to force Galileo to recant and spend the last years of his life under
house arrest.
(A) No change
(B) were seen
(C) are seen
(D) has been seen

- The relationship between goby fish and striped shrimp are truly symbiotic, for neither can survive
without the other.
(A) No change
(B) were
(C) have
(D) is
- A study by two marketing professors has found that the best way for a newspaper to increase profits
is to spend more of its budget on improving news coverage.
(A) No change
(B) have found
(C) were found
(D) are found
Top Score
GET READY TO STUDY
Chapter 5 Verb-subject Agreement

2) Subject + P.P / Gerund...+ Verb


Examples:
The plans proposed by the students for the home-combing celebration are perfectly clear.
The people sitting behind me in the movie theater were talking throughout the film.
SAT Question:
- The light emitted by high-intensity- discharge car headlights are very effective in activating the
reflective pains of road markers, thereby making driving at night safer.
(A) No change
(B) were
(C) was
(D) is
- Biologists working to rescue the giant condor from extinction uses radio telemetry and satellite
signals to track banded birds.
(A) No change
(B) using radio telemetry and satellite signals to track
(C) use radio telemetry and satellite signals to track
(D) to track, by use of radio telemetry and satellite signals,

3) Subject + Relative Clause…+ Verb


 The relative clause is a sentence which starts in relative pronoun: (which / who /
whose / whom / that).
Examples:
John Moffet and Paolo Morales, who were swimmers on the United States team, set world
record.
Exercises:
James and Maria who had been studying for an exam need / needs a break.
Each player on the team that wins the tournament receives / receive a trophy.
The bowl which was full of gravy has / have slipped off the table and broke.
The goods that imported from China is / are very cheap.
SAT Question:
- Paule Marshall, whose Barbadian background has influenced her writing, describes many details of
life in the Caribbean Islands vividly in her novels and short stories.
(A) No change
(B) describe
(C) have described
(D) were described

Top Score
GET READY TO STUDY
Chapter 5 Verb-subject Agreement

4) Subject, Appositive, Verb


Examples:
Joseph Pulitzer, a Hungarian born immigrant to the United States and an innovator in the
field of popular Journalism, establishes the Pulitzer Prize in journalism and literature.
Exercises:
The newly elected Prime Minister, to the dismay of opponents from other parties, has / have
argued for the strict regulation of campaign financing.
SAT Question:
- A sweetener, normally either sugar or syrup, are used in almost all bread for taste or as an aid yeast
growth.
(A) No change
(B) is used
(C) are using
(D) have used

5) Compound subjects linked by "and"


 Noun, noun, and noun…Plural verb

Examples:
The television set and the washing machine were damaged by the short circuit.

Exercises:
A diamond necklace and an emerald ring was / were stolen from the hotel safe.
Interviews with prominent activists and a letter from the Prime Minister form / forms the
core of the objections to the plan.
Mistletoe and the fir tree appear / appears on innumerable cards every Christmas.
SAT Question:
- Turquoise-inlaid frog and bird figurines seems to have played an important role in prehistoric
ceremonies in what is now the southwestern United States.
(A) No change
(B) seems to have played important roles
(C) seem to have played important roles
(D) seeming to be playing important roles

Top Score
GET READY TO STUDY
Chapter 5 Verb-subject Agreement

6) Singular indefinite pronouns


The singular indefinite pronouns take singular verbs, even if followed by a plural noun.
(Someone / anyone / everyone / either / neither / no one / none / each, etc.)
Examples:
Neither of the students is prepared for the interview.
Each of the supervisors wants to leave early.

Exercises:

I am not convinced that either of your excuses pass / passes the test.
Anybody foolish enough to go near the hot lava is / are likely to get burned or to inhale
noxious fumes.
The survey shows that none of the respondents wants / want to live close to the radio
antenna.

SAT Question:
- In neighborhoods throughout the United States, one can encounter hundreds of different rope-
jumping games, each with its own rules.
(A) No change
(B) each having their own rules
(C) when they each have their own rules
(D) which has its own rules

- The Empire State Building, the Sears Tower, the Canadian National Tower - each of these structures
was the tallest in the world at the time they were built.
(A) No change
(B) it was
(C) it is
(D) they have been

7) Neither…nor / Either…or
 In these sentences the verb agrees with the subject that follows nor/or.
Examples:
Either your parents or your sister is required to attend.
Either your sister or your parents are required to attend.
Neither the passengers nor the driver was hurt.
Neither the driver nor the passengers were hurt.
Top Score
GET READY TO STUDY
Chapter 5 Verb-subject Agreement

Exercises:
Neither black pepper nor pimento has / have the pungency of green chili.
Either the two beds or the wardrobe has / have to be left behind as the removal van is too
full.
The surgeon insisted that neither he nor his nurses was / were responsible for the error.
SAT Question:
- Of the two options, neither the system of appointing judges to the bench nor the process of electing
judges are entirely satisfactory.
(A) No change
(B) have
(C) is
(D) were

8) Collective nouns
 Collective nouns such as jury, team, army, orchestra, equipment, etc. ...normally take
singular verbs.
Examples:
The orchestra performs best in the new auditorium.
Exercises:
An army march / marches on its / their stomach.
The new group, consisting of just four young but talented musicians, makes / make the old
school band seem out-dated.
The jury, as directed by the judge, was / were sequestered for four days.
SAT Question:
- The math team, which included Tom and me, was stuck at the airport overnight because of inclement
weather.
(A) No change
(B) were
(C) are
(D) have

9) A number of / the number of


 The expression the number of takes a singular verb.
 The expression a number of takes a plural verb.
Examples:
A number of artists are now using this technique.
The number of children contracting these dangerous diseases is falling.

Top Score
GET READY TO STUDY
Chapter 5 Verb-subject Agreement

Exercises:
I wonder whether the number of philosophy students is / are about to increase.
A number of the trainees that we recruited has / have already dropped out.
The number of blogs commenting on international affairs has / have continued to rise, but at
a lower rate.
The number of students taking standardized tests increase / increases every year.
SAT Question:
- In a nearby resort community fewer accidents have been reported that the number of speeding tickets
to be issued have decreased since speed bumps were installed.
(A) No change
(B) issued have been on the decreased
(C) issued has decreased
(D) that were issued have now decreased

10) Latin and Greek plurals


 Certain words of foreign origin are so often used in the plural that we forget they have
singular forms.
Latin and Greek Plurals
Singular Plural
Memorandum memoranda
Datum data
Criterion criteria
Phenomenon phenomena
Medium Media
Analysis Analyses
Radius radii
Atrium atria

Examples:
Your data are not sufficient to justify your conclusion.
Exercises:

The phenomena currently under investigation by scientists concerns / concern the


interactions of laser light with biological materials.
The criteria used to select the finalist was / were unclear to the audience.
Our research failed because the data on Etruscan antiquities was / were impossible to obtain
in the time available.

Top Score
GET READY TO STUDY
Chapter 5 Verb-subject Agreement

SAT Question:
- Careful analysis of pictures of the Moon reveal that parts of the Moon's surface are markedly similar
to parts of the Earth's.
(A) No change
(B) reveals that
(C) reveals that:
(D) revealed, that

11) Gerund subject is singular


Example:
Reading stories is my favorite hobby.

Exercises:
Eating a lot of sugary foods results / result in tooth decay.
SAT Question:
- Some doctors believe that taking vitamins on a daily basis help decrease a patient's susceptibility to
infection.
(A) No change
(B) believes that taking vitamins on a daily basis help
(C) believe that taking vitamins on a daily basis helps
(D) believes that taking vitamins on a daily basis helps

12) Inverted subject and verb


 There is + singular / There are + plural / Here is + singular / Here are + Plural
Examples:
In the corner there are some red balls.
Here are your books, Sam.
SAT Question:
- Although science offers the hope of preventing serious genetic diseases, there is difficult ethical
questions raised by the possibility of altering human heredity.
(A) No change
(B) there was
(C) there are
(D) there were
 When the sentence begins the prepositional phrase:
(In–on–at–with–for–across–alongside–throughout–above–from, etc.)

Top Score
GET READY TO STUDY
Chapter 5 Verb-subject Agreement

 Prep phrase + verb + subject


 Prep phrase (,) subject + verb
Examples:
For the lunch went the girl with her dad.
For the lunch, the girl went with her dad.
SAT Question:
- In the central courtyard was over a dozen different varieties of lilies, meticulously maintained by the
gardener.
(A) No change
(B) were
(C) has
(D) have
- Not until the recent scandal has the newspapers published anything even vaguely negative about the
company or its executives.
(A) No change
(B) have
(C) are
(D) were
- Also supported by the commission was the proposed health clinics and the proposed center to
distribute information on job-training opportunity.
(A) No change
(B) were
(C) has
(D) is
 Comparisons:
Subject + Verb THAN Verb + Subject
- Venezuela devotes a higher percentage of its budget to education than do other large Latin American
countries such as Mexico and Brazil.
(A) No change
(B) than doing
(C) than does
(D) than did

NB:
 All Italic words, even if in Plural form, are singulars.
 Names of the countries (Emirates / Netherlands), organizations (United Nations), or
subjects and sciences (Mathematics–Cryonics–News)
Exercises:
The Great Expectations, which written by Charles Dickens, is / are the one of the greatest
novel in the history.
Faroe Islands is / are an archipelago and autonomous country within the Kingdom of
Denmark.

Top Score
GET READY TO STUDY
Chapter 5 Verb-subject Agreement

Practice
1- At the 1984 Olympic Games, John Moffet and 5- Innovative use of computers in the classroom
Paolo Morales, who were swimmers on the allows students to undertake projects that
United States team, set world record. encourages them to be both analytical and
(A) No change intellectually adventurous.
(B) have been (A) No change
(C) was (B) encouraging them
(D) is (C) encourage they
(D) encourage them
2- A newly formed organization of homeowners
and business people have met with the 6- Freedom of action and expression are at the
transportation department to voice its concerns foundation not only of government but also of
about plans for a shopping mall in the our expectations concerning human relations at
community. all levels of society.
(A) No change (A) No change
(B) has met (B) is
(C) were met (C) have
(D) was met (D) were
3- No one of the students in the advanced chemistry 7- Absent from the speech were any mention of the
class have passed a single test with a grade better students and laboratory technicians upon whose
than a C, but the second half of the course will contributions the chemist had depended heavily.
be easier. (A) No change
(A) No change (B) was
(B) have managed to pass a single test with (C) are
better than a C grade (D) have
(C) have passed a single test any better than a
8- Never before had a group of artists been as
grade of C
isolated from society and from official patronage
(D) has passed a single test with better than a C
as was the so called Impressionists.
grade
(A) No change
4- A flurry of do-it-yourself books on the market (B) than was
today are inspiring homeowners to do their own (C) as were
repairs. (D) as has
(A) No change
9- After carefully studying of both the articles, Dr.
(B) is inspiring homeowners into doing their own
Rodriguez and Nurse Alba found that the only
repairing
difference between them were their titles.
(C) is inspiring homeowners to do their own
(A) No change
repairs
(B) them were the titles
(D) inspiring homeowners to repair their own
(C) the articles were the titles
homes
(D) the articles was their titles

Top Score
GET READY TO STUDY
Chapter 5 Verb-subject Agreement

10- Decorative arts, art forms that have a mainly 14- No two of the specimens was sufficiently alike
practical or ornamental purpose, and include to warrant them being called members of a
ceramics, basketry, furniture making and single species.
(A) No change
glassblowing.
(B) was sufficiently alike to warrant the calling
(A) No change of them
(B) including (C) was sufficiently alike to warrant their
(C) includes being called
(D) include (D) were sufficiently alike to warrant calling
them
11- The credit for making Franz Kafka
internationally famous as a writer belong to his 15- The uncompromising tone of a recent city hall
friend, novelist Max Brod, who edited Kafka's ordinance concerning the blocking of
unpublished manuscript and then had them emergency vehicles in traffic jams carry a stern
published, despite Kafka's dying wishes to the warning to motorists.
contrary. (A) No change
(A) No change (B) are carrying
(B) belonging (C) have carried
(C) belongs (D) carries
(D) to belong 16- Experts agree that permanently modifying
12- Cryonics—the technique used to store human eating and exercise habits rather than merely
dieting for brief periods are the key to
bodies at extremely low temperatures with the
controlling weight.
hope of one day reviving them—are being
(A) No change
performed today, but their technology is still in
(B) were the key controlling
its infancy.
(C) have been the key controlling
(A) No change
(D) is the key to controlling
(B) are being performed today, while the
technology 17- The scientific writings of Edward O. Wilson,
(C) is being performed today, but their Stephan Jay Gould and Richard Dawkins,
technology which has continued the discussion of genetic
(D) is being performed today, but the issues raised by Charles Darwin, are familiar to
technology many high school and college student.
(A) No change
13- The governor's aides are convinced that the
(B) which have continued
announcement of the investigation, coming just
(C) which was continued
days before the filing deadline, were calculated
(D) which is continued
to discourage the governor from running for
reelection. 18- Air pollution caused by industrial fumes has
(A) No change been studied for years, but only recently has the
(B) was calculated harmful effects of noise pollution become
(C) have been calculated known.
(D) are calculated (A) No change
(B) but only recently have the harmful effects
(C) but only recently have the harmful effect
(D) but only recently is the harmful effects

Top Score
GET READY TO STUDY
Chapter 5 Verb-subject Agreement

19- The main reasons students give for failing to 24- Ongoing research by several scientists suggest
participate in the political process is that they that regular periods of meditation reduce blood
have demanding assignments and work at part- pressure and are likely to contribute to other
time jobs. improvements in health.
(A) No change (A) No change
(B) is demanding assignments and they work (B) suggest, that
at (C) suggests that
(C) are that they have demanding assignments (D) suggesting that
and that they work at
25- The theory that the mass extinctions of 250
(D) is having demanding assignments and
million years ago were caused by increased
having to work at
volcanic activity and a warming climate are
20- Evidence from surveys and interviews show based around newly discovered evidence.
friendships made in high school tend to last (A) No change
longer than those made in college. (B) climate were based around
(A) No change (C) climate is based on
(B) show high school friendships that tend to (D) climate they based this on
last
26- Inscribed on the wall is the names of those who
(C) shows that friendships made in high school
left their homes in the village to travel to the
tend to last
United States.
(D) shows friendships in high school tends to
(A) No change
last
(B) was
21- The newly elected Prime Minister, to the (C) has been
dismay of opponents from other parties, have (D) are
argued for the strict regulation of campaign
27- Jason is the only one of the three people I knew
financing.
well in middle school who still write to me
(A) No change
regularly.
(B) has argued
(A) No change
(C) were argued
(B) writing
(D) are argued
(C) writes
22- Only by tapping their last reserves of energy (D) are writing
were the team members able to salvage what
28- There is, in spite of the large bay window and
was beginning to look like a lost cause.
the new floors, many reasons, such as the poor
(A) No change
location and the small basement, for Nancy not
(B) was the team members
to buy the house.
(C) has been members
(A) No change
(D) is the members
(B) There was
23- In those cities in which public transportation is (C) There are
adequate, fewer traffic problems occur and (D) There has been
pedestrians are rarely involved in accidents.
(A) No change
(B) was rarely involved in
(C) is rarely involved in
(D) has been rarely involved in

Top Score
GET READY TO STUDY
Chapter 5 Verb-subject Agreement

Homework
1- All the talk about controlling noise, keeping 6- Ronald Takaki, grandson of Japanese
rivers clean, and planting trees have not immigrants who worked as plantation laborers
impressed people enough to be bringing about in Hawaii, have been recognized as an expert in
major changes in laws and lifestyles. the field of multicultural studies.
(A) No change (A) No change
(B) have not made enough of an impression on (B) and has been recognized to be an expert in
people to bring the field of multicultural studies
(C) has not impressed people enough to bring (C) is recognized as an expert in the field of
(D) has not made enough people impressed for the multicultural studies
bringing (D) he is recognized in the field of
2- The opposing opinion expressed were that the multicultural studies as an expert
school should be torn down and, on the other 7- The pace at which industrial and
hand, to keep it as a historical landmark. communications technologies are progressing
(A) No change in developing countries are so rapid that many
(B) was that the school should be torn down pr governments cannot anticipate the harm these
kept technologies may do to the environment.
(C) were that the school should be torn down and (A) No change
that it should be kept (B) is progressing in developing countries are
(D) were if they should tear the school down and (C) are progressing in developing countries is
keeping it (D) is progressing in developing countries is
3- The loyalty of British families to specific brands
of toothpaste have been cited by historians as 8- Yet to be discussed in the conference is more
evidence of the growing influence of advertising than a dozen proposals for changes in the
during the early Twentieth Century. procedural rules for choosing new officers.
(A) No change (A) No change
(B) has been cited (B) was more than a dozen proposals
(C) were cited (C) are more than a dozen proposals
(D) are cited (D) are most than a dozen proposals
4- Some parents believe that a weekly allowance 9- The validity of IQ tests as accurate measures
help children to appreciate the importance of of human intelligence have been the subject of
good money management skills. much debate.
(A) No change (A) No change
(B) help children appreciating (B) are often the subject of much debate
(C) helping children to appreciate (C) has been the subject of much debate
(D) helps children to appreciate (D) are a debate that is frequently held
5- Michael Jordan, Wayne Gretzky and Jim Brown,
each of these athletes was considered to be the 10- Bob hope, long considered a hero by many
best at his sport at the time they were playing. members of the military, were very patriotic.
(A) No change (A) No change
(B) they are (B) are
(C) they have been (C) was
(D) he was (D) have been

Top Score
GET READY TO STUDY
Chapter 5 Verb-subject Agreement

11- The harmful effects of excessive alcohol intake 16- The same theory that allows linguists to explain
on the hepatic system is extensively why certain verbs take only transitive or
documented by medical studies. intransitive forms also help psychologists to
(A) No change explain how the human mind processes sensory
(B) is documented more extensively by information from the everyday world.
medical studies (A) No change
(C) are extensively documented by medical (B) helps psychologists to explain
studies (C) helping psychologists to explain
(D) has documented extensively by medical (D) to help psychologists to explain
studies 17- A careful reading of Galileo's written work and
12- The surgical method of inserting the valve, correspondence, which include dozens of
which includes making a small incision letters to his daughter, suggest that his
between the ribs, are intended to shorten experiment testing the theory of gravitation was
recovery time and reduce complications in fact conducted at the leaning Tower of Pisa.
associated with traditional open-heart surgery. (A) No change
(A) No change (B) includes dozens of letters to his daughter,
(B) include making a small incision between suggests
the ribs, are intended (C) include dozens of letters to his daughter,
(C) includes making a small incision between suggests
the ribs, is intended (D) include dozens of letters to his daughter,
(D) include making a small incision between suggesting
the ribs, is intended 18- Most linguists are convinced that the ability to
13- Neither Roger nor his sister was able to speak, while uniquely human, is simply a
understand what the conductor was saying combination of cognitive skills that have been
because they did not know how to speak passed on to our species through evolution.
French. (A) No change
(A) No change (B) the ability to speak, while uniquely human,
(B) were able to understand what the are simply
conductor was (C) the ability to speak, while uniquely human,
(C) was able to understand what the conductor being simply
were (D) the ability to speak, while uniquely human,
(D) was able to understand what the conductor have been simply
is 19- Many existentialistic works, such as Samuel
14- The reluctant of the top ambassadors to initiate Becket's Malone dies, which subordinates the
diplomatic exchange with neighboring role of plot to the role of introspection.
countries were baffling to many observers. (A) No change
(A) No change (B) which subordinate
(B) was baffling to many observers (C) subordinate
(C) are baffling to many observers (D) subordinates
(D) have been baffling to many observers 20- Although there is a lot more than twenty copies
15- The change in seasons, while beautiful, often of the book left in the storeroom, they will sell
cause people to fall ill. out quickly, so I recommend that we order
more now.
(A) No change (A) No change
(B) often causing
(C) often causes (B) their are
(D) therefore causes (C) there are
(D) there have been

Top Score
GET READY TO STUDY
Chapter 5 Verb-subject Agreement

21- The effort required to study for seven final 27- Traditional Jamaican music, enriched with
exams within the span of four days are far more rock, jazz and other modern rhythm from
than the typical student can manage. American, were the basis for reggae.
(A) No change (A) No change
(B) is far more (B) have been a basis for
(C) were far more (C) become the basis of
(D) being far more (D) was the basis for
22- The investment required to develop new 28- In the Nineteenth Century, reproductions of
technologies are often so enormous that very cathedrals or castles made entirely of ice was
few energy companies are able to undertake often a popular feature in North American
innovative ventures. winter carnivals.
(A) No change (A) No change
(B) were often so enormous (B) often were popular features
(C) is often so enormous (C) often was featured popularly
(D) having been so enormous (D) often being popular features
23- The rapid growth in the rate of construction of 29- Early rock and roll music, like jazz and the
private homes and apartment buildings have blues, celebrated characteristics of urban life
been stimulated by low interest rates. that was only disparaged: loud, repetitive city
(A) No change sounds were reproduced as raucous melodies
(B) private homes and apartment buildings has and insistent rhythms.
(C) private homes and apartment buildings are (A) NO CHANGE
(D) private homes and apartment buildings (B) is
having (C) were
24- Noam Chomsky has long argued that the media (D) has
plays a large role in skewing our perception of 30- The Bridge of Sighs, a partially enclosed bridge
reality, presenting images and framing stories built in Venice in the Sixteenth Century,
in such a manner as to misrepresent the truth. connect the Ducal Palace with the state prison.
(A) No change (A) NO CHANGE
(B) playing a large role (B) have connected
(C) is playing a large role (C) connecting
(D) play a large role (D) connects
25- Each of the stories end with a moral but some 31- Angered by the administrator's recent refusal to
of these lessons are more obvious than others, address their concerns about working
spelled out so as to make them impossible to conditions, most members of the hospital staff
overlook. is expressing their discontent by showing up
(A) No change late for work.
(B) ends with a moral (A) NO CHANGE
(C) ending with a moral (B) are expressing
(D) to end with a moral (C) expressing
26- Having such acute senses of hearing, smell and (D) has been expressing
sight, zebras often provide early warning to 32- After extensive remodeling, each of the historic
other grazers that predators are approaching. hotel's 75 rooms have an Internet connection
(A) No change for use by guests.
(B) providing (A) NO CHANGE
(C) provides (B) having
(D) is providing (C) has
(D) is having

Top Score
GET READY TO STUDY
Chapter 5 Verb-subject Agreement

Mini Exam
Prisma App Makes Art Out of Photos . 1 .. .

Pokemon GO is the most popular app in (A) NO CHANGE


(B) adding
the world right now. But another new app is also (C) add
(D) has added
getting a lot of attention. And it’s free. Prisma lets
users . 1 . adds filters to photos to interesting, . 2 .. .

beautiful and strange effect. The app is going (A) NO CHANGE


(B) adds
viral this summer with over 1 million daily users (C) adding
in its first month. Prisma is a free app that . 2 . (D) to add

add filters to your photos to turn them into . 3 .. .

beautiful works of art. The app . 3 . has 35 filters (A) NO CHANGE


(B) having
to use with your photos. . 4 . All of the filters is (C) have
included free with the app. (D) is having

Each filter . 5 . give a different look to . 4 .. .

photos. Many of the filters . 6 . is in the style of (A) NO CHANGE


(B) Each of the filters are
famous artists. They range from pencil drawings (C) All of the filters are
to modern art to classic art with many styles in (D) Many filters is

between. Filters have names such as Colored Sky, . 5 .. .


Tokyo, Papered Art, Heisenberg, Mosaic, Illegal (A) NO CHANGE
(B) have given
Beauty, Dreams and Running in the Rain.
(C) gives
Using Prisma is about as easy as an app (D) gave

can get. Download the Prisma app to your phone . 6 .. .


or tablet and give it permission to use your photos (A) NO CHANGE
(B) are
and camera. No need to set up an account. Users
(C) were
. 7 . do not have to log in or link any social media (D) being
accounts to use the app. The phone must be . 7 .. .
connected to the Internet for Prisma to work. The (A) NO CHANGE
app works in the Cloud, not on your phone. Users (B) did
(C) does
who . 8 . has a limited data plan may want to use (D) doing
Prisma when connected to a Wi-Fi network. . 8 .. .

(A) NO CHANGE
(B) were
(C) is
(D) have

Top Score
GET READY TO STUDY
Chapter 5 Verb-subject Agreement
Another settings possibility . 9 . are to . 9 .. .
save every photo automatically, although this is a
(A) NO CHANGE
bad idea. You will likely try many filters and if (B) were
you save every photo you create, you will be (C) is
(D) have been
filling up your phone fast. Better
.10.. .
to manually save the photos you like. If you
forget, you can always create the artwork easily (A) NO CHANGE
(B) suggest that
again. (C) suggest's that
Prisma's privacy policy and terms of use (D) suggest, that

.10. suggests that the app may add advertising in .11.. .


the future. Prisma .11. do not gain ownership of (A) NO CHANGE
the images users .12. creates through the app. (B) did
(C) doing
But Prisma does get an unlimited license to use
(D) does
those images. Be sure to read the privacy policy
.12.. .
and terms of service if these issues concern
(A) NO CHANGE
you.
(B) created
(C) creating
(D) create

Top Score
GET READY TO STUDY
Chapter 5 Verb-subject Agreement

Answers
Explanation:
Page 1: has / were / arrives / doesn't / don't / debates

Page 2: was / are / has / gives / B /D /A

Page 3: D / C / receives / has / are / A

Page 4: has / B / were / form / appear / C


Page 5: passes / is / wants / A / B
Page 6: has / has / were / C / marches / its / makes / was / A

Page 7: is / have / has / increases / C / concern / were / were

Page 8: B / results / C / C

Page 9: B / B / B / A / is / is

Practice:
1 A / 2 B / 3 D / 4 C / 5 D / 6 B / 7 B / 8 C / 9 D / 10 D / 11 C / 12 D / 13 B / 14 D / 15 D / 16 D / 17 B / 18 B
/ 19 C / 20 C / 21 B / 22 A / 23 A / 24 C / 25 C / 26 D / 27 C / 28 C

Homework:
1 C / 2 B / 3 B / 4 D / 5 D / 6 C / 7 C / 8 C / 9 C / 10 C / 11 C / 12 C / 13 A / 14 B / 15 C / 16 B / 17 C / 18 A /
19 C / 20 C / 21 B / 22 C / 23 B / 24 D / 25 B / 26 A / 27 D / 28 B / 29 C / 30 D / 31 B / 32 C

Mini Exam:
1 C / 2 B / 3 A / 4 C / 5 C / 6 B / 7 A / 8 D / 9 C / 10 B / 11 D / 12 D

Top Score
GET READY TO STUDY
Chapter 6 Number Agreement

Number Agreement
Plural + V. to be + Plural
Singular + V. to be + Singular
My friend and his wife from England are a member in House of Common. X
My friend and his wife from England are e ers in se n √
NB:
The articles "a" and "an" are used only for singular nouns.
So, "are a…." X / "were a…." X / "are an…." X / "were an…." X
Exercises:
- Fairness, freedom and equality are an important requirement for any developed country.
………………………………………………………………………………………………….
- When they were children, Ali and Nader dreamed to be a doctor.
…………………………………………………………………………………………………
- I think that traveling with friends will be good chances.
…………………………………………………………………………………………………
SAT Question:
Most fashion designers agree that diligence, creativity, and a keen understanding of popular
tastes are a requirement for sustained success in their profession.
(A) No change
(B) is a requirement
(C) are requirements
(D) being requirements
Plural + as + plural
Singular + as + Singular
As + plural………., plural
As + singular ………., singular
Exercises:
- As a volunteer, Lily and Cecilia contributed in a charity.
…………………………………………………………………………………………………
- Mark and Sam are working as an engineer.
…………………………………………………………………………………………………
- We treat each manager in the firm as normal employees.
…………………………………………………………………………………………………

Top Score
GET READY TO STUDY
Chapter 6 Number Agreement

SAT Question:
- As a witness to the signing of the new bill, - In the aggressive society created by William
the two delegates were required to take an Golding in Lord of the Flies, both Ralph and
oath of loyalty. Jack emerge early on as the leader of the lost
(A) No change boys.
(B) As witnesses to the signing (A) No change
(C) Being a witness to the signing (B) as the leaders
(D) To be the witness for the signing (C) to be a leader
(D) being a leader

Plural + to become/becoming + plural


Singular + to become/becoming + Singular

- My brother and I promised our parents that we will become a doctor or an engineer.
…………………………………………………………………………………………………
- The determination of Chad and Niger of becoming a developed country is achievable.
…………………………………………………………………………………………………
- Susan and Peter were inspired to become a professional writer after hearing a famous
journalist speak about the challenges of investigative reporting.
(A) No change
(B) one of the professional writers
(C) as professional writer
(D) professional writers

Their / our + Plural

The students brought out their book.


…………………………………………………………………………………………………
We have to be patient to achieve our dream.
…………………………………………………………………………………………………
- The tour guide warned us to lower our head when entering the old castle, since the old
doorway was very low.
(A) No changes
(B) to lower us heads
(C) in lowering our heads
(D) to lower our heads

Top Score
GET READY TO STUDY
Chapter 6 Number Agreement

Practices
1. The people sitting behind me in the movie 6. Amusement parks, particularly those
theater were talking throughout the film featuring water rides, are a fun place for
and would not keep their voice down even families to visit during the hot summer
after being asked to do so.
months.
(A) No change
(B) his or her voice (A) No change
(C) their voices (B) is a fun place
(D) the voice of them (C) are fun places
2. As quiet, stuffy place where people just (D) and are fun places
borrowed books, libraries have been 7. A significant factor to include in the budget
changing their images dramatically over the when buying a car are the costs of fuel
last few years.
maintenance and the insurance payments.
(A) No change
(B) places where people (A) No change
(C) place when people (B) A significant factors
(D) places when people (C) Significant factors
3. Throughout the season the coach (D) As a significant factor
would often remind us that the best 8. A majority of the students who attended the
teammates are not those who strive to set job fair expressed interest in becoming
individual records but those who fit in as a a doctor or lawyer.
member of a cohesive unit. (A) No change
(A) No change
(B) doctors and lawyers
(B) as members of
(C) as a members of (C) a doctor and lawyers
(D) being a member of (D) doctors and also lawyers
4. The Stegosaurus, plant-eating dinosaurs 9. Many of the senator's speeches, which were
with protective bony plates and tail spikes, enthusiastically received by the large
was once common in what is now crowds that came to see him, addressed an
Colorado. important topic, including health care and
(A) No change global warming.
(B) plant-eating dinosaur (A) No change
(C) a dinosaurs which eating plant (B) addresses an important topic
(D) having been a plant - eating dinosaurs (C) addressing an important topic
(D) addressed important topics
5. People sharing housing with others often 10. No matter where they came from or what
find that cooking and cleaning are a their previous lifestyle is, the refugees were
difficult task to coordinate, especially when grateful for having been granted political
housemates have very different schedules. asylum in the United States.
(A) No change (A) No change
(B) which are a difficult task (B) their previous lifestyle are
(C) is a difficult task (C) their previous lifestyles are
(D) are difficult tasks (D) their previous lifestyles were

Top Score
GET READY TO STUDY
Chapter 6 Number Agreement

11. Growing up in a family where music was 15. Carla and Gideon are taking a course that
a daily part of life, Steve and Rick shared is supposed to help them both become a
a determination to become singing duos safer driver.
known nationwide. (A) NO CHANGE
(A) No change (B) to be a safer driver
(B) to become a singing duo (C) become safer drivers
(C) of becoming singing duos (D) becoming a safer driver
(D) that they would become singing duos
16. The transformation of an animal into a
12. In the early Twentieth Century, telephone person, or the reverse, are parts of many a
companies supported the cause of folktale.
financial independence for women by (A) NO CHANGE
providing many with respectable jobs as a (B) reverse, are a part
switchboard operator. (C) reverse, is part
(A) NO CHANGE (D) reverse, they are parts
(B) as a switchboard operators
17. The museum experienced a greatly
(C) as switchboard operators
increased number of visitors in one year,
(D) to be a switchboard operator
and analysts cited the museum's
13. As a proponent of school budget cuts, the expansion, free admission, and new
district's superintendent and budged family-oriented education center to be
director have had to face probing, often reasons for they appeal.
hostile, questions from teachers and (A) NO CHANGE
parents. (B) to be a reason for their
(A) NO CHANGE (C) as reasons for its
(B) As a proponents (D) as reason for its
(C) While proponent
18. All of these cats have a kink in their tail.
(D) As proponents
(A) NO CHANGE
14. Much of the success of both Jack Benny (B) All of these cats have a kink in its tail
and Bob Hope as a master of comic (C) Each of these cats has a kink in its tail
monologues can be attributed to the (D) Each of these cats has a kink in their
creative work of their talented joke tail
writers.
19. Little Rock and Memphis have a museum
(A) NO CHANGE
commemorating the important roles these
(B) as a master of comic
cities played in changing the social
(C) as masters of comic
landscape of the United States.
(D) to be a master of comic
(A) NO CHANGE
(B) has a museum commemorating
(C) commemorating with a museum
(D) have museums commemorating

Top Score
GET READY TO STUDY
Chapter 6 Number Agreement

Homework
1. Knowing the roots of words that are hard to 6. In the book, dancers from various
spell helps students to become a better backgrounds talk about their very different
speller. paths to success as a performer in ballet and
modern dance productions, Broadway
(A) No change
shows, and music videos.
(B) is helpful to students who want to be a (A) No change
better speller (B) for succeeding as a performer
(C) helps students to become better spellers (C) to success as performers
(D) helps a student be better spellers (D) to success as to perform
2. Cattle ranchers often view the prairie dog 7. A porcupine's loosely attached quills
as pests, eating pasture grasses and digs easily break away from the porcupine and
holes that can injure cattle. become embedded in the bodies of any
(A) No change animal that is foolish enough to attack
(B) pests, they eat such a well - defended creature.
(C) pests that eat (A) No change
(D) a pest because it eats (B) and becoming embedded in the bodies
3. A big vegetable salad is generally more (C) and becomes embedded in the body
nutritious than a low-fat pasta dish, but (D) and become embedded in the body
either meal would be good choices for the 8. Howard Gardner, an observer of Chinese
health - conscious eater. elementary education, has questioned the
(A) No change view that requiring young children to copy
(B) are good choices models prevents them from becoming a
(C) is a good choices creative artist later in life.
(D) would be good choice (A) No change
4. As a child growing up on a farm, my sister (B) prevent them from becoming a
and I visited many county fairs, where my creative artist
parents would exhibit their prize livestock. (C) prevents them from becoming creative
(A) No change artists
(B) A child who grew up on a farm (D) preventing them from becoming a
(C) Farm children and growing up creative artists
(D) Growing up on a farm 9. In this production of Hamlet, the actors
5. It appears that either Jane or Marek will be periodically turn their back to the real
elected to be presidents of the student appear to address an imaginary audience
union. on the other side of the stage.
(A) No change (A) No change
(B) as presidents (B) their backs to the real appear
(C) presidents (C) his or her back to the real appear
(D) president (D) his or her backs to the real appear

Top Score
GET READY TO STUDY
Chapter 6 Number Agreement

10. The second-year students at this college 14. All morning long my friend and
have evolved into insightful, critical I waited patiently in the courtroom to hear
thinkers from an inexperienced and whether we would be selected as a juror in
uncertain teenager just a year before. the upcoming trial.
(A) No change (A) No change
(B) from the inexperienced and uncertain (B) will be selected as juror
teenagers they were just a year ago (C) will be selected as jurors
(C) even though just a year before they (D) would be selected as jurors
were an inexperienced and uncertain 15. Preparing to be a foreign service officer
teenager after graduation, both Mary and Keisha
(D) in just a year from an inexperienced have decided to major in international
and uncertain teenager affairs.
11. Although Debussy and Ravel are (A) NO CHANGE
generally considered as impressionist (B) Because they wanted to prepare to be
composer, their compositional styles are a foreign service officer
quite distinct from one another. (C) When preparing to be a foreign
(A) No change service officer
(B) to be an impressionist composer, their (D) Preparing to be a foreign service
compositional style is officers
(C) impressionist composers, their 16. Horse psychology, a science that
compositional styles are investigates the reasons for the behavior
(D) impressionist composers, whose of horses, helps trainers both motivate
compositional styles are their charge and prevent problems.
12. Many adults today consider themselves (A) NO CHANGE
a member of a special group of caretakers (B) their charges
who care for both their teenage children (C) his or her charges
and their aging parents. (D) a charges
(A) No change 17. Some people are convinced that dowsing,
(B) the member of a method of finding underground water
(C) members of with a Y- shaped stick, is effective, but
(D) as a member of others condemn the procedures as mere
13. Sui Sin Par and Edith Eaton are both superstition.
a name used by an early Twentieth- (A) NO CHANGE
Century writer whose psychological (B) mere superstitions
realism continues to bring her diameters to (C) a mere superstitions
life for today's readers. (D) some superstitions
(A) No change
(B) names
(C) the name
(D) two names

Top Score
GET READY TO STUDY
Chapter 6 Number Agreement

Mini Exam
Scientists in Poland Work to Save Honeybees . 1 .. .
Honeybees are in danger worldwide. (A) NO CHANGE
Their . 1 . homes - known as . 2 . (B) home's
(C) homes'
"colony's" - are suffering from what scientists call (D) home
Colony Collapse Disorder. Scientists have linked . 2 .. .
the disorder to infections from harmful (A) NO CHANGE
microorganisms and pesticides - products meant (B) “colonies”
(C) “colonies'”
to kill other insects. Researchers also have (D) “colony”
blamed a sharp . 3 . drops in the honeybee
. 3 .. .
population to loss of . 4 . the bees’ natural habitat.
(A) NO CHANGE
Polish researchers are working on ways to (B) drops'
(C) drop
save honeybees. One of the . 5 . diseases' that
(D) drop's
affect honeybee colonies in Poland is nosemosis,
. 4 .. .
also called nosema. It is a common fungi-based
(A) NO CHANGE
. 6 . diseases that affects adult bees. (B) the bee's
(C) the bees
Aneta Ptaszynska is a . 7 . researchers
(D) the bee
with Maria Curie-Sklodowska University.
. 5 .. .
"Infected worker bees live for a very short
(A) NO CHANGE
time in the summer, about eight to 12 days, while (B) diseases
(C) disease
they normally live 36 days. So the productivity of
(D) disease's
the whole bee family decreases and bees also
. 6 .. .
have problems with passing (the) winter... ."
(A) NO CHANGE
Scientists believe some pesticides weaken (B) disease's
the bees’ natural resistance against disease. This (C) disease
(D) diseases'
makes it difficult for them to fight parasitic
. 7 .. .
organisms. After eating food or drinking water
(A) NO CHANGE
containing pesticides, the bees begin to act (B) researcher
strangely and soon die. Scientists believe nosema (C) researcher's
(D) researchers'
is one of the main . 8 . reasons' the number of
. 8 .. .
bees has dropped by 50 percent in Poland over
(A) NO CHANGE
the past 15 years. (B) reason
(C) reason's
(D) reasons

Top Score
GET READY TO STUDY
Chapter 6 Number Agreement
. 9 .. .
Polish scientists say they have found a
natural . 9 . substances in flowers that is more (A) NO CHANGE
(B) substances'
than 90 percent effective in saving bees. They (C) substance
have created .10. drug's from it. The drug is now (D) substance's

being tested. Scientists are warning .10.. .


that insecticides designed to kill mosquitoes (A) NO CHANGE
carrying the Zika virus can also harm bees. They (B) a drugs
(C) drugs
say such products should be used in affected
(D) a drug
areas only when it is dark - after bees are asleep.
.11.. .
And they say only chemicals that kill the
(A) NO CHANGE
mosquito larvae should be used. (B) beekeepers
Michael Steinkampf is a .11. beekeeper's. (C) beekeeper
(D) beekeepers'
"This particular .12. strains are
very toxic to mosquito larva but when it's put in .12.. .

the water it doesn't hurt the bees at all." (A) NO CHANGE


(B) strain is
(C) a strains are
(D) strain's is

Top Score
GET READY TO STUDY
Chapter 6 Number Agreement

Answers
Explanation
Page 1: are important requirements / are good players / dreamed to be doctors / will be good chance
/ C / As volunteers / as engineers / as normal employee / of becoming developed countries / D /

Page 2: B / B / become doctors or engineers / their books / our dreams / D

Practices
1 C / 2 B / 3 B / 4 B / 5 D / 6 C / 7 C / 8 B / 9 D / 10 D / 11 B / 12 C / 13 D / 14 C / 15 C / 16 C / 17
C / 18 C / 19 D

Homework
1 C / 2 D / 3 D / 4 D / 5 D / 6 C / 7 D / 8 C / 9 B /10 B / 11 C / 12 C / 13 B / 14 D / 15 D / 16 B

17 B

Mini Exam
1 A / 2 B / 3 C / 4 A / 5 B / 6 C / 7 B / 8 D / 9 C / 10 D / 11 C / 12 B

Top Score
GET READY TO STUDY
Chapter 7 Fragment

Fragment
Fragment is a group of words lacks one of its elements: Subject, verb, or object.

Subject + gerund………. X

Biologists working to rescue the giant condor from extinction to track.

e t ger nd er √

The people sitting behind me in the movie theater were talking throughout the film.
SAT Questions
- Most of the world's sharks living in temperate and tropical regions though the Greenland shark lives
in the cold Arctic waters and the huge basking shark is at home in the seas around Antarctica.
(A) No change
(B) lives
(C) live
(D) being living
- It has long been known that the sea otters living along the West Coast of North America help keep
kelp forests in their habitat healthy and vital.
(A) NO CHANGE
(B) living along the West Coast of North America, they help
(C) that live along the West Coast of North America and help to
(D) that live along the West Coast of North America, where they help

Subject (,) Appositive (,) Verb

- That Naomi, the book's main character and possibly its hero, is a mere child during the Revolution
makes the story even more appealing to young readers.
Subject (,) Appositive (,) /. /which/who/and/but/pronoun + Verb ×

Example

- Ethel Payne, the tough investigative reporter which is well known for her coverage of the Civil
Rights movement, and received critical acclaim for a diary she kept while living in Japan after the
Second World War. X
SAT Question
- Joseph Pulitzer, a Hungarian born immigrant to the United States and an innovator in the field of
popular Journalism. He established the Pulitzer Prize in journalism and literature.
(A) No change
(B) , establishing
(C) , established
(D) , establish

Top Score
GET READY TO STUDY
Chapter 7 Fragment

Subject (,) Appositive (,) Gerund ×

Example

- American Sign Language, which comprises hand signs, facial expressions, and finger spelling,
including many regional differences and slang.
SAT Question
- James Houston, whose glass sculptures often depict Arctic scenes such as fishing or kayaking,
drawing his inspiration from Inuit stone carvings he saw while living on Baffin Island.
(A) No change
(B) drew
(C) draw
(D) being drawing

Subject + who/which/that + Verb + Verb √

- The man who has no control on his temper is under control at all times is likely to think clearly and
to accomplish more in his business and social relations.
- At the 1984 Olympic Games, John Moffet and Paolo Morales who were swimmers on the United
States team set world record. √

Subject (,) who/which/that + verb ……….. . ×

Examples

- The man who followed me along the day yesterday. X


- The Indus scripts which are very difficult to decode. X
SAT Question
- Besides publishing ten poetry collections for - Medical insurance coverage that requires high
adults, Gary Soto, who is writing numerous monthly premiums and that is beyond the
collections of shorts stories and novels for financial means of many people
children (A) No change
(A) No change (B) that requires high monthly premiums and it
(B) Soto, the writer of is
(C) Soto writing (C) requiring high monthly premiums are
(D) Soto has written (D) that requires high monthly premiums is

Top Score
GET READY TO STUDY
Chapter 7 Fragment

Linker + Subject + Verb (,) Subject + verb √


Linker + Subject + gerund (,) Subject + gerund ×

Example

- After Ali had studied English for about 10 years, he finding that French is so easier.
SAT Question
- Because they build nests on telephone poles as readily as they do in trees, Ospreys adapting well to
various environment.
(A) No change
(B) adapt
(C) is adapting
(D) adapts

Top Score
GET READY TO STUDY
Chapter 7 Fragment

Practices
1. America's first roller coaster ride, which opened 6. Ken Alice N'doye, who earned a reputation as
in 1884 at Coney Island, Brooklyn, and capable caterer and then opened her own bakery, but first
of a top speed of only six miles per hour. learned to cook by preparing food for her own
(A) No change family.
(B) are capable (A) No change
(C) is capable (B) who first learned
(D) being capable (C) first learned
2. After studying plants that thrive in warm (D) and first learned
climates and those that do not, scientists
concluding that a plant's heat tolerance is 7. The meteor shower being best seen from the
determined by a particular gene. naval observatory on Saturday just before dawn.
(A) No change (A) No change
(B) concluded (B) was
(C) has concluded (C) were
(D) who concluded (D) having been
3. Used primarily in the United States and Canada, 8. The Navajo code talkers, recruited by the United
American Sign Language, which comprises hand States Marine Corps during the second World
signs, facial expressions, and finger spelling, War, they designed a secret code that was used
including many regional differences and slang. to transmit military messages.
(A) No change (A) No change
(B) include (B) designs
(C) which include (C) designed
(D) includes (D) who designed
4. Visitors are quick to notice that the main
building of the critically acclaimed Denver Art 9. An increased concentration of carbon dioxide
Museum consisting of geometric, titanium - clad and certain other gases in the Earth's atmosphere,
projections that resemble Rocky Mountain contributing to global warming.
peaks. (A) No change
(A) No change (B) atmosphere, they contribute to
(B) consists (C) atmosphere, which contributes to
(C) which consists (D) atmosphere contributes to
(D) being consisting 10. Legendary Nineteenth Century endurance rider
5. Many years after Ernest Just received the Frank T. Hopkins, who rode an American
NAACP's Spingam medal for his mustang to victory in a 3000 mile race across
groundbreaking discoveries in cell biology, the the Arabian desert, and the hero of Disney
nation issuing a postage stamp to honor him. movie Hidalgo.
(A) No change (A) No change
(B) have issued (B) and who has become
(C) which issued (C) also has become
(D) issued (D) is

Top Score
GET READY TO STUDY
Chapter 7 Fragment

11. Richard Rodriguez pointed out that Mission 15. The poet Claude McKay was a native of
San Luis Rey, a Spanish church with an Arabic Jamaica who spent most of his life in the
dome getting its name to honor a French King. United States but writing some of his poems in
(A) No change the Jamaican dialect.
(B) Mission San Luis Rey, a Spanish church (A) No change
with an Arabic dome, was named to honor (B) Although a native of Jamaica, the poet
a French king Claude McKay spent most of his life in the
(C) Mission San Luis Rey's being, a Spanish United States, he wrote
church with an Arabic dome got named to (C) Although the poet Claude McKay spent
honor a French king most of his life in the United States, he
(D) the name mission San Luis Rey having was a native of Jamaica and wrote
been a Spanish church with an Arabic (D) Because he was a native of Jamaica who
dome honoring a French king. spent most of his life in the United States,
12. Since scientific advances are central to the poet Claude McKay writing
progress, basic research deserving continuing 16. Abidjan, a port city, no longer the capital of
support. Ivory coast, but still the country's economic
(A) No change center
(B) basic research being what deserves (A) No change
continuing support (B) A port city, Abidjan, is no longer the
(C) basic research deserves continuing support capital of Ivory coast, it is still the
(D) continuing support is deserved by basic country's economic center
research (C) Although it is no longer the capital of the
Ivory coast, the port city of Abidjan is still
13. All the demand on soprano Kathleen Battle for the country's economic center
operatic performances, solo concerts, and (D) No longer the capital of Ivory coast, but
special guest appearances, tempting her to sing the port city of Abidjan is still the
too often and straining her voice. country's economic center
(A) No change 17. Jean Piaget, a Swiss psychologist, and the first
(B) appearances not only tempt her to sing too scientist that made systematic studies of how
often plus they strain children learn
(C) appearances, tempting her into singing too (A) No change
often and she therefore strains (B) as a Swiss psychologist, it was Jean Piaget
(D) appearances tempt her to sing too often who was the first scientist making
and strain (C) Swiss psychologist Jean Piaget, the first
14. Althea Gibson, the first African American scientist in having made
tennis player to be recognized as a world (D) The Swiss psychologist Jean Piaget was
champion, beginning to play amateur tennis in the first scientist to make
the 1940s
(A) No change
(B) began playing
(C) and she began playing
(D) she had began playing

Top Score
GET READY TO STUDY
Chapter 7 Fragment

18. Digital technology, like every marketer knows, 23. Because of geothermal changes beneath Old
it is synonymous with speed, precision, and the Faithful, the frequency and duration of the
future. geyser's eruptions are now more predictable.
(A) No change
(A) No change
(B) eruptions, they are now more predictable
(B) technology, similar to what every marketer (C) eruptions are predicted more easier now
knows as (D) eruptions, they can now be predicted more
(C) technology, as every marketer knows, is easily
(D) technology is what every marketer knows 24. The Olmec people, widely regarded as the
as creators of the first civilization in
19. Clara Barton, an American nurse, whose Mesoamerica, the area encompassing much of
Mexico and Central America
influence as a reformer in the field of health
(A) No change
care almost equals that of Florence Nightingale (B) people, which are widely
(A) No change (C) people, they are widely
(B) Clara Barton, who was an American nurse (D) people are widely
and whose influence as a reformer 25. Numismatics, or coin collecting, and becoming
(C) Clara Barton was an American nurse popular in the United States around 1857, when
whose influence as a reformer the replacement if the large cent by the new
(D) An American, Clara Barton who was a flying - eagle cent led enthusiasts to start
nursing reformer and whose influence collecting the earlier coin
(A) No change
20. Hawaii's Haleakala, being more than 10,000 (B) becoming
feet high, and the world's largest dormant (C) will become
volcano (D) became
(A) No change 26. Decorative arts, are forms that have a mainly
(B) Haleakala which is more than 10,000 feet practical or ornamental purpose, and include
high, being ceramics, basketry, furniture making and
glassblowing
(C) Haleakala, more than 10,000 feet high, is
(A) No change
(D) Haleakala, more than 10,000 feet high; it is (B) including
21. The port city of Shanghai, which along with (C) includes
Hong Kong is a leading Chinese banking and (D) include
manufacturing center. 27. Because the United States in the early 1800s
(A) No change was largely rural, many invention by women of
(B) Shanghai is, along with Hong Kong, that period relating to agriculture.
(A) No change
(C) Shanghai, along with Hong Kong being (B) related to
(D) Shanghai and Hong Kong are (C) is related for
22. The first African American woman to win the (D) relates for
Pulitzer Prize for poetry, Gwendolyn Brooks 28. Below the bend, the river, flowing more
with her special interest in encouraging young swiftly, as it cuts through sand hills covered
with pine trees.
poets
(A) No change
(A) No change
(B) river, flowing more swiftly, cutting
(B) had a special interest in encouraging (C) river flows more swiftly as it cuts
(C) who had a special interest in encouraging (D) river flows more swiftly, it cuts
(D) she had a special interest to encourage

Top Score
GET READY TO STUDY
Chapter 7 Fragment

Homework
1- While studying mutations in corn, biologist 5- In 1911 the Great Blue Norther, a massive
Barbara McClintock discovered that certain storm in the mid western United States, it
genes that were able to move their positions on caused temperatures in some cities to drop
chromosomes and control the activity of other more than 60 degrees in a single day
genes. (A) No change
(A) No change (B) caused
(B) that had been (C) causing
(C) were able (D) having caused
(D) able 6- Although she spoke only Chinese at the time
2- George Orwell's term doublespeak referring to and had to memorize her English lines
the intentional use of language to confuse or to phonetically, Gong Li gave a memorable
mislead, as when one says revenue performance in the film Memoirs of a Geisha.
enhancement instead of tax increase. (A) No change
(A) No change (B) Gong Li, whose performance in the film
(B) which refers to intentionally using Memoirs of a Geisha was memorably
language given
(C) refers to the intentional use of language (C) But the performance in the film Memoirs
(D) is when it refers to language used of a Geisha given by Gong Li was
internationally memorable
3- One might expect global warming to cause (D) The film Memoirs of a Geisha had a
plants to bloom earlier in the spring, but a memorable performance, it was given by
recent study was shown that plants in northern Gong Li
latitudes blooming later as the plant warms. 7- The National Film Registry established by
(A) No change Congress in 1988 in an effort to protect the
(B) that plants in northern latitudes they are artistic integrity of motion pictures.
blooming later. (A) No change
(C) that plants in northern latitudes are (B) having been established
blooming later (C) was established
(D) plants in northern latitudes that bloomed (D) whose establishment
more later
8- The foundation planning to bring together
4- Some of the Smithsonian Institution's most
scholars and industrial experts to evaluate new
prized items, from Duke Ellington's musical
approaches to manufacturing and trade
transcripts to First Ladies gowns, coming from
efficiency.
unsolicited donations.
(A) No change
(A) No change
(B) The foundation is planning
(B) they come from
(C) The foundation, which plans
(C) came from
(D) As planned, the foundation
(D) which came from

Top Score
GET READY TO STUDY
Chapter 7 Fragment

9- Iceland, known for its colorful tales, which 13- The Four Comers Monument in the
poetically depict the dangerous, complex lives
southwestern region of the United States,
of the country's earliest settlers.
(A) No change marking the place where the boundaries of
(B) Iceland is known for its colorful tales, they Arizona, Colorado, New Mexico and Utah
(C) Iceland is known for its colorful tales, intersect.
which (A) No change
(D) Iceland, its colorful tales are known to (B) States marks the place
10- The new political science curriculum at some
(C) States, the place
high schools in India, encouraging teachers to
(D) States, it marks
use cartoons and news paper clippings to
provoke classroom debate on contemporary 14- Sandra Cisneros, she was born in Chicago but
issues. lived intermittently in Mexico as a child and so
(A) No change grew up between two cultures.
(B) India, which encourages teachers to use (A) No change
cartoons and news paper clippings, and (B) born in Chicago, she
provokes (C) though born in Chicago,
(C) India is encouraging teachers to use (D) though born in Chicago, she
cartoons and news paper clippings and 15- Since civilization began, some cities have been
then provoking built according to a deliberate plan, whereas
(D) India, so encourages teachers to use others have developed naturally.
cartoons and news paper clippings as (A) No change
provoking (B) some cities being built
11- Critics often equate the popular appeal of work (C) there has been cities built
of an with inferiority, forgetting that (D) they have built some cities
Shakespeare wrote his extraordinary plays
16- In a nearby resort community fewer accidents
primarily to please his audience. have been reported that the number of speeding
(A) No change
tickets to be issued have decreased since speed
(B) Critics often equate the popular appeal of
bumps were installed.
work of an with inferiority, they forget (A) No change
(C) Often equaling the popular appeal of a
(B) issued have been on the decreased
work of art with inferiority are critics, they
(C) issued has decreased
forget
(D) issued decreasing
(D) The popular appeal of a work of art often
equated by critics as being the same as 17- Josephine Baker (1906 -1975), one of the most
inferiority, they forget versatile performers of the Twentieth Century,
12- To encourage young people to read, a number and who acquired fame as a dancer, singer,
of authors, who sell their books with Broadway actress, and movie star.
companion video games that require player to (A) No change
use information from the books to advance in (B) she has acquired
the games? (C) and she has acquired
(A) No change (D) acquired
(B) authors, they sell
(C) authors selling
(D) authors are selling

Top Score
GET READY TO STUDY
Chapter 7 Fragment

18- Medieval Icelandic sagas, which range from 24- Mary Wollstonecraft's Vindications of the
epic tales about legendary figures to factual Rights of Woman, published in 1792, being one
accounts of Icelandic history. of the first texts that addresses women's rights.
(A) No change
(A) No change
(B) Sagas, they range
(C) Sagas ranges (B) published in 1792, was one of the first
(D) Sagas range texts to address
19- It being agreed by world leaders that there (C) published in 1792 as one of the first texts
should be a major organized effort to address that address
poverty, illiteracy, and disease; they then set (D) it was published in 1792, one of the first
goals by which progress could be measured. texts to address
(A) No change
25- Sir Ronald Ross, winner of the 1902 Nobel
(B) World leaders agreed
(C) World leaders who should agree Prize for physiology or Medicine, and who
(D) World leaders, in agreement identified the Anopheles mosquito as the
20- Before he was elected governor of New transmitter of human malaria.
Mexico in 1918, Octaviano Larrazolo had (A) No change
helped to ensure that the constitution of this (B) he has identified
new state guaranteed equal rights for Hispanic (C) and who is identifying
Americans. (D) identified
(A) No change 26- Home fuel cells, already available in Japan,
(B) Larrazolo has helped
which is a compact, pollution - free device that
(C) Larrazolo, helped
(D) Larrazolo, helping produce electricity at a fraction of the regular
21- Some experts think that the rise of multi- cost.
language Internet forums has made learning a (A) No change
foreign language easier than ever before. (B) a compact, pollution - free device
(A) No change (C) is a compact, pollution - free device
(B) Forums have made (D) are compact, pollution - free devices
(C) Forums, which have made
(D) Forums, having made 27- The Medieval Warm Period was a lime of
22- One of the most influential philosophers of the unusual mild temperatures in the North
Twentieth Century, W.V.O. Quine, who was Atlantic region, it lasted from approximately
known for both his analytic precision and his 800 to 1300 C.E.
great wit (A) No change
(A) No change (B) Period had been a lime of unusual mild
(B) Quine, he was known both for
temperatures in the North Atlantic region,
(C) Quine was known for both
(D) Quine was both known for it lasted
(C) Period, a time of unusual mild
23- Maria Merian, who undertook an expedition to
study insects in Suriname in 1699, when she temperatures in the North Atlantic region
was fifty-two years old. and lasting
(A) No change (D) Period, a time of unusual mild
(B) Merian undertook an expedition to study temperatures in the North Atlantic region,
(C) Merian undertaking an expedition to study lasted
(D) Merian has undertaken an expedition to
study

Top Score
GET READY TO STUDY
Chapter 7 Fragment

28- Formed by volcanic eruption over the last five 29- Crude oil, often called petroleum, which means
million years, the Hawaiian Islands containing "rock oil".
an incredibly wide variety of species many (A) NO CHANGE
found nowhere else on Earth. (B) oil, which is often called
(A) No change (C) oil is often called
(B) contain (D) oil, that is,
(C) contains
(D) would contain

Top Score
GET READY TO STUDY
Chapter 7 Fragment

Mini Exam
Rio Olympic Games Open This Weekend . 1 .. .
Olympic athletes, high - level officials, (A) NO CHANGE
and . 1 . other people, have been arriving in Rio (B) other people have been
de Janeiro for the opening of the 2016 summer (C) other people who have been
games. The opening ceremony is Friday night, (D) other people having been
local time, but some events already . 2 . starting.
The first medals . 3 . which will be awarded on . 2 .. .
Saturday. (A) NO CHANGE
The Rio Olympics . 4 . officially beginning (B) starts
Friday, when the Olympic torch is carried into (C) started
Maracana Stadium to light the Olympic cauldron. (D) to start
The cauldron will stay lit for over two weeks until
the closing ceremony on August 21. During that . 3 .. .
period, 301 gold medals will be awarded to the
(A) NO CHANGE
winners. (B) who will be awarded
At the last summer Olympics, the 2012 (C) being awarded
London Games, the United States won 46 gold (D) will be awarded
medals and 104 medals in all. China was second,
with 38 gold medals and a total of 88. Britain . 4 .. .
finished in third place, with Russia in fourth.
(A) NO CHANGE
One major question this year is: How (B) which officially begin
many medals will Russia win at the Rio Games? (C) which officially begins
Just before the Olympics started, the (D) officially begin
International Olympic Committee (IOC)
. 5 . considered banning every member of . 5 .. .
Russia’s team because of a doping scandal.
(A) NO CHANGE
Investigators for the World Anti - Doping (B) considering
Agency (WADA) . 6 . but found evidence of (C) which considered
Russian athletes using unapproved substances at (D) to consider
the 2014 Winter Olympics in Sochi. And the
. 7 . agency which accused the Russian . 6 .. .
government of being involved and covering up
(A) NO CHANGE
the activity.
(B) and
But in the end, the IOC decided not to ban (C) so
all 387 Russian qualifiers for the Rio Games. (D) Delete the underlined portion.
Many Russian athletes will have a chance to
compete. But most of the country′s track and field . 7 .. .
athletes . 8 . banned by the International
(A) NO CHANGE
Association of Athletics Federation.
(B) agency who
(C) agency
(D) agency was

. 8 .. .

(A) NO CHANGE
(B) were banned
(C) was banned
(D) who banned

Top Score
GET READY TO STUDY
Chapter 7 Fragment
At a meeting in Rio this week, WADA . 9 .. .
and the IOC . 9 . and criticized each other.
(A) NO CHANGE
WADA officials said the IOC .10. should have (B) but criticized
banned all the Russian Olympians. The IOC said (C) criticized
WADA should have acted faster on (D) criticizing
the allegations of Russian cheating. .10.. .
More than 10,000 men and women from
over 200 nations .11. and will compete at Rio de (A) NO CHANGE
(B) which should have
Janeiro. The first medals, in men’s cycling and (C) which have
women’s shooting, .12. which will be awarded on (D) having
Saturday. .13. Chris Froome, who won the Tour
.11.. .
de France last month, will be representing Britain.
American swimmer Michael Phelps has (A) NO CHANGE
won the most Olympic medals ever. He has 22 (B) competing
(C) who will compete
medals, including 18 gold. He will carry the flag (D) will compete
of the United States at the opening ceremony on
Friday. Aside from the question of which country .12.. .
will win the most medals, some people are (A) NO CHANGE
worried about the health of the competitors. (B) will be awarded
Athletes are being careful .14. about the (C) was awarded
(D) awarding
time which they spend out of doors because of
concerns about the Zika virus. Zika spreads when .13.. .
mosquitos bite human skin. The virus is linked to
(A) NO CHANGE
microcephaly, a birth defect in which a baby’s (B) Chris Froome won the Tour de France last
head is smaller than the heads of other children of month,
the same age and sex. (C) Chris Froome which won the Tour de
France last month,
Olympic organizers say they are not
(D) Chris Froome who won the Tour de
overly concerned about Zika since the games take France last month and
place during Rio de Janeiro′s winter. Athletes
competing in water events, like open-water .14.. .
swimming, rowing and sailing are concerned (A) NO CHANGE
about polluted water. The Associated Press (B) about the time they spend
reports that those athletes run a high risk of (C) about the time spending
getting sick. (D) about the time and spend
Water at some of the competition areas is
polluted by untreated waste water. It contains
bacteria and viruses carried by human waste.
While some athletes will wear gloves or other
protection, it will be hard for the open - water
swimmers to keep water from getting into their
mouths. Yet Olympic organizers say athletes will
not get sick.

Top Score
GET READY TO STUDY
Chapter 7 Fragment

Answers
Explanation
Page 1: C / A / C
Page 2: B / D / D
Page 3: B

Practice:
1 C / 2 B / 3 D / 4 B / 5 D / 6 C / 7 B / 8 C / 9 D / 10 D / 11 B / 12 C /13 D / 14 B / 15 C / 16 C / 17 D / 18 C
/ 19 C / 20 C / 21 B / 22 B / 23 A / 24 D / 25 D / 26 D / 27 B / 28 C

Homework
1 C / 2 C / 3 C / 4 C / 5 B / 6 A / 7 C / 8 B / 9 C / 10 C / 11 A / 12 D / 13 B/ 14 C / 15 A / 16 C / 17 D / 18 D
/ 19 B / 20 A / 21 A / 22 C / 23 B / 24 B / 25 D / 26 D / 27 D / 28 C / 29 C

Mini Exam
1 B / 2 C / 3 D / 4 D / 5 A / 6 D / 7 C / 8 B / 9 C / 10 A / 11 D / 12 B / 13 A / 14 B

Top Score
GET READY TO STUDY
Chapter 8 Verb Tense

Verb Tense
One tense for all verbs in one sentence
EX: As the mayor was evaluating the proposed tax, he was less interested in the revenue they
would generate than in whether they would disproportionately affect certain income groups.

SAT Question:
- Few people could have guessed, as they watch Anna win the tennis match with apparent case,
how pessimistic she had been about her chances of beating her opponent.
(A) No change
(B) as they have watched
(C) as they watched
(D) as he or she watched
Subject + P.P is INCORRECT:
EX: John told me that Mark broken my car. (Decide the error)
SAT Question:
- One subject of Felipe Alfau's second novel, published more than 40 years after it written, is the
illusory nature of the passage of time.
(A) No change
(B) after it was written
(C) after it is written
(D) after it being written
Present Perfect: Subject + Have / Has + P.P
Time markers: (since / for / over the past time/ now that)
EX: I haven't met my cousin since his birthday.
Over the past year, our company has published many projects in the area.
SAT Question:
- Since 1980's, most people entering politics feel that loss of privacy was a fair price to pay for the
chance to participate in policy making.
(A) No change
(B) entering politics have felt that
(C) entering politics has felt that
(D) entering politics having felt that

Top Score
GET READY TO STUDY
Chapter 8 Verb Tense

V. to Have + Third Conjugation NOT Second Conjugation


EX: The Olympic Games have began for more than three weeks. (Decide the error)

SAT Question:
- In 1988 a Soviet icebreaking ship helped free three gray whales that had become trapped in the
Arctic ice after they had swam into the coastal water of Alaska to feed.
(A) No change
(B) have swum
(C) had swum
(D) has swum

Past Simple: Second Conjugation of the verb

Time markers: (Yesterday / in the past / early / last…… / …..ago / past date / any old event "World
War"/ another verb in the past)
SAT Question:
1) After 1986, trading on the London Stock Exchange has been computerized to such an extent that
personal contact between buyers and sellers became unnecessary.
(A) No change
(B) have been computerized
(C) was computerized
(D) having been computerized
2) Nellie Lutcher did not achieve success quickly: she had been giving live performance for over a
decade before she will record her first hit song.
(A) No change
(B) she records
(C) she recorded
(D) she has recorded

3) In the years following the Second World War, millions of soldiers returned to civilian life, the
demand for housing will cause a massive expansion of suburb.
(A) No change
(B) caused
(C) have caused
(D) causes

Top Score
GET READY TO STUDY
Chapter 8 Verb Tense

Present simple: The first Conjugation of the verb


Time markers: (always / usually / often / sometimes / rarely / seldom / never / facts / conclusion of
fiction story)
- At the conclusion of the novel The Great Gatsby, Nick Caraway, a young Midwesterner recently
arrived to New York, moodily watches the blinking green light at the tip of Long Island.
(A) No change
(B) arrives to New York, moodily watched
(C) arrives to New York, moodily watches
(D) had arrived to New York, moodily watches
- Linker + Tense…………………., the same Tense
- Tense…………………..Linker + the same Tense
SAT Question:
1. Although the author has presented her book to the publisher as a fact-based memoir, she later
confessed that it had been entirely fabricated.
(A) No change
(B) the author had presented their book
(C) the author presented her book
(D) the author have presented her book
2. A passion fruit, which is about the size of an egg, has a brittle outer shell that became slightly
wrinkled when the fruit is ripe.
(A) No change
(B) that becomes
(C) that become
(D) that will become

If Conditionals
a) If + present simple…….., will / may / can / shall / should + infinitive
If you feel hot, you may have a soft drink.
SAT Question:
- If hordes of insects attack crops, growers will not make any profits.
(A) No change
(B) growers would not make
(C) growers will have not made
(D) growers will not to make

Top Score
GET READY TO STUDY
Chapter 8 Verb Tense

b) If + past simple…….., would / might / could / should + infinitive


If you were informed how to fix it, you would finish your work on PC.
OR Were you to be informed how to fix it, you would finish your work on PC.
SAT Question:
- If you can acquire the necessary calories by drinking gasoline instead of by eating food, you
would be able to run 26 miles on about one-twelfth of a gallon gas.
(A) No change
(B) we can acquire
(C) you will acquire
(D) you acquired

- Were they to be told the purpose of the study, the participants might find it difficult to answer
the questions spontaneously.
(A) No change
(B) If they would have been told
(C) With it being told them
(D) Telling them

c) If + past perfect (Had + p.p)………, would have + p.p


If she had studied hard, she would have gotten higher scores.
OR Had she studied hard, she would have gotten higher scores.
SAT Question:
- If the bridge did not become treacherous in the aftermath of an ice storm, the bus would not
have had to take the long detour instead of the main highway.
(A) No change
(B) had not become
(C) had not became
(D) would not become

NB:
If + modal verb x

Top Score
GET READY TO STUDY
Chapter 8 Verb Tense

Practice
1. Nora Ephron first became known in the late 1960s 6. We had initially planned to remove all of the old
as magazine journalist who has wrote feature furniture from the storage area, but we decided
articles about culture institutions and media against doing so because it would have took too
personalities. much time.
(A) No change (A) No change
(B) have wrote (B) will have took
(C) had written (C) have took
(D) has written (D) would have taken
2. In 1936 mathematician Alan Turing proved that a 7. After the prince characterized modern architecture
digital computer, which at the time exists only in as ugly, he has been severely criticized for having
theory, could be programmed to perform the been so outspoken in public.
function of any other information-processing (A) No change
device. (B) have been
(A) No change (C) was
(B) existed (D) is
(C) has existed 8. Until time zones were standardized in the United
(D) to exist States, each railway station will have to keep
3. Katherine felt that she has not had several clocks, each showing the time of day
any understanding of the highly intricate workings according to a different railroad company.
of the stock market until her uncle took her to the (A) No change
New York Stock exchange. (B) would have to
(A) No change (C) had to
(B) have not had (D) have to
(C) had not had 9. Twenty-five years after Alex Haley's Roots
(D) having not had stimulate many people to research their family
4. Those who defend sequoia trees from loggers histories; new technology has been developed to
justified doing so on the ground that such trees make the task easier.
irreplaceable. (A) No change
(A) No change (B) after Alex Haley's Roots have stimulate
(B) justify to do so (C) after Alex Haley's Roots stimulated
(C) justify doing so (D) after Alex Haley's Roots stimulates
(D) justified to do so 10. By the beginning of 1755, events are reaching a
5. Whenever Umberto sang in the shower, his voice stage that made war between Britain and France
rings throughout the house. all but inevitable.
(A) No change (A) No change
(B) rang (B) will reach
(C) ringing (C) reached
(D) has rung (D) had reached

Top Score
GET READY TO STUDY
Chapter 8 Verb Tense

11. From 1566 until 1576 Santa Elena, now an 16. As the price of petroleum rises and concern about
excavation site in South Carolina, was the capital the environmental effects of petrochemicals
of Spanish Florida; however, it has become a mounted, the plastics industry is racing to create
settlement by 1735. plastics not made from petroleum.
(A) No change (A) No change
(B) ; however, it had become (B) mounts
(C) ; however, it becomes (C) mounting
(D) ; however, it is becoming (D) having mounted
12. At first we panicked when we discovered we had 17. According to a recent study, the domestication of
missed our flight, but then we took a bus to cats had began in the Middle East over 100,000
another airport, where there are several planes years ago.
leaving for Denver that evening. (A) No change
(A) No change (B) have begun
(B) were several (C) has begun
(C) is several (D) had begun
(D) was several 18. After 1986, trading on the London Stock
13. Only after the flood water had rose two feet was Exchange has been computerized to such an
the mayor willing to order the evacuation of some extent that personal contact between buyers and
homes. sellers became unnecessary.
(A) No change (A) No change
(B) had risen two feet was the mayor (B) have been computerized to such
(C) had risen two feet were the mayor (C) having been computerized to such
(D) had rose two feet were the mayor (D) was computerized to such
14. Although he had never played organized sports, 19. The prevailing attitude in Seventeenth-Century
whenever Justin, who was uncommonly tall, England was that schools and universities should
attends a basketball game, fans would ask him teach nothing that would discredit the established
autograph. religion or the authority of kings and magistrates.
(A) No change (A) No change
(B) attends a basketball game, fans will ask him (B) should teach nothing that will
(C) attended a basketball game, fans will ask him (C) should only teach that which will not
(D) attended a basketball game, fans would ask (D) shall teach nothing that will
him 20. During television's early years, many stations
15. Something about the project must have broadcast programming only until midnight, after
energized bureaucrats; only six month after plans which television screens will display a static
are submitted, the playground was complete. image known as a test pattern.
(A) No change (A) No change
(B) only six month after plans were submitted (B) after which television screens displayed
(C) only six month after plans will be submitted (C) after which television screens had displayed
(D) only six month after plans was submitted (D) television screens display at that time

Top Score
GET READY TO STUDY
Chapter 8 Verb Tense

21. Since last September Patricia has been working at 26. Alain Lock's 1925 book on the African American
the convenience store down the road. art movement in New York City's Harlem
(A) No change described artist who were living in Harlem and
(B) works the exceptional works they produce.
(C) will be working (A) NO CHANGE
(D) worked (B) it produces
22. At the age of seventy-one legendary country (C) they produced
music artist Johnny Cash astonished his fans by (D) they are producing
performing a song that the rock group Nine Inch 27. Professor Brown started yesterday's lecture by
Nails originally has recorded. suggesting that if Benjamin Franklin were alive
(A) NO CHANGE today, he probably worked for an advertising
(B) have recorded agency.
(C) recorded (A) NO CHANGE
(D) records (B) probably will work
23. Next Monday a committee chosen by the school (C) probably would work
board met to review the final list of applicants for (D) probably works
the position of principal at the new high school. 28. Although Duke Ellington did not collaborate with
(A) NO CHANGE other composers on many of his jazz
(B) will meet compositions, he works closely with pianist and
(C) has met composer Billy Strayhorn on a number of
(D) meets successful works over the years.
24. If we have knowledge of the future, our lives (A) NO CHANGE
would be more predictable but less interesting. (B) has worked
(A) NO CHANGE (C) worked
(B) Were we to have (D) working
(C) If we have had 29. Last month a committee created in response to
(D) If we would have employees' demands for safer working conditions
25. By the end of the day, the recruiter for the new has sponsored a workshop on industrial safety.
internship program had spoke to each senior who (A) NO CHANGE
wanted to pursue a career in city government. (B) have sponsored
(A) NO CHANGE (C) had sponsored
(B) has spoke (D) sponsored
(C) had spoken
(D) having spoken

Top Score
GET READY TO STUDY
Chapter 8 Verb Tense

Homework
1. If every nation were completely self-sufficient and 5. Now that Michiko finished the research, she feels
operated under a free-market economy, the world’s reasonably confident about writing her paper on
food supply will be governed solely by the the rise of the progressive movement in the
economics of supply and demand. United States.
(A) No change (A) No change
(B) would have been (B) has finished
(C) was (C) had finished
(D) would be (D) was finished
2. After the most popular surfboard designer in the 6. After Morris spent ten minutes giving an answer,
United States retires, many surfers were Claudette found he had given her only one item of
convinced of having no alternative but to ride information beyond what she already knew.
boards of inferior design. (A) No change
(A) No change (B) what she knows already.
(B) retiring, many surfers were convinced that (C) beyond her knowledge at the current time
they would have (D) to add to what she knew already presently
(C) retired, many surfers are convinced that they 7. Over the past two years, apparel manufacturers
would have have worked to meeting the revised federal
(D) retired, many surfers were convinced that they standards for the design of uniforms.
would have (A) No change
3. After each of the architects have finished to (B) apparel manufacturers worked to meet
present a plan for the building’s renovation, the (C) apparel manufacturers have worked to meet
committee considered the proposals and made its (D) apparel manufacturers work to meet
decision. 8. When Marie Curie shared the 1903 Nobel Prize
(A) No change for Physics with two other scientists-her husband
(B) have finished presenting Pierre Curie and Henri Becquerel-she had been
(C) has finished presenting the first woman to win the prize.
(D) had finished presenting (A) No change
4. Mr. Lee and his grandchildren practiced (B) has been
traditional Chinese calligraphy together so that the (C) was
children would be knowing an art cherished by (D) is
earlier generations of their family. 9. Winslow Homer, one of America's foremost
(A) No change artists, spent his last 27 years and painted on the
(B) would know scenic Maine coast.
(C) will know (A) No change
(D) will be knowing (B) spent his last 27 years having painted
(C) spent his last 27 years painting
(D) spending his last 27 years painting

Top Score
GET READY TO STUDY
Chapter 8 Verb Tense

10. Dr. Chien-Shiung Wu has disproved a widely 15. Samuel Adams was by no means the first
accepted theory of physics when she showed that American to espouse the democratic cause, but
identical nuclear particles do not always act alike. he has been the first who conceived the party
(A) No change machinery that made it practical.
(B) having disproved (A) No change
(C) disproved (B) had been the first who conceived
(D) disproves (C) was the first to conceive
11. The candidate called for medical insurance (D) having been the first to conceive
reform, but to me he seemed less interested in the 16. Born of Ibuza parents in Nigeria, novelist Buchi
plight of uninsured citizens than in whether Emecheta moved to England in 1962, since which
enough is registered to vote. she has lived in North London.
(A) No change (A) No change
(B) enough registered to vote (B) 1962 and has lived since then in North
(C) enough was registered to vote London.
(D) enough has been registered to vote (C) 1962 and lived since then in North London.
12. Jerome often referred to art history textbooks (D) 1962, and living in North London since that
while he was sculpting; whenever he learned a time.
new method in art class, he seeks out the work of 17. Despite its cultural importance, the Daily Gazette
sculptors who had used it in the past. lost 70 percent of its subscribers since 1920 and,
(A) No change by 1955, was losing as much as $200,000 a year.
(B) is seeking (A) No change
(C) sought (B) have lost
(D) has sought (C) has lost
13. Many admire Louisa May Alcott for her (D) loses
detailed descriptions of Nineteenth-Century 18. During television's early years, many stations
domestic life in novels such as little woman, but broadcast programming only until midnight, after
few have read the lurid thrillers she writes early in which television screens will display a static
her career. image known as a test pattern.
(A) No change (A) No change
(B) has written (B) after which television screens displayed
(C) wrote (C) after which television screens had displayed
(D) will write (D) television screens display at that time
14. When one debates the merits of the proposed 19. After the prince characterized modern architecture
reduction in our tax base, you should take into as ugly, he has been severely criticized
consideration the effect it will have on the schools for having been so outspoken in public.
and the other public services. (A) No change
(A) No change (B) he is severely criticized
(B) you should consider the effect (C) he will be severely criticized
(C) one should take the effect (D) he was severely criticized
(D) one takes into consideration the affect

Top Score
GET READY TO STUDY
Chapter 8 Verb Tense

20. By the end of 1945, Alice, thirty years old but 25. Evidence from several sources suggests that the
already an expert in the field of family dynamics, poet Emily Dickinson, when young, is quite
had begun writing her memories. sociable, although she was later deemed a
(A) NO CHANGE reclusive eccentric.
(B) has begun writing (A) NO CHANGE
(C) began writing (B) being
(D) beginning writing (C) has been
21. By the beginning of 1755, events are reaching a (D) was
stage that made war between Britain and France
all but inevitable. 26. Only after the fierce winds of the blizzard had fell
(A) NO CHANGE to occasional gusts did the bus driver feel
confident that she would navigate the snow-
(B) could reach
covered back roads.
(C) having reached (A) NO CHANGE
(D) had reached (B) been fell
22. If they would have been painted this afternoon, (C) fallen
the walls would be completely dry by tomorrow (D) falling
evening.
(A) NO CHANGE 27. In May, construction began on a second building
that will be attaching to the existing one.
(B) Were they to be painted
(A) NO CHANGE
(C) After painting them
(B) would be attaching
(D) They would be painted
(C) attaches
23. In 1882 there were only seven female doctors (D) to attach
practicing in France, when in 1903 the number
rising to 95. 28. If not for bees, which are responsible for
(A) NO CHANGE pollinating the vast majority of Earth’s flowers,
many plants were unable to produce fruits or
(B) but by 1903 the number had risen
seeds.
(C) there 1903 the number rose (A) NO CHANGE
(D) however, in 1903 when the number had risen (B) have been unable
24. Yesterday Robin and John would use a computer (C) are unable
presentation to illustrate the highlights of their (D) unable
report on alternative energy sources for heating 29. In 1961 Rita Moreno gained fame and won an
and cooling our homes. Academy Award for her portraying Anita in the
(A) NO CHANGE film adaptation of the groundbreaking Broadway
(B) used musical West Side Story.
(C) will use (A) NO CHANGE
(D) had used (B) gained fame and win
(C) gaining fame and winning
(D) gained fame and wins

Top Score
GET READY TO STUDY
Chapter 8 Verb Tense

Mini Exam
. 1 .. .
Solar Impulse 2, an aircraft powered only by (A) NO CHANGE
(B) taken off
the sun's rays, . 1 . took off from Cairo on last
(C) have taken off
Sunday morning on what should be the last leg of (D) is taking off
its history-making flight around the world.
. 2 .. .
If all . 2 . went as planned, pilot Bertrand (A) NO CHANGE
(B) goes
Piccard will steer the plane across the Arabian
(C) had gone
Peninsula and land in Abu Dhabi early Tuesday, (D) will go
completing the first global airplane flight using . 3 .. .
only energy from the sun — not a single drop of (A) NO CHANGE
(B) took
aviation fuel.
(C) taken
Piccard and the project's co-founder, Andre (D) been took
Borschberg, have . 3 . taking turns flying Solar . 4 .. .
Impulse 2 since departing from the United Arab (A) NO CHANGE
(B) begin
Emirates in March 2015 to . 4 . beginning their (C) began
35,000-kilometer journey around Earth. The (D) begun
plane . 5 . could have carried only one pilot. . 5 .. .
Piccard says circling the world in the (A) NO CHANGE
(B) can
pioneering aircraft . 6 . was both an adventure and (C) could
a project that can lead to a better world: "If we (D) carried

want to protect the environment, if we want to . 6 .. .


fight climate change ... if we want to give access (A) NO CHANGE
(B) has been
to energy to the poorest people in the world, to (C) is
the most remote villages, we . 7 . needed (D) being

renewable energy." . 7 .. .
(A) NO CHANGE
(B) would need
(C) will need
(D) to need

Top Score
GET READY TO STUDY
Chapter 8 Verb Tense
Solar Impulse 2, built of lightweight . 8 .. .
(A) NO CHANGE
materials, . 8 . being an enormous but slender (B) was
aircraft. Its wingspan is wider than that of a (C) has been
(D) is
Boeing 747 jumbo jet, but the entire plane weighs
less than 2,300 kilograms, including 17,000 solar . 9 .. .
(A) NO CHANGE
cells that . 9 . had converted sunlight into (B) converts
electrical energy. (C) convert
(D) converted
The energy .10. was stored in lithium
.10 . .
batteries, which .11. had allowed the plane to
(A) NO CHANGE
keep flying during the night. Its slow flying speed (B) is stored
(C) being stored
— only 10 percent to 15 percent as fast as a
(D) to be stored
jetliner, or near top speed for an ordinary car —
.11.. .
makes it necessary to stay aloft for days on end. (A) NO CHANGE
Stops on its global journey have 12. to (B) allowed
(C) to allow
includ Oman, India, Myanmar, China, Japan,
(D) allows
Hawaii, several U.S. cities and Spain. The plane
.12.. .
13. has arrived in Cairo about two weeks ago. (A) NO CHANGE
(B) includes
(C) include
(D) included

.13.. .
(A) NO CHANGE
(B) arrives
(C) arrived
(D) arrive

Top Score
GET READY TO STUDY
Chapter 8 Verb Tense

Answers
Explanation:
Page 1: C / B / B
Page 2: C / C / C / B
Page 3: C / C / B / A
Page 4: D / A / B

Practice:
1 C / 2 B / 3 C / 4 C / 5 B / 6 D / 7 C / 8 C / 9 C / 10 D / 11 B / 12 B / 13 B / 14 D / 15 B / 16 B / 17 D / 18 D /
19 A / 20 B / 21 A / 22 C / 23 B / 24 B / 25 C / 26 C / 27 A / 28 C / 29 D

Homework:
1 D / 2 D / 3 D / 4 B / 5 B / 6 A / 7 C / 8 C / 9 C / 10 C / 11 C / 12 C / 13 C / 14 D / 15 C / 16 B / 17 C / 18 B /
19 D / 20 A / 21 D / 22 B / 23 B / 24 B / 25 D / 26 C / 27 B / 28 C / 29 A

Mini Exam:
1 A / 2 B / 3 C / 4 B / 5 B / 6 C / 7 C / 8 D / 9 C / 10 B / 11 D / 12 D / 13 C

Top Score
GET READY TO STUDY
Chapter 9 Wordiness

Wordiness
Avoid passive voice:

The share of library materials that is in nonprint formats is increasing steadily; in 2010, at least 18.5
million e-books were available for them to circulate.
(A) NO CHANGE
(B) to be circulated by them.
(C) for their circulating.
(D) for circulation.

NOMINALIZATION causes wordiness:

When the choice is between the verb and its noun, choose the verb:

Example:

"invented" is better than "was the inventor of"

"explored" is better than "is the explorer of"

Taking time off from her job as an attorney, it was Courtney's intention to teach math to middle school
students in Boston.
(A) No change
(B) Courtney's intention was to teach
(C) Courtney had the intention for teaching
(D) Courtney intended to teach

By employing exotic harmonies and making unusual use of instruments, Mahler was a pathfinder from
romanticism to modern music.
(A) No change
(B) Mahler created a path
(C) Mahler was the creator of a path
(D) was how Mahler created a path

Two sentences for one info

Avoid the choice which includes two portions give one idea:

Example:

I met some helpful people. √ I met some people, and they were helpful. X

Top Score
GET READY TO STUDY
Chapter 9 Wordiness

In a blind taste test, people are asked which out of two or more products that are unidentified that they
prefer.
(A) No change
(B) which they would prefer out of two or more products that would not be identified
(C) which of two or more unidentified products they prefer
(D) out of two or more products, which are unidentified, what their preference would be

A beautifully written narrative of the author's boyhood in South America, his descriptions of animal life
in the plains region are fascinating.
(A) No change
(B) he fascinatingly describes animal life in the plains region
(C) the book contains fascinating descriptions of animal life in the plains region
(D) the book contains descriptions of animal life in the plains region and they are fascinating

REDUNDANCY

Using two words that mean the same thing or Synonyms.


Example:
Original: Adrian fulfilled all our hopes and dreams when she saved the whole entire planet.
Revised: Adrian fulfilled all our hopes when she saved the planet.
Common Redundancy

while - at the same time - simultaneously


and also
extra - additional- superfluous
imminent - in the future – soon
both - two
the reason - because - why - the fact
similar – the same way
annually - every year - per year - each year
after – afterwards
impact – influence
far – away
overwhelming – tremendous
Morning – Am
Quick – prompt

Top Score
GET READY TO STUDY
Chapter 9 Wordiness

Because European filmmaking shut down during the First World War is the reason Why the film industry
in the United States rose to prominence.
(A) No change
(B) Because European filmmaking all shut down during the First World War,
(C) European filmmaking all but shut down during the First World War,
(D) The Fact that European filmmaking nearly shut down during the First World War is why

After carefully reviewing the notes he had taken at the meeting, Bob afterwards prepared a report to be
distributed to all members of the council.
(A) No change
(B) after prepared
(C) prepared
(D) to prepare

Stock phrase

You can replace stock phrase with one or two words

Example:

Original: The fact that I do not like ……………………….

Revised: I dislike …………………..

Original: I have finished my project in a professional manner.

Revised: I have finished my project professionally.

But more pragmatically, the discipline encourages students to analyze complex material, question
conventional beliefs, and express thoughts in a concise manner.
(A) NO CHANGE
(B) speaking in a more pragmatic way,
(C) speaking in a way more pragmatically,
(D) in a more pragmatic-speaking way,

Top Score
GET READY TO STUDY
Chapter 9 Wordiness

Practice
1- For all their talk about ecology, major 4- When for the first time the United States
companies have so far spent very little to imported more oil than it exported,
fight pollution. Americans should have realized that an
(A) No change energy crisis was imminent and could
(B) In spite of the fact of their having happen in the future.
talked. (A) No change
(C) In addition to their talking. (B) could happen imminently in the future.
(D) Although there is talk between one and (C) will be imminent and happening soon.
the other. (D) might be imminent.
2- The inventor studied the tiny hooks on 5- Small marine crustaceans known as krill are
cockleburs that had stuck lightly to his often fed to farm animals, but there is not
clothing and then created a fabric fastener much human consumption.
that similarly in much the same way. (A) No change
(A) No change (B) animals, but consumption is not done
(B) clothing and then created a fabric much by people.
fastener that operated similarly. (C) animals but are rarely eaten by people.
(C) clothing, then creating a fabric fastener (D) animals, but eating them is rarely done
operating similarly in much the same by humans.
way. 6- Most often defenders of art have justified
(D) clothing, then a fabric fastener that its existence with their pointing out a
operated similarly was created by him function that nothing but art itself could
3- In 1912 the mayor of Tokyo gave perform.
thousands of cherry trees to the city of (A) No change
Washington D.C., which holds the annual (B) by them pointing out a function that
national cherry blossom festival every year only art could perform
to commemorate this gift. (C) through the pointing out of a function
(A) No change performed by art alone
(B) which annually holds the (D) by pointing to some function that art
(C) which holds the alone can perform
(D) and they hold the

Top Score
GET READY TO STUDY
Chapter 9 Wordiness

7- The most painting in the exhibit of work by 10- With Americans concerning sugar in
local artists was done by a seventy–year– record amounts, nutritionists are urging
old woman, who painted an exquisite self– the public to reduce its consumption of
portrait of herself. sodas, which have largely replaced other,
(A) No change more healthful, beverages.
(B) was an exquisite self–portrait of a (A) No change
seventy-year-old woman who painted (B) nutritionists have been urging that the
herself public reduces its consumption of
(C) was an exquisite self–portrait of a sodas; those
seventy-year-old woman (C) the public ought to reduce its
(D) was done by a seventy–year–old consuming of sodas, as urged by
woman, and it is her own exquisite nutritionists, because they
self–portrait (D) less soda should be consumed by the
8- A wiki is a web site that uses simple pubic urge nutritionists, which
software designed to allow the site’s
content be altered or changed by anyone 11- Earth Day, a holiday instituted to raise
who has access. awareness of environmental issues, is
(A) No change celebrated at schools across the country.
(B) to be altered or be changed (A) No change
(C) to be altered (B) which was a holiday having been
(D) be changed instituted
(C) it had been instituted as a holiday
9- Rapid transit system, though often costly,
(D) was instituted as a holiday
can be a practical solution to growing
12- Many of the ships used by oceanographic
urban traffic congestion.
institutions are small vessels that have
(A) No change
outlived the purpose which they have been
(B) often in spite of their costliness
built for originally.
(C) even though they often cost a lot
(A) No change
(D) although it often has a lot of cost
(B) vessels that have outlived their
original purpose
(C) vessels, and these had outlived their
original purpose in being built
(D) vessels, having outlived their original
purpose

Top Score
GET READY TO STUDY
Chapter 9 Wordiness

13- In addition to scientific talent, Santiago 14- It is often disappointing for people who
Ramon had artistic talent, as is evidenced travel alone and find that cabins and
by the excellent drawings in his papers hotels are priced for two or more guests..
about the nervous system. (A) NO CHANGE
(A) NO CHANGE (B) people traveling alone are often
(B) which the excellent drawings are seen disappointed to
to be evidence of and appeared (C) when people are traveling alone, it is
(C) the excellent drawings which are often disappointing to
evidence of this are (D) the disappointment for people
(D) his excellent drawings as evidence, traveling alone is often when they
appearing

Top Score
GET READY TO STUDY
Chapter 9 Wordiness

Answers
Explanation:
Page 1: D / D / B

Page 2: C / C

Page 3: B/ C / A

Practice:
1 A / 2 B / 3 C / 4 D / 5 C / 6 D / 7 C / 8 C / 9 A / 10 A / 11 A / 12 B / 13 D / 14 B

Top Score
GET READY TO STUDY
Chapter 10 Transitions

Transitions
What are transitions and how are they used?
• Transitions are phrases or words used to connect one idea to the next.
• Transitions are used by the author to help the reader progress from one significant idea
to the next.
• Transitions also show the relationship within a paragraph (or within a sentence)
between the main idea and the support the author gives for those ideas.
• Before using a particular transitional, be sure you understand its meaning and usage
completely and be sure that it's the right match for the logic in context.
• Different transitions do different things.

Addition: All these word could be replaced by "and"

Indeed Moreover Also


Further What is more As well
Furthermore In addition/Additionally In broad terms
Besides Next Not to mention

These transition words add information and express agreement with preceding material.

Examples:
- Reading is a good way to learn new vocabulary words. Furthermore, it can improve
your ability to spell words correctly.
- Grilling makes meat taste really good. As well, it allows fatty juices to drain away from
the meat.
- Jim had to be careful not to step upon the tiny piglets. Also, he tried to protect the
pretty little things from injury.
- Religion offers people an opportunity to reflect on their personal existence. Moreover, it
brings communities together in a spirit of peace and cooperation.
- He stood me up and left me standing in the rain. What's more, I broke a heel trying to
hail a cab.
- Double-decker buses can move a lot of people through dense urban areas. In addition /
Additionally, they provide a good view from which to see a city.
- He doesn't mind being unemployed right now. Besides, it gives him more time to ride
his skateboard.
- I'd love to own a horse and go riding every day. On the other hand, taking care of the
animal is a very big responsibility.

Top Score
GET READY TO STUDY
Chapter 10 Transitions

Contrast and concession: All these word could be replaced by "but"

However Conversely Otherwise


But Nevertheless Nonetheless
Yet Despite By/In contrast
On the other hand Alternatively Even so
Notwithstanding Unfortunately
These transition words express that there is evidence to the contrary and thus introduce a
change the line of reasoning (contrast).

Examples:
- We had a small drop in sales last month. However, we are still above the plan.
- He is a great swimmer, but he prefers to play golf.
- I’ve asked you a thousand times not to leave your dirty socks on the floor. Yet, you keep
doing it.
- House prices have gone up this year. In contrast, car prices seem to be stagnating.
- I was in so much pain I didn’t want to get up in the morning. Nevertheless, I went to
football practice as usual.
- Joan lived in Tokyo for ten years. Nevertheless, she can't speak any Japanese.
- Sigmund Freud once said: The great question which I have not been able to
answer, despite my thirty years of research into the feminine soul, is: What does a
woman want?
- If you love and encourage your children, they will develop good self-esteem. Conversely,
if you are harsh and critical, they may become angry and insecure.
- The Buddha tells us that to keep the body in good health is a duty. Otherwise, we shall
not be able to keep our mind strong and clear.

Effect/Result: All these word could be replaced by "so"

As a result Consequently In consequence


Thereafter Hence Thus
Accordingly Therefore For these reasons
These transition words are time words that are used to show that after a particular time
there was a consequence or an effect.

Examples:
- I’ve done a pranic healing course. As a result, I’ve been able to cure my neighbor’s sick
cat.
- Zack has skipped school on many occasions. consequently, he’s failed his French test.
- He has a race tomorrow. Therefore, he can’t stay out late tonight.
- You didn’t tell me you wanted to come. Thus, we booked only three seats not four.
- He has a race tomorrow. Hence, he can’t stay out late tonight.
- Plenty of tourists visit the area in summer. Accordingly, selling hand-made objects is
the main source of income for locals.
- The athlete was caught using drugs. For this reason, he was kicked off the team.

Top Score
GET READY TO STUDY
Chapter 10 Transitions

Exemplifying:

For example For instance Specially


Particularly In particular

These transition words are used to follow a specific idea after a general idea as an example or
support to it.

Examples:
- Some birds migrate to avoid harsher winter climates. For instance, Swallows leave the
UK in early winter and fly south.
- My father loves going to restaurants which serve exotic foods. For example, last week
he went to a restaurant which serves deep-fried rattlesnake.
- A person could change a lot in a few years. Especially, when they were that young.
- The neighboring country is pleasant enough. Particularly, along the river, but the town
itself is purely industrial, and contains no pre-eminent buildings.

Similarity:

Similarly In the same way Likewise

These transition words add information, reinforce ideas, and express agreement with
preceding material.

Examples:
- You’re not allowed to use your phone here. Similarly, you have to switch it off when
you’re in the library.
- Cutting down on sugar will help you lose weight. In the same way, doing more exercise
will help you get rid of a few kilos.
- The banks advise against sending cash. Likewise, sending British cheques may cause
problems

Conclusion:

To conclude–Eventually–At last–Lastly–Finally–In short–Ultimately–To these ends…etc.


These transition words conclude, summarize, or restate ideas, or indicate a final general
statement.

Places of the transition words:

a) _________________ . Transition, Subject + Verb


b) _________________ . Subjects, transition, Verb
c) _________________ . Full Sentence……….. , transition

Top Score
GET READY TO STUDY
Chapter 10 Transitions

Practice
Beginners

1) Having a car can be very convenient. Also, it's 7) Beekeeping is an important occupation.
expensive to buy and maintain one. Consequently, it is tiring because the beekeeper
(A) No change must keep himself covered at all times.
(B) Moreover (A) No change
(C) Consequently (B) Thus
(D) However (C) Moreover
2) Hector decided not to use a map. Consequently, (D) On the other hand
he got lost and never found his way out of the 8) Our lives have become complicated. However,
forest. There he died. we've decided to simplify things and move to a
(A) No change log cabin in the country.
(B) Finally (A) No change
(C) For example (B) Accordingly
(D) Likewise (C) Likewise
3) His skin burns very easily. Besides, he's decided (D) Also
to stay under an umbrella and wear lots of 9) He spends too much time doing office work even
sunscreen. when he's out with his girlfriend. As a result,
(A) No change she's thinking about ending the relationship.
(B) Otherwise (A) No change
(C) Therefore (B) Moreover
(D) Nevertheless (C) Despite
4) Jim is in love with Graciella. However, he's not (D) Alternatively
sure if she loves him. 10) Nim could easily get a job as a nurse if she
(A) No change moved to the United States; Moreover, she would
(B) Accordingly miss her family in Thailand if she moved away.
(C) Similarly (A) No change
(D) Moreover (B) For example
5) Living in a cold climate is difficult for some (C) On the other hand
people. In addition, there are many fun winter (D) Finally
activities, such as sledding, that you can't do in a 11) The birds in our backyard couldn't find enough to
warm climate. eat. For instance, we set up a bird feeder and now
(A) No change they're okay.
(B) On the other hand (A) No change
(C) Moreover (B) Thus
(D) Thus (C) However
6) Bill doesn't mind working late. What's more, he's (D) In addition
a little scared of walking home by himself at this 12) They practice their instruments; However, they
time of the night. study theory and music history.
(A) NO change (A) No change
(B) In contrast (B) Therefore
(C) In addition (C) In addition
(D) Therefore (D) Specially

Top Score
GET READY TO STUDY
Chapter 10 Transitions

13) Gas prices rose too high for Matt to 19) She doesn't want to move to San Francisco
afford. Consequently, he sold his car and made because it's too expensive to live there.
his daily commute by bike. Therefore, she likes living in a cold weather state.
(A) No change (A) No change
(B) Likewise (B) Besides
(C) Furthermore (C) However
(D) Besides (D) Nevertheless
14) Walter has a lot of work to do. Moreover, he's 20) Owning a house can cost a lot of money. Thus,
too tired to finish it. home ownership is a big part of living the
(A) No change American dream.
(B) Thus (A) No change
(C) As a result (B) In addition
(D) However (C) Therefore
15) Hector was a very good student who studied (D) Nevertheless
constantly. Consequently, he found a very good 21) Marijuana is less toxic than alcohol or tobacco.
job after he graduated from college. Nevertheless, some people believe it should be
(A) No change legalized.
(B) Likewise (A) No change
(C) In addition (B) For example
(D) In the same way (C) Therefore
16) Driving while distracted increases your chances (D) What's more
of getting into an accident. What's more, it 22) Her singing abilities aren't as good as the others
endangers the lives of other drivers. in the choir. Also, she enjoys singing.
(A) No change (A) No change
(B) Despite (B) Therefore
(C) Finally (C) Nevertheless
(D) For instance (D) For example
17) It's important to be cautious when using a 23) The rainforests are being destroyed. Despite,
chainsaw. Hence, accidents and injuries still many species of wildlife are becoming extinct.
happen. (A) No change
(A) No change (B) Therefore
(B) Besides (C) Likewise
(C) Moreover (D) For instance
(D) Nevertheless 24) I missed my family; likewise, they missed their
18) Recycling is an easy way to conserve natural son.
resources. Thus, it can help reduce garbage (A) No change
disposal costs. (B) However
(A) No change (C) Finally
(B) Despite (D) Furthermore
(C) Also 25) I love to go to the gym. Especially, my sister
(D) Likewise prefers to exercise at home.
(A) No change
(B) Moreover
(C) In the same way
(D) In contrast

Top Score
GET READY TO STUDY
Chapter 10 Transitions

26) The current political situation is worrying. In 30) I don't like Italian food. Also, I don't eat pasta or
addition, people do not seem to be too worried pizza.
about it. (A) No change
(A) No change (B) Therefore
(B) However (C) Moreover
(C) Therefore (D) Despite
(D) Similarly 31) Students should attend class. Otherwise, they
27) Caracas is fun and the weather there is beautiful. may lose their status.
Besides, it is a very dangerous city to live in. (A) No change
(A) No change (B) But
(B) Therefore (C) Moreover
(C) Likewise (D) For example
(D) On the other hand 32) I need to wear reading glasses. In addition, I hate
28) We have to take care of the environment. how I look in them.
However, we need to educate our children so (A) No change
they do not make the same mistake. (B) Therefore
(A) No change (C) Nevertheless
(B) Nevertheless (D) Likewise
(C) Moreover 33) Silk possesses qualities invaluable in nest
(D) As a result building. However, birds incorporate or introduce
29) There are lots of beautiful people around me. For silk into their nests, including it as a component.
instance, I have a wonderful family that supports (A) No change
and loves me dearly. (B) For this reason
(A) No change (C) Likewise
(B) Nevertheless (D) Moreover
(C) Thus
(D) At least

Top Score
GET READY TO STUDY
Chapter 10 Transitions

Intermediate

1) Poachers have hunted and killed too many 6) The United States suffered terribly under
elephants for their tusks. For instance, a weakened economy and a lack of
they have become an endangered species government oversight in 2008. For
in some parts of the world. example, the Democrats easily won in the
(A) No change November 2008 elections.
(B) Hence (A) No change
(C) However (B) Thus
(D) In addition (C) Otherwise
2) Homelessness produces intense feelings of (D) What's more
despair in those who don't have a place to 7) The use of fire by early humankind made
live. Nevertheless, it produces feelings of cold climates more inhabitable. However,
guilt among people who are not homeless. it made people more productive by
(A) No change lengthening the number of hours that
(B) However people could do work.
(C) Furthermore (A) No change
(D) Consequently (B) Furthermore
3) Astronomy has been an interest of human (C) Nevertheless
beings for centuries. Moreover, our (D) Therefore
curiosity has led to a better understanding 8) Life in the United States is easy compared
of our place in the solar system and to other parts of the world; Also, the stress
human space exploration. of daily American life makes one wonder
(A) No change how easy it really is.
(B) However (A) No change
(C) Thus (B) In consequence
(D) Likewise (C) Similarly
4) There were too many accidents on this (D) On the other hand
road when the speed limit was 55 miles 9) An adequate water supply is important to
per hour. Similarly, the city decided to a farmer's success. Nonetheless, irrigation
lower it to 45 miles per hour. The limit for is used in places where water is in short
snowmobiles was also lowered. supply.
(A) No change (A) No change
(B) As well (B) As well
(C) For instance (C) For instance
(D) Therefore (D) Therefore
5) In primate troops, males “defend, control, 10) There are many benefits to
and lead the troop.” Despite, the troops are exercising. Thus, you must take some
dominated by adult males. precautions to avoid injury.
(A) No change (A) No change
(B) Nevertheless (B) However
(C) Consequently (C) Moreover
(D) For example (D) Therefore

Top Score
GET READY TO STUDY
Chapter 10 Transitions

11) No one likes to feed parking meters. 15) To fail, one must first make an attempt. In
Nevertheless, they are a low-cost that attempt, experience expands, new
alternative to parking garages, and they ideas blossom, and viewpoints change.
help keep city streets free of abandoned Nevertheless, even if one does not always
vehicles. accomplish what he or she sets out to do,
(A) No change one gains new knowledge from the
(B) Moreover attempt.
(C) Hence (A) No change
(D) Likewise (B) Therefore
12) Kurtis was told not to go out fishing before (C) In other words
the storm. For this reason, the coast guard (D) However
came to his assistance when he called for 16) This timber wolf was hunted almost to the
help. point of extinction. Nevertheless, the
(A) No change government put it on an endangered
(B) Nevertheless species list and now it's protected by law.
(C) Also (A) No change
(D) Similarly (B) In the same way
13) We have ignored environmental issues for (C) For this reason
many years. For instance, we have a (D) But
plethora of problems related to the matter. 17) Ninety per cent of the energy used in the
(A) No change US comes from fossil fuels, oil, coal, and
(B) Consequently natural gas, but problems arise from other
(C) However sources. For example, nuclear power
(D) What's more plants leave radioactive by-products,
14) Many people seem too busy to take the making storage difficult.
time to understand one another. However, (A) No change
my father sees taking time to listen as (B) Nevertheless
essential to any relationship. (C) Furthermore
(A) No change (D) Subsequently
(B) Also
(C) For instance
(D) Thus

Top Score
GET READY TO STUDY
Chapter 10 Transitions

Advanced

1) Questions about the Model 3 are more 4) To be legitimate authorities, all


pointed now that General Motors Co governments must uphold man’s rights
(GM.N) has confirmed it expects its and do justice. Otherwise, the people owe a
$35,000 Chevy Bolt electric car to launch lawless and tyrannical ruler no allegiance
at the end of this year. However, German at all.
competitors are speeding up plans to offer (A) No change
luxury electric cars. (B) However
(A) No change (C) Therefore
(B) Also (D) Moreover
(C) Finally 5) The humorous story is American; the
(D) For example comic story, English; the witty story,
2) Prime Minister Shinzo Abe launched a French. The humorous story depends for
bold economic revival drive in late 2012, its effect upon the manner of the telling;
known as Abenomics, with fiscal the comic story and the witty story upon
expansion, monetary stimulus and what the matter. The humorous story may be
was supposed to be sweeping reforms. spun out to great length, and may wander
Moreover last arrow has missed its mark around as much as it pleases, and arrive
with critics nowhere in particular. However, the comic
(A) No change and witty stories must be brief and end
(B) For instance with a point.
(C) However (A) No change
(D) Therefore (B) Thus
3) North Korea said the rocket was carrying (C) Additionally
a satellite. It was launched early Sunday (D) In broad terms
morning near the northwestern border 6) The catastrophist theories hypothesized or
with China. The North said the launch is maintained that mountains and species
part of its peaceful program to send were created by sudden dramatic events or
satellite into orbit. What's more, many catastrophes. Moreover, Darwin theorized
experts said the space program is being that nature was the result of cumulative,
used to hide its test of nuclear weapons gradual change.
and ballistic missiles systems. (A) No change
(A) No change (B) However
(B) Especially (C) Thus
(C) In addition (D) Likewise
(D) In contrast

Top Score
GET READY TO STUDY
Chapter 10 Transitions

7) The silence of the poor reaffirms their 12) Picasso admitted that at the time he was
sense of despair. They feel powerless to working on Les Demoiselles “he was much
alter their condition. Yet, they listen but interested in Iberian” or ancient Spanish
don’t say anything. sculpture. But he may have been
(A) No change influenced by ancient Spanish art.
(B) Conversely (A) No change
(C) Consequently (B) Accordingly,
(D) Also (C) However,
8) The city of Paris might come to have some (D) Moreover,
symbolic value for you, bringing a mood of 13) While the education provided the Indians
joy to your mind. Nevertheless, the in the colleges of the northern provinces
relationship between the city and the included all the white men’s sciences, it
mood is not an inherent, built-in one; it is did not prepare these young men for life in
purely coincidental. the woods. Thus, it did not meet the
(A) No change Indian elders’ educational goals. It is clear
(B) Hence that the Indians and the gentlemen of
(C) In addition Virginia have different educational goals.
(D) For instance (A) No change
9) Presidents, like everyone else, hate to be (B) However
criticized in public. For instance, they all (C) For example
have experienced hostility between (D) Conversely
themselves and the press. 14) The arboreal cacti lack access to moisture
(A) No change because they grow high up in the canopy
(B) In contrast with no root connections to the soil.
(C) In consequence However, both kinds of cacti have had to
(D) Likewise develop features to cut down or reduce the
10) This important trade involving vast loss of moisture.
quantities of textiles was so vital to the (A) No change
economy that not even a war could stop it. (B) Nevertheless
Particularly, it continued or kept on taking (C) In addition
place through the Revolutionary War. (D) Therefore
(A) No change 15) As founder and president of the Children’s
(B) Nonetheless Defense Fund, Marian Wright Edelman
(C) Finally has ensured that the young cannot vote or
(D) As a result make campaign contributions. Therefore,
11) She yearned for the fame that only that they are not ignored in Washington.
world could give. Furthermore, she felt (A) No change
contempt for the world of money, opinion (B) Similarly
and power. (C) Nevertheless
(A) No change (D) Moreover
(B) However
(C) Moreover
(D) Thus

Top Score
GET READY TO STUDY
Chapter 10 Transitions

16) The family itself had to remain honest and consequence such as sweet fluid or the
upright. Moreover, the geomancer’s job smiling presence of the mother, will very
was not only to identify bad and good sites likely repeat the act with increasing
but also to advise on how to mitigate evil frequency as time and reinforcement goes
influences or to improve good ones. on.
Response consequences serve as (A) No change
reinforcers of the behavior, then, and tend (B) Therefore
to perpetuate the behavior. Yet, an infant (C) However
who spontaneously makes a sound, which (D) Likewise
is then followed by an attractive

Official Guide Questions

1) Greek yogurt is slightly lower in sugar and 4) Philosophy is now being seen by many
carbohydrates than conventional yogurt is. students and prospective employers as in
Also, because it is more concentrated, fact a very useful and practical major,
Greek yogurt contains slightly more offering students a host of transferable
protein per serving. skills with relevance to the modern
(A) NO CHANGE workplace. In broad terms, philosophy is
(B) In other words, the study of meaning and the values
(C) Therefore, underlying thought and behavior.
(D) For instance, (A) NO CHANGE
2) The ice sheet begins to show evidence of (B) For example,
thawing in late summer. For example, in (C) In contrast,
the summer of 2012, virtually the entire (D) Nevertheless,
Greenland Ice Sheet underwent thawing 5) A 1994 survey concluded that only 18
at or near its surface by mid-July, the percent of American colleges required at
earliest date on record. least one philosophy course. Therefore,
(A) NO CHANGE between 1992 and 1996, more than 400
(B) However, independent philosophy departments were
(C) As such, eliminated from institutions.
(D) Moreover, (A) NO CHANGE
3) The spaces are usually stocked with (B) Thus,
standard office equipment, such as (C) Moreover,
photocopiers, printers, and fax machines. (D) However,
In these locations, however, the spaces
often include small meeting areas and
larger rooms for hosting presentations.
(A) NO CHANGE
(B) In addition to equipment,
(C) For these reasons,
(D) Likewise,

Top Score
GET READY TO STUDY
Chapter 10 Transitions

6) Public libraries in the United States have 10) Among the possibilities to reconfigure a
experienced reducing in their operating building’s lighting is the installation of
funds due to cuts imposed at the federal, full-pane windows to allow the greatest
state, and local government levels.
degree of sunlight to reach office interiors.
However, library staffing has been cut by
almost four percent since 2008, and the Thus, businesses can install light tubes,
demand for librarians continues to these are pipes placed in workplace roofs
decrease. to capture and funnel sunlight down into a
(A) NO CHANGE building’s interior.
(B) Consequently, (A) NO CHANGE
(C) Nevertheless, (B) Nevertheless,
(D) Previously,
(C) Alternatively,
7) The first time I visited the Art Institute of (D) Finally,
Chicago, I expected to be impressed by its
11) Although advocates of organic food
famous large paintings. On one hand, I
preserve that organic produce is healthier
couldn’t wait to view painter Georges
than conventionally grown produce
Seurat’s 10-foot-wide A Sunday Afternoon
because it has more vitamins and
on the Island of La Grande Jatte in its full
minerals, this assertion is not supported
size.
by scientific research. For instance, one
(A) NO CHANGE
review published in The American Journal
(B) For instance,
of Clinical Nutrition provided analysis of
(C) However,
the results of comparative studies
(D) Similarly,
conducted over a span of 50 years;
8) Two years or less of sea otters can researchers consistently found no evidence
completely eliminate sea urchins in a
that organic crops are more nutritious
coastal area. Nevertheless, without sea
than conventionally grown ones in terms
otters present, kelp forests run the danger
of becoming barren stretches of coastal of their vitamin and mineral content.
wasteland known as urchin barrens. (A) NO CHANGE
(A) NO CHANGE (B) However,
(B) However, (C) In addition,
(C) Hence, (D) Likewise,
(D) Likewise,
12) The programmers, artists, and others on
9) He painted the mural’s first two sections,
the team all share the same vision.
featuring images of a tropical rainforest
Likewise, anyone considering a career as a
and a Maya pyramid, during the day. Also,
video game designer must be skilled
to avoid scrutiny, Siqueiros painted the
writers and speakers.
final section of the mural, the centerpiece
(A) NO CHANGE
at night.
(B) Nevertheless,
(A) NO CHANGE
(C) Consequently,
(B) However,
(D) However,
(C) Although,
(D) Moreover,

Top Score
GET READY TO STUDY
Chapter 10 Transitions

13) In the name of health, spending $1.60 benefits, in terms of either nutritional
for every dollar they would have spent value or safety, from organic food.
on food that is 14 grown in a manner
(A) NO CHANGE
that is considered conventional.
(B) furthermore,
Scientific evidence, therefore, suggests
(C) however,
that consumers do not reap significant
(D) subsequently

Top Score
GET READY TO STUDY
Chapter 10 Transitions

Answers
Beginners:
1 D / 2 A / 3 C / 4 A / 5 B / 6 B / 7 D / 8 B / 9 A / 10 C / 11 B / 12 C / 13 A / 14 D / 15 A / 16 A / 17 D / 18
C / 19 B / 20 D / 21 C / 22 C / 23 B / 24 A / 25 D / 26 B / 27 D / 28 C / 29 A / 30 B / 31 A / 32 C / 33 B

Intermediate:
1 B / 2 C / 3 C / 4 D / 5 C / 6 B / 7 D / 8 D / 9 D / 10 B / 11 A / 12 B / 13 B / 14 A / 15 C / 16 C
17 A

Advanced:
1 B / 2 C / 3 D / 4 A / 5 A / 6 B / 7 C / 8 A / 9 C / 10 D / 11 B / 12 B / 13 A / 14 D / 15 C / 16 B

Official Guide Questions:


1 A / 2 B / 3 B / 4 A / 5 C / 6 B / 7 B / 8 B / 9 B / 10 C / 11 A / 12 C / 13 C

Top Score
GET READY TO STUDY
Chapter 11 Data Analysis

Relevance and linking details:


You must take in your consideration the following words and their functions when you answer this kind of
questions:

1- "related / relevant / completes the description…."


In the questions that contain these words, you must read a sentence before and a sentence after the
"UNDERLINED SENTENCE", and then you must match the main idea of the choices with the main
idea of the two sentences.

Example 1:

Farmers have found a number of uses for Which choice provides the most relevant
acid whey. [4] They can add it to livestock feed detail?
A) NO CHANGE
as a protein supplement, and people can make B) supplement and convert it into gas to use as
their own Greek-style yogurt at home by fuel in electricity production.
C) supplement, while sweet whey is more
straining regular yogurt. desirable as a food additive for humans.
D) supplement, which provides an important
element of their diet.
The answer here is (B) because it shows "a number of uses for acid whey": "They can add it …..and convert
it…." (A) talks about "Greek-style yogurt", (C) talks about "sweet whey", (D) blur the talk about the uses of acid
whey.

Example 2:

As the ice melts, the land and water under Which choice best completes the description
the ice become exposed, and since land and water of a self-reinforcing cycle?
A) NO CHANGE
are darker than snow, the surface absorbs even B) raises the surface temperature.
more heat, which is related to the rising C) begins to cool at a certain point.
D) leads to additional melting.
temperatures.

Here the answer is (D) because it talks about what will happen "As the ice melts": "leads to additional melting"
(A) talks about "rising temperatures", (B) talks about "surface temperature", (C) talks about " to cool at a
certain point"

Top Score
GET READY TO STUDY
Chapter 11 Data Analysis

2- inked ideas / to link the idea of …


In the questions that contain these words, the writer wants to end the paragraph with a sentence related
to the first sentence of the next paragraph. So, you must match them together with the same idea.

Example:

Yet some of the earliest known works of art, The writer wants to link the first paragraph with
including paintings and drawings tens of the ideas that follow. Which choice best
thousands of years old found on cave walls accomplishes this goal?
in Spain and France, portrays animals. Nor has
artistic homage to our fellow creatures entirely A) NO CHANGE
died out in the millennia since, despite the many B) with special attention being paid to domestic
years that have passed between then and now. animals such as cats.
The State Hermitage Museum in St. C) even though most paintings in museums are of
Petersburg, one of Russia’s greatest art museums, people, not animals.
has long had a productive partnership with a D) as the example of one museum in Russia
much loved animal: the cat. shows.

Here the answer is (D) because it talks about "museum in Russia" and the first sentence of the next paragraph
talks about "one of Russia’s greatest art museums" (A) talks about "many years passed", (B) talks about
"domestic animals", (C) talks about "the paintings in museums are of people, not animals".

3- "sets up / introduces……. "


In the questions that contain these words, you must match the "UNDERLINED SENTENCE" with the
next sentence.

Example:

Studies have shown that employees are


happier in environment in which temperatures are Which choice provides the most appropriate
carefully controlled. New buildings may be introduction to the passage?
designed with these studies in mind, but many A) NO CHANGE
older buildings were not, resulting in spaces that B) that affords them adequate amounts of
often depend primarily on artificial lighting. natural light.
While employers may balk at the expense of C) that is thoroughly sealed to prevent energy
reconfiguring such buildings to increase the loss.
amount of natural light, the investment has been D) in which they feel comfortable asking
shown to be well worth it in the long run. managers for special accommodations.

Top Score
GET READY TO STUDY
Chapter 11 Data Analysis

Here the answer is (B) because it talks about "the natural light" the idea which matches with the next
sentence which is talking about "natural lighting / amount of natural light".
(A) is incorrect because it talks about the "temperatures", (C) is incorrect because it talks about "energy
loss", (D) is incorrect because it talks about "special accommodations".
4- "supports / logically follows……"
In the questions that contain these words, you must match the "UNDERLINED SENTENCE" with the
previous sentence.

Example:
Employees who feel less than 100 percent Which choice best supports the statement made in the
previous sentence?
and are sleep deprived are also less prone to work
A) NO CHANGE
at their maximal productivity. One company in
B) saw a 5 percent increase in productivity
California gained a huge boost in its employees’
C) saved a great deal on its operational costs
morale when it moved from an artificially lit
D) invested large amounts of time and capital
distribution facility to one with natural illumination.

Here, the answer is (B) because it talks about "productivity" the idea which matches with the previous
sentence which is talking about "Maximal productivity".
(A) is incorrect because it talks about the "employees' morale", (C) is incorrect because it talks about
"operational costs", (D) is incorrect because it talks about "time and capital".

5- "Transition"
The sentence makes "transition" from sentence to another sentence when it is related to the previous sentence
and the next sentence.
Example :

Just weeks after the scientists added the nitrates and . 5 . Which choice provides the best transition from
phosphates, the water in Lake 227 turned bright green: the previous paragraph to this one?
it was thick with the same type of algal blooms that had
plagued Lake Erie. A) NO CHANGE
B) The Experimental Lakes Area is located in a
. 5 . One mission of the Experimental Lakes Area is sparsely inhabited region that experiences few
to conduct research that helps people better understand effects of human and industrial activity.
threats to the environment. The scientists divided the C) To isolate the cause of the algae, Schindler and
lake in half by placing a nylon barrier through the Brunskill performed another experiment, this
narrowest part of its figure-eight shape. In one half of time using Lake 226.
Lake 226, they added phosphates, nitrates, and a source D) The process by which water becomes enriched
of carbon; in the other, they added just nitrates and by dissolved nutrients, such as phosphates, is
carbon. called eutrophication.

Top Score
GET READY TO STUDY
Chapter 11 Data Analysis
To solve this kind of questions, as mentioned before, you must choose the answer that related to the previous
sentence and related to the next sentence.
- The previous sentence says that scientists added the nitrates and phosphates, the water in Lake 227 turned
bright green.
- The next sentence says that The scientists divided the lake in half and says In one half of Lake 226, they
added phosphates, nitrates, and a source of carbon; in the other, they added just nitrates and carbon.
So, as it shown for us the first sentence is talking about an experiment in Lake 227 and the second is talking
about another experiment in Lake 226. Therefore, the best answer is (C).

6- "writer's main or primary idea or claim / main idea of the passage or


paragraph….."
In the questions that contain these words, you must identify the main idea of the paragraph by
selecting the important words of the rest of the paragraph.

Example 1:
In sum, the Internet does not replace the Which choice most clearly ends the passage
need for librarians, and librarians are hardly with a restatement of the writer’s primary
obsolete. Like books, librarians have been around
claim?
A) NO CHANGE
for a long time, but the Internet is extremely B) Although their roles have diminished
useful for many types of research. significantly, librarians will continue to be
employed by public libraries for the
foreseeable future.
C) The growth of electronic information has led
to a diversification of librarians’ skills and
services, positioning them as savvy resource
specialists for patrons.
D) However, given their extensive training and
skills, librarians who have been displaced by
budget cuts have many other possible
avenues of employment.

Here, the answer here is (C) because the main claim of the paragraph is that "the internet doesn't replace the
need for librarians" which means that librarians are as important as internet the idea that matches the answer
(C) " the growth of electronic information has led to a diversification of librarians’ skills and services"
(A) is incorrect because it says that the librarians became not needed like books meanwhile the internet is
useful, (B) is incorrect because it says that the librarians' roles disappeared, (D) is incorrect because it talks
the employability of the librarians.

Top Score
GET READY TO STUDY
Chapter 11 Data Analysis
Example 2:

.19. [1] Termite mounds can reach up to thirty feet .19. Which choice most effectively establishes
high. [2] To provide some perspective: termites are the main topic of the paragraph?
a quarter of an inch long and build structures that
A) Termites are able to survive in many
are approximately 1,500 times their height. [3] The
human-scale equivalent would be building a different climates.
skyscraper over 8,000 feet tall, but the tallest B) Termites have engineered more
skyscraper in the world is not even 3,000 feet tall. complicated structures than those designed
[4] The enormous mounds support local ecosystems, by humans.
allowing water to penetrate deep into the ground and C) Some termites build immense mounds that
creating moist oases where plants can flourish in are structural marvels and that benefit a
arid terrain. [5] Animals also benefit from the
multitude of other organisms.
presence of the mounds. [6] Many animals,
including lizards, aardvarks, and mongooses, D) Tourists visiting countries where termites
burrow into these mounds and use them as build mounds are often in awe of these
homes. extraordinary structures.

Here, the answer here is (C) because it talks about the structure of the mounds and how it is beneficial to
other organisms which is the main idea of the paragraph: " Termite mounds can reach up to thirty feet high",
"Animals also benefit from the presence of the mounds", " Many animals, including lizards, aardvarks, and
mongooses, burrow into these mounds and use them as homes" (A) is incorrect because it talks about the
ability of Termites to live in different conditions, (B) is incorrect because it compares the Termites' structure
to humans' structures, (D) is incorrect because it says that the Termite mounds visited by many tourists.

7- "Explains why / gives reason:"


The sentence explains why or gives reason when it contains one of these words: "because, for , as … etc"
These tails can exceed 150 million kilometers in Which choice most clearly explains why comet tails
length are visible from Earth. Because tails are move away from the sun?
caused by solar winds, they are always moving A) NO CHANGE
away from the sun. B) Tails, because of their cause, solar winds, are
always moving away from the sun.
C) They are being caused by solar winds, and tails
are always moving away from the sun.
D) Tails are always moving away from the sun,
being caused by solar winds.

Here the question asks you to choose the sentence which explains why, so the best answer here is (A).

Top Score
GET READY TO STUDY
Chapter 11 Data Analysis

Exercise 1
Because philosophy teaches students not what to . 1 . Which choice most effectively sets up the
think but how to think, the age-old discipline offers information that follows?
consistently useful tools for academic and A) Consequently, philosophy students have been
receiving an increasing number of job offers.
professional achievement.. 1 . A 1994 survey
B) Therefore, because of the evidence, colleges
concluded that only 18 percent of American increased their offerings in philosophy.
colleges required at least one philosophy course. C) Notwithstanding the attractiveness of this course
Moreover, between 1992 and 1996, more than 400 of study, students have resisted majoring in
independent philosophy departments were philosophy.
eliminated from institutions. D) However, despite its many utilitarian benefits,
colleges have not always supported the study of
philosophy.
………………………………………………………………………………………………………………….
. 2 . Free to all who utilize their services, public . 2 . Which choice most effectively sets up the
libraries and librarians are especially valuable, examples given at the end of the sentence?
because they offer free resources that may be A) NO CHANGE
B) During periods of economic recession,
difficult to find elsewhere, such as help with online
C) Although their value cannot be measured,
job searches as well as résumé and job material D) When it comes to the free services libraries
development. provid
………………………………………………………………………………………………………………….
A couch, for example, is seven inches long, and . 3 . . 3 . Which choice gives a second supporting
that is based on a seven-foot-long couch. Each room example that is most similar to the example already
represents a distinctive style of European, in the sentence?
American, or Asian interior design from the A) NO CHANGE
thirteenth to twentieth centuries. B) a tea cup is about a quarter of an inch.
C) there are even tiny cushions on some.
D) household items are also on this scale.
………………………………………………………………………………………………………………….
In return for the servers’ work, the position paid . 4 . Which choice most logically follows the
quite well for the time: $17.50 a month, plus tips, previous sentence?
meals, room and board, laundry service, and travel A) The growth of Harvey’s business coincided with
expenses. .4 . the expansion of the Santa Fe Railway, which
served large sections of the American West.
B) Harvey would end up opening dozens of
restaurants and dining cars, plus 15 hotels, over
his lucrative career.
C) These benefits enabled the Harvey Girls to save
money and build new and exciting lives for
themselves in the so-called Wild West.
D) The compensation was considered excellent at
the time, though it may not seem like much
money by today’s standards.

Top Score
GET READY TO STUDY
Chapter 11 Data Analysis

It also reinforces the belief that it is easier to replace . 5 . Which choice provides information that best
goods than to mend them, as repair shops are rare supports the claim made by this sentence?
and . 5 . repair methods are often specialized. In A) NO CHANGE
2009, an enterprising movement, the Repair Café, B) obsolete goods can become collectible items.
challenged this widely accepted belief. C) no one knows whether something will fall into
disrepair again.
D) new designs often have “bugs” that must be
worked out.

. 6 . Despite having worked for railroad companies, . 6 . Which choice provides the most logical
Fred Harvey, an English-born entrepreneur, decided introduction to the sentence?
to open his own restaurant business to serve rail A) NO CHANGE
customers. B) He had lived in New York and New Orleans, so
C) To capitalize on the demand for good food,
D) DELETE the underlined portion.

The room is simple but spacious, with a small sink . 7 . Which choice most closely matches the stylistic
and counter along one wall, a cast-iron wood stove pattern established earlier in the sentence?
and some hanging pots and pans against another A) NO CHANGE
wall, and . 7 . a small table under a window of the B) a small table is under the third wall’s window.
third wall. Aside from a few simple wooden chairs C) the third wall has a window and small table.
placed near the edges of the room, the floor is open D) the third wall has a small table against it and a
and obviously well worn. window.

Employee naps might also lead to reduced health . 8 . Which choice provides a supporting example
care costs for companies, since regular napping that
leads to long-term health benefits, . 8 . and it reinforces the main point of the sentence?
improves workers’ average weekly attendance. A) NO CHANGE
B) including a lower risk of cardiovascular problems
such as heart attack and stroke.
C) which are essential in an era of rising health care
costs.
D) in addition to making employees more efficient.

With 30 percent of United States workers not getting . 9 . The writer wants a concluding sentence that
enough sleep at night, according to the Wall Street restates the main argument of the passage. Which
Journal, US companies 1 lose $63.2 billion annually choice best accomplishes this goal?
due to the drop in employee productivity resulting A) NO CHANGE
from sleep deprivation. Sleep-deprived workers B) Clearly, employers should consider reducing
generally have lower morale and are less able to employees’ hours when they are overworked.
retain information than their better-rested C) Companies should consider employee schedules
colleagues. carefully when implementing a napping policy.
. 9 . These executives are among the most successful D) More businesses should follow their lead and
leaders in their respective fields. embrace napping on the job.

Top Score
GET READY TO STUDY
Chapter 11 Data Analysis
.10 Nature has been around forever. First-century .10. Which choice most effectively sets up the
Roman naturalist Pliny the Elder stated in his paragraph?
Natural History that the Moon “replenishes the A) NO CHANGE
Earth; when she approaches it, she fills all bodies, B) People all over the world farm by the Moon.
while, when she recedes, she empties them.” C) Farming by the Moon is not new.
Chinese and Egyptian people performed agricultural D) Talk of the Moon’s influence is far-reaching.
tasks according to the lunar calendar for millennia,
and, to this day, the vaunted Old Farmer’s Almanac
includes regional lunar calendars and advice on
when to conduct farm chores. The almanacs editor,
Janice Stillman, says, “That information is of value
to our readers who practice these traditional
methods—and claim great success.”

Chinese and Egyptian people performed agricultural .11. Which choice provides the most specific
tasks according to the lunar calendar for millennia, information
and, to this day, the vaunted Old Farmer’s Almanac on the type of advice a lunar calendar offers?
includes regional lunar calendars and advice on 11. A) NO CHANGE
when to conduct farm chores. B) actions relevant to farming.
C) points in time at which to undertake certain tasks.
D) optimal times to plant, weed, prune, and harvest.

Nast’s cartoons depicted Tweed as a great big .12. Which choice adds the most relevant supporting
bloated thief. One of the artist’s most famous images information to the paragraph?
showed Tweed with a bag of money in place of his A) head; like many other Nast cartoons, that one was
.12 head. published in Harper’s Weekly.
B) head; Nast would later illustrate Tweed’s escape
from prison.
C) head, one depiction that omits Tweed’s signature
hat.
D) head, an image that perfectly captured Tweed’s
greedy nature.

The research demonstrated a clear correlation .13. At this point, the writer wants to add a second
between introducing phosphates and the growth of policy outcome of the research described.
blue-green algae. Subsequently, legislators in Which choice best accomplishes this goal?
Canada passed laws banning phosphates in laundry A) Lake 226 continued to develop blooms of blue-
detergents, which had been entering the water green algae for eight consecutive years after the
supply 13. experiment took place.
B) In the United States, many individual states have
also adopted legislation to eliminate, or at least
reduce, phosphorous content in laundry
detergents.
C) In 1974, Schindler initiated a study of the effects
of acid rain, using Lake 223 to examine how
sulfuric acid altered aquatic ecosystems.
D) Aerial photos of the lakes taken before and
during algal blooms helped convey the effects of
phosphates in water to the public.

Top Score
GET READY TO STUDY
Chapter 11 Data Analysis

Sentence Placement

A) Sentence Movement:
To make this paragraph most logical, sentence "8" should be placed
A) where it is now. "The sentence supports the previous sentence OR introduces the following sentence"
B) after sentence 2. "The sentence supports sentence 2 OR introduces sentence 3"
C) before sentence 4. "The sentence introduces sentence 4 OR supports sentence 3"
D) after sentence 4. "The sentence supports sentence 4 OR introduces sentence 5"
Example 1:

[1] Termite mounds can reach up to thirty feet high. arid terrain. [5] Animals also benefit from the
[2] To provide some perspective: termites are a presence of the mounds. [6] Many animals,
quarter of an inch long and build structures that are including lizards, aardvarks, and mongooses,
approximately 1,500 times their height. [3] The burrow into these mounds and use them as homes.
human-scale equivalent would be building a To make the paragraph most logical, sentence 4
skyscraper over 8,000 feet tall, but the tallest should be placed
skyscraper in the world is not even 3,000 feet tall. A) where it is now.
[4] The enormous mounds support local ecosystems, B) after sentence 1.
allowing water to penetrate deep into the ground and C) before sentence 3.
creating moist oases where plants can flourish in D) after sentence 5.

Here, the answer is (A) because sentence [4] supports the idea of the previous sentence "the mounds has the
same height as the skyscraper" by saying "The enormous mounds". Moreover, it introduces the next sentence
by providing a benefit for the mounds "plants can flourish in arid terrain" which is supported by another
benefit in the next sentence "Animals also benefit from the presence of the mounds".

Example 2:

[1] One of the big reasons behind workers’ lack of companies continue to demand long hours from
sleep is the work itself. [2] To combat the problem workers, and managers should champion napping as
of sleep deprivation in a demanding work a means to keep employees happy, healthy, and
environment, some companies have begun allowing functional.
workers to take naps. [3] The hours the average To make this paragraph most logical, sentence 3
American spend working have increased should be placed
dramatically since the 1970s, making it hard for A) where it is now.
many workers to get a good night’s sleep. [4] B) before sentence 1.
Although employees who sleep on the job are often C) after sentence 1.
considered lazy and unproductive, napping in the D) after sentence 4.
workplace has been shown to improve workers’
efficiency and quality of life. [5] As long as

Top Score
GET READY TO STUDY
Chapter 11 Data Analysis
Here, the answer is (C) because sentence [3] says that increasing working hours made it difficult to the
worker to sleep well at night, the idea which supports the idea in the sentence [1] "One of the big reasons
behind workers’ lack of sleep is the work itself"
(A) is incorrect because sentence [3] in this place will INTERRUPT the discussion of "taking nap" in
sentence [2] and in sentence [4].

B) Writer Consideration:
The writer wants to add the following sentence to the paragraph.
……………………………………………………………………….
……………………………………………………………………….
The best placement for the sentence is immediately
A) before sentence 1. "The given sentence must set up sentence 1"
B) after sentence 1. "The given sentence must support sentence 1 OR set up sentence 2"
C) after sentence 2. "The given sentence must support sentence 2 OR set up sentence 3"
D) after sentence 3. "The given sentence must support sentence 3 OR set up sentence 4"

Example 1:
[1] Thus, even though I already had all the The writer wants to add the following sentence to
equipment I needed in my home office, I decided to the paragraph.
try using a coworking space in my city. [2] Because
After filling out a simple registration form and
I was specifically interested in coworking’s reported
taking a quick tour of the facility, I took a seat at a
benefits related to creativity, I chose a facility that
table and got right to work on my laptop.
offered a bright, open work area where I wouldn’t
be isolated. [3] Throughout the morning, more The best placement for the sentence is immediately
people appeared. [4] Periods of quiet, during which A) before sentence 1.
everyone worked independently, were broken up B) after sentence 1.
occasionally with lively conversation. C) after sentence 2.
D) after sentence 3.

Here, the answer is (C) because the given sentence talks about a facility and the registration to work inside
the idea which supports the idea in sentence [2] "I chose a facility that offered a bright, open work area
where I wouldn’t be isolated." But the given sentence doesn't support or set up any other sentences.

Top Score
GET READY TO STUDY
Chapter 11 Data Analysis

Example 2:
[1] Perhaps the epitome of traditional methodology work that dictionary founder Frederic G. Cassidy
is the Dictionary of American Regional English, had expected to be finished by 1976 was not, in fact,
colloquially known as DARE. [2] Its fifth and final completed in his lifetime. [6] The wait did not
alphabetical volume—ending with “zydeco”— dampen enthusiasm among scholars. Scholars
released in 2012, the dictionary represents decades consider the work a signal achievement in
of arduous work. [3] Over a six-year period from linguistics.
1965 to 1970, university graduate students
conducted interviews in more than a thousand To improve the cohesion and flow of this paragraph,
communities across the nation. [4] Their goal was to the writer wants to add the following sentence.
determine what names people used for such Data gathering proved to be the quick part of
everyday objects and concepts as a submarine the project.
sandwich (a “hero” in New York City but a The sentence would most logically be placed after
“dagwood” in many parts of Minnesota, Iowa, and A) sentence 2.
Colorado) and a heavy rainstorm (variously a “gully B) sentence 3.
washer,” “pour-down,” or “stump mover”). [5] The C) sentence 4.
D) sentence 5.

C) Paragraph Movement:
To make the passage most logical, paragraph 2 should be placed
A) where it is now. "First sentence of paragraph 2 must match the last sentence of paragraph 1
OR last sentence of paragraph 2 must match the first sentence of paragraph 3"
B) after paragraph 3. "First sentence of paragraph 2 must match the last sentence of paragraph 3
OR last sentence of paragraph 2 must match the first sentence of paragraph 4"
C) after paragraph 4. "First sentence of paragraph 2 must match the last sentence of paragraph 4
OR last sentence of paragraph 2 must match the first sentence of paragraph 5"
D) after paragraph 5. "First sentence of paragraph 2 must match the last sentence of paragraph 5
OR last sentence of paragraph 2 must match the first sentence of paragraph 6"

Top Score
GET READY TO STUDY
Chapter 11 Data Analysis

—1— —4—
The first time I visited the Art Institute of The plainer rooms are more sparsely
Chicago, I expected to be impressed by its famous furnished. Their architectural features, furnishings,
large paintings. On one hand, I couldn’t wait to
and decorations are just as true to the periods they
view painter, Georges Seurat’s, 10-foot-wide A
Sunday Afternoon on the Island of La Grande Jatte represent. One of my favorite rooms in the whole
in its full size. It took me by surprise, then, when my exhibit, in fact, is an 1885 summer kitchen. The
favorite exhibit at the museum was one of its tiniest: room is simple but spacious, with a small sink and
the Thorne Miniature Rooms. counter along one wall, a cast-iron wood stove and
—2— some hanging pots and pans against another wall,
Viewing the exhibit, I was amazed by the and a small table under a window of the third wall.
intricate details of some of the more ornately Aside from a few simple wooden chairs placed near
decorated rooms. I marveled at a replica of a salon the edges of the room, the floor is open and
(a formal living room) dating back to the reign of obviously well worn.
French king Louis XV. Built into the dark paneled —5—
walls are bookshelves stocked with leather-bound As I walked through the exhibit, I overheard
volumes. The couch and chairs, in keeping with the a visitors’ remark, “You know, that grandfather
style of the time, are characterized by elegantly clock actually runs. Its glass door swings open, and
curved arms and legs, they are covered in luxurious the clock can be wound up.” Dotted with pin-sized
velvet. A dime-sized portrait of a French aristocratic knobs, another visitor noticed my fascination with a
woman hangs in a golden frame. tiny writing desk and its drawers. “All of those little
—3— drawers pull out. And you see that hutch? Can you
This exhibit showcases sixty-eight miniature believe it has a secret compartment?” Given the
rooms inserted into a wall at eye level. Each exquisite craftsmanship and level of detail I’d
furnished room consists of three walls; the fourth already seen, I certainly could.
wall is a glass pane through which museum goers
To make the passage most logical, paragraph 2
observe. The rooms and their furnishings were
should be placed
painstakingly created to scale at 1/12th their actual
A) where it is now.
size, so that one inch in the exhibit correlates with
B) after paragraph 3.
one foot in real life. A couch, for example, is seven
C) after paragraph 4.
inches long, and that is based on a seven-foot-long
D) after paragraph 5.
couch. Each room represents a distinctive style of
European, American, or Asian interior design from
the thirteenth to twentieth centuries.
Here, the answer is (B) because the first sentence of paragraph 2 "Viewing the exhibit, I was amazed by the
intricate details of some of the more ornately decorated rooms" matches with the last sentence of paragraph
3 "Each room represents a distinctive style of European, American, or Asian interior design from the
thirteenth to twentieth centuries." As both are talking about rooms in an exhibit with some distinctive
decorations and amazing styles. Moreover, the first sentence of paragraph 3 "This exhibit showcases sixty-
eight miniature rooms inserted into a wall at eye level" supports the last sentence of paragraph 1 "It took me
by surprise, then, when my favorite exhibit at the museum was one of its tiniest: the Thorne Miniature
Rooms" as both talk about the Miniature Rooms.

Top Score
GET READY TO STUDY
Chapter 11 Data Analysis

Exercise 2
[1] The main environmental problem caused by the oxygen content of streams and rivers as it
production of Greek yogurt is the creation of acid decomposes. [6] Yogurt manufacturers, food
whey as a by-product. [2] Because it requires up to scientists; and government officials are also working
four times more milk to make than conventional together to develop additional solutions for reusing
yogurt does, Greek yogurt produces larger amounts whey. 1 .
of acid whey, which is difficult to dispose of. [3] To
address the problem of disposal, farmers have found . 1 . To make this paragraph most logical, sentence 5
a number of uses for acid whey. [4] They can add it should be placed
to livestock feed as a protein supplement, and A) where it is now.
people can make their own Greek-style yogurt at B) after sentence 1.
home by straining regular yogurt. [5] If it is C) after sentence 2.
improperly introduced into the environment, acid- D) after sentence 3.
whey runoff can pollute waterways, depleting the

[1] Box’s research is important because the fires of contributing to the melting of the ice sheet. [6]
2012 may not be a one-time phenomenon. [2] Members of the public will be able to track his
According to scientists, rising Arctic temperatures team’s progress—and even help fund the
are making northern latitudes greener and thus more expedition—through a website Box has created. 2 .
fire prone. [3] The pattern Box observed in 2012
. 2 . To make this paragraph most logical, sentence 4
may repeat itself, with harmful effects on the Arctic
should be placed
ecosystem. [4] Box is currently organizing an
A) where it is now.
expedition to gather this crucial information. [5] The
B) after sentence 1.
next step for Box and his team is to travel to
C) after sentence 2.
Greenland to perform direct sampling of the ice in
D) after sentence 5.
order to determine just how much the soot is

[1] 1-MCP works by limiting a fruit’s production of respond as well to 1 MCP treatment. [6] Take
ethylene, a chemical that causes fruit to ripen and Bartlett pears, for instance: unless they are treated
eventually rot. [2] While 1-MCP keeps apples firm with exactly the right amount of 1-MCP at exactly
and crisp for months, it also limits their scent the right time, they will remain hard and green until
production. [3] This may not be much of a problem they rot, and consumers who experience this will be
with certain kinds of apples that are not naturally unlikely to purchase them again. . 3 .
very fragrant, such as Granny Smith, but for apples
that are prized for their fruity fragrance, such as . 3 . To make this paragraph most logical, sentence 4
McIntosh, this can be a problem with consumers, should be placed
who will reject apples lacking the expected aroma. A) where it is now.
[4] But some fruits do not respond as well to 1-MCP B) after sentence 1.
as others do, and some even respond adversely. [5] C) after sentence 2.
Furthermore, some fruits, particularly those that D) after sentence 5.
naturally produce a large amount of ethylene, do not

Top Score
GET READY TO STUDY
Chapter 11 Data Analysis

[1] The State Hermitage Museum in St. . 4 . To make this paragraph most logical,
Petersburg, one of Russia’s greatest art museums, sentence 5 should be placed
has long had a productive partnership with a much A) where it is now.
loved animal: the cat. [2] For centuries, cats have B) after sentence 1.
guarded this famous museum, ridding it of mice, C) after sentence 3.
rats, and other rodents that could damage the art, D) after sentence 6.
not to mention scared off visitors. [3] Peter the
Great introduced the first cat to the Hermitage in
the early eighteenth century. [4] Later Catherine
the Great declared the cats to be official guardians
of the galleries. [5] Continuing the tradition,
Peter’s daughter Elizaveta introduced the best and
strongest cats in Russia to the Hermitage. [6]
Today, the museum holds a yearly festival
honoring these faithful workers. . 4 .

[1] One significant benefit of 3-D printing as well as its preciousness and fragility. [5] In many
technology is its ability to create scale reproductions cases, scientists had to rearrange bones virtually,
of fossils. [2] But now 3-D scale models can be using artists’ renderings. 5 .
rearranged with ease, which is a huge boon to
scientists. [3] A team led by Drexel University . 5 . To make this paragraph most logical, sentence 2
professor Kenneth Lacovara is making models of should be placed
dinosaur bones one-tenth the bones’ original sizes 5 A) where it is now.
in order to learn how they fit together when the B) before sentence 1.
animals were alive. [4] In the past, such research C) after sentence 4.
was limited by the weight and bulk of the fossils D) after sentence 5.

[1] One innovative playwright has woven the to attend the performance. [6] By subverting the
deficiencies of the system into her crowdfunding presumption that money used for her project is an
model. [2] Though the price for her tickets was altruistic donation, the playwright showed that 31
higher than that of tickets for comparable shows, it our work has monetary value to those who enjoy it.
was still affordable to most theatergoers—and 6.
reflected the real cost of the performance. [3] She . 6 . To make this paragraph most logical, sentence 2
presented the total cost for producing her play on a should be placed
crowdfunding site. [4] Then she divided the total A) where it is now.
cost by the number of people she expected to attend B) after sentence 3.
the performance. [5] The result of the calculation C) after sentence 4.
was the minimum donor price, and only donors who D) after sentence 5.
paid at least the minimum ticket price were allowed

Top Score
GET READY TO STUDY
Chapter 11 Data Analysis
[1] The recent precipitous decline of print survive, investigative journalism must continue to
journalism as a viable profession has exacerbated adapt to the digital age. 7 .
long-held concerns about the state of investigative
reporting in the United States. [2] Facing lower print 7 . For the sake of the logic and cohesion of the
circulation and diminished advertising revenue, paragraph, sentence 3 should be
many major newspapers have reduced or eliminated A) placed where it is now.
investigative resources. [3] Newspapers, the B) placed before sentence 1.
traditional nurturing ground for investigative C) placed after sentence 1.
journalism, have been hit especially hard by the D) DELETED from the paragraph.
widespread availability of free news online. [4] To

[1] A report by the Space Foundation estimated that testimony before the US Senate: “For . . . a penny
NASA contributed $180 billion to the economy in on a dollar—we can transform the country from a
2005. [2] More than 60 percent of the contribution 8 sullen, dispirited nation, weary of economic
coming from commercial goods and services created struggle, to one where it has reclaimed its twentieth-
by companies using space-related technology. [3] century birthright to dream of tomorrow.” 8 .
This translates as excellent returns from an agency
that received approximately 17.7 billion in tax 8 . To make this paragraph most logical, sentence 1
dollars in 2014. [4] This investment by taxpayers should be placed
enhances not only the national economy but also the A) where it is now.
United States’ competitiveness in the international B) after sentence 2.
market. [5] Moreover, the benefits of NASA C) after sentence 3.
funding extend beyond the purely economic, as D) after sentence 4.
astrophysicist Neil deGrasse Tyson indicated in his

[1] The library is working hard to publicize the The writer plans to add the following sentence to
project and encourage the public to try the recipes. this paragraph.
[2] It has formed a club dedicated to cooking The judges reported that the entries were
manuscript recipes. [3] Some recipes don’t fare well delicious.
in the twenty-first century (one club member called
her 1800s gingerbread a “molasses-laden brick”), 9 . To make this paragraph most logical, the
while others had worked just fine. [4] In another sentence
instance of library outreach, a competition at the should be placed
2013 Iowa State Fair, contestants baked desserts in A) after sentence 1.
three categories— almond cheesecake, summer B) after sentence 2.
mince pie, and Marlborough pie—using recipes C) after sentence 3.
from the Szathmary collection. 9 . D) after sentence 4.

Top Score
GET READY TO STUDY
Chapter 11 Data Analysis
[1] A four-year study by a team of Canadian scientists returned to the nests many times over nine
scientists, headed by student Laura McKinnon of the days to check how many eggs remained in the nests.
Université du Québec, provide evidence in support [6] A nest was said to have survived if, at the end of
of this hypothesis. [2] The scientists created the nine days, it contained at least one undisturbed
artificial nests that resembled a typical shorebird’s quail egg. 10.
nest. [3] Then each year, during the shorebirds’ 10. To make this paragraph most logical, sentence 5
breeding season, forty of the nests were placed in should be placed
each of seven locations that ranged in latitude from A) where it is now.
the low Arctic to the high Arctic. [4] Each nest had B) after sentence 1.
been baited with four quail egg’s, which are similar C) after sentence 2.
in size and shape to a shorebird’s eggs. [5] The D) after sentence 6.

[1] It used to be that a move to a metropolis such as ambitious young people getting by on “pluck and
New York City was an inevitable step for aspiring luck” in the big city were commonplace. [7] These
artists. [2] Back when geography was everything, an days, however, they are more fiction than fact. 11.
artist had to get her painting, song, poem, or dance
in front of as large an audience as possible. [3] To 11. To make this paragraph most logical, sentence 3
some degree, these tales may have been true. [4] should be placed
That was much easier in a city with a teeming A) where it is now.
population. [5] Geographical proximity helped B) after sentence 1.
artists meet other artists, be inspired by them, and C) after sentence 4.
compete with them. [6] Stories of talented, D) after sentence 6.

[1] Most students learn the trade through a union, which create supportive local communities
four-year apprenticeship with experienced and advocates for the rights and welfare of all its
boilermakers. [2] Compared to liberal arts members. [6] Local unions foster the development
universities or colleges, boilermakers receive more of young boilermakers by sponsoring many of the
specialized, vocational training. [3] Students gain aforementioned apprenticeships. 12.
practical experience and hands-on knowledge
through working on projects with advanced 12. To make this paragraph most logical, sentence 4
professionals, as well as putting in a minimum of should be placed
144 classroom hours per year. [4] In class, students A) where it is now.
learn about the physical and chemical properties of B) before sentence 1.
the liquids and gases with which they will be C) after sentence 5.
working. [5] Once they are prepared to join the D) after sentence 6.
workforce, they can join the boilermakers’

Top Score
GET READY TO STUDY
Chapter 11 Data Analysis

Writer considerations:

A. Deleting Sentence:
The writer is considering deleting the underlined sentence.
Should the writer do this?
A) Yes / Deleted because NEGATIVE REASON
B) Yes / Deleted because ANOTHER NEGATIVE REASON
C) No / Kept because POSITIVE REASON
D) No / Kept because ANOTHER POSITIVE REASON

B. Adding Sentence:
The writer is considering adding the following sentence.
…………………………………………………
Should the writer make this addition here?

A) Yes because POSITIVE REASON


B) Yes because ANOTHER POSITIVE REASON
C) No because NEGATIVE REASON
D) No because ANOTHER NEGATIVE REASON

Positive key words: Adding: Yes / Deleting: No


1- Transition / Supports / logically follows / Reinforces / Acknowledges / Continues the explanation.
*Adding: We will choose the answer Yes which contains one of these words when the added sentence
related to the PREVIOUS sentence.
*Deleting: We will choose the answer No which contains one of these words when the added sentence
related to the PREVIOUS sentence.
Example:
Studies have offered several possible reasons that . 3 . At this point, the writer is considering adding
the following sentence.
bees are vanishing. One reason that is often cited is
Prolonged exposure to neonicotinoids has been
the use of pesticides called neonicotinoids, which shown to increase bees’ vulnerability to disease and
parasitic mites.
are absorbed by plants and linger much longer than
Should the writer make this addition here?
do topical pesticides. . 3 . Chemicals such as A) Yes, because it provides support for the claim
made in the previous sentence.
herbicides and fungicides may also play a role,
B) Yes, because it introduces a new idea that will
contaminating the pollen that bees typically feed on become important later in the passage.
C) No, because it would be better placed elsewhere
and inhibiting healthy insect maturation.
in the passage.
D) No, because it contradicts the main idea of the
passage.
Here, the added sentence is talking about the same idea of the previous sentence, so the answer (A) is the
best answer.

Top Score
GET READY TO STUDY
Chapter 11 Data Analysis

2- Introduces / Sets up:


*Adding: We will choose the answer Yes which contains one of these words when the added
sentence related to the NEXT sentence.
*Deleting: We will choose the answer No which contains one of these words when the added
sentence related to the NEXT sentence.
Example:
Yogurt manufacturers, food scientists, and . 6 . The writer is considering deleting the
government officials are also working together to underlined sentence.
develop additional solutions for reusing whey. Should the writer do this?
. 6 . Though these conservation methods can be A) Yes, because it does not provide a transition
costly and time-consuming, they are well worth the from the previous paragraph.
effort. Nutritionists consider Greek yogurt to be a B) Yes, because it fails to support the main
healthy food: it is an excellent source of calcium argument of the passage as introduced in the
and protein, serves as a digestive aid, and contains first paragraph.
few calories in its unsweetened low- and non-fat C) No, because it continues the explanation of how
forms. acid whey can be disposed of safely.
D) No, because it sets up the argument in the
paragraph for the benefits of Greek yogurt.
Here, we find that the sentence is related to the next sentence which is talking about the benefits of
the Greek yogurt: it is an excellent source of calcium and protein, serves as a digestive aid, and contains
few calories in its unsweetened low- and non-fat forms. So, the best answer here is (D).
3- Examples: when the answer has this word and the sentence takes that structure (…….., ……..,
and ……..), the answer will be correct.
Example:
The share of library materials that is in nonprint . 1 . At this point, the writer is considering adding
the following information.
formats . 1 . is increasing steadily; in 2010, at least
—e-books, audio and video materials, and online
18.5 million e-books were available for them to journals—
circulate. As a result, librarians must now be Should the writer make this addition here?
proficient curators of electronic information, A) Yes, because it provides specific examples of
the materials discussed in the sentence.
compiling, catalog, and updating these collections.
B) Yes, because it illustrates the reason for the
But perhaps even more importantly, librarians increase mentioned later in the sentence.
function as first responders for their communities’ C) No, because it interrupts the flow of the
sentence by supplying irrelevant information.
computer needs.
D) No, because it weakens the focus of the passage
by discussing a subject other than librarians

Here, we find the the added sentence is containing example of non-print formats, so the best answer
is (A).

Top Score
GET READY TO STUDY
Chapter 11 Data Analysis

This example shows for us that the answer (B) is not a correct answer because the added sentence
doesn't give reason as it doesn't contain one of the words: because, as, for …etc.

4- Supply / explain / provide / shows / indicate / demonstrates / Define:


*Adding: We will choose the answer Yes If the choice has one of these words followed by the
same information in the added sentence.
*Deleting: We will choose the answer No If the choice has one of these words followed by the
same information in the added sentence.
Example:
Such a proposition may seem counterintuitive, but, . 4 . At this point, the writer is considering adding
in fact, allowing employees to nap could save the following sentence.
Even fifteen-minute power naps improve alertness,
companies hours of lost productivity. Studies reveal
creativity, and concentration.
that napping improves memory and boosts Should the writer make this addition here?
wakefulness for the remainder of the day. . 4 . A) Yes, because it demonstrates that the benefits of
napping can be gained without sacrificing large
Napping can also have a positive effect on mood
amounts of work time.
and overall job satisfaction, while constant B) Yes, because it explains the methodology of the
drowsiness reduces reaction time and hampers one’s studies mentioned in the previous sentence.
C) No, because a discussion of the type of nap
ability to concentrate.
workers take is not important to the writer’s
main point in the paragraph.
D) No, because it contradicts the writer’s
discussion of napping in the previous sentences.

In the answer (B) the word "explains" followed by the idea: the methodology of the studies mentioned in the
previous sentence, but the added sentence doesn't contain the same information. However, in the answer (A)
the word "demonstrates" followed by the idea: the benefits of napping can be gained without sacrificing
large amounts of work time, which matches to the idea of the added sentence. So, the best answer here is (A)

Top Score
GET READY TO STUDY
Chapter 11 Data Analysis

Negative Key Words:


1- Blurs / Undermines / Weakens the focus / Contradicts / Distracts / Detracts / doesn't related:
*Adding: We will choose the answer No which contains one of these words when the added
sentence doesn't related to the main idea of the paragraph.
*Deleting: We will choose the answer Yes which contains one of these words when the added
sentence doesn't related to the main idea of the paragraph.

Law and business specifically benefit from the .42. At this point, the writer is considering adding
complicated theoretical issues raised in the study of the following sentence.
The ancient Greek philosopher Plato, for example,
philosophy, but philosophy can be just as useful in
wrote many of his works in the form of dialogues.
engineering or any field requiring complex analytic Should the writer make this addition here?
skills. .42. A) Yes, because it reinforces the passage’s main
point about the employability of philosophy
majors.
B) Yes, because it acknowledges a common
counterargument to the passage’s central claim.
C) No, because it blurs the paragraph’s focus by
introducing a new idea that goes unexplained.
D) No, because it undermines the passage’s claim
about the employability of philosophy majors.

Here, the added sentence is talking about the philosopher Plato and his way to write his works, the
idea which is not related to the paragraph which talking about the importance of the philosophy for
many majors, so the best answer is (C).
2- Repeats / Reformulates information:
*Adding: We will choose the answer No which contains information repeated in the same passage
or paragraph.
*Deleting: We will choose the answer Yes which contains information repeated in the same
passage or paragraph.

3- Interrupt:
*Adding: We will choose the answer No which contains information doesn't related to the previous
or next sentence.
*Deleting: We will choose the answer Yes which contains information doesn't related to the
previous or next sentence.

Top Score
GET READY TO STUDY
Chapter 11 Data Analysis

Example:
I marveled at a replica of a salon (a formal living . 8 . At this point, the writer is considering adding
the following sentence.
room) dating back to the reign of French king
Some scholars argue that the excesses of King
Louis XV. . 8 . Built into the dark paneled walls Louis XV’s reign contributed significantly to the
are bookshelves stocked with leather-bound conditions that resulted in the French Revolution.
volumes. Should the writer make this addition here?
A) Yes, because it provides historical context for
the Thorne Miniature Rooms exhibit.
B) Yes, because it explains why salons are often
ornately decorated.
C) No, because it interrupts the paragraph’s
description of the miniature salon.
D) No, because it implies that the interior
designer of the salon had political
motivations.

Here, the added sentence is talking about the rule of the King Louis XV whose reign was the main
reason of the French Revolution, while the previous sentence is describing the replica salon, and the
next sentence complete the description, which results to the interruption of the description of the
replica salon, so the best answer is (C).
Doesn't / Fails: these words convert all POSITIVE words to NEGATIVE words.

Top Score
GET READY TO STUDY
Chapter 11 Data Analysis

Exercise 3
In Chicago showed that inadequate natural light . 1 . At this point, the writer is considering adding
could result in eye strain, headaches, and fatigue, as the following sentence.
Workers in offices with windows sleep an average
well as interference with the body’s circadian
of 46 minutes more per night than workers in offices
rhythms. . 1 . Circadian rhythms, which are without windows.
controlled by the body's biological clocks, influence Should the writer make this addition here?
A) Yes, because it supplies quantitative data that
body temperature, hormone release, cycles of sleep
will be examined in the rest of the paragraph.
and wakefulness, and other bodily functions. B) Yes, because it explains the nature of the bodily
functions referred to in the next sentence.
C) No, because it interrupts the discussion of
circadian rhythms.
D) No, because it does not take into account
whether workers were exposed to sunlight
outside the office.

His restaurants were immediately successful, but . 2 . The writer is considering deleting the previous
Harvey was not content to follow conventional sentence. Should the writer make this change?
A) Yes, because it introduces information that is
business practices. . 2 . Although women did not
irrelevant at this point in the passage.
traditionally work in restaurants in the nineteenth B) Yes, because it does not logically follow from
century, Harvey decided to try employing women as the previous paragraph.
C) No, because it provides a logical introduction to
waitstaff.
the paragraph.
D) No, because it provides a specific example in
support of arguments made elsewhere in the
passage.

Paleontologists are using modern technology to . 3 . At this point, the writer is considering adding
gain a greater understanding of the distant past. the following sentence.
With the aid of computed tomography (CT) Fossils provide paleontologists with a convenient
way of estimating the age of the rock in which the
scanning and 3-D printing, researchers are able to
fossils are found.
create accurate models of prehistoric fossils. 3 . Should the writer make this addition here?
These models have expanded researchers’ A) Yes, because it supports the paragraph’s
knowledge of ancient species and promise to argument with an important detail.
advance the field of paleontology in the years to B) Yes, because it provides a logical transition
come. from the preceding sentence.
C) No, because it is not directly related to the main
point of the paragraph.
D) No, because it undermines the main claim of
the paragraph.

Top Score
GET READY TO STUDY
Chapter 11 Data Analysis
Transportation planners work closely with a number . 4 . The writer is considering deleting the
underlined sentence.
of community stakeholders, such as government
Should the sentence be kept or deleted?
officials and other interested organizations and A) Kept, because it provides supporting evidence
individuals. For instance, representatives from the about the benefits of walking.
B) Kept, because it provides an additional example
local public health department might provide input
of a community stakeholder with whom
in designing a network of trails and sidewalks to transportation planners work.
encourage people to walk more. 4 . According to C) Deleted, because it blurs the paragraph’s focus
on the community stakeholders with whom
the American Heart Association, walking provides
transportation planners work.
numerous benefits related to health and well-being. D) Deleted, because it doesn’t provide specific
examples of what the numerous benefits of
walking are.

In 1990, chef Louis Szathmary, a voracious . 5 . The writer is considering deleting the
collector of cookbooks, donated approximately underlined portion (ending the sentence with a
period).
20,000 culinary artifacts to the University of Iowa
Should the writer make this deletion?
library. . 5 . The gift included more than 100 A) Yes, because the underlined portion detracts
manuscript recipe books —collections of recipes from the paragraph’s focus on the Szathmary
collection.
handwritten by the people who used them.
B) Yes, because the information in the underlined
portion is provided in the previous sentence.
C) No, because the underlined portion defines a
term that is important to the passage.
D) No, because the underlined portion gives an
example of a particular culinary artifact.

What was less well-known, until recently at least, . 6 . At this point, the writer is considering adding
the following information.
was how this relationship among sea otters, sea
since the start of the Industrial Revolution, resulting
urchins, and kelp forests might help fight global in a rise in global temperatures
Should the writer make this addition here?
warming. The amount of carbon dioxide in the
A) Yes, because it establishes the relationship
atmosphere has increased 40 percent . 6 .. between the level of carbon dioxide in the
atmosphere and global warming.
B) Yes, because it explains the key role sea otters,
sea urchins, and kelp forests play in combating
global warming.
C) No, because it contradicts the claim made in the
previous paragraph that sea otters help keep
kelp forests healthy.
D) No, because it mentions the Industrial
Revolution, blurring the focus of the paragraph.
Top Score
GET READY TO STUDY
Chapter 11 Data Analysis
Tyson’s expansive vision for the agency hints at . 7 . At this point, the writer is considering adding
another mission of NASA’s, illuminated in this the following sentence.
observation by Apollo 14 astronaut Edgar Mitchell: In addition, NASA has facilities in Washington,
“You develop an instant global consciousness, a DC, Florida, Texas, California, and other states.
people orientation, an intense dissatisfaction with Should the writer make this addition here?
the state of the world, and a compulsion to do A) Yes, because it serves as a counterargument to
something about it.” . 7 . With world population the quotation from astrophysicist Neil deGrasse
topping seven billion, humanity is in need of some Tyson.
perspective. Therefore, we should continue to B) Yes, because it reinforces the passage’s point
support NASA not only for practical reasons but about the importance of NASA’s work.
also because it is a necessary vehicle for increasing C) No, because it undermines the passage’s claim
our awareness of how we can fulfill our about the economic benefits of NASA’s work.
responsibilities to the planet and each other. D) No, because it blurs the paragraph’s focus by
introducing information that does not support the
paragraph’s claim about the importance of
NASA’s work.

There are also respected traditions about when these . 8 . At this point, the writer is considering adding
different coffee drinks should be consumed: while the following sentence.
cappuccino is a popular morning drink, espresso, These espresso-based coffee drinks have
either plain or diluted with water, is usually the become increasingly popular in the United
drink of choice for coffee drinkers in Italy in the States over
afternoon and evening. . 8 . the past several decades.
Should the writer make this addition here?
A) Yes, because it adds force to the writer’s
argument about the popularity of coffeehouses in
Italy.
B) Yes, because it elaborates on the statement in the
previous sentence about different types of drinks.
C) No, because it suggests that coffee drinking is
not as popular in Italy as the writer claims it is.
D) No, because it digresses from the main topic of
how coffee is regarded in Italy.

Henderson had been working with musician and A) Yes, because it mentions the musical instrument
composer Don Redman to develop arrangements of that was associated with Armstrong.
songs that used a call-and-response . 9 . structure. B) Yes, because it clarifies a term used to describe
Redman’s arrangements.
. 9 . The writer is considering revising the C) No, because it interrupts the discussion of
underlined portion to the following. Redman’s arrangements with irrelevant
structure that, for example, featured a melody information.
played by the saxophone section followed by an D) No, because it diverges from the paragraph’s
answer from the trumpet section. point about Henderson.
Should the writer make this revision?
Top Score
GET READY TO STUDY
Chapter 11 Data Analysis

Answers
Exercise 1 Exercise 2 Exercise 3
1- D 1- C 1- C
2- B 2- D 2- C
3- B 3- B 3- C
4- C 4- C 4- C
5- A 5- D 5- C
6- C 6- D 6- A
7- A 7- C 7- D
8- B 8- A 8- D
9- D 9- D 9- B
10- C 10- A
11- D 11- D
12- D 12- A
13- B

Top Score
GET READY TO STUDY
Chapter 1 Evidence-based Questions

Evidence-based Questions
a. Paired questions:
In this type of questions the test maker gives you a question asking about information in the passage without
mentioning the part where the information exists in the passage:
Ex: Which reaction does Akira most fear from Chie?
A) She will consider his proposal inappropriate.
B) She will mistake his earnestness for immaturity.
C) She will consider his unscheduled visit an
imposition.
D) She will underestimate the sincerity of his
emotions.

For sure to answer such a question you need to read the passage until you find the answer. But the test maker
unintentionally will help you rather than trick you by giving you a question is related to this one and asks
you that which lines did you find your answer? As shown below:
Which choice provides the best evidence for the
answer to the previous question?
A) Line 33 (“His voice . . . refined”)
B) Lines 49-51 (“You . . . mind”)
C) Lines 63-64 (“Please . . . proposal”)
D) Lines 71-72 (“Eager . . . face”)

So, it became easier because the test maker gave us only four parts of the passage to find my answer of the
first question. Follow these steps:
1- Read carefully each answer of the first question.
2- Read carefully each reference given in the second question.
3- Match perfectly one answer of the first question with one reference of the second question.
Line 33: "His voice was soft, refined.”
A) She will consider his proposal inappropriate. Lines 49-51: "You know how children speak so
earnestly, so hurriedly, so endearingly about things
B) She will mistake his earnestness for immaturity.
that have no importance in an adult’s mind?"
C) She will consider his unscheduled visit an Lines 63-64: “We have an understanding. Please
imposition. don’t judge my candidacy by the unseemliness of
this proposal.
D) She will underestimate the sincerity of his
emotions. Lines 71-72: "Eager to make his point, he’d been
looking her full in the face."
As we read the references given in the second question, we will find that only one reference can match only
one answer of the first question. If you read the lines 63 – 64 you will find that Akira is begging Chie to
don’t judge his demand as unseemly thing or inappropriate, so the answer A of the first question matches the
answer C in the second question.

Top Score
GET READY TO STUDY
Chapter 1 Evidence-based Questions

This kind of questions comes in two forms:

1- Evidence-based questions (With specific Idea):


In this form of evidence-based questions, the test maker will ask you about a specific Idea in the first
question. So, in the references mentioned in the second question, you must find this specific idea first, and
then you must match the reference with one answer of the answers given in the first question.

The mental consequences of our online 2009, prominent developmental psychologist Patricia
info-crunching are not universally bad. Greenfield reviewed more than 40 studies of the
Certain cognitive skills are strengthened by our use effects of various types of media on intelligence and
of computers and the Net. These tend to involve learning ability. She concluded that “every medium
5 more primitive mental functions, such as hand-eye 30 develops some cognitive skills at the expense of
coordination, reflex response, and the processing of others.” Our growing use of the Net and other
visual cues. One much-cited study of video gaming screen-based technologies, she wrote, has led to the
revealed that after just 10 days of playing action “widespread and sophisticated development of
games on computers, a group of young people had visual-spatial skills.” But those gains go hand in hand
10 significantly boosted the speed with which they 35 with a weakening of our capacity for the kind of
could shift their visual focus between various “deep processing” that underpins “mindful
images and tasks. knowledge acquisition, inductive analysis, critical
It’s likely that Web browsing also strengthens thinking, imagination, and reflection.”
brain functions related to fast-paced problem
15 solving, particularly when it requires spotting The author of Passage 1 indicates which of the
patterns in a welter of data. A British study of the following about the use of screen-based technologies?
way women search for medical information online A) It should be thoroughly studied.
indicated that an experienced Internet user can, at B) It makes the brain increasingly rigid.
least in some cases, assess the trustworthiness and C) It has some positive effects.
20 probable value of a Web page in a matter of D) It should be widely encouraged.
seconds.
Which choice provides the best evidence for the
The more we practice surfing and scanning, the more
answer to the previous question?
adept our brain becomes at those tasks.
But it would be a serious mistake to look narrowly
A) Lines 3-4 (“Certain . . . Net”)
at such benefits and conclude that the Web is making B) Lines 23-25 (“But . . . smarter”)
25 us smarter. In a Science article published in early C) Lines 25-29 (“In a . . . ability”)
D) Lines 29-31 (“She . . . others”)

Here, the first question asks about “the use of screen-based technologies”. So, we must search for this
specific idea in the references mentioned in the second question. We will find that lines 3 – 4 only talks
about the use of screen-based technology by saying “Certain cognitive skills are strengthened by our use of
computers and the Net.” As Computer and Net are “screen-based technologies”. When we match this lines with
the answers given in the first question, we will find that the answer C only says what said in lines 3 – 4.

Top Score
GET READY TO STUDY
Chapter 1 Evidence-based Questions

2- Evidence-based questions (With General Idea):

In this form of evidence-based questions, the test maker asks you about general information exists in the
passage. So, in this case you will use only the match process to match the answer given in the first question
with the lines given in the second question because there is no specific idea to determine which reference is
the correct one.
20 Cortisol is best known as a stress hormone. When afford to be exploratory and playful. Once you spend
cortisol courses through our bodies, it prepares us to 55 a calorie on that, it’ s a calorie you can’ t use to
handle fearful situations, increasing the brain’ s grow.” The babies fed high-cortisol milk develop a
consumption of glucose and suppressing the nervous temperament, focusing their limited energy
digestive system. The cortisol in a mother’ s body on putting on weight. They grow faster, despite
25 can also end up in her milk. Babies appear to be getting less energy from their inexperienced
sensitive to the hormone as they nurse. Scientists 60 mothers.
found that drinking milk causes infants to build Cortisol in breast milk may influence human
receptors in their intestines for detecting cortisol. infants as well. But Melissa Thompson, an
The same shift doesn’ t happen when babies drink anthropologist, cautioned that the differences
30 formula. between monkeys and humans make comparisons
Among the macaques, some mothers delivered a 65 difficult. Infant monkeys, for example, cling to their
lot of cortisol to their babies, while others delivered mothers and nurse whenever they want. Human
a little. High-cortisol milk made babies put on mothers balance breast-feeding with many other
weight faster, and they were more nervous and less tasks. “ We should expect the relationship between
35 confident. To make sense of these results, the maternal stress, breast milk and infant temperament
scientists looked for factors that might determine 70 in humans to be relatively complex,” said Dr.
how much cortisol a mother produced in her milk. Thompson.
One stood out: how many other offspring she had.
mothers had high cortisol levels in their milk. The passage most strongly suggests that
40 Hormone levels were much lower in mothers who A) human breast milk is complex and not
had had about 10 babies. completely understood.
When female monkeys start having babies, they B) infant formula needs to be improved.
can’ t store as much energy in their milk. New C) mothers should try to decrease their stress and
mothers are still small, so their bodies can’ t provide cortisol levels.
45 many of the raw ingredients for milk. Their D) research findings about monkeys are irrelevant
mammary glands are also underdeveloped, so they to humans.
can’ t convert those ingredients efficiently into milk. Which choice provides the best evidence for the
Monkey mothers who have had more babies are able
answer to the previous question?
to supply new infants with more energy. Dr. Hinde
A) Lines 26-28 (“ Scientists found … cortisol” )
50 suspects that the cortisol that newer mothers give
B) Lines 35-37 (“ To make … milk” )
their babies serves as a warning that they shouldn’ t
C) Lines 48-49 (“ Monkey mothers … energy” )
expect a lot of milk, or energy. She sums up the
D) Lines 68-71 (“ We should … Thompson” )
message: “ Prioritize growth, kiddo. You can’ t really

After reading carefully all answers given in the first question, we must read carefully each reference given in the
second question to match them with the answers of the first question. We will find that the Answer A of the first
question matches the lines 68 – 71 in the answer D in the second question.

Top Score
GET READY TO STUDY
Chapter 1 Evidence-based Questions

Exercises 1
1
While everyone is busy fighting over what should be the shelves of friends, one book leading to another, back
taught in the classroom, something is being overlooked. and forth in history and across languages and cultures.
That is the state of reading, and books, and literature in our 40 After graduation, they continue to read and in the fullness
country, at this time. Why, ask yourself, is everyone so hot of time produce a new generation of readers. Oh happy
20 under the collar about what to put on the required-reading land! I wish we all lived there.
shelf? It is because, while we have been arguing so fiercely
about which books make the best medicine, the patient has
been slipping deeper and deeper into a coma. - The country described is noteworthy because
Let us imagine a country in which reading was a popular
25 voluntary activity. There, parents read books for their own
(A) parents demonstrate their enjoyment of reading
edification and pleasure and are seen by their children at (B) children learn to read at an early age
this silent and mysterious pastime. These parents also read (C) children and parents share many activities
to their children, give them books for presents, talk to them (D) writing is viewed as a valuable skill
about books, and underwrite, with their taxes, a public
30 library system that is open all day, every day. In school, - Which choice provides the best evidence for
the children study certain books together but also have an the answer to the previous question?
active reading life of their own. Years later, it may even
(A) Lines 15 – 16 ("While . . . overlooked")
be hard for them to remember if they read Jane Eyre at
home and Judy Blume1 in class or the other way around. (B) Lines 21 – 23 ("while . . . comma")
35 In college, young people continue to be assigned certain (C) Lines 24 – 27 ("Let . . . pastime")
books, but far more important are the books they discover (D) Lines 35 – 37 ("In college . . . library")
for themselves browsing in the library, in bookstores, on

2
I sat on the Lees’ red chair for the first time on anyone, I stopped analyzing the situation in such linear
May 19, 1988. Earlier that spring I had come to Merced, terms. Now, when I play the tapes late at night, I imagine
25 California, because I had heard that there were some 50 what they would sound like if I could splice them together,
misunderstandings at the county hospital between its so the voices of the Hmong and those of the American
Hmong patients and medical staff. One doctor called them doctors could be heard on a single tape, speaking a
“collisions,” which made it sound as if two different kinds common language.
of people had rammed into each other, head on, to the
30 accompaniment of squealing brakes and breaking glass. - Which of the following views can describe the
As it turned out, the encounters were messy but rarely conflict between Hmong patients and medical
frontal. Both sides were wounded, but neither side seemed staff?
to know what had hit it or how to avoid another crash. (A) Efforts to prevent conflicts are not always
I have always felt that the action most worth watching successful.
35 occurs not at the center of things but where edges meet. (B) Conflict can occur in many different guises.
I like shorelines, weather fronts, international borders. (C) In most conflicts, both parties are to blame.
These places have interesting frictions and incongruities, (D) You can learn more about two parties in conflict as
and often, if you stand at the point of tangency, you can an observer than as an involved participant.
see both sides better than if you were in the middle of either
40 one. This is especially true when the apposition is cultural. - Which choice provides the best evidence for
When I first came to Merced, I hoped that the culture of the answer to the previous question?
American medicine, about which I knew a little, and the (A) Lines 23 – 25 ("I sat . . . California")
culture of the Hmong, about which I knew nothing, would (B) Lines 27 – 30 ("One . . . glass")
somehow illuminate each other if I could position myself (C) Lines 38 – 40 ("if . . . one")
45 between the two and manage not to get caught in the (D) Lines 46 – 49 ("But . . . night")
crossfire. But after getting to know the Lees family and their
daughter’s doctors and realizing how hard it was to blame

(A) 22

Top Score
GET READY TO STUDY
Chapter 1 Evidence-based Questions

3
“I don’t want to trouble you. Normally I would consent, I become Naomi’s yoshi.* We’ll live in the
40 approach you more properly but I’ve received word House of Fuji. Without your consent, I must go to
of a position. I’ve an opportunity to go to America, as 70 America, to secure a new home for my bride.”
dentist for Seattle’s Japanese community.” Eager to make his point, he’d been looking her full
“Congratulations,” Chie said with amusement. in the face. Abruptly, his voice turned gentle. “I see
“That is an opportunity, I’m sure. But how am I I’ve startled you. My humble apologies. I’ll take no
45 involved?” more of your evening. My address is on my card. If
Even noting Naomi’s breathless reaction to the 75 you don’t wish to contact me, I’ll reapproach you in
name card, Chie had no idea. Akira’s message, two weeks’ time. Until then, good night.”
delivered like a formal speech, filled her with
maternal amusement. You know how children speak - Why does Akira say his meeting with Chie is “a matter
50 so earnestly, so hurriedly, so endearingly about of urgency” (line 32)?
things that have no importance in an adult’s mind? (A) He fears that his own parents will disapprove of
That’s how she viewed him, as a child. Naomi.
It was how she viewed Naomi. Even though (B) He worries that Naomi will reject him and marry
Naomi was eighteen and training endlessly in the arts someone else.
55 needed to make a good marriage, Chie had made no (C) He has been offered an attractive job in another
effort to find her a husband. country.
Akira blushed. (D) He knows that Chie is unaware of his feelings for
“Depending on your response, I may stay in Naomi.
Japan. I’ve come to ask for Naomi’s hand.”
60 Suddenly Chie felt the dampness of the night. - Which choice provides the best evidence for the answer
“Does Naomi know anything of your . . . to the previous question?
ambitions?” (A) Line 39 (“I don’t . . . you”)
“We have an understanding. Please don’t judge
(B) Lines 39-42 (“Normally . . . community”)
my candidacy by the unseemliness of this proposal. I
(C) Lines 58-59 (“Depending . . . Japan”)
65 ask directly because the use of a go-between takes
much time. Either method comes down to the same (D) Lines 72-73 (“I see . . . you”)
thing: a matter of parental approval. If you give your

4
Due to the constancy of the underlying measurements, The authors imply that the response of various
both field and LiDAR data could provide the needed officials to attempts to measure their countries’ carbon
65 information if they covered every hectare on Earth. But, in stock through field surveys has been
the case of field surveys, this is impossible. The surveys that A) unhelpful, because they fear that jobs for their
do exist measure a tiny amount of actual forest, and so what countries’ scientists will be lost.
might be verified is widely spaced. And to avoid fraud and B) helpful, because their countries have invested
protect landowners, many governments keep their plot significantly in technology to allow studies to
70 locations secret. Satellite LiDAR data remain sparse, expand.
providing only extrapolated, coarse-resolution carbon C) helpful, because their countries stand to benefit
estimates with very high uncertainties, and there is no from universal carbon data that the studies will
prospect of wall-toy-wall coverage in the near future. By uncover.
2020, airborne LiDAR could give us a direct measurement of D) unhelpful, because they do not make their
75 3-D forest structure for every hectare in the tropics: a countries’ land holdings readily available for
standardized database from which to build a carbon study.
economy.
Which choice provides the best evidence for the
answer to the previous question?
A) lines 63-65 (“Due ... Earth”)
B) lines 66-68 (“The surveys ... spaced”)
C) lines 68-70 (“And ... secret”)
D) lines 70-73 (“Satellite ... future”)

Top Score
GET READY TO STUDY
Chapter 1 Evidence-based Questions

5
showers. This frequent experience of gift-giving can gift-recipients, but nevertheless tend to overspend
10 engender ambivalent feelings in gift-givers. Many each time they set out to purchase a meaningful gift.
relish the opportunity to buy presents because In the present research, we propose a unique
gift-giving offers a powerful means to build stronger 40 psychological explanation for this overspending
bonds with one’s closest peers. At the same time, problem—i.e., that gift-givers equate how much they
many dread the thought of buying gifts; they worry spend with how much recipients will appreciate the
15 that their purchases will disappoint rather than gift (the more expensive the gift, the stronger a
delight the intended recipients. gift-recipient’s feelings of appreciation). Although a
Anthropologists describe gift-giving as a positive 45 link between gift price and feelings of appreciation
social process, serving various political, religious, and might seem intuitive to gift-givers, such an
psychological functions. Economists, however, offer assumption may be unfounded. Indeed, we propose
20 a less favorable view. According to Waldfogel that gift-recipients will be less inclined to base their
(1993), feelings of appreciation on the magnitude of a gift
gift-giving represents an objective waste of resources. 50 than givers assume.
People buy gifts that recipients would not choose to
buy on their own, or at least not spend as much - The authors indicate that people value gift-giving
money to purchase (a phenomenon referred to as because they feel it
25 ‘‘the deadweight loss of Christmas”). To wit, givers (A) functions as a form of self-expression.
are likely to spend $100 to purchase a gift that (B) is an inexpensive way to show appreciation.
receivers would spend only $80 to buy themselves. (C) requires the gift-recipient to reciprocate.
This ‘‘deadweight loss” suggests that gift-givers are (D) can serve to strengthen a relationship.
not very good at predicting what gifts others will
30 appreciate. That in itself is not surprising to social - Which choice provides the best evidence for the answer
psychologists. Research has found that people often to the previous question?
struggle to take account of others’ perspectives— (A) Lines 10-13 (“Many . . . peers”)
their insights are subject to egocentrism, social (B) Lines 22-23 (“People . . . own”)
projection, and multiple attribution errors. (C) Lines 31-32 (“Research . . . perspectives”)
35 What is surprising is that gift-givers have (D) Lines 44-47 (“Although . . . unfounded”)
considerable experience acting as both gift-givers and

6
little queer. We have worn private clothes so battles to fight, your rent to pay, your bazaars to
45 long. . . . But we have not come here to laugh, or to organize. That excuse shall not serve you, Madam.
talk of fashions—men’s and women’s. We are here,
on the bridge, to ask ourselves certain questions. - Woolf characterizes the questions in lines 53-57 (“For
And they are very important questions; and we have we . . . men”) as both
very little time in which to answer them. The (A) controversial and threatening.
50 questions that we have to ask and to answer about (B) weighty and unanswerable.
that procession during this moment of transition are (C) momentous and pressing.
so important that they may well change the lives of (D) provocative and mysterious.
all men and women forever. For we have to ask
ourselves, here and now, do we wish to join that - Which choice provides the best evidence for the answer
55 procession, or don’t we? On what terms shall we join to the previous question?
that procession? Above all, where is it leading us, the (A) Lines 46-47 (“We . . . questions”)
procession of educated men? The moment is short; it
(B) Lines 48-49 (“And . . . them”)
may last five years; ten years, or perhaps only a
(C) Line 57 (“The moment . . . short”)
matter of a few months longer. . . . But, you will
60 object, you have no time to think; you have your (D) Line 62 (“That . . . Madam”)

Top Score
GET READY TO STUDY
Chapter 1 Evidence-based Questions

b. Single question:
In this type of question, the test maker gives you four references from the passage and gives you an idea and
asks you to support this idea by one of these references.
EX: Which choice best supports the conclusion that
public transportation is compatible with the use
of personal electronic devices?
A) Lines 59-63 (“The . . . subways”)
B) Lines 63-67 (“Part . . . annoyances”)
C) Lines 68-70 (“Even . . . ago”)
D) Lines 77-81 (“Already . . . homes”)

The idea you want to support mostly starts after the word “that” in the head of the question:
“public transportation is compatible with the use of personal electronic devices”
So, you must find the lines which support this idea.
A) Lines 59-63 (“The “Millenials,” who reached
adulthood around the turn of the century and now
outnumber baby boomers, tend to favor cities over
suburbs, and are far more willing than their parents to
ride buses and subways.”)
B) Lines 63-67 (“Part of the reason is their ease
with iPads, MP3 players, Kindles, and smart phones: you
can get some serious texting done when you’re not
driving, and earbuds offer effective insulation from all
“public transportation is compatible with but the most extreme commuting annoyances.”)
the use of personal electronic devices” C) Lines 68-70 (“Even though there are more
teenagers in the country than ever, only ten million have
a driver’s license (versus twelve million a generation
ago).”)
D) Lines 77-81 (“Already, dwellings in older
neighborhoods in Washington, D.C., Atlanta, and Denver,
especially those near light-rail or subway stations, are
commanding enormous price premiums over suburban
homes”)
Here we will find that the lines from 63 to 67 can support the idea that one of the most important advantages of the
public transportations is to use personal electronic devices easily. So the answer is B

Top Score
GET READY TO STUDY
Chapter 1 Evidence-based Questions

Exercise 2
1
Which choice best supports the claim that Quilotoa was records of the island written on palm leaves in Old
not responsible for the Little Ice Age? Javanese, Samalas erupted catastrophically before
A) Lines 3-4 (“Identifying . . . tricky”) The end of the 13th century, devastating surrounding
B) Lines 26-28 (“Such a . . . cooling”) 40 villages—including Lombok’s capital at the time,
C) Lines 43-46 (“The researchers . . . atop the Pamatan—with ash and fast-moving sweeps of hot
volcano”) rock and gas called pyroclastic flows.
D) Lines 71-75 (“But . . . closer match”) The researchers then began to reconstruct the
formation of the large, 800-meter-deep caldera [a
About 750 years ago, a powerful volcano erupted
45 basin-shaped volcanic crater] that now sits atop the
somewhere on Earth, kicking off a centuries-long
volcano. They examined 130 outcrops on the flanks
cold snap known as the Little Ice Age. Identifying the
of the volcano, exposing sequences of pumice—ash
volcano responsible has been tricky.
hardened into rock—and other pyroclastic material.
5 That a powerful volcano erupted somewhere in
The volume of ash deposited, and the estimated
the world, sometime in the Middle Ages, is written in
50 height of the eruption plume (43 kilometers above
polar ice cores in the form of layers of sulfate
sea level) put the eruption’s magnitude at a
deposits and tiny shards of volcanic glass. These
minimum of 7 on the volcanic explosivity index
cores suggest that the amount of sulfur the mystery
(which has a scale of 1 to 8)—making it one of the
10 volcano sent into the stratosphere put it firmly
largest known in the Holocene.
among the ranks of the strongest climate-perturbing
55 The team also performed radiocarbon analyses on
eruptions of the current geological epoch, the
carbonized tree trunks and branches buried within
Holocene, a period that stretches from 10,000 years
the pyroclastic deposits to confirm the date of the
ago to the present. A haze of stratospheric sulfur
eruption; it could not, they concluded, have
15 cools the climate by reflecting solar energy back into
happened before 1257 C.E., and certainly happened
space.
60 in the 13th century.
In 2012, a team of scientists led by geochemist
It’s not a total surprise that an Indonesian volcano
Gifford Miller strengthened the link between the
might be the source of the eruption, Miller says. “An
mystery eruption and the onset of the Little Ice Age
equatorial eruption is more consistent with the
20 by using radiocarbon dating of dead plant material
apparent climate impacts.” And, he adds, with sulfate
from beneath the ice caps on Baffin Island and
65 appearing in both polar ice caps—Arctic and
Iceland, as well as ice and sediment core data, to
Antarctic—there is “a strong consensus” that this
determine that the cold summers and ice growth
also supports an equatorial source.
began abruptly between 1275 and 1300 C.E. (and
Another possible candidate—both in terms of
25 became intensified between 1430 and 1455 C.E.).
timing and geographical location—is Ecuador’s
Such a sudden onset pointed to a huge volcanic
70 Quilotoa, estimated to have last erupted between
eruption injecting sulfur into the stratosphere and
1147 and 1320 C.E. But when Lavigne’s team
starting the cooling. Subsequent, unusually large and
examined shards of volcanic glass from this volcano,
frequent eruptions of other volcanoes, as well as
they found that they didn’t match the chemical
30 sea-ice/ocean feedbacks persisting long after the
composition of the glass found in polar ice cores,
aerosols have been removed from the atmosphere,
75 whereas the Samalas glass is a much closer match.
may have prolonged the cooling through the 1700s.
That, they suggest, further strengthens the case that
Volcanologist Franck Lavigne and colleagues now
Samalas was responsible for the medieval “year
think they’ve identified the volcano in question:
without summer” in 1258 C.E.
35 Indonesia’s Samalas. One line of evidence, they note,
is historical records. According to Babad Lombok,

Top Score
GET READY TO STUDY
Chapter 1 Evidence-based Questions

2
Which choice best supports the claim that increased mass media. As news of democratic triumphs spread
political openness is a widespread, global trend? from country to country, greater access to media by
A) Line 23 (“What . . . transformation”) 40 increasingly literate populations encouraged
B) Lines 26-27 (“Poor . . . standing”) emulation. In today’s digital culture, the force of that
C) Lines 41-42 (“In today’s . . . exploded”) factor has exploded.
D) Lines 56-59 (“According . . . all”) There have been exceptions, of course—not just
countries where democracy has yet to spread but
What caused this global transformation? 45 others where it has experienced reversals.
Obviously local factors were at work, but scholar Larry Diamond, a leading scholar in this field, calls
25 Samuel Huntington noted some big forces as well. the stalling in recent years in countries like Russia,
Poor economic management by many authoritarian Venezuela, or Bangladesh a “democratic recession.”
governments eroded their popular standing. A rising Yet against this is mounting evidence that public
middle class demanded better public services, 50 attitudes have shifted. In Latin America, for example,
greater despite persistent poverty and inequality, and
participation, and eventually more political freedom. constant corruption scandals, opinion polls show
30 Western governments and activists encouraged greater confidence in civilian government than in the
dissent and held out rewards for reform, such as military.
membership in NATO or the EU or access to funds 55 Even autocracies are less autocratic today.
from international financial institutions. A newly According to one study of the world’s democratic
activist Catholic Church under Pope John Paul II electoral systems, Brunei may be the only country
35 empowered opposition in Poland, El Salvador, and where “electoral politics has failed to put down any
the Philippines. Above all, success begat success, a meaningful roots at all.
process accelerated by the new reach and speed of

3
Which choice most clearly provides information crop of corn.
indicating how some rootworms have overcome There are almost certainly genetic differences that
farmers’ efforts to eradicate them? separate the rotation-resistant rootworms from their
A) Lines 15-17 (“These . . . die”) 25 normal peers, but what are they? Researchers at the
B) Lines 18-20 (“But . . . corn”) University of Illinois have been studying the problem
C) Lines 25-28 (“Researchers . . . question”) since 2000 and, despite generating a vast mountain of
D) Lines 41-43 (“And . . . rotation”) data, have failed to find the genes in question.
Consider the western corn rootworm—a beetle “The western corn rootworm has been an enigma for
that’s a serious pest of corn in the United States. The 30 a long time,” says Manfredo Seufferheld. “This insect
adults have strong preferences for laying eggs in corn has the ability to adapt to practically all control
10 fields, so that their underground larvae hatch into a methods deployed against it, including crop rotation.
feast of corn roots. This life cycle depends on a After many years of research about the mechanisms
continuous year-on-year supply of corn. Farmers can of rotation resistance, results were mostly
use this dependency against the rootworm, by 35 inconclusive.”
planting soybean and corn in alternate years. So, Seufferheld looked elsewhere. Rather than
15 These rotations mean that rootworms lay eggs into focusing on the rootworm’s own genes, he studied
corn fields but their larvae hatch among soybean, the genes of the bacteria in its gut . . . and found
and die. some answers. The rotation-resistant varieties have
But the rootworms have adapted to this strategy 40 very different gut bacteria from the normal ones.
by reducing their strong instincts for laying eggs in And when the team killed these microbes with
20 corn. These rotation-resistant females might lay antibiotics, they severely reduced the beetle’s ability
among soybean fields, so their larvae hatch into a to cope with rotation.

Top Score
GET READY TO STUDY
Chapter 1 Evidence-based Questions

Practice
1
Within a few decades, these firms may be aluminium in lunar soil and asteroids, which could
meeting earthly demands for precious metals, such as be used in 3D printers to make spare parts or
20 platinum and gold, and the rare earth elements vital machinery. Others want to turn space dirt into
for personal electronics, such as yttrium and 45 concrete for landing pads, shelters, and roads.
lanthanum. But like the gold rush pioneers who
transformed the western United States, the first space - The author of Passage 1 indicates that space mining
miners won’t just enrich themselves. They also hope could have which positive effect?
25 to build an off-planet economy free of any bonds (A) It could yield materials important to Earth’s
with Earth, in which the materials extracted and economy.
processed from the moon and asteroids are delivered (B) It could raise the value of some precious metals on
for space-based projects. Earth.
In this scenario, water mined from other (C) It could create unanticipated technological
30 worlds could become the most desired commodity. innovations.
“In the desert, what’s worth more: a kilogram of gold (D) It could change scientists’ understanding of space
or a kilogram of water?” asks Kris Zacny of resources.
HoneyBee Robotics in New York. “Gold is useless.
Water will let you live.” - Which choice provides the best evidence for the answer
35 Water ice from the moon’s poles could be sent to to the previous question?
astronauts on the International Space Station for (A) Lines 18-22 (“Within . . . lanthanum”)
drinking or as a radiation shield. Splitting water into
(B) Lines 24-28 (“They . . . projects”)
oxygen and hydrogen makes spacecraft fuel, so
(C) Lines 29-30 (“In this . . . commodity”)
ice-rich asteroids could become interplanetary
40 refuelling stations. (D) Lines 41-44 (“Companies . . . machinery”)
Companies are eyeing the iron, silicon, and

2
A study published earlier this month suggests that, in In the second paragraph,the author implies that the
addition to making us feel connected with others, all those study shows hugs to be
hugs may have prevented us from getting sick. At first, this A) the only proven method of preventing sickness.
15 finding probably seems counterintuitive (not to mention B) less successful than social support in protecting
bizarre). You might think, like I did, that hugging hundreds people from colds and other illnesses.
of C) not conclusively effective at helping people
strangers would increase your exposure to germs and maintain good health.
therefore D) one of several ways to guard against some illnesses.
the likelihood of falling ill. But the new research
out of Carnegie Mellon indicates that feeling connected to Which choice provides the best evidence for the
20 others, especially through physical touch, protects us from answer to the previous question?
stress-induced sickness. This research adds to a large A) line 12-14(“A study...sick”)
amount of evidence for the positive influence of social B) line 16-18(“You...ill”)
support on health. C) line 18-21(“But...sickness”)
D) line 21-23(“This...health”)

Top Score
GET READY TO STUDY
Chapter 1 Evidence-based Questions

3
Honey bees are hosts to the pathogenic large fatal either directly or due to a secondary infection of
ectoparasitic mite Varroa destructor (Varroa mites). immunocompromised or nutritionally deficient bees.
These mites feed on bee hemolymph (blood) and can
kill bees directly or by increasing their susceptibility In line 42, the authors state that a certain hypothesis “can
5 to secondary infection with fungi, bacteria or viruses. best be tested by a trial.” Based on the passage, which of the
Little is known about the natural defenses that keep following is a hypothesis the authors suggest be tested in a
the mite infections under control. trial?
Pyrethrums are a group of flowering plants which A) Honeybees that are exposed to both pyrethrums and
include Chrysanthemum coccineum, Chrysanthemum mites are likely to develop a secondary infection by a
10 cinerariifolium, Chrysanthemum marschalli, and virus, a bacterium, or a fungus.
related species. These plants produce potent B) Beekeepers who feed their honeybee colonies a diet of a
insecticides with anti-mite activity. The naturally single crop need to increase the use of insecticides to
occurring insecticides are known as pyrethrums. prevent mite infestations.
A synonym for the naturally occurring pyrethrums is C) A honeybee diet that includes pyrethrums results in
15 pyrethrin and synthetic analogues of pyrethrums are honeybee colonies that are more resistant to mite
known as pyrethroids. In fact, the human mite infestations.
infestation known as scabies (Sarcoptes scabiei) is D) Humans are more susceptible to varroa mites as a result
treated with a topical pyrethrum cream. of consuming nutritionally deficient food crops.
We suspect that the bees of commercial bee
20 colonies which are fed mono-crops are nutritionally Which choice provides the best evidence for the
deficient. In particular, we postulate that the problem answer to the previous question?
is a diet deficient in anti-mite toxins: pyrethrums, A) Lines 3-5 (“These mites . . . viruses”)
and possibly other nutrients which are inherent in B) Lines 16-18 (“In fact . . . cream”)
such plants. Without, at least, intermittent feeding on C) Lines 19-21 (“We suspect . . . deficient”)
25 the pyrethrum producing plants, bee colonies are D) Lines 24-28 (“Without . . . bees”)
susceptible to mite infestations which can become

4
In the present research, we propose a unique and larger gifts as representing smaller and larger
40 psychological explanation for this overspending 65 signals of thoughtfulness and consideration.
problem—i.e., that gift-givers equate how much they The notion of gift-givers and gift-recipients being
spend with how much recipients will appreciate the unable to account for the other party’s perspective
gift (the more expensive the gift, the stronger a seems puzzling because people slip in and out of
gift-recipient’s feelings of appreciation). Although a these roles every day, and, in some cases, multiple
45 link between gift price and feelings of appreciation 70 times in the course of the same day. Yet, despite the
might seem intuitive to gift-givers, such an extensive experience that people have as both givers
assumption may be unfounded. Indeed, we propose and receivers, they often struggle to transfer
that gift-recipients will be less inclined to base their information gained from one role (e.g., as a giver)
feelings of appreciation on the magnitude of a gift and apply it in another, complementary role (e.g., as
50 than givers assume. 75 a receiver). In theoretical terms, people fail to utilize
Why do gift-givers assume that gift price is closely - The passage indicates that the assumption made by gift-
linked to gift-recipients’ feelings of appreciation? givers in lines 41-44 may be
Perhaps givers believe that bigger (i.e., more
(A) insincere.
expensive) gifts convey stronger signals of
(B) unreasonable.
55 thoughtfulness and consideration. According to
Camerer (1988) and others, gift-giving represents a (C) incorrect.
symbolic ritual, whereby gift-givers attempt to signal (D) substantiated.
their positive attitudes toward the intended recipient - Which choice provides the best evidence for the answer
and their willingness to invest resources in a future to the previous question?
60 relationship. In this sense, gift-givers may be (A) Lines 53-55 (“Perhaps . . . consideration”)
motivated to spend more money on a gift in order to (B) Lines 55-60 (“According . . . relationship”)
send a “stronger signal” to their intended recipient. (C) Lines 63-65 (“As . . . consideration”)
As for gift-recipients, they may not construe smaller (D) Lines 75-78 (“In . . . relations”)

Top Score
GET READY TO STUDY
Chapter 1 Evidence-based Questions

5
Which choice best supports the view of the “skeptics” “Our experiment does not reveal the psychology
(line 20)? behind people’s decisions,” Aral says, “but an
A) Lines 55-58 (“Comments . . . reports”) intuitive explanation is that people are more
B) Lines 58-60 (“And . . . them”) skeptical of negative social influence. They’re more
C) Lines 63-65 (“The ratings . . . them”) 70 willing to go along with positive opinions from other
D) Lines 76-79 (“He . . . manipulated”) people.”
Duncan Watts, a network scientist at Microsoft
At least when it comes to comments on news Research in New York City, agrees with that
sites, the crowd is more herdlike than wise. conclusion. “[But] one question is whether the
55 Comments that received fake positive votes from the 75 positive [herding] bias is specific to this site” or true
researchers were 32% more likely to receive more in general, Watts says. He points out that the
positive votes compared with a control, the team category of the news items in the experiment had a
reports. And those comments were no more likely strong effect on how much people could be
than the control to be down-voted by the next viewer manipulated. “I would have thought that ‘business’ is
60 to see them. By the end of the study, positively 80 pretty similar to ‘economics,’ yet they find a much
manipulated comments got an overall boost of about stronger effect (almost 50% stronger) for the former
25%. However, the same did not hold true for than the latter. What explains this difference? If we’re
negative manipulation. The ratings of comments that going to apply these findings in the real world, we’ll
got a fake down vote were usually negated by an up need to know the answers.”
65 vote by the next user to see them.

6
There they go, our brothers who have been speak from a pulpit. Nobody will dare contradict us
educated at public schools and universities, 35 then; we shall be the mouthpieces of the divine
mounting those steps, passing in and out of those
15 doors, ascending those pulpits, preaching, teaching, - Woolf indicates that the procession she describes in the
administering justice, practising medicine, passage
transacting business, making money. It is a solemn (A) has come to have more practical influence in recent
sight always—a procession, like a caravanserai years.
crossing a desert. . . . But now, for the past twenty (B) has become a celebrated feature of English public
20 years or so, it is no longer a sight merely, a life.
photograph, or fresco scrawled upon the walls of (C) includes all of the richest and most powerful men in
time, at which we can look with merely an esthetic England.
appreciation. For there, trapesing along at the tail (D) has become less exclusionary in its membership in
end of the procession, we go ourselves. And that recent years.
25 makes a difference. We who have looked so long at
the pageant in books, or from a curtained window - Which choice provides the best evidence for the answer
watched educated men leaving the house at about to the previous question?
nine-thirty to go to an office, returning to the house (A) Lines 12-17 (“There . . . money”)
at about six-thirty from an office, need look passively
(B) Lines 17-19 (“It . . . desert”)
30 no longer. We too can leave the house, can mount
(C) Lines 23-24 (“For . . . ourselves”)
those steps, pass in and out of those doors, . . . make
money, administer justice. . . . We who now agitate (D) Lines 30-34 (“We . . . pulpit”)
these humble pens may in another century or two

Top Score
GET READY TO STUDY
Chapter 1 Evidence-based Questions

7
Researchers report in the journal Cell that the enzyme NAMPT (nicotinamide phosphoribosyltransferase).
phenomenon is likely linked to two enzymes-SIRT3 and The two- to four-fold surge in NAMPT in turn triggers the
SIRT4-in mitochondria (the cell’s powerhouse that, among 35 production of a molecule called NAD (nicotinamide adenine
10 other tasks, converts nutrients to energy). They found that a dinucleotide), which plays a key role in cellular metabolism
cascade of reactions triggered by lower caloric intake raises and signaling.
the levels of these enzymes, leading to an increase in the The uptick in NAD levels activates the SIRT3 and SIRT4
strength and efficiency of the cellular batteries. By genes, increasing levels of their corresponding SIRT3 and
invigorating the mitochondria, SIRT3 and SIRT4 extend the 40 SIRT4 enzymes, which then flood the interior of the
15 life of cells, by preventing flagging mitochondria from mitochondria. Sinclair says he’s not sure exactly how SIRT3
developing tiny holes (or pores) in their membranes that and SIRT4 beef up the mitochondria’s energy output, but that
allow proteins that trigger apoptosis, or cell death, to seep out events leading to cell death are at the very least delayed when
into the rest of the cell. there are vast quantities of the enzymes.
“We didn’t expect that the most important part of this
20 pathway was in the mitochondria,” says David Sinclair, an Based on the passage, SIRT-3 and SIRT-4 indirectly
affect cell longevity by
assistant professor of pathology at Harvard Medical School
A) strengthening the mitochondrial membrane.
and a study co-author. “We think that we’ve possibly found
B) fixing damage to the cell’s DNA.
regulators of aging.”
C) initiating cell death.
In 2003 Sinclair’s lab published a paper in Nature that
D) diminishing the efficiency of the cell’s
25 described the discovery of a gene that switched on in the
mitochondria.
yeast cell in response to calorie restriction, which Sinclair
calls a “master regulator in aging.” Since then, his team has Which choice provides the best evidence for the
been searching for an analogous gene that plays a similar role answer to the previous question?
in the mammalian cell. A) lines 7-9 (“Researchers ... mitochondria”)
30 The researchers determined from cultures of human B) lines 13-18 (“By ... cell.”)
embryonic kidney cells that lower caloric intake sends a C) lines 38-41 (“The ... mitochondria”)
signal that activates a gene inside cells that codes for the D) lines 41-44 (“Sinclair ... enzymes”)

8
45 SIRT3 and SIRT4 are part of a family called sirtuins 65 interesting hypothesis connecting the mitochondria to
(SIRT1, which helps extend cell life by modulating the longevity, but that it needs to be more directly tested in the
number of repair proteins fixing DNX damage both inside context of dietary restriction. “If the NAMPToverexpressing
and outside the cell’s nucleus,is also a member). SIRT is mice are long-lived and disease resistant, that
short for sir-2 homologue-a well-studied protein that is will provide more support for this idea.”
50 known to extend yeast cell longevity. According to Sinclair,
all of the mammalian SIRT genes (and their proteins) are The author implies that the results of Sinclair’s study
possible drug targets for therapies aimed at extending life, as will enable future scientists to
well as staving off age-related illnesses, such as Alzheimer’s A) reverse the aging process.
disease, cancers and metabolic disorders, like diabetes. B) diagnose patients with age-related illnesses from
55 “I think SIRT3 is the next most interesting sirtuin from a an earlier age.
drug development standpoint,” Sinclair says. “It does protect C) create mice that are essentially immortal.
cells, but there's growing evidence that it may mediate the D) more effectively treat a number of age-related
benefits of exercise as well.” illnesses.
Sinclair’s lab is now working on developing what he calls
60 a possible “supermouse” with elevated levels of NAMPT to Which choice provides the best evidence for the
see if it lives longer and is more disease-resistant than answer to the previous question?
normal mice. A) lines 45-50 (“SIRT ... longevity.”)
Matt Kaeberlein, a pathologist at the University of B) lines 50-54 (“According ... diabetes.”)
Washington in Seattle, says that Sinclair’s team has an C) lines 59-62 (“Sinclair’s ... mice”)
D) lines 67-69 (“If ... idea”)

Top Score
GET READY TO STUDY
Chapter 1 Evidence-based Questions

9
With respect to self-esteem, the research is also mixed, Based on the passage, the connection between social
50 especially considering the different types of self-esteem. In media use and self-esteem is
certain cases, excessive Facebook use may be related to A) complicated due to the many variables that must
lower general self-esteem. For example, Mehdizadeh be accounted for.
found a significant negative correlation between self-esteem B) strongly linked because social media can reveal
and the number of times students checked Facebook per day self-perception.
55and between self-esteem and the time spent on Facebook per C) inconsistent although multiple studies suggest
session. Those with lower self-esteem also presented less there is a correlation.
self-promoting content on Facebook pages.However, D) contradictory because not enough research has
Facebook can enhance self-esteem specifically related to the been undertaken.
social functions of the self (social self-esteem) when
60 individuals receive positive comments and feedback from Which choice provides the best evidence for the
their friends online. In addition, researchers have found that answer to the previous question?
introverted individuals and individuals with low self-esteem A) line 49-52(“With...self-esteem”)
may benefit from the social opportunities provided by the B) line 52-56(“For...session”)
Facebook interface, without the anxiety of interacting C) line 57-61(“However...online”)
65 face-to-face. D) line 61-65(“In...face-to-face”)

10
A student claims that nitrogenous bases pair but of two. These two chains are both coiled around
randomly with one another. Which of the following 15 a common fiber axis. It has often been assumed that
statements in the passage contradicts the student’s since there was only one chain in the chemical
claim? formula there would only be one in the structural
A) Lines 5-6 (“To each . . . types”) unit. However, the density, taken with the X-ray
B) Lines 9-10 (“So far . . . irregular”) evidence, suggests very strongly that there are two.
C) Lines 23-25 (“The bases . . . other”) 20 The other biologically important feature is the
D) Lines 27-29 (“One member . . . chains”) manner in which the two chains are held together.
This is done by hydrogen bonds between the bases.
5 To each sugar is attached a nitrogenous base, which The bases are joined together in pairs, a single base
can be of four different types. Two of the possible from one chain being hydrogen-bonded to a single
bases—adenine and guanine—are purines, and the 25 base from the other. The important point is that only
other two—thymine and cytosine—are pyrimidines. certain pairs of bases will fit into the structure.
So far as is known, the sequence of bases along the One member of a pair must be a purine and the other
10 chain is irregular. The monomer unit, consisting of a pyrimidine in order to bridge between the two
phosphate, sugar and base, is known as a nucleotide. chains. If a pair consisted of two purines, for
The first feature of our structure which is of 30 example, there would not be room for it.
biological interest is that it consists not of one chain,

Top Score
GET READY TO STUDY
Chapter 1 Evidence-based Questions

11
30 SCRABBLE skill has been found to correlate positively with performed a task in which they were shown groups of letters
the amount of time people spend engaging in SCRABBLE and judged whether they formed words. (fMRI measures
related activities. In one study, using official SCRABBLE brain activity by detecting changes in blood flow within
rating as an objective measure of skill, researchers found that different regions of the brain.) The major finding of this
groups of “elite” and “average” SCRABBLE players differed 65 study was that competitive SCRABBLE players recruited
35 in the amount of time they had devoted to things like brain regions associated with working memory and visual
studying word lists, analyzing previous SCRABBLE games, perception to perform this task to a greater degree than the
and anagramming—and not by a little. Overall, the elite control subjects did.
group had spent an average of over 5000 hours on What might explain SCRABBLE experts’ superiority in
SCRABBLE study, compared to only about 1300 hours for 70 working memory and visuospatial reasoning? For the same
40 the average group. basic reason that basketball players tend to be tall, a likely
Clearly, expert SCRABBLE players are to some degree explanations is that people high in working memory and
“made”. But there is evidence that basic cognitive abilities visuospatial reasoning abilities are people who tend to get
play a role, too. In a study recently published in Applied into, and persist at, playing SCRABBLE: because it gives
Cognitive Psychology, Michael Toma and his colleagues 75 them an advantage in the game. This explanation fits with
45 found that elite SCRABBLE players outperformed college what behavioral geneticists call gene-environment
students from a highly selective university on tests of two correlation, which is the idea that our genetic makeup
cognitive abilities: working memory and visuospatial influences our experiences.
reasoning. Working memory is the ability to hold in mind
information while using it to solve a problem, as when The author implies that people who experience great
50 iterating through possible moves in a SCRABBLE game. success in a given activity are often born with
Visuospatial reasoning is the ability to visualize things and to A) advantages uniquely suited to that activity.
detect patterns, as when imagining how tiles on a B) a drive to succeed in any activity.
SCRABBLE board would intersect after a certain play. Both C) parents who value success in that activity.
abilities are influenced by genetic factors. D) a desire to participate in many activities.
55 Further evidence pointing to a role of these abilities in
Which choice provides the best evidence for the
SCRABBLE expertise comes from a recent brain imaging
answer to the previous question?
study by Andrea Protzner and her colleagues at the
A) Line 30-32 (“SCRABBLE...activities”)
University of Calgary. Using functional magnetic resonance
B) Line 37-40 (“Overall...group”)
imaging (fMRI), these researchers recorded the brain activity
C) Line 51-53 (“Visuospatial...play”)
60 of SCRABBLE players and control subjects as they
D) Line 70-75 (“For...game”)

12 he gestured with it so that the two women at the next


Which choice best supports the conclusion that table would notice the diamond ring on the fourth
Mr. Peters wants to attract attention? 85 finger of his right hand. Both of these things, and
A) Lines 80-81 (“Apparently . . . change”) also the fact that his hands showed signs of the
B) Lines 81-85 (“He straightened . . . hand”) manicurist, one can blame on the young man who
C) Lines 90-91 (“The young . . . Mr. Peters”) had his picture taken with a derby hat on the back of
D) Lines 91-93 (“He was . . . forty-five”) his head, and also sitting with a girl in the curve of
90 the moon. The young man had never for one second
80 Apparently he himself was not aware that there deserted Mr. Peters. He was always there, tugging at
had been any change. He straightened his tie Mr. Peters’ elbow, making him do things that were
self-consciously and when Irma handed him a menu, not becoming in a man of forty-five.

Top Score
GET READY TO STUDY
Chapter 1 Evidence-based Questions

13
As case studies go, understanding the distribution of that trait is present in another, geographically proximate,
cultural elements in Native American communities during tribe. In other words, selection biases have not disrupted the
the historical period of the Great Plains would seem a most distribution of these decorations to the extent that they
challenging one. Famously, there is a mixture of powerful 50 deviate significantly from a pattern predictable on the basis
5 internal and external factors, creating -- for a relatively brief of the geographic relationships between tribes alone. These
period in time -- a seemingly distinctive set of shared results would seem to reaffirm the role of intergroup relations
elements from a linguistically diverse set of peoples. This is in creating a pattern of shared cultural similarity over the
known across the world as the “Great Plains culture.” [In our region during the 19th century, which many have previously
study], quantitative analyses show how different processes 55 discussed.
10 operated on two sets of cultural traits among nine High Behavioral variation between tribes in terms of their
Plains groups. Moccasin decorations exhibit a pattern practice of the religious ceremony of the Sun Dance also
consistent with geographically-mediated between-group indicated a statistical relationship with geography, again
interaction. However, group variations in the religious reiterating the role of inter-tribe transmission in creating the
ceremony of the Sun Dance also reveal evidence of purifying 60 phenomenon historically labeled as the “Great Plains
15 cultural selection associated with historical biases, dividing culture.” However, patterns of inter-tribe variation in Sun
down ancient linguistic lines. The latter shows that while the Dance elements also exhibited a statistically significant
conglomeration of “Plains culture” may have been a product relationship with linguistic affinities between different tribes.
of merging new ideas with old, combined with cultural Ordinarily, this pattern might simply be attributed to the fact
interchange between groups, the details of what was 65 that tribes with more mutually comprehensible languages
20 accepted, rejected or elaborated in each case reflected were able to more effectively transmit the behavioral variants
preexisting ideological biases. Although culture may sometimes among themselves. Here, however, language patterns were
be a “melting pot,” the analyses show that even in highly found not to correlate with geographic patterns and,
fluid situations, cultural mosaics may be indirectly shaped by moreover, the statistical relationship between Sun Dance
historical factors that are not always obvious. . . . 70 patterning and linguistic affiliation was found to still hold
25 “Culture” is the conglomeration of information, even when geography was controlled for. The relationship
knowledge, ideas, and beliefs, shared by communities and between Sun Dance variations and language patterns is,
transmitted by social interaction. This shared property therefore, in this instance puzzling.
characterizes all human societies. Communities that came to
occupy the High Great Plains during the 19th century What does the author imply about similarities in
30 exemplify the manner in which humans can take existing moccasin design among Great Plains tribes in the 19th
ideas, elaborate them, combine them with new ones, pass century?
them successfully between groups and create novel, distinct A) The moccasin designs are as similar as the tribes’
patterns, visible over temporal and spatial scales. Attempting Sun Dances.
to examine the role of specific factors in creating cultural B) Similar languages played a small but vital role in
35 patterning under such historically-contingent, transient, and determining the similarity of the tribes’ moccasin
dynamic conditions, however, presents a challenge. designs.
Here, analyses have shown the presence of distinct C) The similarities in moccasin design can be
processes operating and ultimately influencing the explained in the context of the tribes’ relative
representation of cultural traits in different tribes. Patterns of geographies.
40 similarity and difference in moccasin designs among D) The Great Plains tribes did not have common
different tribes show a statistically significant relationship with
the model of geographic relationships between tribes. Which choice provides the best evidence for the
No statistical effect of language affiliation on the distribution answer to the previous question?
of moccasin decorations was detected. This indicates that the A) lines 4–7 (“Famously . . . peoples”)
45 representation of moccasin decoration types among groups B) lines 39–42 (“Patterns . . . tribes”)
is, in this case, most strongly determined by whether or not C) lines 43–44 (“No . . . detected”)
D) lines 61–63 (“However . . . tribes”)

Top Score
GET READY TO STUDY
Chapter 1 Evidence-based Questions

14
Based on the table, is the percentage of adenine in restrictive, and the only pairs of bases possible are:
each organism’s DNA the same or does it vary, and 35 adenine with thymine, and guanine with cytosine.
which statement made by the authors is most Adenine, for example, can occur on either chain; but
consistent with that data? when it does, its partner on the other chain must
A) The same; “Two of . . . pyrimidines” (lines 6-8) always be thymine.
B) The same; “The important . . . structure” The phosphate-sugar backbone of our model is
(lines 25-26) 40 completely regular, but any sequence of the pairs of
C) It varies; “Adenine . . . thymine” (lines 36-38) bases can fit into the structure. It follows that in a
D) It varies; “It follows . . . information” long molecule many different permutations are
(lines 41-45) possible, and it therefore seems likely that the precise
sequence of bases is the code which carries the
5 To each sugar is attached a nitrogenous base, which 45 genetical information. If the actual order of the bases
can be of four different types. Two of the possible on one of the pair of chains were given, one could
bases—adenine and guanine—are purines, and the write down the exact order of the bases on the other
other two—thymine and cytosine—are pyrimidines. one, because of the specific pairing. Thus one chain
So far as is known, the sequence of bases along the is, as it were, the complement of the other, and it is
10 chain is irregular. The monomer unit, consisting of 50 this feature which suggests how the deoxyribonucleic
phosphate, sugar and base, is known as a nucleotide. acid molecule might duplicate itself.
The first feature of our structure which is of
biological interest is that it consists not of one chain,
but of two. These two chains are both coiled around
15 a common fiber axis. It has often been assumed that Base Composition of DNA
since there was only one chain in the chemical
formula there would only be one in the structural Percentage of base
unit. However, the density, taken with the X-ray in organism’s DNA
evidence, suggests very strongly that there are two. Organism adenine guanine cytosine thymine
20 The other biologically important feature is the
(%) (%) (%) (%)
manner in which the two chains are held together.
This is done by hydrogen bonds between the bases. Maize 26.8 22.8 23.2 27.2
The bases are joined together in pairs, a single base Octopus 33.2 17.6 17.6 31.6
from one chain being hydrogen-bonded to a single Chicken 28.0 22.0 21.6 28.4
25 base from the other. The important point is that only Rat 28.6 21.4 20.5 28.4
certain pairs of bases will fit into the structure. Human 29.3 20.7 20.0 30.0
One member of a pair must be a purine and the other Grasshopper 29.3 20.5 20.7 29.3
a pyrimidine in order to bridge between the two Sea urchin 32.8 17.7 17.3 32.1
chains. If a pair consisted of two purines, for Wheat 27.3 22.7 22.8 27.1
30 example, there would not be room for it.
Yeast 31.3 18.7 17.1 32.9
We believe that the bases will be present almost
entirely in their most probable forms. If this is true,
E. coli 24.7 26.0 25.7 23.6
the conditions for forming hydrogen bonds are more

Top Score
GET READY TO STUDY
Chapter 1 Evidence-based Questions
15
Which choice best supports the claim that Nawab 40 a word. The landowner, who was cheerfully filing his
performs his duties for Harouni well? nails in front of a crackling rosewood fire, told him
A) Lines 28-32 (“By his . . . Lahore”) to go ahead.
B) Lines 40-42 (“The landowner . . . ahead”) “Sir, as you know, your lands stretch from here to
C) Lines 46-49 (“In your . . . should”) the Indus, and on these lands are fully seventeen tube
D) Line 58 (“I’ve . . . years”) 45 wells, and to tend these seventeen tube wells there is
in an Atlantic gale. By his superhuman efforts he but one man, me, your servant. In your service I have
almost managed to maintain K. K. Harouni in the earned these gray hairs”—here he bowed his head to
30 same mechanical cocoon, cooled and bathed and show the gray—“and now I cannot fulfill my duties
lighted and fed, that the landowner enjoyed in as I should. Enough, sir, enough. I beg you, forgive
Lahore. 50 me my weakness. Better a darkened house and proud
Harouni of course became familiar with this hunger within than disgrace in the light of day.
ubiquitous man, who not only accompanied him on Release me, I ask you, I beg you.”
35 his tours of inspection, but morning and night could The old man, well accustomed to these sorts of
be found standing on the master bed rewiring the speeches, though not usually this florid, filed away at
light fixture or in the bathroom poking at the water 55 his nails and waited for the breeze to stop.
heater. Finally, one evening at teatime, gauging the “What’s the matter, Nawabdin?”
psychological moment, Nawab asked if he might say “Matter, sir? O what could be the matter in your
service. I’ve eaten your salt for all my years.

16
25 home. No one need wonder at the disorganization, at And now man himself stands appalled at the results
the fragmentary condition of everything, when we of his own excesses, and mourns in bitterness that
remember that man, who represents but half a 55falsehood, selfishness, and violence are the law of life.
complete being, with but half an idea on every The need of this hour is not territory, gold mines,
subject, has undertaken the absolute control of all railroads, or specie payments but a new evangel of
30 sublunary matters. womanhood, to exalt purity, virtue, morality, true
People object to the demands of those whom they religion, to lift man up into the higher realms of
choose to call the strong-minded, because they say 60 thought and action.
“the right of suffrage will make the women We ask woman’s enfranchisement, as the first step
masculine.” That is just the difficulty in which we are toward the recognition of that essential element in
35 involved today. Though disfranchised, we have few government that can only secure the health, strength,
women in the best sense; we have simply so many and prosperity of the nation. Whatever is done to lift
reflections, varieties, and dilutions of the masculine 65 woman to her true position will help to usher in a
gender. The strong, natural characteristics of new day of peace and perfection for the race.
womanhood are repressed and ignored in
40 dependence, for so long as man feeds woman she - Stanton contends that the situation she describes in the
will try to please the giver and adapt herself to his passage has become so dire that even men have begun to
condition. To keep a foothold in society, woman (A) lament the problems they have created.
must be as near like man as possible, reflect his (B) join the call for woman suffrage.
ideas, opinions, virtues, motives, prejudices, and (C) consider women their social equals.
45 vices. She must respect his statutes, though they strip
(D) ask women how to improve civic life.
her of
every inalienable right, and conflict with that higher
- Which choice provides the best evidence for the answer to
law written by the finger of God on her own soul. . . .
the previous question?
. . . [M]an has been molding woman to his ideas
by direct and positive influences, while she, if not a (A) Lines 25-30 (“No one . . . matters”)
50 negation, has used indirect means to control him, (B) Lines 53-55 (“And now . . . life”)
and in most cases developed the very characteristics (C) Lines 56-60 (“The need . . . action”)
both in him and herself that needed repression. (D) Lines 61-64 (“We ask . . . nation”)

Top Score
GET READY TO STUDY
Chapter 1 Evidence-based Questions

17
25 my darling, my cherished-in-secret, Imagination, the and prying as was Edward’s malignity, it could never
tender and the mighty, should never, either by baffle the lynx-eyes of these, my natural sentinels.
softness or strength, have severed me. But this was Day by day did his malice watch my tact, hoping it
not all; the antipathy which had sprung up between 55 would sleep, and prepared to steal snake-like on its
myself and my employer striking deeper root and slumber; but tact, if it be genuine, never sleeps.
30 spreading denser shade daily, excluded me from I had received my first quarter’s wages, and was
every glimpse of the sunshine of life; and I began to returning to my lodgings, possessed heart and soul
feel like a plant growing in humid darkness out of the with the pleasant feeling that the master who had
slimy walls of a well. 60 paid me grudged every penny of that hard‑earned
Antipathy is the only word which can express the pittance—(I had long ceased to regard
35 feeling Edward Crimsworth had for me—a feeling, in Mr. Crimsworth as my brother—he was a hard,
a great measure, involuntary, and which was liable to
be excited by every, the most trifling movement, - The passage indicates that when the narrator began
look, or word of mine. My southern accent annoyed working for Edward Crimsworth, he viewed
him; the degree of education evinced in my language Crimsworth as a
40 irritated him; my punctuality, industry, and (A) harmless rival.
accuracy, fixed his dislike, and gave it the high (B) sympathetic ally.
flavour and poignant relish of envy; he feared that I (C) perceptive judge.
too should one day make a successful tradesman. (D) demanding mentor.
Had I been in anything inferior to him, he would not
- Which choice provides the best evidence for the
45 have hated me so thoroughly, but I knew all that he
answer to the previous question?
knew, and, what was worse, he suspected that I kept
the padlock of silence on mental wealth in which he (A) Lines 28-31 (“the antipathy . . . life”)
was no sharer. If he could have once placed me in a (B) Lines 38-40 (“My southern . . . irritated him”)
ridiculous or mortifying position, he would have (C) Lines 54-56 (“Day . . . slumber”)
50 forgiven me much, but I was guarded by three (D) Lines 61-62 (“I had . . . brother”)
faculties—Caution, Tact, Observation; and prowling

18
be high, but decisions will get made. “If you are In most cases, whoever cares most about the
45 both maximizers, neither of you will be able to result should choose, Dr. Schwartz says. This isn’t
relinquish your standards,” Dr. Schwartz says. the same as always letting the maximizer decide. If
David Gerzof Richard makes quick, decisive 70 the maximizer is paralyzed with indecision, it can
choices. His wife, Brooke, likes to research every work best if the satisficer chooses, Dr. Schwartz
option. The spouses say they didn’t learn to make says.
50 decisions together until after an event early in their Which best summarizes the story of Mr. and Ms.
marriage that they refer to as “the car.” Richards?
Mr. Richard decided the couple needed a new car A) A maximizer and satisficer will have many
to replace their old one. He spent a few days arguments in their marital life.
researching SUVs, found a good deal on an Audi Q5 B) Although it can be hard, a maximizer and
55 and signed the lease—without telling his wife. Ms. satisficer can learn to make decisions together.
Richard wasn’t happy when he told her. At her C) A maximizer may feel compelled to accept his
insistence, the spouses spent the weekend testdriving or her spouse’s rash decisions.
five more cars. The couple stuck with the D) While they might disagree at first, maximizers
Q5—Ms. Richard agreed that it remained the best and satisficers always agree in the end.
60 deal—but they both learned something about how
their decision-making styles could complement each Which choice provides the best evidence for the
other’s. “His decision-making makes it so we can answer to the previous question?
get it done faster and don’t lose opportunities,” Ms. A) Lines 44-46 (“If you … says”)
Richard says. “And my decision-making makes sure B) Lines 49-51 (“The spouses … car”)
65 we are truly not forgetting to consider what is C) Lines 58-62 (“The couple … other’s”)
important.” D) Lines 69-72 (“If the … says”)

18
Top Score
GET READY TO STUDY
Chapter 1 Evidence-based Questions
Some argue that because the free markets allow losing money? Show too little and you’re a “greed is
5 for personal choice, they are already ethical. Others good” corporate raider; too much and you’re wasting
have accepted the ethical critique and embraced 40 money on unproductive capital. Aristotle thought
corporate social responsibility. But before we can there was a golden mean between the two extremes,
label any market outcome as “immoral,” or sneer at and finding it was a matter of fine judgment. But if
economists who try to put a price on being ethical, ethics is about character, it’s not clear what those
10 we need to be clear on what we are talking about. characteristics should be.
There are different views on where ethics should 45 There is yet another approach: instead of rooting
apply when someone makes an economic decision. ethics in character or the consequences of actions, we
Consider Adam Smith, widely regarded as the can focus on our actions themselves. From this
founder of modern economics. He was a moral perspective some things are right, some wrong—we
15 philosopher who believed sympathy for others was should buy fair trade goods, we shouldn’t tell lies in
the basis for ethics (we would call it empathy 50 advertisements. Ethics becomes a list of
nowadays). But one of his key insights in The Wealth commandments, a catalog of “dos” and “don’ts.”
of Nations was that acting on this empathy could be
counter-productive—he observed people becoming - In the passage, the author anticipates which of the
20 better off when they put their own empathy aside, following objections to criticizing the ethics of free
and interacted in a self-interested way. Smith justifies markets?
selfish behavior by the outcome. Whenever planners (A) Smith’s association of free markets with ethical
use cost-benefit analysis to justify a new railway line, behavior still applies today.
or someone retrains to boost his or her earning (B) Free markets are the best way to generate high
25 power, or a shopper buys one to get one free, they are profits, so ethics are a secondary consideration.
using the same approach: empathizing with (C) Free markets are ethical because they are made
someone, and seeking an outcome that makes that possible by devalued currency.
person as well off as possible—although the person (D) Free markets are ethical because they enable
they are empathizing with may be themselves in the individuals to make choices.
30 future.
Instead of judging consequences, Aristotle - Which choice provides the best evidence for the
said ethics was about having the right answer to the previous question?
character—displaying virtues like courage and (A) Lines 4-5 (“Some . . . ethical”)
honesty. It is a view put into practice whenever
(B) Lines 7-10 (“But . . . about”)
35 business leaders are chosen for their good character.
(C) Lines 21-22 (“Smith . . . outcome”)
But it is a hard philosophy to teach—just how much
loyalty should you show to a manufacturer that keeps (D) Lines 52-54 (“When . . . way”)

Top Score
GET READY TO STUDY
Chapter 1 Evidence-based Questions

Answers
Exercise 1 Exercise 2 Practice
1- A-C 1- D 1- A-A
2- D-C 2- D 2- D-C
3- C-B 3- B 3- C-D
4- D-C 4- C-C
5- D-A 5- A
6- C-B 6- D-C
7- A-B
8- D-B
9- A-A
10- D
11- A-D
12- B
13- C-B
14- D
15- A
16- A-B
17- B-D
18- B-C
19- D-A

Top Score
GET READY TO STUDY
Chapter 2 Author’s Indications

Author’s Indications
In this type of questions the test maker asks you about what the author said about Or why the author mentioned a
specific idea or word or group of words. And this kind of question comes in two forms:

a. Indication with references:


In this form the test maker gives you the word or group of words or the idea in the question with reference
(specific lines):
Ex:
plenipotentiaries. It was by far the most splendid and right thumb) presided over the Congress, and
important assembly ever convoked to discuss and 40 Prince Talleyrand (the index finger) represented
30 determine the affairs of Europe. The Emperor of France.
Russia, the King of Prussia, the Kings of Bavaria,
Denmark, and Wurttemberg, all were present in It can reasonably be inferred that Irma, the waitress,
person at the court of the Emperor Francis I in the thinks Lymie is “through eating” (line 37) because
Austrian capital. When Lymie put down his fork and A) he has begun reading his book.
35 began to count them off, one by one, on the fingers B) his plate is empty.
of his left hand, the waitress, whose name was Irma, C) he is no longer holding his fork.
thought he was through eating and tried to take his D) he has asked her to clear the table.
plate away. He stopped her. Prince Metternich (his

Here the test maker said that the authors indicated to the word or group of words or idea “through eating” to
support or to show which idea with mentioning the lines in which you find this word or group of words or
the idea?
To solve this question you must stick to these steps:
1- Go to the line in which the indicated words exist.
2- Read the sentence in which the words exist from the beginning till the end of the same sentence.
3- Match the idea included this sentence with one of the answers given:

When Lymie put down his fork and A) he has begun reading his book.
35 began to count them off, one by one, on the fingers B) his plate is empty.
of his left hand, the waitress, whose name was Irma, C) he is no longer holding his fork.
thought he was through eating and tried to take his D) he has asked her to clear the table.
plate away.

So, from here we can find that Irma, the waitress, thought that Lymie “through eating” because he put down
his fork. The answer is C.

Exercise:
The first feature of our structure which is of
biological interest is that it consists not of one chain, In the second paragraph (lines 12-19), what do the
but of two. These two chains are both coiled around authors claim to be a feature of biological interest?
15 a common fiber axis. It has often been assumed that A) The chemical formula of DNA
since there was only one chain in the chemical B) The common fiber axis
formula there would only be one in the structural C) The X-ray evidence
unit. However, the density, taken with the X-ray D) DNA consisting of two chains
evidence, suggests very strongly that there are two.

Top Score
GET READY TO STUDY
Chapter 2 Author’s Indications

b. Indications without references:


In this form the test maker gives you the word or group of words or the idea in the question without
reference (without specific lines):
Ex:
The first feature of our structure which is of The authors’ main purpose of including the
biological interest is that it consists not of one chain, information about X-ray evidence and density is to
but of two. These two chains are both coiled around A) establish that DNA is the molecule that carries the
15 a common fiber axis. It has often been assumed that genetic information.
since there was only one chain in the chemical B) present an alternate hypothesis about the composition
formula there would only be one in the structural of a nucleotide.
unit. However, the density, taken with the X-ray C) provide support for the authors’ claim about the
evidence, suggests very strongly that there are two. number of chains in a molecule of DNA.
D) confirm the relationship between the density of DNA
and the known chemical formula of DNA.
Here the test maker asks about the purpose behind indicating to “X-ray evidence” and “density” but without saying in
which lines they exist. To solve this question stick to these steps:
1- Determine the place of the mentioned words in the passage by scan skimming the passage.
2- Read the sentence in which the words exist from the beginning till the end of the same sentence.
3- Match the idea included this sentence with one of the answers given:

A) establish that DNA is the molecule that carries the


15 a common fiber axis. It has often been assumed that genetic information.
B) present an alternate hypothesis about the composition
since there was only one chain in the chemical
of a nucleotide.
formula there would only be one in the structural C) provide support for the authors’ claim about the
unit. However, the density, taken with the X-ray number of chains in a molecule of DNA.
evidence, suggests very strongly that there are two. D) confirm the relationship between the density of DNA
and the known chemical formula of DNA.

When you read the sentence included the words “X-ray” and “density” you will find that the author wants to prove
that they thought that there was only one chain but by using X-ray evidence they find that there are two chains not
one, the idea which matches the answer C.
Exercise:
A party of four, two men and two women, came secret treaty concluded between England, France,
into the restaurant, all talking at once, and took and Austria, when the pretensions of Prussia and
possession of the center table nearest Lymie. Russia, acting in concert, seemed to threaten a
The women had shingled hair and short tight skirts renewal of the attack. The results of the Congress
which exposed the underside of their knees when were stated clearly at the bottom of page 67 and at
they sat down. One of the women had the face of a the top of page 68, but before Lymie got halfway
young boy but disguised by one trick or another through them, a coat that he recognized as his
(rouge, lipstick, powder, wet bangs plastered against father’s was hung on the hook next to his chair.
the high forehead, and a pair of long pendent Lymie closed the book and said, “I didn’t think you
earrings) to look like a woman of thirty-five, which were coming.”
as a matter of fact she was. The men were older. They
laughed more than there seemed any occasion for,
The narrator indicates that Lymie finally closes the
while they were deciding between soup and shrimp
history book because
cocktail, and their laughter was too loud. But it was
A) his father has joined him at the table.
the women’s voices, the terrible not quite sober pitch
B) the people at the other table are too disruptive.
of the women’s voices which caused Lymie to skim C) he has finished the chapter about the Congress.
over two whole pages without knowing what was on D) he is preparing to leave the restaurant.
them. Fortunately he realized this and went back.
Otherwise he might never have known about the

Top Score
GET READY TO STUDY
Chapter 2 Author’s Indications

Exercises
1
1 Every day, millions of shoppers hit the stores in The authors most likely use the examples in lines 1-9 of
full force—both online and on foot—searching the passage (“Every . . . showers”) to highlight the
frantically for the perfect gift. Last year, Americans A) regularity with which people shop for gifts.
spent over $30 billion at retail stores in the month of B) recent increase in the amount of money spent on gifts.
5 December alone. Aside from purchasing holiday C) anxiety gift shopping causes for consumers.
gifts, most people regularly buy presents for other D) number of special occasions involving gift-giving.
occasions throughout the year, including weddings,
birthdays, anniversaries, graduations, and baby
showers.

2
Anthropologists describe gift-giving as a positive The “social psychologists” mentioned in paragraph 2
social process, serving various political, religious, and (lines 17-34) would likely describe the “deadweight
psychological functions. Economists, however, offer loss” phenomenon as
20 a less favorable view. According to Waldfogel (1993), A) predictable.
gift-giving represents an objective waste of resources. B) questionable.
People buy gifts that recipients would not choose to C) disturbing.
buy on their own, or at least not spend as much D) unprecedented.
money to purchase (a phenomenon referred to as
25 ‘‘the deadweight loss of Christmas”). To wit, givers
are likely to spend $100 to purchase a gift that
receivers would spend only $80 to buy themselves.
This ‘‘deadweight loss” suggests that gift-givers are
not very good at predicting what gifts others will
30 appreciate. That in itself is not surprising to social
psychologists. Research has found that people often
struggle to take account of others’ perspectives—
their insights are subject to egocentrism, social
projection, and multiple attribution errors.
3
As you know from your own experience, and there short is it leading us, the procession of the sons of
are facts that prove it, the daughters of educated men educated men?
65 have always done their thinking from hand to
mouth; not under green lamps at study tables in the The range of places and occasions listed in
cloisters of secluded colleges. They have thought lines 72-76 (“Let us . . . funerals”) mainly serves to
while they stirred the pot, while they rocked the emphasize how
cradle. It was thus that they won us the right to our A) novel the challenge faced by women is.
70 brand-new sixpence. It falls to us now to go on B) pervasive the need for critical reflection is.
thinking; how are we to spend that sixpence? Think C) complex the political and social issues of the day are.
we must. Let us think in offices; in omnibuses; while D) enjoyable the career possibilities for women are.
we are standing in the crowd watching Coronations
and Lord Mayor’s Shows; let us think . . . in the
75 gallery of the House of Commons; in the Law Courts;
let us think at baptisms and marriages and funerals.
Let us never cease from thinking—what is this
“civilization” in which we find ourselves? What are
these ceremonies and why should we take part in
80 them? What are these professions and why
should we make money out of them? Where in

Top Score
GET READY TO STUDY
Chapter 2 Author’s Indications
4
The phosphate-sugar backbone of our model is The authors’ use of the words “exact,” “specific,” and
40 completely regular, but any sequence of the pairs of “complement” in lines 47-49 in the final paragraph
bases can fit into the structure. It follows that in a functions mainly to
long molecule many different permutations are A) confirm that the nucleotide sequences are known
possible, and it therefore seems likely that the precise for most molecules of DNA.
sequence of bases is the code which carries the B) counter the claim that the sequences of bases
45 genetical information. If the actual order of the bases along a chain can occur in any order.
on one of the pair of chains were given, one could C) support the claim that the phosphate-sugar
write down the exact order of the bases on the other backbone of the authors’ model is completely
one, because of the specific pairing. Thus one chain regular.
is, as it were, the complement of the other, and it is D) emphasize how one chain of DNA may serve as a
50 this feature which suggests how the deoxyribonucleic template to be copied during DNA replication.
acid molecule might duplicate itself.

5
The chemical formula of deoxyribonucleic acid when it does, its partner on the other chain must
(DNA) is now well established. The molecule is a always be thymine.
very long chain, the backbone of which consists of a The phosphate-sugar backbone of our model is
regular alternation of sugar and phosphate groups. 40 completely regular, but any sequence of the pairs of
5 To each sugar is attached a nitrogenous base, which bases can fit into the structure.
can be of four different types. Two of the possible
bases—adenine and guanine—are purines, and the The authors use the word “backbone” in lines 3 and 39
other two—thymine and cytosine—are pyrimidines. to indicate that
So far as is known, the sequence of bases along the A) only very long chains of DNA can be taken from an
10 chain is irregular. The monomer unit, consisting of organism with a spinal column.
phosphate, sugar and base, is known as a nucleotide. B) the main structure of a chain in a DNA molecule is
The first feature of our structure which is of composed of repeating units.
biological interest is that it consists not of one chain, C) a chain in a DNA molecule consists entirely of
but of two. These two chains are both coiled around phosphate groups or of sugars.
15 a common fiber axis. It has often been assumed that D) nitrogenous bases form the main structural unit of
since there was only one chain in the chemical DNA.
formula there would only be one in the structural
unit. However, the density, taken with the X-ray
evidence, suggests very strongly that there are two.
20 The other biologically important feature is the
manner in which the two chains are held together.
This is done by hydrogen bonds between the bases.
The bases are joined together in pairs, a single base
from one chain being hydrogen-bonded to a single
25 base from the other. The important point is that only
certain pairs of bases will fit into the structure.
One member of a pair must be a purine and the other
a pyrimidine in order to bridge between the two
chains. If a pair consisted of two purines, for
30 example, there would not be room for it.
We believe that the bases will be present almost
entirely in their most probable forms. If this is true,
the conditions for forming hydrogen bonds are more
restrictive, and the only pairs of bases possible are:
35 adenine with thymine, and guanine with cytosine.
Adenine, for example, can occur on either chain; but

Top Score
GET READY TO STUDY
Chapter 2 Author’s Indications

Practice
1
Critics of new media sometimes use science itself War and Peace in one sitting: “It was about Russia.”
to press their case, citing research that shows how Genuine multitasking, too, has been exposed as a
“experience can change the brain.” But cognitive myth, not just by laboratory studies but by the
neuroscientists roll their eyes at such talk. Yes, every familiar sight of an SUV undulating between lanes as
time we learn a fact or skill the wiring of the brain the driver cuts deals on his cell phone.
changes; it’s not as if the information is stored in the
pancreas. But the existence of neural plasticity does The author of Passage 2 refers to the novel
not mean the brain is a blob of clay pounded into War and Peace primarily to suggest that
shape by experience. Woody Allen
Experience does not revamp the basic A) did not like Tolstoy’s writing style.
information-processing capacities of the brain. B) could not comprehend the novel by speed-reading it.
Speed-reading programs have long claimed to do just C) had become quite skilled at multitasking.
that, but the verdict was rendered by Woody Allen D) regretted having read such a long novel.
after he read Leo Tolstoy’s famously long novel

2
15 The male element has held high carnival thus far; remember that man, who represents but half a
it has fairly run riot from the beginning, complete being, with but half an idea on every
overpowering the feminine element everywhere, subject, has undertaken the absolute control of all
crushing out all the diviner qualities in human 30 sublunary matters.
nature, until we know but little of true manhood and
20 womanhood, of the latter comparatively nothing, for Stanton uses the phrase “high carnival” (line 15)
it has scarce been recognized as a power until within mainly to emphasize what she sees as the
the last century. Society is but the reflection of man A) utter domination of women by men.
himself, untempered by woman’s thought; the hard B) freewheeling spirit of the age.
iron rule we feel alike in the church, the state, and the C) scandalous decline in moral values.
25 home. No one need wonder at the disorganization, at D) growing power of women in society.
the fragmentary condition of everything, when we
3
The motorcycle increased his status, gave him According to the passage, what does Nawab consider
weight, so that people began calling him “Uncle,” and to be the best result of getting the motorcycle?
asking his opinion on world affairs, about which he A) People start calling him “Uncle.”
88 knew absolutely nothing. He could now range B) He’s able to expand his business.
further, doing a much wider business. Best of all, C) He’s able to educate his daughters.
now he could spend every night with his wife, who D) He can spend more time with his wife.
had begged to live not on the farm but near her
family in Firoza, where also they could educate at
least the two eldest daughters.

4
What I don’t understand is why I The question the narrator asks in lines 30-31
am so intent on going to this particular place. Who (“Will it . . . railway”) most nearly implies that
wants the North Pole! What good is it! Can you eat A) balloons will never replace other modes of
30 it? Will it carry you from Gothenburg to Malm like transportation.
a railway? The Danish ministers have declared from B) the North Pole is farther away than the cities usually
their pulpits that participation in polar expeditions is reached by train.
beneficial to the soul’s eternal well-being, or so I read C) people often travel from one city to another without
in a newspaper. considering the implications.
D) reaching the North Pole has no foreseeable benefit to
humanity.

Top Score
GET READY TO STUDY
Chapter 2 Author’s Indications

5
To create its special herd of goats, GTC used 70 the protein. Et voilà—human medicine! And, for
55 microinjection, the same technique that produced GTC, liquid gold. ATryn hit the market in 2006,
GloFish and AquAdvantage salmon. The company’s becoming the world’s first transgenic animal drug.
scientists took the gene for human antithrombin and Over the course of a year, the “milking parlors” on
injected it directly into fertilized goat eggs. Then they GTC’s 300-acre farm in Massachusetts can collect
implanted the eggs in the wombs of female goats. 75 more than a kilogram of medicine from a single
60 When the kids were born, some of them proved to be animal.
transgenic, the human gene nestled safely in their
cells. The researchers paired the antithrombin gene Which of the following does the author suggest about
with a promoter (which is a sequence of DNA that the “female goats” mentioned in line 59?
controls gene activity) that is normally active in the A) They secreted antithrombin in their milk after giving
65 goat’s mammary glands during milk production. birth.
When the transgenic females lactated, the promoter B) Some of their kids were not born with the antithrombin
turned the transgene on and the goats’ udders filled gene.
with milk containing antithrombin. All that was left C) They were the first animals to receive microinjections.
to do was to collect the milk, and extract and purify D) Their cells already

6
When the transgenic females lactated, the promoter The phrase “liquid gold” (line 71) most directly
turned the transgene on and the goats’ udders filled suggests that
with milk containing antithrombin. All that was left A) GTC has invested a great deal of money in the
to do was to collect the milk, and extract and purify microinjection technique.
70 the protein. Et voilà—human medicine! And, for B) GTC’s milking parlors have significantly increased milk
GTC, liquid gold. ATryn hit the market in 2006, production.
becoming the world’s first transgenic animal drug. C) transgenic goats will soon be a valuable asset for dairy
Over the course of a year, the “milking parlors” on farmers.
GTC’s 300-acre farm in Massachusetts can collect D) ATryn has proved to be a financially beneficial product
75 more than a kilogram of medicine from a single for GTC.
animal.

7
About 750 years ago, a powerful volcano erupted The author uses the phrase “is written in” (line 6)
somewhere on Earth, kicking off a centuries-long most likely to
cold snap known as the Little Ice Age. Identifying the A) demonstrate the concept of the hands-on nature of the
volcano responsible has been tricky. work done by scientists.
5 That a powerful volcano erupted somewhere in B) highlight the fact that scientists often write about their
the world, sometime in the Middle Ages, is written in discoveries.
polar ice cores in the form of layers of sulfate C) underscore the sense of importance that scientists have
deposits and tiny shards of volcanic glass. regarding their work.
D) reinforce the idea that the evidence is there and can be
interpreted by scientists.

8
Another possible candidate—both in terms of As used in line 68, the phrase “Another possible candidate”
timing and geographical location—is Ecuador’s implies that
70 Quilotoa, estimated to have last erupted between A) powerful volcanic eruptions occur frequently.
1147 and 1320 C.E. But when Lavigne’s team B) the effects of volcanic eruptions can last for centuries.
examined shards of volcanic glass from this volcano, C) scientists know of other volcanoes that erupted during
they found that they didn’t match the chemical the Middle Ages.
composition of the glass found in polar ice cores, D) other volcanoes have calderas that are very large.
75 whereas the Samalas glass is a much closer match.
That, they suggest, further strengthens the case that
Samalas was responsible for the medieval “year
without summer” in 1258 C.E.

Top Score
GET READY TO STUDY
Chapter 2 Author’s Indications

9
Why do gift-givers assume that gift price is closely send a “stronger signal” to their intended recipient.
linked to gift-recipients’ feelings of appreciation? As for gift-recipients, they may not construe smaller
Perhaps givers believe that bigger (i.e., more and larger gifts as representing smaller and larger
expensive) gifts convey stronger signals of 65 signals of thoughtfulness and consideration.
55 thoughtfulness and consideration. According to
Camerer (1988) and others, gift-giving represents a The authors refer to work by Camerer and others
symbolic ritual, whereby gift-givers attempt to signal (line 56) in order to
their positive attitudes toward the intended recipient A) offer an explanation.
and their willingness to invest resources in a future B) introduce an argument.
60 relationship. In this sense, gift-givers may be C) question a motive.
motivated to spend more money on a gift in order to D) support a conclusion.

10
“Madame,” said Akira, “forgive my disruption, “Depending on your response, I may stay in
but I come with a matter of urgency.” Japan. I’ve come to ask for Naomi’s hand.”
His voice was soft, refined. He straightened and 60 Suddenly Chie felt the dampness of the night.
stole a deferential peek at her face. “Does Naomi know anything of your . . .
35 In the dim light his eyes shone with sincerity. ambitions?”
Chie felt herself starting to like him. “We have an understanding. Please don’t judge
“Come inside, get out of this nasty night. Surely my candidacy by the unseemliness of this proposal. I
your business can wait for a moment or two.” 65 ask directly because the use of a go-between takes
“I don’t want to trouble you. Normally I would much time. Either method comes down to the same
40 approach you more properly but I’ve received word thing: a matter of parental approval. If you give your
of a position. I’ve an opportunity to go to America, as consent, I become Naomi’s yoshi.* We’ll live in the
dentist for Seattle’s Japanese community.” House of Fuji. Without your consent, I must go to
“Congratulations,” Chie said with amusement. 70 America, to secure a new home for my bride.”
“That is an opportunity, I’m sure. But how am I Eager to make his point, he’d been looking her full
45 involved?” in the face. Abruptly, his voice turned gentle. “I see
Even noting Naomi’s breathless reaction to the I’ve startled you. My humble apologies. I’ll take no
name card, Chie had no idea. Akira’s message, more of your evening. My address is on my card. If
delivered like a formal speech, filled her with 75 you don’t wish to contact me, I’ll reapproach you in
maternal amusement. You know how children speak two weeks’ time. Until then, good night.”
50 so earnestly, so hurriedly, so endearingly about He bowed and left. Taking her ease, with effortless
things that have no importance in an adult’s mind? grace, like a cat making off with a fish.
That’s how she viewed him, as a child.
It was how she viewed Naomi. Even though In the passage, Akira addresses Chie with
Naomi was eighteen and training endlessly in the arts A) affection but not genuine love.
55 needed to make a good marriage, Chie had made no B) objectivity but not complete impartiality.
effort to find her a husband. C) amusement but not mocking disparagement.
Akira blushed. D) respect but not utter deference.

11
20 The other biologically important feature is the Based on the passage, the authors’ statement “If a
manner in which the two chains are held together. pair consisted of two purines, for example, there
This is done by hydrogen bonds between the bases. would not be room for it” (lines 29-30) implies that a
The bases are joined together in pairs, a single base pair
from one chain being hydrogen-bonded to a single A) of purines would be larger than the space
25 base from the other. The important point is that only between a sugar and a phosphate group.
certain pairs of bases will fit into the structure. B) of purines would be larger than a pair consisting
One member of a pair must be a purine and the other of a purine and a pyrimidine.
a pyrimidine in order to bridge between the two C) of pyrimidines would be larger than a pair of
chains. If a pair consisted of two purines, for purines.
30 example, there would not be room for it. D) consisting of a purine and a pyrimidine would be
larger than a pair of pyrimidines.

Top Score
GET READY TO STUDY
Chapter 2 Author’s Indications

12
Close at hand is a bridge over the River Thames, on the bridge, to ask ourselves certain questions.
an admirable vantage ground for us to make a And they are very important questions; and we have
survey. The river flows beneath; barges pass, laden very little time in which to answer them. The
with timber, bursting with corn; there on one side are 50 questions that we have to ask and to answer about
5 the domes and spires of the city; on the other, that procession during this moment of transition are
Westminster and the Houses of Parliament. It is a so important that they may well change the lives of
place to stand on by the hour, dreaming. But not all men and women for ever. For we have to ask
now. Now we are pressed for time. Now we are here ourselves, here and now, do we wish to join that
to consider facts; now we must fix our eyes upon the 55 procession, or don’t we? On what terms shall we join
10 procession—the procession of the sons of educated that procession? Above all, where is it leading us, the
men. procession of educated men? The moment is short; it
There they go, our brothers who have been may last five years; ten years, or perhaps only a
educated at public schools and universities, matter of a few months longer. . . . But, you will
mounting those steps, passing in and out of those 60 object, you have no time to think; you have your
15 doors, ascending those pulpits, preaching, teaching, battles to fight, your rent to pay, your bazaars to
administering justice, practising medicine, organize. That excuse shall not serve you, Madam.
transacting business, making money. It is a solemn As you know from your own experience, and there
sight always—a procession, like a caravanserai are facts that prove it, the daughters of educated men
crossing a desert. . . . But now, for the past twenty 65 have always done their thinking from hand to
20 years or so, it is no longer a sight merely, a mouth; not under green lamps at study tables in the
photograph, or fresco scrawled upon the walls of cloisters of secluded colleges. They have thought
time, at which we can look with merely an esthetic while they stirred the pot, while they rocked the
appreciation. For there, trapesing along at the tail cradle. It was thus that they won us the right to our
end of the procession, we go ourselves. And that 70 brand-new sixpence. It falls to us now to go on
25 makes a difference. We who have looked so long at thinking; how are we to spend that sixpence? Think
the pageant in books, or from a curtained window we must. Let us think in offices; in omnibuses; while
watched educated men leaving the house at about we are standing in the crowd watching Coronations
nine-thirty to go to an office, returning to the house and Lord Mayor’s Shows; let us think . . . in the
at about six-thirty from an office, need look passively 75 gallery of the House of Commons; in the Law Courts;
30 no longer. We too can leave the house, can mount let us think at baptisms and marriages and funerals.
those steps, pass in and out of those doors, . . . make Let us never cease from thinking—what is this
money, administer justice. . . . We who nowagitate “civilization” in which we find ourselves? What are
these humble pens may in another century or two these ceremonies and why should we take part in
speak from a pulpit. Nobody will dare contradict us 80 them? What are these professions and why
35 then; we shall be the mouthpieces of the divine should we make money out of them? Where in
spirit—a solemn thought, is it not? Who can say short is it leading us, the procession of the sons of
whether, as time goes on, we may not dress in educated men?
military uniform, with gold lace on our breasts,
swords at our sides, and something like the old Woolf uses the word “we” throughout the passage
40 family coal-scuttle on our heads, save that that mainly to
venerable object was never decorated with plumes of A) reflect the growing friendliness among a group
white horsehair. You laugh—indeed the shadow of of people.
the private house still makes those dresses look a B) advance the need for candor among a group of
little queer. We have worn private clothes so people.
45 long. . . . But we have not come here to laugh, or to C) establish a sense of solidarity among a group of
talk of fashions—men’s and women’s. We are here, people.
D) reinforce

Top Score
GET READY TO STUDY
Chapter 2 Author’s Indications
13
Follow the money and you will end up in space. In lines 9-17, the author of Passage 1 mentions
That’s the message from a first-of-its-kind forum on several companies primarily to
mining beyond Earth. A) note the technological advances that make space mining
Convened in Sydney by the Australian Centre for possible.
5 Space Engineering Research, the event brought B) provide evidence of the growing interest in space
together mining companies, robotics experts, lunar mining.
scientists, and government agencies that are all C) emphasize the large profits to be made from space
working to make space mining a reality. mining.
The forum comes hot on the heels of the D) highlight the diverse ways to carry out space mining
10 2012 unveiling of two private asteroid-mining firms. operations.
Planetary Resources of Washington says it will
launch its first prospecting telescopes in two years,
while Deep Space Industries of Virginia hopes to be
harvesting metals from asteroids by 2020. Another
15 commercial venture that sprung up in 2012,
Golden Spike of Colorado, will be offering trips to
the moon, including to potential lunar miners.

14
No man likes to acknowledge that he has made a 30 spreading denser shade daily, excluded me from
mistake in the choice of his profession, and every every glimpse of the sunshine of life; and I began to
man, worthy of the name, will row long against wind feel like a plant growing in humid darkness out of the
and tide before he allows himself to cry out, “I am slimy walls of a well.
5 baffled!” and submits to be floated passively back to
land. From the first week of my residence in X— The references to “shade” and “darkness” at the end
felt my occupation irksome. The thing itself—the of the first paragraph mainly have which effect?
work of copying and translating business-letters— A) They evoke the narrator’s sense of dismay.
was a dry and tedious task enough, but had that been B) They reflect the narrator’s sinister thoughts.
10 all, I should long have borne with the nuisance; I am C) They capture the narrator’s fear of confinement.
not of an impatient nature, and influenced by the D) They reveal the narrator’s longing for rest.
double desire of getting my living and justifying to
myself and others the resolution I had taken to
become a tradesman, I should have endured in
15 silence the rust and cramp of my best faculties; I
should not have whispered, even inwardly, that I
longed for liberty; I should have pent in every sigh by
which my heart might have ventured to intimate its
distress under the closeness, smoke, monotony, and
20 joyless tumult of Bigben Close, and its panting desire
for freer and fresher scenes; I should have set up the
image of Duty, the fetish of Perseverance, in my
small bedroom at Mrs. King’s lodgings, and they two
should have been my household gods, from which
25 my darling, my cherished-in-secret, Imagination, the
tender and the mighty, should never, either by
softness or strength, have severed me. But this was
not all; the antipathy which had sprung up between
myself and my employer striking deeper root and

Top Score
GET READY TO STUDY
Chapter 2 Author’s Indications
15
Antipathy is the only word which can express the At the end of the second paragraph, the comparisons
35 feeling Edward Crimsworth had for me—a feeling, in of abstract qualities to a lynx and a snake mainly
a great measure, involuntary, and which was liable to have the effect of
be excited by every, the most trifling movement, A) contrasting two hypothetical courses of action.
look, or word of mine. My southern accent annoyed B) conveying the ferocity of a resolution.
him; the degree of education evinced in my language C) suggesting the likelihood of an altercation.
40 irritated him; my punctuality, industry, and D) illustrating the nature of an adversarial relationship.
accuracy, fixed his dislike, and gave it the high
flavour and poignant relish of envy; he feared that I
too should one day make a successful tradesman.
Had I been in anything inferior to him, he would not
45 have hated me so thoroughly, but I knew all that he
knew, and, what was worse, he suspected that I kept
the padlock of silence on mental wealth in which he
was no sharer. If he could have once placed me in a
ridiculous or mortifying position, he would have
50 forgiven me much, but I was guarded by three
faculties—Caution, Tact, Observation; and prowling
and prying as was Edward’s malignity, it could never
baffle the lynx-eyes of these, my natural sentinels.
Day by day did his malice watch my tact, hoping it
60 would sleep, and prepared to steal snake-like on its
slumber; but tact, if it be genuine, never sleeps.

16
We suspect that the bees of commercial bee How do the words “can,” “may,” and “could” in the
20 colonies which are fed mono-crops are nutritionally third paragraph (lines 19-41) help establish the tone
deficient. In particular, we postulate that the problem of the paragraph?
is a diet deficient in anti-mite toxins: pyrethrums, A) They create an optimistic tone that makes clear the
and possibly other nutrients which are inherent in authors are hopeful about the effects of their research on
such plants. Without, at least, intermittent feeding on colony collapse disorder.
25 the pyrethrum producing plants, bee colonies are B) They create a dubious tone that makes clear the authors
susceptible to mite infestations which can become do not have confidence in the usefulness of the research
fatal either directly or due to a secondary infection of described.
immunocompromised or nutritionally deficient bees. C) They create a tentative tone that makes clear the authors
This secondary infection can be viral, bacterial or suspect but do not know that their hypothesis is correct.
30 fungal and may be due to one or more pathogens. D) They create a critical tone that makes clear the authors
In addition, immunocompromised or nutritionally are skeptical of claims that pyrethrums are inherent in
deficient bees may be further weakened when mono-crops.
commercially produced insecticides are introduced
into their hives by bee keepers in an effort to fight
35 mite infestation. We further postulate that the proper
dosage necessary to prevent mite infestation may be
better left to the bees, who may seek out or avoid
pyrethrum containing plants depending on the
amount necessary to defend against mites and the
40 amount already consumed by the bees, which in
higher doses could be potentially toxic to them.

Top Score
GET READY TO STUDY
Chapter 2 Author’s Indications

Answers
Exercise Practice
1- A 1- B
2- A 2- A
3- D 3- D
4- D 4- D
5- B 5- B
6- D
7- D
8- C
9- A
10- D
11- B
12- C
13- B
14- A
15- D
16- C

Top Score
GET READY TO STUDY
Chapter 3 Main Idea

Main Idea
In this type of questions the test maker asks you about the main idea of specific part of the passage or the passage as
whole or the author’s tone:
a. Main Purpose for Specific Part:
In this form the test maker asks you about the main idea of specific paragraph or specific lines of the
passage:
Ex:
The Alcazar Restaurant was on Sheridan Road page there was some marginal notation, either in ink
near Devon Avenue. It was long and narrow, with or in very hard pencil. And unless someone had
tables for two along the walls and tables for four upset a glass of water, the marks on page 177 were
down the middle. The decoration was art moderne, from tears.
except for the series of murals depicting the four
seasons, and the sick ferns in the front window. The main purpose of the first paragraph is to
Lymie sat down at the second table from the cash A) introduce the passage’s main character by showing
register, and ordered his dinner. The history book, his nightly habits.
which he propped against the catsup and the glass B) indicate the date the passage takes place by presenting
sugar bowl, had been used by others before him. period details.
Blank pages front and back were filled in with maps, C) convey the passage’s setting by describing a place
drawings, dates, comic cartoons, and organs of the and an object.
body; also with names and messages no longer clear D) foreshadow an event that is described in detail later in
and never absolutely legible. On nearly every other the passage.

To solve this question you must stick to these steps:


1- Read the answers first and carefully, selecting the main words of each.
A) Main character and his nightly habits.
B) Date and period details.
C) Setting, place and an object.
D) Event will be described later.
2- Read the specific part till you find that the idea of this part matches one of these answers.
By reading just 4 lines of this paragraph, we find that this paragraph describe the place where the
passage takes place, by describing Alcazar Restaurant. So, the answer is C
Exercise 1:
Another man might have thrown up his demur even when asked to fix watches, though that
hands—but not Nawabdin. His twelve daughters enterprise did spectacularly badly, and in fact earned
acted as a spur to his genius, and he looked with him more kicks than kudos, for no watch he took
satisfaction in the mirror each morning at the face of apart ever kept time again.
a warrior going out to do battle. Nawab of course
knew that he must proliferate his sources of The main purpose of the first paragraph is to
revenue—the salary he received from K. K. Harouni A) characterize Nawab as a loving father.
for tending the tube wells would not even begin to B) outline the schedule of a typical day in
suffice. He set up a little one-room flour mill, run off Nawab’s life.
a condemned electric motor—condemned by him. C) describe Nawab’s various moneymaking
He tried his hand at fish-farming in a little pond at ventures.
the edge of his master’s fields. He bought broken D) contrast Nawab’s and Harouni’s lifestyles.
radios, fixed them, and resold them. He did not

Top Score
GET READY TO STUDY
Chapter 3 Main Idea
1
noticeable. Mr. Peters’ hair was turning gray and his The primary impression created by the narrator’s
scalp showed through on top. He had lost weight description of Mr. Peters in lines 74-79 is that he is
also; he no longer filled out his clothes the way he A) healthy and fit.
used to. His color was poor, and the flower had B) angry and menacing.
disappeared from his buttonhole. In its place was an C) nervous and hesitant.
American Legion button. D) aging and shriveled.
2
No man likes to acknowledge that he has made a The main purpose of the opening sentence of the passage
mistake in the choice of his profession, and every man, is to
worthy of the name, will row long against wind and tide A) establish the narrator’s perspective on a controversy.
before he allows himself to cry out, “I am baffled!” and B) provide context useful in understanding the narrator’s
submits to be floated passively back to land. From the emotional state.
first week of my residence in X—felt my occupation C) offer a symbolic representation of Edward
irksome. Crimsworth’s plight.
D) contrast the narrator’s good intentions with his
malicious conduct.
3
There is yet another approach: instead of rooting The main purpose of the fifth paragraph (lines 45-56) is
ethics in character or the consequences of actions, we to
can focus on our actions themselves. From this A) develop a counterargument to the claim that greed is
perspective some things are right, some wrong—we good.
should buy fair trade goods, we shouldn’t tell lies in B) provide support for the idea that ethics is about
advertisements. Ethics becomes a list of character.
commandments, a catalog of “dos” and “don’ts.” C) describe a third approach to defining ethical
When a finance official refuses to devalue a currency economics.
because they have promised not to, they are defining D) illustrate that one’s actions are a result of one’s
ethics this way. According to this approach character
devaluation can still be bad, even if it would make
everybody better off.
4
Some of the largest ocean waves in the world are And they can reach staggering heights—some as tall
nearly impossible to see. Unlike other large waves, as skyscrapers.
these rollers, called internal waves, do not ride the
ocean surface. Instead, they move underwater, The first paragraph serves mainly to
undetectable without the use of satellite imagery or A) explain how a scientific device is used.
sophisticated monitoring equipment. Despite their B) note a common misconception about an event.
hidden nature, internal waves are fundamental parts C) describe a natural phenomenon and address its
of ocean water dynamics, transferring heat to the importance.
ocean depths and bringing up cold water from below. D) present a recent study and summarize its findings.

5
These human quirks mean we can never make The main idea of the final paragraph is that
purely “rational” decisions. A new wave of behavioral A) human quirks make it difficult to predict people’s
economists, aided by neuroscientists, is trying to ethical decisions accurately.
understand our psychology, both alone and in B) people universally react with disgust when faced with
groups, so they can anticipate our decisions in the economic injustice.
marketplace more accurately. But psychology can C) understanding human psychology may help to define
also help us understand why we react in disgust at ethics in economics.
economic injustice, or accept a moral law as D) economists themselves will be responsible for
universal. Which means that the relatively new reforming the free market.
science of human behavior might also define ethics
for us. Ethical economics would then emerge from
one of the least likely places: economists themselves.

Top Score
GET READY TO STUDY
Chapter 3 Main Idea
6
Ken settled on the Chukar Partridge as a they were safe from predators. They really only used
model species, but he might not have made his the ground for feeding and traveling. So he brought
discovery without a key piece of advice from the local in some hay bales for the Chukars to perch on and
rancher in Montana who was supplying him with then left his son in charge of feeding and data
birds. When the cowboy stopped by to see how collection while he went away on a short work trip.
things were going, Ken showed him his nice, tidy
laboratory setup and explained how the birds’ first In the second paragraph (lines 12-32), the incident
hops and flights would be measured. The rancher involving the local rancher mainly serves to
was incredulous. “He took one look and said, in A) reveal Ken Dial’s motivation for undertaking his
pretty colorful language, ‘What are those birds doing project.
on the ground? They hate to be on the ground! Give B) underscore certain differences between laboratory and
them something to climb on!’ ” At first it seemed field research.
unnatural—ground birds don’t like the ground? But C) show how an unanticipated piece of information
as he thought about it Ken realized that all the influenced Ken Dial’s research.
species he’d watched in the wild preferred to rest on D) introduce a key contributor to the tree-down theory.
ledges, low branches, or other elevated perches where

7
This hypothesis can best be tested by a trial The main purpose of the fourth paragraph (lines 42-50) is to
wherein a small number of commercial honey bee
colonies are offered a number of pyrethrum A) summarize the results of an experiment that confirmed
producing plants, as well as a typical bee food source the authors’ hypothesis about the role of clover in the
such as clover, while controls are offered only the diets of wild-type honeybees.
clover. Mites could then be introduced to each hive B) propose an experiment to investigate how different diets
with note made as to the choice of the bees, and the affect commercial honeybee colonies’ susceptibility to
effects of the mite parasites on the experimental mite infestations.
colonies versus control colonies. C) provide a comparative nutritional analysis of the honey
produced by the experimental colonies and by the
control colonies.
D) predict the most likely outcome of an unfinished
experiment summarized in the third paragraph (lines 19-
41).

8
For years, for a lifetime, the machinery The sentence in lines 10-13 (“For years . . . other”)
of my destiny has worked in secret to prepare for this mainly serves to
moment; its clockwork has moved exactly toward A) expose a side of the narrator that he prefers to keep
this time and place and no other. hidden.
B) demonstrate that the narrator thinks in a methodical
and scientific manner.
C) show that the narrator feels himself to be influenced
by powerful and independent forces.
D) emphasize the length of time during which the
narrator has prepared for his expedition.

9
When the kids were born, some of them proved to be The most likely purpose of the parenthetical
transgenic, the human gene nestled safely in their information in lines 63-64 is to
cells. The researchers paired the antithrombin gene A) illustrate an abstract concept.
with a promoter (which is a sequence of DNA that B) describe a new hypothesis.
controls gene activity) that is normally active in the C) clarify a claim.
goat’s mammary glands during milk production. D) define a term.

Top Score
GET READY TO STUDY
Chapter 3 Main Idea
10
We are not witnessing the abandonment of the young professionals moved to new downtown
suburbs, or a movement of millions of people back to condos in the recession years because few such
the city all at once. The 2010 census certainly did not residences were being built. But there is no reason to
turn up evidence of a middle-class stampede to the believe that the demographic trends prevailing prior
nation’s cities. The news was mixed: Some of the to the construction bust will not resume once that
larger cities on the East Coast tended to gain bust is over. It is important to remember that
population, albeit in small increments. Those in the demographic inversion is not a proxy for population
Midwest, including Chicago, tended to lose growth; it can occur in cities that are growing, those
substantial numbers. The cities that showed gains in whose numbers are flat, and even in those
overall population during the entire decade tended to undergoing a modest decline in size.
be in the South and Southwest. But when it comes to
measuring demographic inversion, raw census Which choice best summarizes the first paragraph of
numbers are an ineffective blunt instrument. A closer the passage (lines 1-35)?
look at the results shows that the most powerful A) The 2010 census demonstrated a sizeable growth in
demographic events of the past decade were the the number of middle-class families moving into
movement of African Americans out of central cities inner cities.
(180,000 of them in Chicago alone) and the B) The 2010 census is not a reliable instrument for
settlement of immigrant groups in suburbs, often measuring population trends in American cities.
ones many miles distant from downtown. C) Population growth and demographic inversion are
Central-city areas that gained affluent residents in distinct phenomena, and demographic inversion is
the first part of the decade maintained that evident in many American cities.
population in the recession years from 2007 to 2009. D) Population growth in American cities has been
They also, according to a 2011 study by Brookings, increasing since roughly 2000, while suburban
suffered considerably less from increased populations have decreased.
unemployment than the suburbs did. Not many
11
Emma Woodhouse, handsome, clever, and rich, Which choice best summarizes the first two paragraphs
with a comfortable home and happy disposition, of the passage (lines 1-14)?
seemed to unite some of the best blessings of A) Even though a character loses a parent at an early age,
existence; and had lived nearly twenty-one years in she is happily raised in a loving home.
the world with very little to distress or vex her. B) An affectionate governess helps a character to
She was the youngest of the two daughters of a overcome the loss of her mother, despite the
most affectionate, indulgent father, and had, in indifference of her father.
consequence of her sister’s marriage, been mistress of his C) Largely as a result of her father’s wealth and
house from a very early period. Her mother had affection, a character leads a contented life.
died too long ago for her to have more than an D) A character has a generally comfortable and fulfilling
indistinct remembrance of her caresses, and her life, but then she must recover from losing her
place had been supplied by an excellent woman as mother.
governess, who had fallen little short of a mother in
affection.

12
In this scenario, water mined from other What function does the discussion of water in
worlds could become the most desired commodity. lines 35-40 serve in Passage 1?
“In the desert, what’s worth more: a kilogram of gold A) It continues an extended comparison that begins in
or a kilogram of water?” asks Kris Zacny of the previous paragraph.
HoneyBee Robotics in New York. “Gold is useless. B) It provides an unexpected answer to a question raised
Water will let you live.” in the previous paragraph.
Water ice from the moon’s poles could be sent to C) It offers hypothetical examples supporting a claim
astronauts on the International Space Station for made in the previous paragraph.
drinking or as a radiation shield. Splitting water into D) It examines possible outcomes of a proposal put
oxygen and hydrogen makes spacecraft fuel, so forth in the previous paragraph.
ice-rich asteroids could become interplanetary
refuelling stations.

Top Score
GET READY TO STUDY
Chapter 3 Main Idea

b. Main Idea for the Passage as a Whole


Frequently, this question comes as the first question of the passage. But as we answer the exam we will it let as the
last question of the passage so as we can explore the most of the passage through answering another questions with
specific references. But if we want to answer this question as a first question of the passage, we must differentiate
between four types of the passages.
1- Historical, social, and scientific passages:
In these three types of the passage, the main idea MOSTLY found in the blurb and the first paragraph of the
passage. But it is preferable to let this question till the end of the passage.
2- Literature:
It is too hard to find the main idea of the passage exists in the first paragraph, so you need to read most of passage
or to answer the question as the last question of the passage.
Ex:

Earlier this year a series of papers in The Lancet University (the Meta-Research Innovation Center at
reported that 85 percent of the $265 billion spent each Stanford), which will seek to study research practices
year on medical research is wasted. This is not because and how these can be optimized. It will examine the best
of fraud, although it is true that retractions are on the means of designing research protocols and agendas to
rise. Instead, it is because too often absolutely nothing ensure that the results are not dead ends but rather that
happens after initial results of a study are published. No they pave a path forward.
follow-up investigations ensue to replicate or expand on The center will do so by exploring what are the best
a discovery. No one uses the findings to build new ways to make scientific investigation more reliable and
technologies. efficient. For example, there is a lot of interest on
The problem is not just what happens after collaborative team science, study registration, stronger
publication scientists often have trouble choosing the study designs and statistical tools, and better peer review,
right questions and properly designing studies to answer along with making scientific data, analyses and protocols
them. Too many neuroscience studies test too few widely available so that others can replicate experiments,
subjects to arrive at firm conclusions. Researchers thereby fostering trust in the conclusions of those studies.
publish reports on hundreds of treatments for diseases Reproducing other scientists’ analyses or replicating
that work in animal models but not in humans. Drug their results has too often in the past been looked down
companies find themselves unable to reproduce on with a kind of “me-too” derision that would waste
promising drug targets published by the best academic resources-but often they may help avoid false leads that
institutions. The growing recognition that something has would have been even more wasteful.
gone awry in the laboratory has led to calls for, as one
might guess, more research on research (aka, meta- The main purpose of the passage is to
research)-attempts to find protocols that ensure that A) argue that scientific studies need to be more
peer-reviewed studies are, in fact, valid. efficient.
B) describe the results of a scientific study.
It will take a concerted effort by scientists and other
C) explain the history of scientific inquiry.
stakeholders to fix this problem. We need to identify and D) highlight the fraudulent nature of many research
correct system-level flaws that too often lead us astray. studies.
This is exactly the goal of a new center at Stanford

When we read the 4 answers carefully and try to match one of which with the first paragraph, we will find that the
answer number A only answer can match the idea of the first paragraph which talks about the government’s
ineffective way of dealing with the findings of the scientific studies; therefore, much money is lost because of this
carelessness, and so these scientific studies need to be more efficient.

Top Score
GET READY TO STUDY
Chapter 3 Main Idea
Exercise 2:
1
MIT business scholars Erik Brynjolfsson and As evidence, Brynjolfsson and McAfee point to a
Andrew McAfee have argued that impressive chart that only an economist could love. In
advances in computer technology—from improved economics, productivity—the amount of economic
industrial robotics to automated translation value created for a given unit of input, such as an
services—are largely behind the sluggish hour of labor—is a crucial indicator of growth and
employment growth of the last 10 to 15 years. Even wealth creation. It is a measure of progress. On the
more ominous for workers, they foresee dismal chart Brynjolfsson likes to show, separate lines
prospects for many types of jobs as these powerful represent productivity and total employment in the
new technologies are increasingly adopted not only United States. For years after World War II, the
in manufacturing, clerical, and retail work but in two lines closely tracked each other, with increases in
professions such as law, financial services, education, jobs corresponding to increases in productivity. The
and medicine. pattern is clear: as businesses generated more value
That robots, automation, and software can replace from their workers, the country as a whole became
people might seem obvious to anyone who’s worked richer, which fueled more economic activity and
in automotive manufacturing or as a travel agent. But created even more jobs.
Brynjolfsson and McAfee’s claim is more troubling
and controversial. They believe that rapid The main purpose of the passage is to
technological change has been destroying jobs faster A) examine the role of technology in workers’ lives
than it is creating them, contributing to the during the last century.
stagnation of median income and the growth of B) advocate for better technology to enhance
inequality in the United States. And, they suspect, workplace conditions.
something similar is happening in other C) argue for changes in how technology is deployed
technologically advanced countries. in the workplace.
D) assess the impact of advancements in technology
on overall job growth.
2
Thanks to baby boomers, the population over 65 people, tend to have a higher disease burden and thus
will have doubled between 2000 and 2030. And higher medical needs. Sicker patients, along with high
when the Affordable Care Act takes full effect, up to labor costs, explain the higher levels of spending
32 million new patients will seek access to medical found in these urban areas—not too many doctors.
care, many of whom will need treatment for ailments There is no question that delivery of care needs
that have gone undiagnosed for years, such as to be better organized, and that some current reforms
cancer, diabetes, arthritis and heart disease. This are likely to improve patient outcomes. That’s true,
surge in demand means the U.S. will have a shortfall for example, with experiments in team-based care.
of at least 90,000 doctors by the end of the decade, However, these improvements in patient care have
according to the Association of American Medical not translated to any reduction in the need for
Colleges Center for Workforce Studies. Many parts physician time.
of the country have too few doctors already.
A small, vocal minority of researchers suggest Which of the following provides the best summary
we don’t need more doctors. That minority clearly is of the passage’s main idea?
having an impact: many clinicians and policy A) The US health care system is about to suffer a
makers say there is 20% to 30% “waste” in our significant collapse, and hundreds of hospitals
health-care system. Elliott Fisher, a Dartmouth will have to be shut down.
professor, says those numbers are backed up by B) There is too much wasteful spending in the
Dartmouth research. The Dartmouth studies base current health care system, which additional
their conclusions about waste on comparisons of doctors cannot correct.
health-care spending in different geographic areas. But C) The US needs to prepare for increasing health
other studies have shown that differences in the health care demands by training more doctors.
status of patients in the different regions explain the D) Accountable-care organizations (ACOs) will
majority of variations in spending. In other words, improve the current health care system and
urban areas, with their high concentrations of poor reduce unnecessary care.

Top Score
GET READY TO STUDY
Chapter 3 Main Idea
3
Is there definitive scientific proof that an organic effect repeated exposure to low levels of chemicals
diet is healthier? Not yet. Robust scientific studies might have on humans over time. And many
comparing food grown organically and food grown pesticides were eventually banned or restricted by
conventionally don’t exist, thanks to a lack of the federal government after years of use when they
funding for this kind of research in humans. were discovered to be harmful to the environment or
But let’s be clear: some convincing scientific human health.
work does exist to suggest that an organic diet has Organic skeptics like to cite a meta-analysis
its benefits. What’s more, it only makes sense that study published in the Annals of Internal Medicine
food free of pesticides and chemicals is safer and last year that suggested organic foods are neither
better for us than food containing those substances, healthier nor more nutritious than their conventional
even at trace levels. This was illustrated in a study counterparts.
published in the journal Environmental Health
Perspectives in 2006. That study, which I led, The author’s main purpose in Passage 1 appears to
showed that within five days of substituting mostly be to
organic produce in children’s diets for conventional A) discuss the implications of new research into
produce, pesticides disappeared from the children’s the health effects of organic foods.
urine. B) persuade readers that eating organic food has
Many say the pesticides found in our food are potential health benefits.
nothing to fear because the levels fall well below C) critique research which claims to show that
federal safety guidelines and thus aren’t dangerous. there are no health benefits from eating
Similarly, they say the bovine growth hormone used organic food.
to increase cows’ milk yield is perfectly safe. But D) argue that more funding is required to perform
federal guidelines don’t take into account what better research about organic food.

4
An insect with paper-thin wings may carry much the sound within two or three days. “But that’s not
the same defense technology as some of the what we found,” says Corcoran, explaining the lack
military’s heavy-duty warships. The finding that a of success bats had in capturing their clicking prey
species of tiger moth can jam the sonar of even through the last nights of the study.
echolocating bats to avoid being eaten seems to be How about the toxic warning theory? If this were
the “first conclusive evidence of sonar jamming in the case, according to Corcoran, bats would not find
nature,” says Aaron Corcoran, a biology PhD the moths palatable or, if they were indeed tasty,
student at Wake Forest University and the lead they would quickly learn they’d been tricked. Either
author of the paper reporting the discovery. “It way, bats should start to ignore the moth’s unique
demonstrates a new level of escalation in the batmoth ultrasonic clicks. Also, bats partook readily when
evolutionary arms race.” offered B. trigona that lacked the ability to click, and
Before Corcoran’s study, scientists were puzzled they kept coming back for more. This attraction also
by why certain species of tiger moths made sound. held true for clicking B. trigona: the predators
Some speculated that the moths use it to startle bats. persisted after their prey despite only reaching them
A few pointed to its potential interference with their about 20 percent of the time. Bats actually launched
echolocation. General consensus, however, fell with four times as many successful attacks against a
a third hypothesis: clicks function to warn a predator control group of silent moths. These findings are
not to eat the clicking prey because it is toxic, or at “only consistent with the jamming hypothesis,”
least pretending to be. Corcoran notes. “But the most distinctive evidence
To test these hypotheses, Corcoran and his team was in the echolocation sequences of the bats.”
pitted the tiger moth Bertholdia trigona against the
big brown bat Eptesicus fuscus, a battle frequently The passage is primarily concerned with
fought after sundown from Central America to A) the ways Eptesicus fuscus bats capture moths.
Colorado. High-speed infrared cameras and an B) the discovery that tiger moths can jam bats’
ultrasonic microphone recorded the action over nine sonar.
consecutive nights. The process of elimination C) how the tiger moths’ clicking defense works.
began. If moth clicks served to startle, previous D) why tiger moths developed defenses against
studies suggested the bats should become tolerant of bats.

Top Score
GET READY TO STUDY
Chapter 3 Main Idea
5
Men being, as has been said, by nature, all free, equal, majority: for that which acts any community, being only the
and independent, no one can be put out of this estate, and consent of the individuals of it, and it being necessary to
subjected to the political power of another, without his own that which is one body to move one way; it is necessary the
consent. The only way whereby any one divests himself of body should move that way whither the greater force carries
his natural liberty, and puts on the bonds of civil society, is it, which is the consent of the majority: or else it is
by agreeing with other men to join and unite into a impossible it should act or continue one body, one
community for their comfortable, safe, and peaceable living community, which the consent of every individual that
one amongst another, in a secure enjoyment of their united into it, agreed that it should; and so everyone is
properties, and a greater security against any, that are not of bound by that consent to be concluded by the majority.
it. This any number of men may do, because it injures not
the freedom of the rest; they are left as they were in the The main idea of Passage l is that
liberty of the state of nature. When any number of men A) it is safer for people to live in a state of nature than to
have so consented to make one community or government, consent to live in a community.
they are thereby presently incorporated, and make one body B) people who choose to become part of a community are
politic, wherein the majority have a right to act and subject to the will of the majority.
conclude the rest. C) minority opinions and majority opinions should be
For when any number of men have, by the consent of given equal consideration in community affairs.
every individual, made a community, they have thereby D) it is unrealistic to expect people to surrender their
made that community one body, with a power to act as one natural liberties when joining a community.
body, which is only b; the will and determination of the
6
Just as the Moon’s history was disrobed by laser ranging could directly assess the carbon stock of every single
50 years ago, Eartl’s tropical forests are giving up their square hectare of tropical forest on Earth. We could do it
secrets to the light. Airborne light detection and just as well as if we were standing there in the flesh with
rangingcalled LiDAR-has over the last ten years become tape measures in hand. And we could do it for far less
a key tool that ecologists use to understand physical than what we have already spent to offset carbon
variation in tropical forests across space and time. Like emissions from forests...
an MRI of the human brain, LiDAR probes the intricate It is easy in principle, though logistically nightmarish, to
three-dimensional architecture of the forest canopy, measure carbon in tropical forests. A strict
unveiling carbon that forests keep out of the atmosphere, constructionist would cut, dry and weigh the biomass of
and also the mounting threats to that carbon storehouse: the world’s forests. But this is a self-defeating enterprise.
drought, fire, clandestine logging and brash gold-mining As a result, it is likely that no one has measured carbon
operations. Even the quintessential natural disturbance of over a single hectare of tropical forest, even with the
the sun-filled light gap-long thought to enhance the most detailed field surveys. For a century ecologists and
incredibly high species diversity of tropical forests-has foresters have relied on allometric1 estimation in lieu of
been deconstructed by laser technology. carbon measurements to translate field surveys of tree
Laser ranging in tropical forests is such a game-changing diameters, heights and wood densities into whole-forest
technology that science results can scarcely get through carbon estimates. Given a volume with known
peerreview before they are dwarfed by still larger-scale dimensions and density, one would estimate its mass in a
studies. In a decade, laser power on commercial-grade similar fashion.
LiDARs has skyrocketed and costs have plummeted.
These improvements in LiDAR technology allow The author’ central claim in the passage is that
airplanes to fly faster, higher and farther, covering more A) LiDAR’s opponents have prevented the technology
forest area in a single day than every ground-based from advancing to a point where it might be
survey that has ever been collected in the history of scientifically useful, favoring traditional methods.
tropical ecology. To estimate the amount of carbon B) Fieldwork and LiDAR are best used in
stored in a 50-hectare tropical forest monitoring plot on combination when mapping carbon in tropical
the ground-the largest field plot in the world-takes a team forests, in order to avoid human error while
of 12 people about eight months: a slog of rain and mud maintaining accuracy
and snakes with tape measures and data log books. C) LiDAR is as important a technology as MRI
Today’s airborne LiDARs can get you to within about scanning or the scientific study of the moon with
10% of the same carbon estimate in eight seconds. lasers.
It is this staggering contrast in scale between LiDAR and D) LiDAR technology is faster, cheaper, and nearly
fieldwork that led us here: Before this decade is out, we as accurate as traditional field methods for
measuring the carbon biomass on Earth.

Top Score
GET READY TO STUDY
Chapter 3 Main Idea
7
A recent research collaboration has discovered that coins, cups, caps, or cars—can be the same as or
crows exhibit strong behavioral signs of analogical different from one another. Because sameness and
reasoning—the ability to solve puzzles like “bird is to air differentness can be detected visually, perhaps that may
as fish is to what?” Analogical reasoning is considered to provide an elegant way to study their apprehension by
be the pinnacle of cognition and it only develops in nonverbal animals.
humans between the ages of three and four. Why might To do so, we present visual stimuli on a touchscreen
crows be promising animals to study? Of course, crows monitor. We reward animals with food for contacting
are reputed to be clever. Aesop’s famous fable “The one button when sets contain identical items and we
Crow and the Pitcher” tells of a crow solving a reward animals for contacting a second button when sets
challenging problem: the thirsty crow drops pebbles into contain non-identical items. Several species of birds and
a pitcher with water near the bottom, thereby raising the mammals learn this task and also transfer their learning
fluid level high enough to permit the bird to drink. Such to new stimuli, showing that they have learned an
tales are charming and provocative, but science cannot abstract concept, which extends beyond the training
rely on them. Recent scientific research sought to items.
corroborate this fable.
It found that crows given a similar problem dropped The main purpose of the passage is to
stones into a tube containing water, but not into a tube A) present research approaches used to determine
containing sand. Crows also chose to drop solid rather that crows may be able to think analogically.
than hollow objects into the water tube.It thus seems that B) show that crows have been proven to be the
crows do indeed understand basic cause-effect relations. smartest of all non-primate animals.
But, what happens when crows are given problems that C) argue that crows need proper training in order to
require more abstract thinking? Before setting our sights perform complex and advanced tasks.
on analogical reasoning,we might begin with a simpler D) explain that scientists’ understanding of crows
abstract task. For example, sameness and differentness likely will progress rapidly following a recent
are key abstract ideas, because two or more items of any study.
kind—
8
Two X-ray space observatories, NASA's Nuclear NuSTAR, an Explorer-class mission launched in June
Spectroscopic Telescope Array (NuSTAR) and the 2012, is designed to detect the highest-energy X-ray light
European Space Agency's XMM-Newton, have teamed in great detail. It complements telescopes that observe
up to measure, for the first time, the spin rate of a black lowerenergy X-ray light, such as XMM-Newton and
hole with a mass two million times that of our sun. NASA's Chandra X-ray Observatory. Scientists use these
The supermassive black hole lies at the dust- and and other telescopes to estimate the rates at which black
gasfilled heart of a galaxy called NGC 1365, and it is holes spin.
spinning almost as fast as Einstein's theory of gravity Until now, these measurements were not certain
will allow.The findings,which appear in a new study in because clouds of gas could have been obscuring the
the journal Nature, resolve a long-standing debate about black holes and confusing the results. With help from
similar measurements in other black holes and will lead XMM-Newton, NuSTAR was able to see a broader range
to a better understanding of how black holes and galaxies of X-ray energies and penetrate deeper into the region
evolve. “This is hugely important to the field of black around the black hole. The new data demonstrate that X-
hole science,” said Lou Kaluzienski, a NuSTAR program rays are not being warped by the clouds, but by the
scientist at NASA Headquarters in Washington. tremendous gravity of the black hole. This proves that
The observations also are a powerful test of Einstein's spin rates of supermassive black holes can be determined
theory of general relativity,which says gravity can bend conclusively.
space-time, the fabric that shapes our universe, and the
light that travels through it. The main purpose of the passage is to
"We can trace matter as it swirls into a black hole A) contradict a longstanding scientific hypothesis.
using Xrays emitted from regions very close to the black B) discuss the findings and implications of a recent
hole," said the coauthor of a new study, NuSTAR study.
principal investigator Fiona Harrison of the California C) show how modern techno1ogy can improve
Institute of Teohnology in Pasadena. “The radiation we scientific observations.
see is warped and distorted by the motions of particles D) inform readers about the history of a
and the black hole's incredibly strong phenomenon.
gravity."

Top Score
GET READY TO STUDY
Chapter 3 Main Idea
9
There is currently a broad global movement away rising value, while air pollution and environmental
from considerations of mere economic success towards a damage lower the score. As a consequence, the
new public policy goal involving a broader notion of downsides of economic growth and modernization ought
quality of life. This movement has also spurred a to be accounted for whilst retaining the benefits of the
rethinking of which statistics inform us best about a GDP, namely a single figure that captures different
country’s situation and how its citizens are faring. For entities and is comparable across nations.
decades, the gold standard was a macroeconomic The second group of measures moves further away
indicator: the GDP - gross domestic product, calculated from the GDP as a yardstick than the previous
per capita. This is the most prominent yardstick that the approaches but does not abandon the sum of goods and
media, politicians and the public consider when they try services altogether. Instead, this group of measures seeks
to assess how a country is performing. However, this to assess national well-being by complementing the GDP
measure was never meant to be a measure of the welfare with a number of key social indicators.
of nations (as its creator Simon Kuznets warned in the For example, the Human Development Index
1930s) and so there is growing skepticism about the comprises the three dimensions health, education, and
GDP’s usefulness as a measure of national well-being. material living conditions, which are measured by life
Slogans such as “beyond GDP” or “redefining progress” expectancy, years of schooling, and GNI [gross national
challenge the preoccupation with the GDP. income], respectively.
Three key strategies have been employed to develop a
better measure of well-being: healing the GDP, A) replacing the GDP outright may seem appealing,
complementing the GDP, and replacing the GDP . but its alternatives would be difficult and
The first group of initiatives tries to deal with the dangerous to implement.
downsides of the GDP by attempting to fix the indicator B) well-being and economics have not been shown
itself. . . One key aim of this group of measures is to to correlate to any significant degree.
account for sustainability and the environmental damage C) the GDP can only measure economics, while
associated with GDP growth. For example, the Index of happiness must be considered primarily in terms
Sustainable Economic Welfare and the Genuine Progress of other factors.
Indicator are both based on the consumption of private D) there is a growing movement to improve the way
households. However, they also reflect additional social in which a country’s well-being is measured.
factors such as household labor and education with a
10
Online social networking sites, such as Facebook, Some researchers initially believed online activities
Google, and Bebo, have grown in popularity in recent negatively impacted relationships and feared that virtual
years and they provide an exciting new area of study in communication would replace face-to-face interactions
the field of psychology. Facebook provides individuals and deteriorate social bonds. Others, however, have
with easy access to view personal information about their found support for the idea that social networking sites
friends, coworkers, and even complete strangers. and the Internet have expanded methods of staying
Facebook has over 500 million active users and every socially connected with others and increased relationship
month over 700 billion minutes are spent on Facebook. closeness and connectedness. Social networking sites,
Among U.S. college students, 96% have a Facebook such as Facebook, allow users to add“friends” and keep
account. Given the popularity of online social track of their status, interests, photos, “likes,” and
networking sites, Facebook in particular, the current updates of others’ personal information in cyberspace.
study was designed to investigate the relationship
between Facebook use, the importance of quality of The primary purpose of the passage is to
friendship to self-esteem, and personality in college A) discuss the benefits and drawbacks of using social
students. media to maintain relationships.
Social relationships are considered by many to be the B) debate the merits of using social media as a
most important component of human life. With the primary form of interpersonal communication.
expansion of the Internet and social networking sites, C) emphasize the positive aspects behind the use of
more people are using technology to communicate with social media to cultivate relationships.
their friends and family online and maintain these D) raise questions about research findings regarding
interpersonal connections in novel ways that were not the negative impact of online activities on
available in previous generations. relationships.

Top Score
GET READY TO STUDY
Chapter 3 Main Idea

c. Focus Shifting
Sometimes, the author shifts his main focus of the passage from idea to another idea. So, in this question we must
identify the main topic of the first part of the passage and the main topic of the second part of the passage.

EX:
The Alcazar Restaurant was on Sheridan Road young boy but disguised by one trick or another
near Devon Avenue. It was long and narrow, with (rouge, lipstick, powder, wet bangs plastered against
tables for two along the walls and tables for four 50 the high forehead, and a pair of long pendent
down the middle. The decoration was art moderne, earrings) to look like a woman of thirty-five, which
5 except for the series of murals depicting the four as a matter of fact she was. The men were older. They
seasons, and the sick ferns in the front window. laughed more than there seemed any occasion for,
Lymie sat down at the second table from the cash while they were deciding between soup and shrimp
register, and ordered his dinner. The history book, 55 cocktail, and their laughter was too loud. But it was
which he propped against the catsup and the glass the women’s voices, the terrible not quite sober pitch
10 sugar bowl, had been used by others before him. of the women’s voices which caused Lymie to skim
Blank pages front and back were filled in with maps, over two whole pages without knowing what was on
drawings, dates, comic cartoons, and organs of the them. Fortunately he realized this and went back.
body; also with names and messages no longer clear 60 Otherwise he might never have known about the
and never absolutely legible. On nearly every other secret treaty concluded between England, France,
15 page there was some marginal notation, either in ink and Austria, when the pretensions of Prussia and
or in very hard pencil. And unless someone had Russia, acting in concert, seemed to threaten a
upset a glass of water, the marks on page 177 were renewal of the attack. The results of the Congress
from tears. 65 were stated clearly at the bottom of page 67 and at
While Lymie read about the Peace of Paris, signed the top of page 68, but before Lymie got halfway
20 on the thirtieth of May, 1814, between France and through them, a coat that he recognized as his
the Allied powers, his right hand managed again and father’s was hung on the hook next to his chair.
again to bring food up to his mouth. Sometimes he Lymie closed the book and said, “I didn’t think you
chewed, sometimes he swallowed whole the food that 70 were coming.”
he had no idea he was eating. The Congress of Time is probably no more unkind to sporting
25 Vienna met, with some allowance for delays, early in characters than it is to other people, but physical
November of the same year, and all the powers decay unsustained by respectability is somehow more
engaged in the war on either side sent noticeable. Mr. Peters’ hair was turning gray and his
plenipotentiaries. It was by far the most splendid and 75 scalp showed through on top. He had lost weight
important assembly ever convoked to discuss and also; he no longer filled out his clothes the way he
30 determine the affairs of Europe. The Emperor of used to. His color was poor, and the flower had
Russia, the King of Prussia, the Kings of Bavaria, disappeared from his buttonhole. In its place was an
Denmark, and Wurttemberg, all were present in American Legion button.
person at the court of the Emperor Francis I in the 80 Apparently he himself was not aware that there
Austrian capital. When Lymie put down his fork and had been any change. He straightened his tie
35 began to count them off, one by one, on the fingers self-consciously and when Irma handed him a menu,
of his left hand, the waitress, whose name was Irma, he gestured with it so that the two women at the next
thought he was through eating and tried to take his table would notice the diamond ring on the fourth
plate away. He stopped her. Prince Metternich (his 85 finger of his right hand. Both of these things, and
right thumb) presided over the Congress, and also the fact that his hands showed signs of the
40 Prince Talleyrand (the index finger) represented manicurist, one can blame on the young man who
France. had his picture taken with a derby hat on the back of
A party of four, two men and two women, came his head, and also sitting with a girl in the curve of
into the restaurant, all talking at once, and took 90 the moon. The young man had never for one second
possession of the center table nearest Lymie. deserted Mr. Peters. He was always there, tugging at
45 The women had shingled hair and short tight skirts Mr. Peters’ elbow, making him do things that were
which exposed the underside of their knees when not becoming in a man of forty-five.
they sat down. One of the women had the face of a

Top Score
GET READY TO STUDY
Chapter 3 Main Idea

Over the course of the passage, the primary focus shifts from
A) Lymie’s inner thoughts to observations made by the other characters.
B) an exchange between strangers to a satisfying personal relationship.
C) the physical setting of the scene to the different characters’ personality traits.
D) Lymie’s experience reading a book to descriptions of people in the restaurant.

1- Read each answer carefully and divide them into two ideas: one idea before the preposition "to", and
another idea after the preposition "to".

A) Lymie’s inner thoughts to observations made by the other characters.


B) an exchange between strangers to a satisfying personal relationship.
C) the physical setting of the scene to the different characters’ personality traits.
D) Lymie’s experience reading a book to descriptions of people in the restaurant.
The first part of the answer (before to) refers to the idea of the first part of the passage. The second part of the answer
(after to) refers to the idea of the second part of the passage.

2- Decide the main idea of the first part of the passage by skimming it.
3- Decide the main idea of the second part of the passage by skimming it.

N.B: We prefer to delay this question till we finish answering the rest of question so as we can understand
the passage better before skimming it.

4- Try to match the main idea of the first part of the passage with the first part of one of each answer
(before to).
5- Try to match the main idea of the second part of the passage with the second part of one of each
answer (after to).
When you read the first part of the passage carefully "paragraph 1 and 2", you will find that the main idea is
to show a man is preoccupied with reading a book: Lines (8–10) "The history book, which he propped
against the catsup and the glass sugar bowl, had been used by others before him." Lines (19-20) " While
Lymie read about the Peace of Paris, signed on the thirtieth of May"….etc.
And, When you read the second part of the passage carefully "paragraph 3 and 4", you will find that the
main idea is to describe the people in the restaurant and his father Mr. Peters: Lines (45-47) "The women
had shingled hair and short tight skirts which exposed the underside of their knees when they sat down."
Line (74-77) "Mr. Peters’ hair was turning gray and his scalp showed through on top. He had lost weight
also; he no longer filled out his clothes the way he used to."
So, the best answer here is (D).

Top Score
GET READY TO STUDY
Chapter 3 Main Idea
Exercise 3:
1
My emotions are complicated and not 35 be interpreted, except that the Pole is something
readily verifiable. I feel a vast yearning that is difficult or impossible to attain which must
simultaneously a pleasure and a pain. I am certain nevertheless be sought for, because man is
of the consummation of this yearning, but I don’t condemned to seek out and know everything
5 know yet what form it will take, since I do not whether or not the knowledge gives him pleasure. In
understand quite what it is that the yearning desires. 40 short, it is the same unthinking lust for knowledge
For the first time there is borne in upon me the full that drove our First Parents out of the garden.
truth of what I myself said to the doctor only an hour And suppose you were to find it in spite of all, this
ago: that my motives in this undertaking are not wonderful place that everybody is so anxious to stand
10 entirely clear. For years, for a lifetime, the machinery on! What would you find? Exactly nothing.
of my destiny has worked in secret to prepare for this 45 A point precisely identical to all the others in a
moment; its clockwork has moved exactly toward completely featureless wasteland stretching around it
this time and place and no other. Rising slowly from for hundreds of miles. It is an abstraction, a
the earth that bore me and gave me sustenance, I am mathematical fiction. No one but a Swedish madman
15 carried helplessly toward an uninhabited and hostile, could take the slightest interest in it. Here I am. The
or at best indifferent, part of the earth, littered with 50 wind is still from the south, bearing us steadily
the bones of explorers and the wrecks of ships, frozen northward at the speed of a trotting dog. Behind us,
supply caches, messages scrawled with chilled fingers perhaps forever, lie the Cities of Men with their
and hidden in cairns that no eye will ever see. teacups and their brass bedsteads. I am going forth of
20 Nobody has succeeded in this thing, and many have my own volition to join the ghosts of Bering and
died. Yet in freely willing this enterprise, in choosing 55 poor Franklin, of frozen De Long and his men.
this moment and no other when the south wind What I am on the brink of knowing, I now see, is
will carry me exactly northward at a velocity of not an ephemeral mathematical spot but myself. The
eight knots, I have converted the machinery of my doctor was right, even though I dislike him.
25 fate into the servant of my will. All this I understand, Fundamentally I am a dangerous madman, and what
as I understand each detail of the technique by which 60 I do is both a challenge to my egotism and a
this is carried out. What I don’t understand is why I surrender to it.
am so intent on going to this particular place. Who
wants the North Pole! What good is it! Can you eat Over the course of the passage, the narrator’s
30 it? Will it carry you from Gothenburg to Malm like attitude shifts from
a railway? The Danish ministers have declared from A) fear about the expedition to excitement about it.
their pulpits that participation in polar expeditions is B) doubt about his abilities to confidence in them.
beneficial to the soul’s eternal well-being, or so I read
C) uncertainty of his motives to recognition of them.
in a newspaper. It isn’t clear how this doctrine is to
D) disdain for the North Pole to appreciation of it.

Top Score
GET READY TO STUDY
Chapter 3 Main Idea
2
The “wisdom of crowds” has become a mantra of by other users.
the Internet age. Need to choose a new vacuum At least when it comes to comments on news
cleaner? Check out the reviews on online merchant sites, the crowd is more herdlike than wise.
Amazon. But a new study suggests that such online 55 Comments that received fake positive votes from the
5 scores don’t always reveal the best choice. A massive researchers were 32% more likely to receive more
controlled experiment of Web users finds that such positive votes compared with a control, the team
ratings are highly susceptible to irrational “herd reports. And those comments were no more likely
behavior”—and that the herd can be manipulated. than the control to be down-voted by the next viewer
Sometimes the crowd really is wiser than you. The 60 to see them. By the end of the study, positively
10 classic examples are guessing the weight of a bull or manipulated comments got an overall boost of about
the number of gumballs in a jar. Your guess is 25%. However, the same did not hold true for
probably going to be far from the mark, whereas the negative manipulation. The ratings of comments that
average of many people’s choices is remarkably close got a fake down vote were usually negated by an up
to the true number. 65 vote by the next user to see them.
15 But what happens when the goal is to judge “Our experiment does not reveal the psychology
something less tangible, such as the quality or worth behind people’s decisions,” Aral says, “but an
of a product? According to one theory, the wisdom intuitive explanation is that people are more
of the crowd still holds—measuring the aggregate of skeptical of negative social influence. They’re more
people’s opinions produces a stable, reliable 70 willing to go along with positive opinions from other
20 value. Skeptics, however, argue that people’s people.”
opinions are easily swayed by those of others. So Duncan Watts, a network scientist at Microsoft
nudging a crowd early on by presenting contrary Research in New York City, agrees with that
opinions—for example, exposing them to some very conclusion. “[But] one question is whether the
good or very bad attitudes—will steer the crowd in a 75 positive [herding] bias is specific to this site” or true
25 different direction. To test which hypothesis is true, in general, Watts says. He points out that the
you would need to manipulate huge numbers of category of the news items in the experiment had a
people, exposing them to false information and strong effect on how much people could be
determining how it affects their opinions. manipulated. “I would have thought that ‘business’ is
A team led by Sinan Aral, a network scientist at 80 pretty similar to ‘economics,’ yet they find a much
30 the Massachusetts Institute of Technology in stronger effect (almost 50% stronger) for the former
Cambridge, did exactly that. Aral has been secretly than the latter. What explains this difference? If we’re
working with a popular website that aggregates news going to apply these findings in the real world, we’ll
stories. The website allows users to make comments need to know the answers.”
about news stories and vote each other’s comments 85 Will companies be able to boost their products by
35 up or down. The vote tallies are visible as a number manipulating online ratings on a massive scale?
next to each comment, and the position of the “That is easier said than done,” Watts says. If people
comments is chronological. (Stories on the site get an detect—or learn—that comments on a website are
average of about ten comments and about three votes being manipulated, the herd may spook and leave
per comment.) It’s a follow-up to his experiment 90 entirely.
40 using people’s ratings of movies to measure how
much individual people influence each other online Over the course of the passage, the main focus shifts
(answer: a lot). This time, he wanted to know how from a discussion of an experiment and its results to
much the crowd influences the individual, and A) an explanation of the practical applications of the
whether it can be controlled from outside. results.
45 For five months, every comment submitted by a
B) a consideration of the questions prompted by the
user randomly received an “up” vote (positive); a
“down” vote (negative); or as a control, no vote at all. results.
The team then observed how users rated those C) an analysis of the defects undermining the
comments. The users generated more than results.
50 100,000 comments that were viewed more than D) a conversation with a scientist who disputes the
10 million times and rated more than 300,000 times results.

Top Score
GET READY TO STUDY
Chapter 3 Main Idea
3
Earlier this year a series of papers in The Lancet and how these can be optimized. It will examine the best
reported that 85 percent of the $265 billion spent each means of designing research protocols and agendas to
year on medical research is wasted. This is not because ensure that the results are
of fraud, although it is true that retractions are on the not dead ends but rather that they pave a path forward.
rise. Instead,it is because too often absolutely nothing The center will do so by exploring what are the best
happens after initial results of a study are published. No ways to make scientific investigation more reliable and
follow-up investigations ensue to replicate or expand on efficient.
a discovery. No one uses the findings to build new For example, there is a lot of interest on collaborative
technologies. team science, study registration, stronger study designs
The problem is not just what happens after and statistical tools, and better peer review, along with
publicationscientists often have trouble choosing the making scientific data, analyses and protocols widely
right questions and properly designing studies to answer available so that others can replicate experiments,
them. Too many neuroscience studies test too few thereby fostering trust in the conclusions of those studies.
subjects to arrive at firm conclusions. Researchers Reproducing other scientists’ analyses or replicating
publish reports on hundreds of treatments for diseases their results has too often in the past been looked down
that work in animal models but not in humans. Drug on with a kind of “me-too” derision that would waste
companies find themselves unable to reproduce resources-but often they may help avoid false leads that
promising drug targets published by the best academic would have been even more wasteful.
institutions. The growing recognition that something has
gone awry in the laboratory has led to calls for, as one Over the course of the passage, the main focus shifts
might guess, more research on research (aka, meta- from
research)-attempts to find protocols that ensure that A) an explanation of a phenomenon to a narrative
peer-reviewed studies are, in fact, valid. illustrating this phenomenon.
It will take a concerted effort by scientists and other B) the identification of a problem to a proposal for
stakeholders to fix this problem. We need to identify and solving this problem.
correct system-level flaws that too often lead us astray. C) a prediction for the future to an explanation
This is exactly the goal of a new center at Stanford underlying this prediction.
University (the Meta-Research Innovation Center at D) the introduction of an argument to a counterclaim
Stanford), which will seek to study research practices refuting this argument.

4
The world is very different now. For man holds in his whatever period is required-not because we seek their
mortal hands the power to abolish all forms of human votes, but because it is right. If a free society cannot help
poverty and all forms of human life. And yet the same the many who are poor, it cannot save the few who are
revolutionary beliefs for which our forebears fought are rich.
still, at issue around the globe. Let the word go forth Finally, to those nations who would make themselves
from this time and place, to friend and foe alike, that the our adversary, we offer not a pledge but a request: that
torch has been passed to a new generation of Americans- both sides begin anew the quest for peace, before the
born in this century, tempered by war, disciplined by a dark powers of destruction unleashed by science engulf
hard and bitter peace, proud of our ancient heritage, and all humanity in planned or accidental self-destruction.
unwilling to witness or permit the slow undoing of those So let us begin anew-remembering on both sides that
human rights to which this nation has always been civility is not a sign of weakness, and sincerity is always
committed, and to which we are committed today at subject to proof. Let us never negotiate out of fear, but
home and around the world. let us never fear to negotiate.
To those new states whom we welcome to the ranks of Let both sides, for the first time, formulate serious and
the free, we pledge our word that one form of colonial precise proposals for the inspection and control of arms,
control shall not have passed away merely to be replaced and bring the absolute power to destroy other nations
by a far more iron tyranny. We shall not always expect to under the absolute control of all nations.
find them supporting our view. But we shall always hope Let both sides seek to invoke the wonders of science
to find them strongly supporting their own freedom-and instead of its terrors. Together let us explore the stars,
to remember that, in the past, those who foolishly sought conquer the deserts, eradicate disease, tap the ocean
power by riding the back of the tiger ended up inside. depths, and encourage the arts and commerce.
To those people in the huts and villages of half the globe And, if a beachhead of cooperation may push back the
struggling to break the bonds of mass misery, we pledge jungle of suspicion,let both sides join in creating a new
our best efforts to help them help themselves, for

Top Score
GET READY TO STUDY
Chapter 3 Main Idea
endeavor-not a new balance of power, but a new world Over the course of the passage, the main focus shifts
of law-where the strong are just, and the weak secure, from
and the peace preserved. A) a description of the history of the United States to
All this will not be finished in the first one hundred a proposal for improving the lives of its citizens.
days. Nor will it be finished in the first one thousand B) a commitment to helping allies of the United
days; nor in the life of this Administration; nor even States to a call for better cooperation among
perhaps in our lifetime on this planet. But let us begin... opposing nations.
C) an argument for improving the lives of people
around the world to a recognition of that
argument’s weaknesses.
D) an explanation of the foreign policies of the
United States to an example illustrating these
policies.

5
The ability to travel mentally through time sets traveling mentally in time may initiate associated bodily
humans apart from many other species, yet little is movements through space. Initial evidence for such a
known about this core cognitive capacity. In particular, thought-action coupling during MTT was reported in a
what shapes the passage of the mind’s journey through study in which spontaneous fluctuations in the direction
time? and magnitude of postural sway were assessed while
A core facet of conscious experience is that one’s mind participants engaged in either retrospective or
periodically wanders from the here-and-now. From prospective mental imagery. The results revealed that the
memories of lost loves to expectations about forthcoming temporal locus of MTT did indeed influence the
vacations, mental time travel (MTT) makes it possible to direction of people’s movements - whereas retrospection
revisit the past and pre-experience the future. Present was accompanied by significant backwards sway,
across cultures and emerging early in childhood, MTT is prospection yielded postural movement in an anterior
believed to serve a pivotal function in human cognition. direction.
When confronted with complex and challenging
judgments, simulating future outcomes (i.e., prospection) Over the course of passage 1, the main focus shifts
on the basis of prior experience (i.e., retrospection) is a from
tactic that optimizes decision-making and behavioral A) a scientific study about mental time travel to an
selection. That the past informs the future in this way example of how humans use mental time travel in
(i.e., recollection-guides-simulation) is evidenced from their daily lives.
research demonstrating that retrospection and B) a description of the evolution of mental time
prospection rely on largely overlapping neural structures travel to an evaluation of its limitations in
and cognitive operations. humans.
However, remarkably little is known about the actual C) an explanation of mental time travel to a
process of MTT and how it impacts people’s behavior. In description of a study about its neurological basis.
this respect, one emerging possibility is that MTT may D) an argument in support of the existence of mental
be represented in the sensory-motor systems that regulate time travel to a counter-argument refuting its
human movement (i.e., MTT is embodied). Put simply, existence in humans.
2

Top Score
GET READY TO STUDY
Chapter 3 Main Idea
6
Magnetic resonance imaging (MRI) uses the body’s time needed for the axial spin to return to its resting
natural magnetic properties to produce detailed images state. The first is called T1 relaxation, the second is
from any part of the body. For imaging purposes the called T2 relaxation.
hydrogen nucleus (a single proton) is used because of its An MR examination is thus made up of a series of
abundance in water and fat. pulse sequences. Different tissues (such as fat and water)
The hydrogen proton can be likened to the planet have different relaxation times and can be identified
earth, spinning on its axis, with a north-south pole. In separately. By using a “fat suppression” pulse sequence,
this respect it behaves like a small bar magnet. Under for example,the signal from fat will be removed, leaving
normal circumstances, these hydrogen proton “bar only the signal from any abnormalities lying within it.
magnets” spin in the body with their axes randomly Most diseases manifest themselves by an increase in
aligned. When the body is placed in a strong magnetic water content, so MRI is a sensitive test for the detection
field, such as an MRI scanner, the proton’s axes all lined of disease. The exact nature of the pathology can be
up. This uniform alignment creates a magnetic vector more difficult to ascertain: for example, infection and
oriented along the axis of the MRI scanner. tumor can in some cases look similar. A careful analysis
MRI scanners come in different field strengths, of the images by a radiologist will often yield the correct
usually between 0.5 and 1.5 tesla. The strength of the answer.
magnetic field can be altered electronically from head to There are no known biological hazards of MRI
toe using a series of gradient electric coils, and, by because, unlike x ray and computed tomography, MRI
alerting the local magnetic field by these small uses radiation in the radiofrequency range which is found
increments different slices of the body will resonate as all around us and does not damage tissue as it passes
different frequencies are applied. When the through.
radiofrequency source is switched off the magnetic
vector returns to its resting state, and this causes a signal Over the course of the passage, the main focus shifts
(also a radio wave) to be emitted.It is this signal which is from
used to create the MR images. Receiver coils are used A) an explanation of how magnetic resonance
around the body part in question to act as aerials to imaging (MRI) works to a discussion of MRI’s
improve the detection of the emitted signal.The intensity practical advantages.
of the received signal is then plotted on a grey scale and B) a story about the origin of magnetic resonance
cross sectional images are built up. imaging (MRI) to a discussion of alternative
Multiple transmitted radiofrequency pulses can be imaging methods used by scientists.
used in sequence to emphasise particular tissues or C) an explanation of tissue relaxation rates to a
abnormalities. A different emphasis occurs because summary of the magnetic properties of hydrogen
different tissues relax at different rates when the protons.
transmitted radiofrequency pulse is switched off. The D) an introduction about the properties of
time taken for the protons to fully relax is measured in radiofrequency waves to a criticism of the health
two ways.The first is the time taken for the magnetic risks of x-ray devices.
vector to return to its resting state and the second is the

Top Score
GET READY TO STUDY
Chapter 3 Main Idea
7
For about 100 years, the scientific community has life and consciousness are examples of emergent
repeatedly changed its collective mind over what viruses complex systems. They each require a critical level of
are. First seen as poisons, then as life-forms, then complexity or interaction to achieve their respective
biological chemicals, viruses today are thought of as states. A neuron by itself, or even in a network of nerves,
being in a gray area between living and nonliving: they is not conscious-whole brain complexity is needed. A
cannot replicate on their own but can do so in truly living virus, too, fails to reach a critical complexity So life
cells and can also affect the behavior of their hosts itself is an emergent, complex state, but it is made from
profoundly. the same fundamental, physical building blocks that
The seemingly simple question of whether or not constitute a virus. Approached from this perspective,
viruses are alive has probably defied a simple answer all viruses, though not fully alive, may be thought of as
these years because it raises a fundamental issue: What being more than inert matter: they verge on life.
exactly defines “life?” A precise scientific definition of In fact, in October [2004], French researchers
life is an elusive thing, but most observers would agree announced findings that illustrate afresh just how close
that life includes certain qualities in addition to an ability some viruses might come. Didier Raoult and his
to replicate. For example, a living entity is in a state colleagues at the University of the Mediterranean in
bounded by birth and death. Living organisms also are Marseille announced that they had sequenced the
thought to require a degree of biochemical autonomy, genome of the largest known virus, Mimivirus, which
carrying on the metabolic activities that produce the was discovered in 1992. The virus, about the same size
molecules and energy needed to sustain the organism. as a small bacterium,infects amoebae.
This level of autonomy is essential to most definitions. Sequence analysis of the virus revealed numerous
Viruses, however, parasitize essentially all genes previously thought to exist only in cellular
biomolecular aspects of life. That is, they depend on the organisms. Some of these genes are involved in making
host cell for the raw materials and energy necessary for the proteins encoded by the viral DNA and may make it
nucleic acid synthesis, protein synthesis, processing and easier for Mimivirus to co-opt host cell replication
transport, and all other biochemical activities that allow systems. As the research team noted in its report in the
the virus to multiply and spread. One might then journal Science, the enormous complexity of the
conclude that even though these processes come under Mimivirus’s genetic complement “challenges the
viral direction, viruses are simply nonliving parasites of established frontier between viruses and parasitic cellular
living metabolic systems. But a spectrum may exist organisms.”
between what is certainly alive and what is not.
A rock is not alive. A metabolically active sack, devoid Over the course of the passage, the main focus shifts
of genetic material and the potential for propagation,is from
also not alive. A bacterium, though, is alive. Although it A) an analysis of historical issues affecting a scientific
is a single cell,it can generate energy and the molecules study to an assessment of that study
needed to sustain itself, and it can reproduce. But what B) the presentation of a question to an analysis of
about a seed? A seed might not be considered alive. Yet factors affecting its answer.
it has a potential for life, and it may be destroyed. In this C) a scientific query to an argument proving that the
regard, viruses resemble seeds more than they do live query is irrelevant and unnecessary
cells. D) a statement of scientific facts to an argument
Another way to think about life is as an emergent about the validity of those facts.
property of a collection of certain non-living things. Both

Top Score
GET READY TO STUDY
Chapter 3 Main Idea
8
Willow trees are well-known sources of salicylic acid, symptoms, the team analyzed leaf samples for the
and for thousands of years, humans have extracted the phytoplasma’s unique DNA fingerprint, which turned up
compound from the tree’s bark to alleviate minor pain, in 94 percent of samples from untreated plants but in
fever, and inflammation. only 47 percent of treated ones. Moreover, symptoms in
Now, salicylic acid may also offer relief to crop plants the treated group were far milder than in untreated
by priming their defenses against a microbial menace plants. In fact, analysis of mildly infected treated plants
known as “potato purple top phytoplasma.” Outbreaks of revealed phytoplasma levels 300 times below those of
the cell-wallless bacterium in the fertile Columbia Basin untreated plants, meaning that the salicylic acid
region of the Pacific Northwest in 2002 and subsequent treatment must have suppressed pathogen multiplication.
years inflicted severe yield and quality losses on potato Significantly, the remaining 53 percent of treated plants
crops. The Agricultural Research Service identified an were symptom- and pathogen-free 40 days after exposure
insect accomplice-the beet leafhopper, which transmits to the infected scions.
the phytoplasma to plants while feeding. Researchers credit salicylic acid with triggering
Carefully timed insecticide applications can deter such “systemic acquired resistance,” a state of general
feeding. But once infected, a plant cannot be cured. Now, readiness against microbial or insect attack. Using
a promising lead has emerged. An ARS-University of quantitative polymerase
Maryland team has found evidence that pretreating chain reaction procedures, the team also identified three
tomato plants, a relative of potato, with salicylic acid can regulatory defense genes whose activity was higher in
prevent phytoplasma infections or at least diminish their treated plants than in untreated ones.
severity Treating crops with salicylic acid to help them Why salicylic acid had this effect isn’t known. Other
fend off bacteria, fungi, and viruses isn’t new, but there questions remain as well, including how treated plants
are no published studies demonstrating its potential in will fare under field conditions. Nonetheless, such
preventing diseases caused by phytoplasmas. investigations could set the stage for providing growers
Wei Wu, a visiting scientist,investigated salicylic of potato, tomato, and other susceptible crops some
acid’s effects, together with molecular biologist Yan insurance against phytoplasmas in outbreak-prone
Zhao and others at ARS’s Molecular Plant Pathology regions.
Laboratory in Beltsville, Maryland. “This work reached
new frontiers by demonstrating that plants could be Over the course of the passage, the main focus shifts
beneficially treated even before they become infected from
and by quantifying gene activity underlying salicylic A) an overview of all research done to date on a
acid’s preventive role:' according to Robert E. Davis, the scientific topic to the future opportunities for
lab’s research leader. studying this topic.
For the study, published in the July 2012 Annals of B) background information needed to understand
Applied Biology, the team applied two salicylic acid an experiment to a description of the experiment
treatments to potted tomato seedlings. The first itself.
application C) a summary of the experiments leading to a
was via a spray solution 4 weeks after the seedlings were particular scientific discovery to a philosophical
planted. The second was via a root drench 2 days before discussion of the discovery’s implications.
phytoplasma-infected scions were grafted onto the plants' D) a description of a scientific inquiry to a
stems to induce disease. A control group of plants was description of the pivotal moments in solving a
not treated. mystery related to that inquiry.
In addition to visually inspecting the plants for disease
2

Top Score
GET READY TO STUDY
Chapter 3 Main Idea
9
Food is energy for the body. Digestive enzymes in the resistant. When cell walls hold strong, foods hoard their
mouth, stomach and intestines break up complex food precious calories and pass through our body intact (think
molecules into simpler structures, such as sugars and corn).
amino acids that travel through the bloodstream to all our Some plant parts have evolved adaptations either to
tissues. Our cells use the energy stored in the chemical make themselves more appetizing to animals or to evade
bonds of these simpler molecules to carry on business as digestion altogether. Fruits and nuts first evolved in the
usual. We calculate the available energy in all foods with Cretaceous
a unit known as the food calorie, or kilocalorie—the (between 145 and 65 million years ago), not long after
amount of energy required to heat one kilogram of water mammals were beginning to run between the legs of
by one degree Celsius. Fats provide approximately nine dinosaurs. Evolution favored fruits that were both tasty
calories per gram, whereas carbohydrates and proteins and easy to digest to better attract animals that could help
deliver just four. Fiber offers a piddling two calories plants scatter seeds. It also favored nuts and seeds that
because enzymes in the human digestive tract have great were hard to digest, however. After all, seeds and nuts
difficulty chopping it up into smaller molecules. need to survive the guts of birds, bats, rodents and
Every calorie count on every food label you have ever monkeys to spread the genes they contain.
seen is based on these estimates or on modest derivations Even foods that have not evolved to survive digestion
thereof. Yet these approximations assume that the differ markedly in their digestibility. Proteins may
19thcentury laboratory experiments on which they are require as much as five times more energy to digest as
based accurately reflect how much energy different fats because our enzymes must unravel the tightly wound
people with different bodies derive from many different strings of amino acids from which proteins are built. Yet
kinds of food. New research has revealed that this food labels do not account for this expenditure. Some
assumption is, at best, far too simplistic. To accurately foods such as honey are so readily used that our digestive
calculate the total calories that someone gets out of a system is hardly put to use.
given food, you would have to take into account a They break down in our stomach and slip quickly across
dizzying array of factors, including whether that food has the walls of our intestines into the bloodstream: game
evolved to survive digestion; how boiling, baking, over.
microwaving or flambéing a food changes its structure Finally, some foods prompt the immune system to
and chemistry; how much energy the body expends to identify
break down different kinds of food; and the extent to and deal with any hitchhiking pathogens. No one has
which the billions of bacteria in the gut aid human seriously evaluated just how many calories this process
digestion and, conversely, steal some calories for involves, but it is probably quite a few. A somewhat raw
themselves. piece of meat can harbor lots of potentially dangerous
Nutrition scientists are beginning to learn enough to microbes. Even if our immune system does not attack
hypothetically improve calorie labels, but digestion turns any of the pathogens in our food, it still uses up energy to
out to be such a fantastically complex and messy affair take the first step of distinguishing friend from foe. This
that we will probably never derive a formula for an is not to mention the potentially enormous calorie loss if
infallible calorie count. a pathogen in uncooked meat leads to illness.
Consider how vegetables vary in their digestibility.
We eat the stems, leaves and roots of hundreds of Over the course of the passage, the main focus shifts
different plants. The walls of plant cells in the stems and from
leaves of some species are much tougher than those in A) an overview of the human digestive tract to an
other species. Even within a single plant, the durability indepth report on how the stomach functions
of cell walls can differ. Older leaves tend to have sturdier B) an explanation of one approach to calories to a
cell walls than young ones. Generally speaking, the discussion on the need for a new approach
weaker or more degraded the cell walls in the plant C) a description of the evolution of plants to a list of
material we eat, the more calories we get from it. different types of plant species
Cooking easily ruptures cells in, say, spinach and D) a theory about the chemical composition of food
zucchini, but cassava (Manihot esculenta) or Chinese to examples supporting this theory
water chestnut (Eleocharis dulcis) is much more

Top Score
GET READY TO STUDY
Chapter 3 Main Idea
10
It seems like something out of a Robert Ludlum spy or it could be a seemingly legitimate phone call or e-mail
novel. Someone tries to coerce you into revealing your designed to coax out this information.
computer security passwords. You might be tempted to The researchers say they have tested their approach
give in, but it is impossible for you to reveal your on 370 players so far and continue to add new
authentication credentials. You do not actually know participants to their study. The test currently requires at
them because they are safely buried deep within your least 30 minutes of training to get reliable results. "It is
subconscious. unlikely that training time can be shrunk much because
Sounds a bit extreme just to make sure no one can log this type of brain memory takes time to get trained,"
on to your laptop or smartphone, but a team of Bojinov says. "It may be possible to reduce the
researchers from Stanford and Northwestern universities authentication time [that follows training], but it is yet to
as well as SRI International is nonetheless experimenting be seen how much." . . .
at the computer-, cognitive- and neuroscience Whether this approach is practical depends upon the
intersection to combat identity theft and shore up cyber system being defended. It is unlikely, for example, that
security-by taking advantage of the human brain’s innate Yahoo or Google would implement this approach to
abilities to learn and recognize patterns. security for their free e-mail services. Would someone
The researchers are studying ways to covertly create want to play a game for several minutes every time they
and store secret data within the brain's corticostriatal want to log onto their e-mail? A government facility
memory system, which is responsible for reminding us housing nuclear weapons, however, could better justify
how to do things. When a person needs to access a the time commitment required to log in using the
computer, network or some other secure system, they sequence learning method, particularly if users log in
would use special authentication software designed to once each day and such an approach promises to improve
tease out that secret data. security, says Nicolas Christin, associate director of
To test this concept, the researchers devised a Carnegie Mellon University's Information Networking
computer game requiring players to tap buttons on a Institute.
keyboard as large black dots descending down their
screen cross a horizontal line-very similar in concept to Over the course of the passage, the main focus shifts
the video game Guitar Hero. During an initial training from
session lasting from 30 minutes to an hour, the dots fall A) a rejection of hypothetical scenarios to the
at different speeds and in various locations, forming dismissal of these scenarios.
patterns that repeat until participants become adept at B) a description of a research study to a discussion
hitting the appropriate buttons at the right time. In effect, of the study’s possible applications.
users' corticostriatal memory becomes adept at repeating C) an explanation of scientific work to an argument
a particular pattern over time, such as dialing a phone about the ethics of using this work in different
number or typing a word on a keyboard without looking ways.
at one's fingers. D) the application of one scientific study to a
The researchers refer to this as "serial interception consideration of that study’s results for an entire
sequence learning" training, during which a person scientific field.
unwittingly learns a specific sequence of keystrokes that
can later be used to confirm that person's identity. To log
on to, for example, a Web site, the user would play the
game the same each time that pattern of dots appears,
proving his identity and allowing him access.
"While the planted secret can be used for
authentication, the participant cannot be coerced into
revealing it since he or she has no conscious knowledge
of it," according to the researchers in a study they
presented August 8 at the USENIX Security Symposium
in Bellevue, Wash. As currently conceived, the implicit
learning approach being studied might protect against
someone either forcing or tricking you to reveal a
password, says lead author Hristo Bojinov, a Stanford
University Ph.D. computer science candidate. Such
coercion could take the form of physical or verbal threats
demanding your password or other security credentials,

Top Score
GET READY TO STUDY
Chapter 3 Main Idea

Answers
Exercise 1 Exercise 2 Exercise 3
1- D 1- D 1- C
2- B 2- C 2- B
3- C 3- B 3- B
4- C 4- 4- B
5- C 5- B 5- C
6- C 6- D 6- A
7- B 7- A 7- B
8- C 8- B 8- B
9- D 9- D 9- B
10- C 10- A 10- B
11- A
12- C

Top Score
GET READY TO STUDY
Chapter 4 Vocabulary

Vocabulary
In vocabulary questions, we can solve the question by using one of two techniques:
a. Contextual Clue:
We can use the contextual clue especially with the modern English vocabulary because the modern English
vocabulary has many synonyms, some of which are near synonyms and some are far synonyms:
For example, the word SIMPLE. You couldn’t respond me directly if I asked you what does the word simple
mean? You must check the sentence in which the word SIMPLE exists to decide its meaning.
Ex1: “The yesterday’s exam was very SIMPLE as I solved all without one mistake.”
The word SIMPLE nearly means
A) Easy
B) Modest
C) Unadorned
D) Straight
For sure here the writer means the exam was easy.
EX2: “This man is very rich, but when you talk to him you will find him so SIMPLE”
The word SIMPLE nearly means
A) Easy
B) Modest
C) Unadorned
D) Straight
Here the author uses the word SIMPLE to describe the rich man as a modest man.
EX3: “I don’t like to wear a colorful t-shirts, and all my clothes are SIMPLE.
The word SIMPLE nearly means
A) Easy
B) Modest
C) Unadorned
D) Straight
The writer means that his clothes are without to much colors or drawings, so he means that his clothes are
unadorned.
EX: “This land is SIMPLE, without any lakes or hills.
The word SIMPLE nearly means
A) Easy
B) Modest
C) Unadorned
D) Straight
As per the context the writer means that the land is straight without lakes or hills

Top Score
GET READY TO STUDY
Chapter 4 Vocabulary
Exercise 1
1-
It seems to us incontestable that our common As used in line 21, “common” most nearly means
happiness, above all that of women, requires that A) average.
they never aspire to the exercise of political rights B) shared.
and functions. C) coarse.
D) similar.

2-
The researchers were also able to devise a As used in line 65, “devise” most nearly means
mathematical model that describes the movement A) create.
and formation of these waves. B) solve.
C) imagine.
D) begin.
3-
"Akira came directly, breaking all tradition. Was As used in line 1 and line 65, “directly” most
that it? Had he followed form—had he asked his nearly means
mother to speak to his father to approach a A) frankly.
go-between—would Chie have been more receptive?" B) confidently.
C) without mediation.
"I ask directly because the use of a go-between takes D) with precision.
much time."

4-
"Akira came directly, breaking all tradition. Was As used in line 2, “form” most nearly means
that it? Had he followed form—had he asked his A) appearance.
mother to speak to his father to approach a B) custom.
go-between—would Chie have been more receptive?" C) structure.
D) nature.

5-
involved today. Though disfranchised, we have few As used in line 36, “best” most nearly means
women in the best sense; we have simply so many A) superior.
reflections, varieties, and dilutions of the masculine B) excellent.
gender. C) genuine.
D) rarest.
6-
waves into account. “If we want to have more and As used in line 19, “capture” is closest in meaning to
more accurate climate models, we have to be able to A) control.
capture processes such as this,” Peacock says. B) record.
C) secure.
D) absorb.
7-
Lady Carlotta stepped out on to the platform of In line 2, “turn” most nearly means
the small wayside station and took a turn or two up A) slight movement.
and down its uninteresting length, to kill time till the B) change in rotation.
train should be pleased to proceed on its way. C) short walk.
D) course correction.

Top Score
GET READY TO STUDY
Chapter 4 Vocabulary
8-
During the drive to the Quabarl mansion In line 55, “charge” most nearly means
Lady Carlotta was impressively introduced to the A) responsibility.
nature of the charge that had been thrust upon her; B) attack.
she learned that Claude and Wilfrid were delicate, C) fee.
sensitive young people. D) expense.

9-
If you credit the demographers, this transit trend As used in line 58, “credit” most nearly means
has legs. The “Millenials,” who reached adulthood A) endow.
around the turn of the century and now outnumber B) attribute.
baby boomers, tend to favor cities over suburbs. C) believe.
D) honor.

10-
has legs. The “Millenials,” who reached adulthood As used in line 61, “favor” most nearly means
around the turn of the century and now outnumber A) indulge.
baby boomers, tend to favor cities over suburbs, and B) prefer.
are far more willing than their parents to ride buses C) resemble.
and subways. D) serve.

11-
wings and take short hops into the air. So when a As used in line 7, “challenged” most nearly means
group of graduate students challenged him A) dared.
to come up with new data on the age-old B) required.
ground-up-tree-down debate, he designed a project C) disputed with.
to see what clues might lie in how baby game birds D) competed with.
learned to fly.

12-
Ken called the technique WAIR, for wing-assisted As used in line 61, “document” most nearly means
incline running, and went on to document it in a A) portray.
wide range of species. It not only allowed young B) record.
C) publish.
D) process.

13-

Some argue that because the free markets allow we need to be clear on what we are talking about.
5 for personal choice, they are already ethical. Others As used in line 6, “embraced” most nearly means
have accepted the ethical critique and embraced A) lovingly held.
corporate social responsibility. But before we can B) readily adopted.
label any market outcome as “immoral,” or sneer at C) eagerly hugged.
10 economists who try to put a price on being ethical, D) reluctantly used.

14-

influence on general practice. And how can woman As used in line 50, “reason” most nearly means
be expected to co-operate unless she know why she A) motive.
ought to be virtuous? unless freedom strengthen her B) sanity.
reason till she comprehend her duty, and see in what C) intellect.
manner it is connected with her real good? D) explanation.

Top Score
GET READY TO STUDY
Chapter 4 Vocabulary
15-

Many moral dilemmas arise when these three As used in line 58, “clashes” most nearly means
versions pull in different directions but clashes are A) conflicts.
not inevitable. B) mismatches.
C) collisions.
D) brawls.

16-

the last century. Society is but the reflection of man As used in line 24, “rule” most nearly refers to
himself, untempered by woman’s thought; the hard A) a general guideline.
iron rule we feel alike in the church, the state, and the B) a controlling force.
home. C) an established habit.
D) a procedural method.

17-

History suggests that those will be hard lines to As used in line 68, “hold” most nearly means
hold, and it may be difficult to persuade the public A) maintain.
that such barren environments are worth preserving. B) grip.
C) restrain.
D) withstand.

18-

“Right,” she says and swiftly checks off a few more As used in line 16, “planting” most nearly means
boxes before planting her sweeping signature on the A) seeding.
bottom of the page. B) placing.
C) settling.
D) cultivating.
19-

Trees are felled to provide the logs for wood stoves, still As used in line 42, “form” most nearly means
a major form of heating in the tow ns and villages of A) structure.
rural France. B) measure.
C) shape.
D) method.
20-

Then, one warm morning, it was a Monday, she As used in line 61, “seized” most nearly means
remembered later, Absalom seized her hand and kissed A) clutched.
it. B) captivated.
C) overpowered.
D) afflicted.
21-

According to Smith, the ideal economic environment for As used in line 18, “open” most nearly means
a society is free-market capitalism, a structure in which A) unrestricted.
open competition among producers yields the best B) exposed.
products for consumers, manufactured and sold at the C) public.
D) accessible.
lowest possible prices.

Top Score
GET READY TO STUDY
Chapter 4 Vocabulary

b. Parts of the word:


We can use the parts of the word to identify the word’s meaning or to get the nearest meaning to it.

1- Prefixes:
The prefix added to the beginning of the word to change its meaning:
Ex:
increase: Go up ….. decrease: Go down / arch: control … monarch: one sided
control.

2- Suffixes:
The suffix added to the end of the word to determine its type: noun, verb, or adjective …..etc.
Ex:
Act + ive = adj (active) / Active + ate = verb (activate) / Activate + tion = noun (activation)
Happy = adj / Happily = adv / Happiness = noun

3- Roots:
The origin meaning of the word
Ex:
Oper = work
Operate – operation - cooperation – operator
Examples:
1- As used in line 16 “bilateral” nearly means to
(A) biological
(B) two-sided
(C) natural
(D) harmonious

The prefix "bi" means "two", so you have to choose the word that has any part refers to the word
"two", so the answer is (B) two-sided.

2- As used in line 17 “symmetry” nearly means to


(A) simplicity
(B) obstinacy
(C) appearance
(D) proportion

The prefix "sym" means "together or with", so you have to choose the word that has any part
refers to the word "with or together", so the answer is (D) proportion.

Top Score
GET READY TO STUDY
Chapter 4 Vocabulary
Exercise 2:
1- As used in line … “urbane” nearly means to 9- As used in line … “aggregate” nearly means to
(A) crowded (A) result
(B) polished (B) difference
(C) rural (C) product
(D) friendly (D) sum
2- As used in line … “decant” nearly means to 10- As used in line.. “therapeutic” nearly means to
(A) bisect (A) curative
(B) speak wildly (B) restful
(C) bequeath (C) warm
(D) pour off (D) stimulating
3- As used in line …“antithesis” nearly means to 11- As used in line .. “transmute” nearly means to
(A) contrast (A) remove
(B) conclusion (B) change
(C) resemblance (C) duplicate
(D) examination (D) carry
4- As used in line … “heretical” nearly means to 12- As used in line … “inoculate” nearly means to
(A) heathenish (A) make harmless
(B) impractical (B) infect
(C) quaint (C) cure
(D) unorthodox (D) overcome
5- As used in line … “coalesce” nearly means to 13- As used in line …“querulous” nearly means to
(A) associate (A) peculiar
(B) combine (B) fretful
(C) cover (C) inquisitive
(D) conspire (D) shivering
6- As used in line…“redundant” nearly means to 14- As used in line …“autonomy” nearly means to
(A) necessary (A) tyranny
(B) plentiful (B) independence
(C) diminishing (C) plebiscite
(D) superfluous (D) dictatorship
7- As used in line … “atrophy” nearly means to 15- As used in line“terminology”nearly means to
(A) soften (A) technicality
(B) waste away (B) finality
(C) grow (C) formality
(D) spread (D) nomenclature
8- As used in line … “analogy” nearly means to 16- As used in line …“invariable” nearly means to
(A) similarity (A) diverse
(B) transposition (B) eternal
(C) variety (C) inescapable
(D) distinction (D) uniform

Top Score
GET READY TO STUDY
Chapter 4 Vocabulary

Official Questions:
1- Test no 1 quest no 12: 4- Test no 3 quest no 16
"This frequent experience of gift-giving can "The “Millenials,” who reached adulthood
engender ambivalent feelings in gift-givers." around the turn of the century and now
outnumber baby boomers, tend to favor
In line 10, the word “ambivalent” most nearly cities over suburbs"
means
As used in line 61, “favor” most nearly
A) unrealistic.
means
B) conflicted.
A) indulge.
C) apprehensive.
B) prefer.
D) supportive.
C) resemble.
D) serve.
2- Test no 1 quest no 18:
"Perhaps givers believe that bigger gifts 5- Test no 3 quest no 47
convey stronger signals of thoughtfulness and "We further postulate that the proper
consideration." dosage necessary to prevent mite infestation
may be better left to the bees"
As it is used in line 54, “convey” most nearly
means As used in line 35, “postulate” most nearly
A) transport. means to
B) counteract. A) make an unfounded assumption.
C) exchange. B) put forth an idea or claim.
D) communicate. C) question a belief or theory.
D) conclude based on firm evidence.
3- Test no 2 quest no 25:
6- Test no 4 quest no 3
"We know that the human brain is highly
"My emotions are complicated and not
plastic; neurons and synapses change as
readily verifiable. I feel a vast yearning that
circumstances change."
is simultaneously a pleasure and a pain. I
am certain of the consummation of this
As used in line 40, “plastic” most nearly
yearning, but I don’t know yet what form it
means
will take"
A) creative.
B) artificial.
As used in lines 1-2, “not readily verifiable”
C) malleable.
most nearly means
D) sculptural.
A) unable to be authenticated.
B) likely to be contradicted.
C) without empirical support.
D) not completely understood.

Top Score
GET READY TO STUDY
Chapter 4 Vocabulary

BASIC WORD PARTS


In addition to reviewing the SAT High-Frequency Word List, what other quick vocabulary-building
tactics can you follow when you face an SAT dead- line?

One good approach is to learn how to build up (and tear apart) words. You know that words are
made up of other words: the room in which you store things is the storeroom; the person whose job
is to keep the books is the bookkeeper.

Just as words are made up of other words, words are also made up of word parts: prefixes,
suffixes, and roots. A knowledge of these word parts and their meanings can help you determine
the mean- ings of unfamiliar words.

Most modern English words are derived from Anglo-Saxon (Old English), Latin, and Greek.
Because few students nowadays study Latin and Greek (and even fewer study Anglo-Saxon!), the
majority of high school juniors and seniors lack a vital tool for unlocking the meaning of unfamiliar
words.

Build your vocabulary by mastering basic word parts. Learning thirty key word parts can help
you unlock the meaning of over 10,000 words. Learning fifty key word parts can help you unlock
the meaning of over 100,000!

Top Score
GET READY TO STUDY
Chapter 4 Vocabulary

COMMON PREFIXES
Prefixes are syllables that precede the root or stem of a word and change or refine its meaning.

Prefix Meaning Illustration


abduct lead away, kidnap
abjure renounce
ab, abs, atro from, away from, down
abject degraded, cast down
atrophy waste away
ambidextrous skilled with both hands
ambi both ambiguous of double meaning
ambivalent having two conflicting emotions
anarchy lack of government
an, a without anemia lack of blood
amoral without moral sense
Anagram similarity, arrangement
ana Again, same
Analogy similarity
antecedent preceding event or word antediluvian
ante before ancient (before the flood) ante-nuptial before the
wedding
antipathy hatred
anti against, opposite antiseptic against infection
antithetical exactly opposite
archetype original
arch chief, first archbishop chief bishop
archeology study of first or ancient times
bicameral composed of two houses (Congress)
bi two biennial every two years
bicycle two-wheeled vehicle
catastrophe disaster
cata down cataract waterfall
catapult hurl (throw down)
circumnavigate sail around (the globe)
circum around circumspect cautious (looking around)
circumscribe limit (place a circle around)
combine merge with
commerce trade with
communicate correspond with coeditor joint editor
com (co, col, con,
with, together collateral subordinate, connected
cor)
conference meeting
corroborate confirm

contravene conflict with


contra, contro against
controversy dispute

Top Score
GET READY TO STUDY
Chapter 4 Vocabulary

debase lower in value


de down, away decadence deterioration
decant pour off
dichotomy division into two parts
di two
dilemma choice between two bad alternatives
diagonal across a figure diameter distance across a
dia across
circle diagram outline drawing
discord lack of harmony
dis not, apart
disparity condition of inequality; difference
dyslexia faulty ability to read
dys faulty, bad
dyspepsia indigestion
ex, e out expel drive out extirpate root out eject throw out
extracurricular beyond the curriculum extraterritorial
extra, extro beyond, outside beyond a nation’s bounds extrovert person interested
chiefly in external objects and actions
Heresy unacceptable or different religious belief
here / heres against, different Heretic who reject a doctrine that is considered to be
correct
hyperbole exaggeration
hyper above; excessively
hyperventilate breathe at an excessive rate
hypo beneath; lower hypoglycemia low blood sugar
inefficient not efficient illegible
in (il, im, ir) not impeccable not capable of sinning; flawless
irrevocable not able to be called back
invite call in
in (il, im, ir) in, on, upon illustration something that makes clear impression
effect upon mind or feelings irradiate shine upon

inter between, among intervene come between international between nations

intramural within a school


intra, intro within
introvert person who turns within himself
macrobiotic tending to prolong life
macro large, long
macrocosm the great world (the entire universe)
megalomania delusions of grandeur
mega great, million
megaton explosive force of a million tons of TNT
meta involving change metamorphosis change of form
microcosm miniature universe microbe minute organism
micro small
microscopic extremely small

mischance unfortunate accident misnomer wrong name


mis / mal bad, improper
malevolent envious / bad man

misanthrope person who hates mankind


mis hatred
misogynist woman-hater

Top Score
GET READY TO STUDY
Chapter 4 Vocabulary

monarchy government by one ruler


mono one
monotheism belief in one god
multifarious having many parts
multi many
multitudinous numerous
neologism newly coined word
neo new
neophyte beginner; novice
noncommittal undecided
non not
nonentity person of no importance
ortho Straight, erect, hard, origin orthodox conventional, fanatic
panacea cure-all
pan all, every
panorama unobstructed view in all directions
permeable allowing passage through
per through, completely
pervade spread throughout
perimeter outer boundary
peri around, near
periphery edge
polygamist person with several spouses
poly many
polyglot speaking several languages
postpone delay
post after posterity generations that follow
posthumous after death
preamble introductory statement
pre before prefix word part placed before a root/stem
premonition forewarning
primordial existing at the dawn of time
prim first
primogeniture state of being the first born
propulsive driving forward
pro forward, in favor of, with
proponent supporter
proto first prototype first of its kind
pseudo FALSE pseudonym pen name
reiterate repeat
re again, back
reimburse pay back
retrospect looking back
retro backward
retroactive effective as of a past date
se away, aside secede withdraw seclude shut away seduce lead astray
semiannual every six months
semi half, partly
semiconscious partly conscious
sub (suc, suf, subway underground road subjugate bring under
under, less control succumb yield; cease to resist
sug, sup, sus)
supernatural above natural things
super, sur over, above supervise oversee
surtax additional tax
synchronize time together synthesize combine together
syn (sym, syl, sys) with, together
sympathize pity; identify with

Top Score
GET READY TO STUDY
Chapter 4 Vocabulary

syllogism explanation of how ideas relate


system network
telemetry measurement from a distance
tele far
telegraphic communicated over a distance
transport carry across
trans across
transpose reverse, move across
ultramodern excessively modern
ultra beyond, excessive
ultracritical exceedingly critical
unfeigned not pretended; real unkempt not combed;
un not
disheveled unwitting not knowing; unintentional
undergird strengthen underneath
under below
underling someone inferior
unison oneness of pitch; complete accord
uni one
unicycle one-wheeled vehicle
vicarious acting as a substitute
vice in place of
viceroy governor acting in place of a king
withhold hold back; keep
with away, against
withstand stand up against; resist

Top Score
GET READY TO STUDY
Chapter 4 Vocabulary

COMMON ROOTS AND STEMS


Roots are basic word elements that have been carried over into English. Stems are variations of
roots brought about by changes in declension or conjugation.
Root or Stem Meaning Illustration
acrimonious bitter; caustic
ac, acr sharp acerbity bitterness of temper
acidulate to make somewhat acid or sour
primeval of the first age
aev, ev age, era coeval of the same age or era
medieval or mediaeval of the middle ages
act deed
ag, act do
agent doer
demagogue false leader of people
agog leader
pedagogue teacher (leader of children)
agrarian one who works in the field agriculture
agri, agrari field
cultivation of fields
aggr sum, total Aggregate total
alias assumed (another) name
ali another
alienate estrange (turn away from another)
altitude height
alt high
altimeter instrument for measuring height
altruistic unselfish, considering others
alter other
alter ego a second self
amorous loving, especially sexually
am love amity friendship
amicable friendly
animadvert cast criticism upon
anim mind, soul unanimous of one mind
magnanimity greatness of mind or spirit
annuity yearly remittance
ann, enn biennial every two years
perennial present all year; persisting for several years
anthropology study of man
anthrop man misanthrope hater of mankind
philanthropy love of mankind; charity
aptitude skill
apt fit
adapt make suitable or fit
aqueduct passageway for conducting water
aqua water
aquatic living in water
archaeology study of antiquities (study of first things)
arch ruler, first monarch sole ruler
anarchy lack of government

Top Score
GET READY TO STUDY
Chapter 4 Vocabulary

astronomy study of the stars


aster star asterisk star-like type character (*)
disaster catastrophe (contrary star)
audible able to be heard
aud, audit hear
auditorium place where people may be heard
Authority rule, power
auth Power, reality
authentic
autocracy rule by one person (self)
auto self automobile vehicle that moves by itself
autobiography story of one’s own life
bellicose inclined to fight
belli war belligerent inclined to wage war
rebellious resisting authority
benefactor one who does good deeds
ben, bon good benevolence charity (wishing good)
bonus something extra above regular pay
bibliography list of books
biblio book bibliophile lover of books
Bible The Book
biography writing about a person’s life
biology study of living things
bio life
biochemist student of the chemistry of living
things
brevity briefness
breve short abbreviate shorten
decadent deteriorating
cad, cas to fall cadence intonation, musical movement
cascade waterfall
capture seize
cap (capt, cept,
to take participate take part
cip)
precept wise saying (originally a command)
decapitate remove (cut off) someone’s head
capit, capt head
captain chief
carnivorous flesh-eating
carn flesh carnage destruction of life
carnal fleshly

recede go back, withdraw


ced, cess to yield, to go
antecedent that which goes before process go forward

celerity swiftness
decelerate reduce swiftness
celer swift
accelerate increase swiftness

Top Score
GET READY TO STUDY
Chapter 4 Vocabulary

century one hundred years


cent one hundred centennial hundredth anniversary
centipede many-footed, wingless animal
chronology timetable of events
chron time anachronism a thing out of time sequence
chronicle register events in order of time
incision a cut (surgical)
cid, cis to cut, to kill homicide killing of a man
fratricide killing of a brother
incite stir up, start up
cit, citat to call, to start excite stir up
recitation a recalling (or repeating) aloud
civilization society of citizens, culture
civi citizen civilian member of community
civil courteous
clamorous loud
clam, clamat to cry out declamation speech
acclamation shouted approval
claustrophobia fear of close places
claud (claus,
to close enclose close in
clos,clud)
conclude finish
agnostic lacking knowledge, skeptical
cognosc, cognit to learn incognito traveling under assumed name cognition
knowledge
accord agreement (from the heart)
cord heart cordial friendly
discord lack of harmony
incorporate organize into a body
corpor body corporeal pertaining to the body, fleshly
corpse dead body
incredulous not believing, skeptical
cred, credit to believe credulity gullibility
credence belief
curator person who has the care of something
cur to care sinecure position without responsibility
secure safe
curr, curs to run excursion journey cursory brief precursor forerunner
debt something owed
deb, debit to owe
indebtedness debt debenture bond
democracy rule of the people
dem people demagogue (false) leader of the people
epidemic widespread (among the people)
epidermis skin
derm skin
pachyderm thick-skinned quadruped

Top Score
GET READY TO STUDY
Chapter 4 Vocabulary

dermatology study of skin and its disorders


diary a daily record of activities, feelings, etc.
di, diurn day
diurnal pertaining to daytime
abdicate renounce
dic, dict to say diction speech
verdict statement of jury
docile obedient; easily taught
doc, doct to teach document something that provides evidence
doctor learned person (originally, teacher)
dominate have power over
domin to rule
domain land under rule dominant prevailing
viaduct arched roadway
duc, duct to lead
aqueduct artificial waterway
dynamic powerful
dynam power, strength dynamite powerful explosive
dynamo engine making electrical power
egoist person who is self-interested
ego I egotist selfish person
egocentric revolving about self
energy power
erg, urg work
metallurgy science and technology of metals
error mistake
err to wander erratic not reliable, wandering
knight-errant wandering knight
eulogize praise
eu good, well, beautiful euphemism substitution of pleasant way of saying
something blunt
fallacious misleading
fall, fals to deceive infallible not prone to error, perfect
falsify lie
transfer bring from one place to another
fer, lat to bring, to bear translate bring from one language to another
conifer bearing cones, as pine trees
infidel nonbeliever, heathen
fid belief, faith
confidence assurance, belief
confine keep within limits
fin end, limit
finite having definite limits
flexible able to bend
flect, flex bend
deflect bend away, turn aside
fortuitous accidental, occurring by chance
fort luck, chance
fortunate lucky
fortitude strength, firmness of mind
fort strong
fortification strengthening

Top Score
GET READY TO STUDY
Chapter 4 Vocabulary

fortress stronghold
fragile easily broken
frag, fract break infraction breaking of a rule
fractious unruly, tending to break rules
fugitive someone who flees
fug flee
refuge shelter, home for someone fleeing
effusive gushing, pouring out
fus pour
diffuse widespread (poured in many directions)
monogamy marriage to one person
bigamy marriage to two people at the same time
gam marriage
polygamy having many wives or husbands at the same
time
genus group of animals with similar traits
gen, gener class, race generic characteristic of a class
gender class organized by sex
digress go astray (from the main point)
grad, gress go, step regress go backwards
gradual step by step, by degrees
epigram pithy statement
graph, gram writing telegram instantaneous message over great distance
stenography shorthand (writing narrowly)
gregarious tending to group together as in a herd
greg flock, herd aggregate group, total
egregious conspicuously bad; shocking
exit way out
it, itiner journey, road
itinerary plan of journey
projectile missile; something thrown forward
jac (jact, jec) to throw trajectory path taken by thrown object
ejaculatory casting or throwing out
perjure testify falsely
jur, jurat to swear
jury group of men and women sworn to seek the truth
laboratory place where work is done
labor, laborat to work collaborate work together with others
laborious difficult
election choice
leg (lect, lig) to choose, to read legible able to be read
eligible able to be selected
legislature law-making body
leg law legitimate lawful
legal lawful
library collection of books
liber, libr book libretto the “book” of a musical play
libel slander (originally found in a little book)
liber free liberation the fact of setting free

Top Score
GET READY TO STUDY
Chapter 4 Vocabulary

liberal generous (giving freely); tolerant


entomology study of insects
log word, study etymology study of word parts and derivations
monologue speech by one person
soliloquy speech by one individual
loqu, locut to talk loquacious talkative
elocution speech
elucidate enlighten
luc light lucid clear
translucent allowing some light to pass through
magnify enlarge
magn great magnanimity generosity, greatness of soul
magnitude greatness, extent
malevolent wishing evil
mal bad malediction curse
malefactor evil-doer
mallet Hammer / maul Malleable flexible, able to be hammered
manufacture create (make by hand)
man hand manuscript written by hand
emancipate free (let go from the hand)
maritime connected with seafaring
mar sea submarine undersea craft
mariner seaman
maternal pertaining to motherhood
mater, matr mother matriarch female ruler of a family, group, or state
matrilineal descended on the mother’s side
missile projectile
mit, miss to send dismiss send away
transmit send across
mobilize cause to move
mob (mot, mov) move motility ability to move
immovable not able to be moved
admonish warn
mon, monit to warn premonition foreboding
monitor watcher (warner)
mortuary funeral parlor
mori, mort to die moribund dying
immortal not dying
amorphous formless, lacking shape
morph shape, form metamorphosis change of shape
anthropomorphic in the shape of man
immutable not able to be changed
mut change mutate undergo a great change
mutability changeableness, inconstancy

Top Score
GET READY TO STUDY
Chapter 4 Vocabulary

nat born innate from birth prenatal before birth nativity birth
navigate sail a ship
nav ship circumnavigate sail around the world
naval pertaining to ships
negation denial
neg deny renege deny, go back on one’s word
renegade turncoat, traitor
nomenclature act of naming, terminology
nomen, nomin name nominal in name only (as opposed to actual)
cognomen surname, distinguishing nickname
novice beginner
nov new renovate make new again
novelty newness
omniscient all knowing omnipotent all powerful
omni all omnivorous eating everything
to work operate work
oper
cooperation working together
pacify make peaceful
pac peace pacific peaceful
pacifist person opposed to war
dispassionate free of emotion
pass feel impassioned emotion-filled
impassive showing no feeling
patriotism love of one’s country (fatherland)
pater, patr father patriarch male ruler of a family, group, or state
paternity fatherhood
pathology study of diseased tissue
path disease, feeling apathetic lacking feeling; indifferent
antipathy hostile feeling
impediment stumbling block; hindrance
ped, pod foot tripod three-footed stand
quadruped four-footed animal
pedagogue teacher of children
ped child
pediatrician children’s doctor
compulsion a forcing to do
pel, puls to drive repel drive back
expel drive out, banish
petition request
pet, petit to seek appetite craving, desire
compete vie with others
philanthropist benefactor, lover of humanity
phil love Anglophile lover of everything English
philanderer one involved in brief love affairs

Top Score
GET READY TO STUDY
Chapter 4 Vocabulary

to place postpone place after


pon, posit
positive definite, unquestioned (definitely placed)
portable able to be carried
port, portat to carry transport carry across
export carry out (of country)
omnipotent all-powerful
poten able, powerful potentate powerful person
impotent powerless
psychology study of the mind
psych mind psychosis mental disorder psychopath mentally ill
person
putative supposed (calculated)
put, putat to trim, to calculate computation calculation
amputate cut off
inquiry investigation
quer (ques, quir, inquisitive questioning query question
to ask
quis)
querulous complaining, whining, or fretful
regicide murder of a ruler
reg, rect rule regent ruler
insurrection rebellion; overthrow of a ruler
derision scorn
rid, ris to laugh
ridiculous deserving to be laughed at
rog, rogat to ask interrogate question
prerogative privilege
interrupt break into
rupt to break bankrupt insolvent rupture a break
sacred holy
sacr holy
sacrilegious impious, violating something holy
science knowledge
sci to know omniscient knowing all
conscious aware
periscope device for seeing around corners
scop watch, see
microscope device for seeing small objects
transcribe make a written copy
scrib, script to write script written text
circumscribe write around, limit
dissect cut apart
sect cut
bisect cut into two pieces
sedentary inactive (sitting)
sed, sess to sit
session meeting
consent agree
resent show indignation
sent, sens to think, to feel
sensitive showing feeling

Top Score
GET READY TO STUDY
Chapter 4 Vocabulary

consecutive following in order


sequi (secut, sequence arrangement
to follow
seque) sequel that which follows
non sequitur something that does not follow logically
absolve free from blame
solv, solut to loosen dissolute morally lax
absolute complete (not loosened)
insomnia inability to sleep
somn sleep somnolent sleepy
somnambulist sleepwalker
philosopher lover of wisdom
soph wisdom
sophisticated worldly wise
spectator observer
spec, spect to look at aspect appearance
circumspect cautious (looking around)
respiratory pertaining to breathing
spir breathe
spirited full of life (breath)
stringent strict
string, strict bind constrict become tight
stricture limit, something that restrains
constructive helping to build
stru, struct build
construe analyze (how something is built)
tangent touching
tang (tact, ting) to touch contact touching with, meeting
contingent depending upon
contemporary at same time
tempor time extemporaneous impromptu
temporize delay
tenable able to be held
ten, tent to hold tenure holding of office
retentive holding; having a good memory
interminable endless
term end
terminate end
terrestrial pertaining to earth
terr land
subterranean underground
thermostat instrument that regulates heat
therm heat
diathermy sending heat through body tissues
distort twist out of true shape or meaning
tors, tort twist torsion act of twisting
tortuous twisting
distract pull (one’s attention) away
tract drag, pull intractable stubborn, unable to be dragged
attraction pull, drawing quality

Top Score
GET READY TO STUDY
Chapter 4 Vocabulary

intrude push one’s way in


trud, trus push, shove
protrusion something sticking out
urban pertaining to a city
urbane polished, sophisticated (pertaining to a city
urb city
dweller)
suburban outside of a city
vacuous lacking content, empty-headed
vac empty
evacuate compel to empty an area
invade enter in a hostile fashion
vad, vas go
evasive not frank; eluding
intervene come between
veni (vent, ven) to come prevent stop
convention meeting
veracious truthful
ver TRUE verify check the truth
verisimilitude appearance of truth
verbose wordy
verb word verbiage excessive use of words
verbatim word for word
vertigo turning dizzy
vers, vert turn revert turn back (to an earlier state)
diversion something causing one to turn aside
deviation departure from the way
via way viaduct roadway (arched)
trivial trifling (small talk at crossroads)
vision sight
vid, vis to see evidence things seen
vista view
invincible unconquerable
vinc (vict, vanq) to conquer victory winning
vanquish defeat
vivisection operating on living animals
viv, vit alive vivacious full of life
vitality liveliness
avocation calling, minor occupation
voc, vocat to call provocation calling or rousing the anger of
invocation calling in prayer
malevolent wishing someone ill
vol wish
voluntary of one’s own will
revolve roll around
volv, volut to roll evolve roll out, develop
convolution coiled state

Top Score
GET READY TO STUDY
Chapter 4 Vocabulary

COMMON SUFFIXES
Suffixes are syllables that are added to a word. Occasionally, they change the meaning of the word; more
frequently, they serve to change the grammatical form of the word (noun to adjective, adjective to noun,
noun to verb).

Suffix Meaning Illustration


capable of (adjective portable able to be carried interminable not able to be
able, ible
suffix) limited legible able to be read
like, pertaining to cardiac pertaining to the heart aquatic pertaining to the
ac, ic
(adjective suffix) water dramatic pertaining to the drama
audacious full of daring
acious, icious full of (adjective suffix) perspicacious full of mental perception
avaricious full of greed
maniacal insane
pertaining to (adjective or
al final pertaining to the end
noun suffix)
logical pertaining to logic

full of (adjective or noun eloquent pertaining to fluid, effective speech suppliant


ant, ent
suffix) pleader (person full of requests) verdant green

dictionary book connected with words


like, connected with
ary honorary with honor
(adjective or noun suffix)
luminary celestial body
consecrate to make holy enervate to make weary
ate to make (verb suffix)
mitigate to make less severe
exasperation irritation
ation that which is (noun suffix)
irritation annoyance
democracy government ruled by the people
cy state of being (noun suffix) obstinacy stubbornness
accuracy correctness
mutineer person who rebels
eer (er, or) person who (noun suffix) lecher person who lusts
censor person who deletes improper remarks
evanescent tending to vanish
escent becoming (adjective suffix)
pubescent arriving at puberty
making, doing (adjective terrific arousing great fear
fic
suffix) soporific causing sleep
capable of (adjective portable able to be carried interminable not able to be
able, ible
suffix) limited legible able to be read
like, pertaining to cardiac pertaining to the heart aquatic pertaining to the
ac, ic
(adjective suffix) water dramatic pertaining to the drama
audacious full of daring
acious, icious full of (adjective suffix)
perspicacious full of mental perception

Top Score
GET READY TO STUDY
Chapter 4 Vocabulary

avaricious full of greed


maniacal insane
pertaining to (adjective or
al final pertaining to the end
noun suffix)
logical pertaining to logic

full of (adjective or noun eloquent pertaining to fluid, effective speech suppliant


ant, ent
suffix) pleader (person full of requests) verdant green

dictionary book connected with words


like, connected with
ary honorary with honor
(adjective or noun suffix)
luminary celestial body
consecrate to make holy enervate to make weary
ate to make (verb suffix)
mitigate to make less severe
exasperation irritation
ation that which is (noun suffix)
irritation annoyance
democracy government ruled by the people
cy state of being (noun suffix) obstinacy stubbornness
accuracy correctness
mutineer person who rebels
eer (er, or) person who (noun suffix) lecher person who lusts
censor person who deletes improper remarks
evanescent tending to vanish
escent becoming (adjective suffix)
pubescent arriving at puberty
making, doing (adjective terrific arousing great fear
fic
suffix) soporific causing sleep

Top Score
GET READY TO STUDY
Chapter 4 Vocabulary

Answers
Exercise 1 Exercise 2 Official Questions
1- B
1- B 1- B 2- D
2- A 2- D 3- C
3- C 3- A 4- B
4- B 4- D 5- B
5- B 5- B 6- D
6- B 6- D
7- C 7- B
8- A 8- A
9- C 9- D
10- B 10- A
11- A 11- B
12- B 12- B
13- B 13- B
14- C 14- B
15- A 15- D
16- B 16- D
17- A
18- B
19- D
20- A
21- A

Top Score
GET READY TO STUDY

You might also like